Resumenes Proedumed Enarm Xl: Temas

Resumen cirugía ENARMDescripción completa

Views 332 Downloads 16 File size 8MB

Report DMCA / Copyright

DOWNLOAD FILE

Recommend stories

Citation preview

RESUMENES PROEDUMED    ENARM XL      “El cansancio es temporal, el orgullo es para siempre”    TEMAS  Patología esofágica    Acalasia      Enfermedad por reflujo gastroesofágico    Esófago de Barret  Enfermedad ácido péptica y sus complicaciones    Úlcera péptica  Colecistitis, colelitiasis y sus complicaciones    Colecistitis    Colelitiasis  Sangrado de tubo digestivo alto y bajo    Hemorragia digestiva    Várices esofágicas  Patología abdominal (resolución quirúrgica de urgencia)    Abdomen agudo*    Apendicitis*    Colecisitits*    Coledocolitiasis y colangitis*    Enfermedad diverticular*    Hemorragia de tubo digestivo alto y bajo*    Hernia inguinal y femoral*    Hernia hiatal*    Hernia umbilical y de pared*    Oclusión intestinal en adultos*    Volvulus  Alteraciones electrolíticas y ácido básicas  Pancreatitis    Pancreatitis aguda  Infecciones óseas y articulares    Ostemielitis    Artritis séptica  Trauma de miembros    Trauma de codo    Esguince de tobillo    Fractura intracapsula del exgtremo proximal del femur    Sx de hombro doloroso  Cervicalgias, lumbalgias y tumores    Cervicalgia    Lumbalgia    Osteocarcoma    Sarcoma de Ewing  Patología articulares    Luxación glenohumeral    Luxación de cadera    Halux Valgus    Lesiones comunes en rodilla    Pie plano    Displasia del desarrollo de la cadera  Patología de oído externo    Otitis externa  Osteomielitis de la base del cráneo  Otitis media aguda    Otitis media crónica    Hematoma auricular  Patología del oido interno    Hipoacusia  Presbiacusia  Otosclerosis 

Patología del sistema vestibular    Generalidades de vértigo*  Vértigo postural paroxístico*  Neuronitis vestibular*    Enfermedad de Meniere*  Patología de la nariz y senos paranasales    Epistaxis    Sinusitis aguda y crónica  Patología de faringe y laringe    Faringoamigalitis aguda    Celulitis y abscesos periamigdalinos    Procesos inflamatorios de laringe    Tumores laringeos    Quiste de conducto tirogloso    Quistes faringeos  Enfermedades corneales y de la órbita    Celulitis y abscesos periorbitarios  Catarata    Catarata  Enfermedades de la retina  Retinopatía diabética    Retinopatías vasculares e hipertensivas    Glaucoma    Glaucoma de ángulo abierto    Glaucoma de ángulo cerrado  Enfermedades de los párpados, aparato lagrimal y conjuntiva    Conjuntivitis generalidades  Conjuntivitis bacteriana del adulto*    Conjuntivitis neonatal*    Toxoplasmosis*    Tracoma*    Orzuelo y chalazión*    Celulitis preseptal y orbitaria*    Perlas de oftalmología*  Patología de colon, ano y recto    Físura anal*  Absceso anorrectal*    Fístula anal*    Incontinencia fecal*    Hemorroides*    Cáncer de colon y recto    Diverticulosis    Trombosis mesentérica*  Cálculo uretrales y uropatía obstructiva    Cálculos urinarios    Urolitiasis add  Incontinencia urinaria    Incontinencia urinaria  Patología de próstata    Cáncer de próstata  Hiperplasia benigna de próstata  *  Prostatitis*    Epididimitis*    Disfunción eréctil    Torsión testicular*    Criptorquidia*  Trauma craneoencefálico    Trauma cráneoencefálico  Trauma de cuello    Esguine cervical  Trauma de tórax    Trauma de tórax    Taponamiento cardiaco    Neumotórax    Hemotórax  Trauma abdominal 

  Trauma abdominal cerrado y abierto  Choque    Choque hipovolémico traumático      Angiología  Insuficiencia arterial aguda*  Enfermedad arterial periférica*  Síndrome de reperfusión*  Síndrome compartimental*  Criterios de amputación*  Enfermedad carotídea*  Insuficiencia venosa*  Enfermedad tromboembólica venosa*  Trombosis venosa profunda*  Pie diabético*  Secuela postrombótica*  Várices*         

   

   

   

PATOLOGÍA ESOFÁGICA  ACALASIA  DEFINICIÓN  Aperistalsis del cuerpo esofágico, Ausencia de relajación del EEI    SALUD PÚBLICA  20‐50 años de edad.    DIAGNÓSTICO  Disfagia temprana, progresiva, a sólidos y líquidos, 2 años antes de buscar atención.    Pérdida de peso variable.  Regurgitación y vómito son utilizados para vaciar el esófago, puede haber broncoaspiración (complicación broncopulmonar más  frecuente [10%].  Prueba inicial: Esofagograma de bario [imagen en punta de lápiz]  Estándar de oro: manometria [ausencia de relajación del EEI]  Endoscopia: descara pseudoacalasia, acalasia secundaria.      TRATAMIENTO  Incurable y el objetivo es el manejo de los síntomas.   

Médico nitratos, bloqueadores de canales de calcio vía SL.  Dilataciones endoscópica  Miotomía de Heller con funduplicatura (de elección)   

ACALASIA  [r]HOMBRE DE 30 AÑOS DE EDAD, ACUDE POR PRESENTAR DISFAGIA Y REGURGITACIÓN PROGRESIVA. USTED SOSPECHA LA POSIBILIDAD  DE ACALASIA.  EL ESTUDIO QUE DEBERÁ SOLICITAR PARA CONFIRMAR SU SOSPECHA DIAGNÓSTICA ES:  A) UN ESOFAGOGRAMA CON BARIO.  B) UNA ENDOSCOPÍA.  C) UNA PH METRÍA DE 24 HRS.  D) UNA MANOMETRÍA.    EN CASO DE CONFIRMAR SU DIAGNÓSTICO EL PACIENTE DEBERÁ SER TRATADO CON  A) ANTAGONISTAS DE CALCIO.  B) TOXINA BOTULINICA.  D) DILATACIONES NEUMÁTICAS.  E) MIOTOMÍA.    RESPUESTAS D, E  Para el ENARM es muy importante comprender qué es lo que te están preguntando.    El estudio inicial en paciente con acalasia es el esofagograma  con bario, el cual demustra la imagen clásica en "punta de lápiz". Este  estudio orienta al diagnóstico pero no lo confirma. La manometría es el estudio ideal para confirmarlo y demostrar ausencia de relajación  del esfinter esofágico inferior y aperistalsis del cuerpo esofágico. Hay que recordar que en estadios iniciales aun no es posible encontrar  la clásica imagen en el esofagograma.  La endoscopia y la TAC son estudios útiles para diferenciar de acalasia de pseudoacalasia.  La pHmetría en 24 hrs mide el reflujo ácido y es gold standar de ERGE, sin utilidad diagnóstica en acalasia.  La miotomía y las dialtaciones son los únicos tratamiento duraderos en acalasia, y son más efectivos en pacientes vírgenes a tratamientos  médicos. La miotomía de Heller con funduplicatura parcial tiene menor riesgo y es más eficaz que las dilatacines, por lo que en este  paciente es gold standar.        UN HOMBRE DE 65 AÑOS DE EDAD SE PRESENTA A SU MÉDICO QUEJÁNDOSE DE DIFICULTAD PARA DEGLUTIR, DOLOR EN EL PECHO DE  VEZ EN CUANDO, Y REGURGITACIÓN DE ALIMENTOS. EN LOS ÚLTIMOS 2 MESES HA PERDIDO ALREDEDOR DE 7 KG. LOS RESULTADOS DE  UN ESTUDIO DE DEGLUCIÓN DE BARIO SE MUESTRAN EN LA IMAGEN. ¿QUÉ PRUEBA SE DEBE REALIZAR PARA BUSCAR POSIBLES CAUSAS  DE SU CONDICIÓN? 

 

  A. 

PHMETRÍA DE 24 HORAS  B. MANOMETRÍA ESOFÁGICA  C. MEDICIÓN DEL NIVEL DE GASTRINA SÉRICA  D. ENDOSCOPIA SUPERIOR  E. PRUEBA DEL ALIENTO CON UREA  RESPUESTA D  (YO)           

ENFERMEDAD POR REFLUJO GASTROESOFÁGICO  DEFINICIÓN  ERGE solo cuando produce síntomas molestos y/o complicaciones (ej. esofagitis, pérdida de peso, estancamiento ponderoestatural])    SALUD PÚBLICA  15% la padecen 1 vez por semana, 7% diariamente.    PATOGENIA  Menanismos antireflujo 

‐Esfinter esofágico interior    Debilidad sin causa evidente    Esclerodermia    Embarazo    Tabaquismo    Anticolinérgicos    Relajantes del m. liso (beta adrenérgicos,aminofilina, nitratos, antagonistas de calcio, inh. de fosfodiesterasa)  ‐Músculo crural diafragmático.    Incompentencia  ‐Localización anatómia del EEI.    DIAGNÓSTICO  Típicos: pirosis y regurgitación  Atípicos tos crónica, disfonía, dolor torácico.    Otros:    eructos, disfagia, hipersalivación, náuseas, hipo.      Endoscopia: si sospecha de estenosis, Barret, esofagitis, adenicarcinoma, datos de complicación (anemia ferropénica, pérdida de peso,  tumoración gástrica, disfagia, vómito persistente)  pHmetría: quienes    no respondan a manejo empírico o si no hubo lesiones endoscópicas.  Impedancia esofágica: nos permite diferencia reflujo ácido de no ácido y es más S y E que pHmetría y manometría para detectar  episodios.      TRATAMIENTO  Gold stándar: fundupliatura vía laparoscópica (Nissen)    Fracaso a menejo médico, joven de 25‐35 años, deseo del paciente.    La funduplicatura no previene adenocarcinoma.    PRONÓSTICO  Escamoso==> columnar o cilíndroco (metaplasia o Barret)   

REFLUJOGASTROESOFÁGICO  [r]HOMBRE DE 51 AÑOS DE EDAD QUIEN PADECE DESDE HACE 10 AÑOS ENFERMEDAD POR REFLUJO GASTROESOFÁGICO  TRATADA SÓLO DE MANERA PARCIAL, ES SOMETIDO A ENDOSCOPÍA, REPORTÁNDOSE EPITELIO ROSA SALMÓN ARRIBA  DE LA UNIÓN GASTROESOFÁGICA.    CON BASE EN EL HALLAZGO ENDOSCÓPICO LO MÁS PROBABLE ES QUE EL PACIENTE PRESENTA EN ESTE MOMENTO:  A) ESOFAGITIS.  B) CARCINOMA ESCAMOCELULAR.  C) METAPLASIA.  D) ACALASIA.    CORRESPONDE AL TRATAMIENTO DE ELECCIÓN EN ESTE CASO:  A) INHIBIDORES DE LA BOMBA DE PROTONES.  B) ANTIHISTAMÍNICOS H2.  C) ESOFAGECTOMÍA.  D) FUNDUPLICATURA    RESPUESTA C, A    Se trata de paciente con ERGE es somentido a endoscopia y se muestras el hallazgo de epitelio color salmón.    Hay que recordar que una metaplasia es el cambio de una células por otra similar, se dice que que hay displasia cuando  hay pérdida de la arquitectura habitual de la célula. El esófago de Barret es una metaplasia que endoscópicamente luce  como  una  sona  de  epitelio  color  salmón,  razón  por  la  cual  el  apciente  presenta  una  metaplasia.  Y  el  tratamiento  conducente es con IBP.      Por medio de endoscopia la esofagitis se puede clasificar en grados:    Grado I‐  Erosiones pequeñas, circulares, no confluyen.    Grado II‐  Erosiones lineales cubiertas por tejido de granulación que sangran fácil si se tocan.      Grado III‐ Erosiones lineales o circulares que confluyen para formar área circular sin epitelio    Grado IV‐ Estenosis (obligada la toma de biopsias)    Grado V‐Esófago de Barret    Respecto al tratamiento hay que recordar que de primera elección están los IBP. 

  Displasia leve: IBP x 1‐2meses y repetir endoscopia.    Displasia de alto grado: vigilancia estricta endoscópica c/ 3 meses.  Los inhibidores H2 solo están indicados junto con funduplicatura en pacientes con displasia leve y después del tratamiento  inicial con IBP.  Esofagectomía inciada en jovenes que no están dispuestos a control endoscópico trimestral.    La funduplicatura está inciada en quienes no teleran los IBP o no responden a ellos.      MUJER DE 38 AÑOS DE EDAD, QUE ACUDE POR PRESENTAR DESDE HACE TRES AÑOS PIROSIS, REGURGITACIÓN Y DISFAGIA  PROGRESIVA QUE ACTUALMENTE ES A SÓLIDOS. SE REALIZA ENDOSCOPÍA, LA CUAL REPORTA ESOFAGITIS CON ESTENOSIS  A NIVEL DEL TERCIO MEDIO DEL ESÓFAGO.     CON BASE EN EL REPORTE ENDOSCÓPICO LA ESOFAGITIS REPORTADA DEBERÁ CONSIDERARSE:  A) GRADO I.  B) GRADO II.  C) GRADO III.  D) GRADO IV.    EL DIAGNÓSTICO NOSOLÓGICO DE BASE MÁS PROBABLE ES:  A) ACALASIA.  B) PSEUDOACALASIA.  C) ENFERMEDAD POR REFLUJO GASTROESOFÁGICO.  D) ENFERMEDAD DE BARRET.    Los  hallazgos  endoscópicos  se  clasifican  según  Savary‐Miller.  Importante  recordar  que  la  estenosis  esofágica  es  característica de un grado IV, aunque podríamos encontrarlo en grado III también.    RESPUESTA    D, C    MUJER DE 58 AÑOS DE EDAD, CON ANTECEDENTE FAMILIAR DE ESÓFAGO DE BARRET Y ALCOHOLISMO DESDE HACE 25  AÑOS,  ACUDE  A  LA  CONSULTA  GENERAL.  DURANTE  EL  INTERROGATORIO  LA  PACIENTE  REFIERE  SINTOMATOLOGÍA  PERSISTENTE  DE  ENFERMEDAD  POR  REFLUJO  GASTROESOFÁGICO  QUE  REMITE  POR  ALGUNOS  DÍAS  DESPUÉS  DE  TRATAMIENTO  ALTERNATIVO.  EN  SU  ÚLTIMA  CONSULTA  REFIERE  DISFAGIA  DE  1  MES  DE  EVOLUCIÓN.  SE  SOLICITA  INTERCONSULTA PARA ENDOSCOPÍA.    EL SIGUIENTE FACTOR JUSTIFICA LA REALIZACIÓN DE UNA ENDOSCOPÍA:      A) LA PRESENCIA DE DISFAGIA.  B) LA EDAD MAYOR A 50 AÑOS.  C) EL ALCOHOLISMO DE 25 AÑOS DE EVOLUCIÓN.  D) EL ANTECEDENTE FAMILIAR DE ESÓFAGO DE BARRET.    La  realización  de  endoscopia  en  paciente  con  ERGE  su  justifica  si:  1)  sospecha de  estenosis,  2)  esófago  de  Barret  o  3)  adenocarcinoma de esófago. O en aquellos que presenten disfagia,    vómito persitente,    hemorragia gastrointestinal,  anemia ferropénica, pérdida de peso sin razón aparente o tumoración epigástrica.    RESPUESTA A           

ESÓFAGO DE BARRETT  DEFINICIÓN  Metaplasia de escamosos ==> cilíndrico==> displasia ==> adenocarcinoma    PATOGENIA  Reflujo ácido==> esofagitis erosiva==> cicatrización    DIAGNÓSTICO  Endoscopia: epitelio rosa salmon (Barrett)  Esofagograma: hernias hiatales, estenosis, ulceraciones.     

TRATAMIENTO  Esófago de Barret metaplásico: IBP o funduplicatura*    *IBP eliminan síntomas pero no garantizan el control del refliujo ácido.    *IBP no eliminan relujo biliar (causa importa de Barret)    *Funduplicatura promueve regresión de epilio cilíndrico  Esófago de Barret con displasia de bajo grado IBP x 3 meses a triple dosis. ==> endoscopia y biopsia.    si se confirma continuar con IBP o funduplicatura*    *revisión endoscópica cada 6m‐12m  Esófago de Barret con displasia de alto grado.    1) Revisión endoscópica cada 3 meses, biopsia de 4 cuadrantes de cada centímetro de Barret.      Después de displasia de alto grado tarda aprox 5 años a progresar a Ca el 50% de pacientes.    2)Esofagectomía en jóvenes      ‐30% de las displasia tienen Ca        ‐50% desarrollar Ca durante la vigilancia endoscópica      ‐En centros especializados hay muy buenos resultados.       Antihistamínicos H2?  Buen día PROEDUMED, en el subtema esófago de Barret, apartado de Manejo Terapéutico dice: "el enfermo podrá seguir con  antihistamínicos H2". ¿Con anthistiamínicos H2 se refiere a inhibidores H2 como ranitidina?  ¿Está inciada para el manejo de esófago de Barret?   

ENFERMEDAD ÁCIDO PÉPTICA Y SUS COMPLICACIONES  ÚLCERA PÉPTICA      DEFINICIÓN  Daño necrótico de la mucosa que supera la muscular de la mucosa, secundaria a pepsia y ác. clorhídrico.    Úlcera péptica complicada aguda: comprometa la vida (sangrado o perforación)    Úlcera péptia complicada crónica: obstrucción?    SALUD PÚBLICA  Perforación 2‐10 de úlcera pépticas.  Una de las 3 causas comunes de hemorragia del tubo digestivo.  FR para sangrado: AINES, edad avanzada, H. pylori, alcohol y tabaco.    PATOGENIA  H pylori y AINES son causa de la mayoría de úlcera duodenales, ác gástrico lesiona mucosa pero no tiene efecto primordial.  Úlceras duodenales: secrecion de HCO reducida, secreción ác basal y nocturna aumentada. H pylori podría estar asociada.  Úlceras gástrias: Puede estar productica por H pylori o AINES,    DIAGNÓSTICO  Sospecha de sangrado de úlcera:==>endoscopia las    1eras 24 hrs o colocacion de SNG con lavado gástrico con solución a temp  ambiente    Antecednte de enf ác. péptica    Hematemesis y melena 50%    Hematemesis 30%    Melena 20%    Hematoquecia 5%    Sx anémico  Sospecha de perforación==>1 era TAC, sino placas simples y contrastadas con material HIDROSOLUBLE    Antec de enf ác. péptica    Dolor transflictivo, súbico abdomen alto, aumenta con movimientos, se irradia a abdomen bajo, hombros.      Datos de iritación peritoneal    Choque, sepsis  Obstrucción secundaria de úlcera péptica (x edema y estenosis) ==>1eropanendoscopia con toma de biopsia, 2do serie EGD    Antec de enf ác péptica    Infección por H pylori no tratada    Desnutrición y pérdida de peso    Vómits postprandiales    Alcalosis metabólica hipoclorémica e hipokalémica    Plenitud postprandial y saciedad temprana    Distensión abdominal    Masa epigástrica timpánica 

    TRATAMIENTO  Sangrado: líquidos, paquete globular si pérdida >30%    IBP 80mg IV en bolo, 8mg7gr x 72hrs, 40mg cada 24 x 3 semans (una vez dada erradicación)    Endoscopia temprana con método hemostático (sin urgencia vital)    Cirugía de emergencia si hay compromiso vigtal o si >2cm.  Perforación: Cristaloides, AB de amplio espetro, SNG a succión.    Cirugía abierta si urgencia o perforación >6mm    Irrigación caviad peritoneal con 30lts de sol salina    SNG 48hrs  Obstrucción pilórica: Corregir anormalidaes HE, desequilibrio A/B, nutrición parenteral total, erradicación,    drenaje.    Erradicación como primer aopci´n, pensar dilatación con balón. 

           

 

   

 

                Erradicación  1era elección: Claritromicina 500m c12, amoxoicilina 1gr cada 12, Omepraz 40mg cada12 x 14 días.    2da elección: tinidazol 1gr c 12, tetraciclina 500mg c6, bismuto 525mg cada 6, omepra 40mg    cada 12 x 14 días.    3era elección: azitromicina 500mg x 3 dias, omeprazol 40mg c 24 con furazolidona 200mg c8    x 10 días.  PRONÓSTICO 

Mortalidad por perforación 5‐30%.    RESPUSTAS  18D, 19C, 20B 

COLECISTITIS, COLELITIASIS Y SUS COMPLICACIONES  COLECISTITIS    DEFINICIÓN  Colecistitis es inflamación de la vesícula generalmente por litos, lodo biliar y en raras ocasiones ninguna de estas.    Colelitiasis es la presencia de litos en vesículas.    SALUD PÚBLICA  M2:1H  FR: mujer, >40 años, obesidad, embarazo, fármacos (fibratos, ceftriaxona), enf del íleon, dislipidemias, enf hepáticas, enf metabólicas,  AO, terapia hormonales sustitutiva con estrógenos.    PATOGENIA  80% son mixtos y de colesterol (>50% de colesterol, sales cálcicas, pigmentos)  20% son pigmentarios (bilirrubinato cálcico y  def de 7 hidroxilasa de colesterol.  Mutación gen MDR3 codifica para bomba de exportación de fosfolípidos    a la bilis (estos disminuyen y aumentan colesterol)    Nucleación    de colesterol  Nucleación rápida del colesterol el la bilis (exceso de factores pronuecleadores o déficil de factores antinucleadores.    Retraso de vaciamiento y estasis biliar  Hipomotilidad vesicular     

 

 

Cálculos de pigmento: hemólisis crónica, cirrosis alcohólica, anemia perniciosa, fibrosis quística, infección crónica de vías biliares,  envejecimiento, enfermedad de íleon, extirpación o derivación de tal segmento.    DIAGNÓSTICO  Colecistitis/colelitiasis: Murphy, dolor CSD, masa CSD, resistencia muscular CSD, náusea, vómito.  Colecistitis/colelitiasis agudas: vesícula palpable, >39°, escalocríos, inestabilidad hemodinámica.    Colasco y peritonitis: irritación abdominal, distensión abdominal, taquicardia, taquipnea, acidosis metabólica, hipotensión, choque    Si agudo precisa un síntoma local y un síntoma sistémico, USG que lo sugiera. BHC reportará leucocitosis, PCR ayudará a corroborar que  hay inflamación.    Amilasa sérica útil para identificar una coledocolitiasis como complicación.    USG reportará colelitiasis aguda:    Pared vesicular >5mm    Líquido perivesicular    Murphy USGnográfico    Alargamiento vesicular 8cm axial, 4cm diametral.    Lito encarcelado    Imagen de doble riel    Sombra acústica    Ecos intamurales  Gammagrafía biliar solo si no es concluyente el USG y la clínica  TAC    Engrosamiento y alargamiento vesicular    Alargamiento vesicular    Áreas de alta densidad en tejido graso perivesicular  RMN    Alargamiento vesicular    Engrosamiento de la pared.  Gammagrafía con Tc HIDA    Exclusión vesicular    Signo de RIM    5‐10% alitiásica (quemados, traumatisados, puerperas, postoperados, NPT prolongada, ==> imagen vesícula tensa, estática, agrandada  sin cálculos.    TRATAMIENTO   

 

 

Ác. ursodesoxicólico, quennodeoxicolico x 1‐2 años.    Diclofenaco 75mg, meperidina, antibiótico.      Litotripsia si: lito único, no calcificado, diámetro 20‐30mm, sin contraindicaciones (panreatitis, alt de la coagulación, quistes o  aneurismas en el trayecto de las ondas de choque).    Grado I  Cole temprana laparoscópica  Grado II  Cole temprana laparoscópica, si inflamación grave drenaje percutáneo  Grado III  Manejo urgente de falla orgánica, drenaje, desinflamar, mejorar condiuciones, colelaparoscópica.    Complicaciones de cole laparoscópica: lesión del conducto biliar, intestino e hígado.    Complicaciones de cole abierta: infecciones, íleo, hemorragia intraperitoneal, atelectasia, trombosis venosas profundas, IVU.    Si coledocolitiasis: colelap+CPRE en el mismo ingreso pero días después.   

 

 

 

 

 

  SANGRADO DE TUBO DIGESTIVO ALTO Y BAJO  HEMORRAGIA DIGESTIVA INFERIOR  DEFINICIÓN  Aquella que se produce distal al ligamento de Treitz (entre el intestino delgado y el ano). Generalmente x malf vasculares, divertículos y  neoplasias. 

  SALUD PÚBLICA  Aprox 65 años.    PATOGENIA  Etiología: Diverticulosis. Angiodisplasia, Neoplasia, Enf perianales, Diverículo de Meckel, Colitis    (Infecciosa, no infecciosa, otras).    Diverticulosis: causa entre 20‐55%. Aunque slo del 3‐15 sangran (muchas diverticulosis no sangran). El sangrado de los divertículos es  como hematoquezia. 70% en colon izquierdo, pero el sangrado es más común en el derecho 50‐90%.  Angiodisplasia: causa de 3‐12% de STDB, principal causa de sangrado gradual e intermitente, el 66% son >70 años, 45% da sangrado  constante pero imperceptible, afecta más colon derecho.  Neoplasias: raramente por si solos sangran, con frec asocian divertículos.  Enf perianales: Hemorroides (en general no producen dolor cuando sangran, prurito, si se trombosa hace coágulo , ulcera y sangra),    fístulas perianales (fred x traumatism por defecación, dolor, comezón, rectorragia).  Divertículo de Meckel: con frec en intestino delgado, sangra más en niños que en adultos, puede confundirse con peritonitis.  Colitis: infecciosa generalmente cuasada por amibiasis, disentería, tuberculosos, campylobacter, E.coli, clostridium difficile.    DIAGNÓSTICO  Se sugiere realizar una colonoscopia a todos los pacientes con sangrado de tubo digestivo bajo. Es diagnóstica y terapéutica.  Glóbulos rojos marcados con TC99: determina hemorragias >0.1ml/min, limitado prque no prmite identifiar la causa del sangrado.  Angiografía: detecta hemorragias de 0.5ml/min. S 30‐50%, E 100%.  Tomografía: detecta causa y extensión de algunas alteraciones, uso limitado porque no siempre hay tomógrafo.    Estudio de video‐cápsulas: identificación del sitio 55‐65%, contraindicaci´n es el uso de marcapasos.    Gold standar en el diagnóstico de angiodisplasias: colonoscopia.    TRATAMIENTO  El de cada uno en particular.      PRONÓSTICO  Tasa de mortalidad 2‐4%    WEBINAR  Pérdidas de tubo digestivo  10%  SV normales  10‐20%  15mmHg por debajo, piel fria, piloereccion  20‐25%  Sistolicas     ileocólica==> mesentérica superior    No tiene vena propia==> Vena ileocólica ==> mesentérica inferior==> porta==> absceso hepático piógeno    Ganglios linfáticos ileocólicos    Inervación simpática y parasimpática    2/3 son retrocecal (indica donde está la punta)  1/3 es pélvico  Ectópico: subhepática==> pasa porque hay alguna alteración en la rotación intestinal en la embriogénesis, estos pacientes no tienen  colon derecho.    Isquemia==> edema, moco==> pus por bacterias==> destrucción de pared apendicular.  1) linfáticos se detienen, 2) drenaje venoso se detiene, 3) flujo arterial se detiene.    Si empeora==> peritonitis  El epiplón fija y trata de bloquear el proceso y forma un plastrón. En pediátricos el epiplón no está bien desarrollado razón por la cual se  tiende a complicar más fácilmente.    Apendicitis aguda puede ser la primera manifestación de enfermedad de Crohn.    La frecuencia de apendicitis aguda recidivante aumentará conforme se disponga de AB con más libertad y queden muñones  apendiculares largos como consecuencia del mayor uso de apendicetomía laparoscópica.      DIAGNÓSTICO  Cuadro atípico: edades extremas, en pediátricos el epiplón no está bien formado, cuando se modifica por medicamentos.      SÍNTOMAS  Abdomen agudo (cólico, localizado periumbilical, incremento rápido, antes de 24 hrs migra a CID, incrementa al toser o caminar.    Náuseas y vómitos no muy numerosos. Puede haber fiebre.  Si fuera muy largo el apéndice puede dar dolor en el otro lado (FII) aunque dolor FII se explica mejor por diverticulitis.      Los tres datos pivote son:    1 Dolor epigástrico con migración a CID    2 Datos de irritación peritoneal (hipersensibilidad FID, rebote, defensa, rigidez) 

  3 Datos de respuesta inflamatoria (leucocitosis con predominio de neotrófilos)    Sospecho apéndice si la clínica aparece en el siguiente orden: Dolor ==> náusea ==> vómito    Otros datos anorexia, fascies dolorosa, taquicardia, posición en gatillo.    Descarto apéndice:    Dolor que no migra      Dolor en FII          FID, dolor que no sigue secuencia (ej. náusea==> vómito==> dolor)  EF:    Hipersensibilidad FID  Defensa muscular involuntaria,    McBurney      Rovsing    Dolor en FID al presionar FII  Cope [psoas]    Aumento del dolor en FID al flexionar activamente la cadera derecha.    Cope [obturador]  Dolor en hipogastrio al flexionar el muslo derecho y rotar la cadera hacia adentro.    Summer    Aumento de tensión de los músculos abdominales a la palpación superficial de FID.  Von Blumber  Rebote    Talopercusión  Percusión en talón derecho y dolor en FID.  Dunphy:      incremento del dolor FID con tos.  Para apendicitis retrocecal: 1) Capurro (pellizcar espina iliaca anterosuperior para adentro) 2) As Schultze?: es para afuera    PARACLÍNICOS:    BHC: Leucos 10,000‐18,000 con neutrofilia (ocasionalmente bandemia). Si solo presentan 1 o 2, pedir EGO, BHC y PIE.  Prueba triple (PCR >8mcg/ml, leucocitos >11,000, neutrofilia >76%) sugiere fuertemente apendicitis.    PCR >55mg/l genera sospecha de apendicitis perforada.    EGO patológico es para ver concomitancias, puede haber leucocitosis en pacientes con apendicitis.  Rx: liquido libre (vidrio despulido). Es el estudio de imagen de inicio y para fines de examen, sigue USG y TAC.    USG: hacer después de rx (sens 86, esp 96), imagen en diana y líquido periapendicular  TAC: gold standar si sospecho de apendicitis pero queda la duda pese a USG. En embarazadas se usa siempre y cuando >20sdg.    Rx es el estudio de imagen de inicio y para fines de examen sigue USG y TAC.      I    Congestiva   Hiperemia + edema  II  Supurativa   Hiperemia + edema + natas fibrinopurulentas  III  Necrótica    Hiperemia + edema + natas fibrinopurulentas + necrosis segmentaria  IV  Perforada    Perforada, peritonitis focal.      Bacteroides fragilis principal asilado.      En pediatría: 6‐10años, M1:F2, dolor abdominal náusea  vómitofiebre. En lactantes hay diarrea. Náusea y vómito antes del  dolor nos da la sospecha de apéndice retrocecal ascendente. En múltiples ocasiones el paciente puede cursar afebril. Leucocitosis  >15,000 con neutrofilia. (Si hay sospecha de IVU tendrá >20 leucos + nitritos positivos,) Niños con 1, 2, 3==> USG ==> TAC.    En embarazadas: Primera causa quirúrgica en el embarazo es apendicitis, es más frecuente en el tercer trimestre, signo pivote es dolor  en flanco o hipoconrio derechos, la Qx se hace donde le duele, si es en el1er trimestre el porcentaje de pérdida del producto es del 30‐ 50% (esta última cifra si se perfora, y en promedio sin importad edad 30%). En segundo trimestre asocia pretérmino en 11%. La fiebre y  taquicardia están presentes en cuadros apendiculares complicados con perforación o absceso apendicular.  Leucocitosis >16mil, con neutrofilia y bandas.        En adulto mayor: atípica, insidiosa, dolor poco intenso, distermia, más de 3 días, distensión, disminución de ruidos intestinales,  meteorismo, parálisis intestinal, poca defensa abdominal, cuadro confusional agudo, generalmente no hay leuocitosis, alt  hidroelectrliticas, aumento de creatinina. Adultos mayores con 1,2,3  TAC    Anciano con ileo paralítico y alt hidroelectrolíticas, pensar que puede ser secundario a apendicitis.      En los viejitos con frecuencia se complican porque no sienten tanto, sin embargo, el síntoma pivote es la diarrea (menos frec, náusea y  vómito)    Ej.  Diarrea==>  medico  gral==>  GEPI==>  BHC  no  elevaciones==>ES  alterados==>  Tx  para  reponer  ES  ==>  lo  complica  más==>  Cr  está  elevada.    No es el dolor el síntoma pivote en ancianos.      "Joven que sangra en la adolescencia, con dolor en fosa iliaca derecha y mesogastrio (pienso en Meckel)"  Gold standar diagnóstico de Meckel: gammagrafía con TC‐99    DIAGNÓSTICO DIFERENCIAL 

Embarazo ectópico (mujer en edad fértil), quimioterapia?, si opero y descubro ectópico (aunque esté perforado), se mitiga el sangrado,  se realiza curetaje y se conserva la tuba.  Solo si está en cuerno de la trompa se realiza histerectomía.        TRATAMIENTO  Tratamiento de elección apendicectomía laparoscópica o abierta dependiente del paciente y el cirujano.    Si hay duda: observación corta y laparoscopia.    Apendicectomía laparoscópica contraindicada en  AB endovenosa y líquidos.    Quiste ovárico torcido ===> Cistectomía y/o salpingooforectomía  Embarazo ectópico==> prueba inmunológica==>Laparoscopia exploradora  Apendicitis==> USG ==> apendicectomía    300 mRADS es lo que genera el tomógrafo, lo permitido en el embarazo son 5 rads.    [r]HOMBRE DE 34 AÑOS ACUDE AL SERVICIO DE URGENCIAS PORQUE DESDE HACE 24 HORAS PRESENTA DOLOR QUE COMIENZA EN  MESOGASTRIO Y QUE POSTERIORMENTE SE LOCALIZA EN CUADRANTE INFERIOR DERECHO CON IRRADIACIÓN A MUSLO Y ESCROTO. SE  ACOMPAÑA DE ASTENIA, ADINAMIA, HIPOREXIA, NÁUSEAS INTENSAS, VÓMITOS EN 3 OCASIONES Y TRES EVACUACIONES DISMINUIDAS  DE CONSISTENCIA. A LA EXPLORACIÓN PRESENTA TEMPERATURA DE 37.9°C, TA 120/90 MMHG, FC 100/MIN, FR 20/MIN, ABDOMEN CON  RESISTENCIA MUSCULAR VOLUNTARIA, DOLOR A LA DESCOMPRESIÓN SÚBITA EN EL CUADRANTE INFERIOR DERECHO DEL ABDOMEN. SE  REPORTA  BIOMETRÍA  CON  HEMOGLOBINA  DE  13MG/DL,  LEUCOS  DE  13000,  70%  DE  NEUTRÓFILOS.  EL  EXAMEN  GENERAL  DE  ORINA  REPORTA 15 A 20 LEUCOCITOS POR CAMPO, BACTERIAS +++.  EL DIAGNÓSTICO MAS PROBABLE ES:      A) LITIASIS URETERAL DERECHA.  B) APENDICITIS AGUDA.  C) COLITIS INFECCIOSA.  D) INFECCIÓN DE VÍAS URINARIA    EL TRATAMIENTO INDICADO EN ESTE CASO ES:  A) ANALGÉSICOS, ANTIESPASMÓDICOS Y REPOSO.  B) APENDICECTOMÍA.  C) ANTIESPASMÓDICOS, ANTIBIÓTICOS.  D) LITOTRIPSIA EXTRACORPÓREA POR ONDAS DE CHOQUE.    RESPUESTA B, B  Los diagnósticos diferenciales factibles por la clínica son litiasis ureteral derecha y apendicitis aguda. Apendicitis aguda tiene dolor que  migra a otro punto y en la litiasis el dolor es irradiado más no migratorio. La apendictis puede hacer leucocitosis, en cambio la litiasis  debería tener EGO con cristales y hematuria.  Pensaríamos en IVU si tuviera en este contexto, >30 eritrocitos x campo y >20 leucocitos por campo      HOMBRE DE 87 AÑOS DE EDAD, PRESENTA CUADRO DE DOLOR ABDOMINAL DE 24 HRS DE EVOLUCIÓN, INICIA EN OMBLIGO, SE RECORRIÓ  A CID, LE SOLICITA LABS QUE DEMUESTRAN LEUCOCITOS DE 12 MIL, HB DE 9.6 CON NEUTROFILIA DE 77% Y SIGNO DE REBOTE POSITIVO.    ¿QUÉ ESTUDIO DE IMAGEN DECIDIRÍA TOMAR?  A) USG  B) RX AP Y LATERAL    C) TAC  D) RMN    LA PRUEBA TRIPLE PARA LA POBLACIÓN GENERAL PARA ESTA PATOLGÍA INCLUYE:  A) CLINICA, LABORATORIOS Y USG  B) PCR, VSG, LEUCOCITOS  C) FECALITO, BORRAMIENTO DE PSOAS Y ASA CENTINELA  D) LEUCOCITOSIS, PCR ELEVADA Y NEUTROFILIA    SI SE TRATARA DE UNA PACIENTE EMBARAZADA, ¿HASTA CUANTO PUEDE DECIR QUE ES NEUTROFILIA FISIOLÓGICA POR LA GRAVIDEZ? 

A) 11MIL  B) 15MIL  C) 16 MIL  D) 18 MIL    RESPUESTAS CDC  En el adulto mayor las causas de dolor abdominal por orden de frecuencia son: enf biliares, obstrucción intestinal, tumores y  causas  vasculares.  La  frecuentes  de  apendicitis  en  el  anciano  es  de  3‐10%  del  total  de  las  apendicitis.    En  anciados  pedir  EGO,  BHC,  ES,  creatinina, ES, rx simple de abdomen, de pie y decúbito, rx de tórax.      La prueba triple sugiere la presencia de apendicitis ante un cuadro clínico susgestivo: PCR >8mcg/ml, leucocitosis >11,000 y neutrofilia  >75%. Es especialmente útil en niños.    La leucocitosis puede estar levada de manera normal en el embarazo en niveles hasta por arriba de 16mil cel/ml.    Niños  con  cualquiera  de  las  3  manifestaciones  cardinales  (dolor  caracterísitco,  datos  de  irritación  peritneal,  datos  de  respuesta  inflamatoria)  solicitar USG, si USG indeterminado TAC  Adultos mayores con cualquiera de las 3 manifestaciones cardinales ==> TAC    [r]HOMBRE    DE  21  AÑOS  DE  EDAD.  ACUDE  A  URGENCIAS  POR  CUADRO  DE  1  DÍA  DE  EVOLUCIÓN,  CARACTRIZADO  POR  DOLOR  ABDOMINAL INTENSO EPIGÁSTRICO Y PERIUMBILICAL, QUE POSTERIOMENTE MIGRÓ HACIA FOSA ILAICA DERECHA, NÁUSEAS, VÓMITO.  A LA EF SE ENCUENTRA FC90 FR 14 T 38 TA 112/70, HIPERSENSIBILDIAD EN CUADRANTE INFERIOR DERECHO, MC BURNEY Y ROVSING  POSITIVO. SE SOLICITAN ESTUDIOS DE LABORTORIO DEMOSTRANDO LEUCOCITOSIS.  ¿CUÁL ES EL ESTUDIO DE ELECCIÓN PARA ESTE PACIENTE?  A) USG  B) TAC  C) RX ABDOMINAL DE PIE  D) RX ABDOMINAL DECÚBITO    RESPUESTA B  El  paciente  tiene  apendicitis  aguda.  Tema  comunmente  preguntand  en  el  ENARM.  La  migración  del  dolor  con  aumento  del  mismo,  presencia  de  náuseas,  vómito  y  anorexia  seugieren  el  diangóstico  de  apendicitis  aguda.  Así  que  la  cracterística  del  dolor,  las  manifestaciones de irritación peritoneal y los datos re respeusta inflamatoria confluyen en el diagnóstico de apendicitis. El patrón de oro,  como estudio de elección para apendictisi es la TAC. La rx abdominal es anormal en menos del 10% de los casos.    EN EL SERVICIO DE URGENCIAS, USTED RECIBE A UN PACIENTE FEMENINO DE 30 AÑOS DE EDAD, LA CUAL INICIA PADECIMIENTOA CTUAL  DE 12 HRS DE EVOLUCIÓN CON PRESENCIA DE DOLOR EN REGIÓN DE MESOGASTRIO QUE MIGRÓ A FOSA ILAICA DERECHA, EL DOLOR HA  IDO EN AUMENTO A TAL GRADO QUE HA PROVOCADO IMPOSIBILDIAD PARA DEAMBULACION, SUMÁNDOSE NÁUSEA QUE PROGRESÓ  AL VÓMITO EN DOS OCASIONES, FIEBRE NO CUANTIFICADA A LA EXPLORACIÓN FÍSICA ENCUENTRA HIPERESTESIA E HIPERBARALGESIA,  CON DATOS DE DOLOR A LA DESCOMPRESIÓN EN FOSA ILIACA DERECHA.     SE REALIZA AP DE ABDOMEN DE PIE Y DECÚBITO, ¿CON QUE FRECUENCIA SE OBSERVA ESTE HALLAZGO?  A) 5%  B) 8%  C) 12%  D) 16%    EN EL CASO PARTICUAR DE ESTE PACIENTE ¿QUÉ ESTUDIO DEBERÁ PRACTICARSE DE MANERA OBLIGADA?  A) TC ABDOMINAL  B) USG  C) LAPAROSCOPIA DIAGNÓSTICA  D) PRUEBA INMUNOLÓGICA DE EMBARAZO    RESPUESTA B, D 

    __  Meckel 60 cms valvula IC 100 cms atras  diverticulo chipote perforaciones , mucosa ectpopica gastrica pancreatica    hcl enzimas  ulcera  samgtado  infantes  obstruccion perforacion    diverticulectomia en cuña cirre intestino priaira,    gols   

gold st gamma Tc 99  __     

COLECISTITIS  [Resumen la Salle]  DEFINICIÓN  Hidrocolecisto: llena de moco  Piocolecisto: con pus, más frec en DM2 y inmunocomprometidos.    Gangrenada: antes de la perforación  Colasco: perforación de la vesícula,    produce peritonitis química, cinluso infectada si ya tenía dos?      SALUD PÚBLICA  Colecistitis: 90% secundaria a litiasis biliar==> ruta fisiopatológica igual al apéndice pero la obstrucción es en el cístico, también puede  haber traslocación bacteriana.    Puede ser alitiásica: ej sepsis, sarcoidosis, LES; SIDA, NOT, quemados.    M3:H1.    F emale  F at  F ourty  F ertility (AO)  F ea :C    PATOGENIA  De arriba a abajo la vesícula tiene: fondo, cuerpo y bolsa de Hartman.    Función de la vesícula: almacernamiento y concentración de bilis, absorción, secreción de moco por cels caliciformes)==> de estas  puede surgir cáncer ==> adenocarcinoma de tipo escirroso==> colecistectomía + disección en cuña donde está adosado al hígado  (segmento IV y V)    Triángulo de admiral:    Lecitina      Ac biliar        Colesterol (más amarillos) ==> generlamente multilitiásica (como esquite)    80% de los litos son de colesterol.  Solo se disuvleven:    10% (pigmento biliar) y otro 10% que son mixtos.      Pólipos vesiculares 1cm opero    Lesión hiperplásica opero. ¿que quiere decir?    Gorrofrigio: es la forma de la vesícula cuando está doblada, se opera porque altera la motilidad.  Vesícula en procelana: calcificada, es premaligna y se opera (cáncer en 7%)  Pared >3mm está patológica. Se mide a la altura del segmento IV y V.    Enfisema: gas producido por bacteriaws  Discinesia: no nacía bien, prueba de Boyden.      Colecistoquinético==> comida al duodeno 1era porción==> secretina + colecistoquinina que promueven el vaciamiento. ==> litos se  desplazan==> dolor, náusea, vómito==> desaparece el dolor porque deja de estimularse la secretina y colecistoquinina.      DIAGNÓSTICO  BHC    Leucocitosis  PFH    aumentadas (TGO TGP, FA, GTT)    Colecistitis==> USG gold standar  Coledocolitiasis==>CPRE (Gold standar), colangiografía.        PRONÓSTICO  Complicaciones: Sx de Mirizzi,    Boberet (ileo biliar) y coledocolitiasis  [Poner imagen]    CONTENIDO ADICIONAL CLAVE [WEBINARS]  Principales bacterias Klebsiella y E. coli 

Gold estándar para qx: cole lap  14% presenta ictericia sin que haya obstrucción de colédoco. Por que vesícula está adherida a IV y V segmente hepático.    La pared de la vesícula normal 3mm.    Ayuno, betalactámico, analgésico (AINES)  si no se enfría el cuadro en 24‐48 hrs colecistectomía abierta con incisión de mini lap.  No laparoscópica porque aumenta la pérdida del producto.   

COLEDOCOLITIASIS  [Resumen la Salle]  DEFINICIÓN  2‐15 % es una complicación de CCL.  Si es antes del años==> litiasis residual  Si después deñ años ==> litiasis recidivante.    Sintomática 25‐50%.  Dolor tipo cólico  Si microlitiasis==> pancreatitis.    Triada de Charcot:    Dolor +    Fiebre + Ictericia    Tríada de Reynolds: Choque + Confusión + Charcot    Coledocolitiasis==>CPRE (Gold standar)      CONTENIDO ADICIONAL CLAVE  Los litos son de pigmentos biliares, no de colesterol como en la colecistitis. Generalmente son único pero grandes.  ICTERICIA es el síntoma pivote.    Se infecta la bilir  Colangitis: ictericia + dolor + fiebre choque séptico , problemas mentales pentada de raynolds  Gold estándar Dx es COLANGIO RETROGRADA ENDOSCOPIA (si se tata de hígado y bilis) si se refiere a hígado es: COLANGIOGRAFÍA  PNACREATICA RETROGRADA ENDOSCÓPICA.      Primerp se hace la CPRE y luego la colecistectomía laparoscópica (se hace en las primeras 72hrs)     

ENFERMEDAD DIVERTICULAR DEL COLON  [Resumen la Salle]  DEFINICIÓN  La enfermedad diverticular engloba tres condiciones: a) estado prediverticular, b) diverticulosis y c) diverticulitis.    La Salle Onlina propone: divertulosis como término genérico para definir presencia de diverticulos.    Enfermedad diverticular cuando hay síntomas, estando dentro de esta dos presentaciones, a) hemorragia y b) diverticulitis.    Pólipos protruyen hacia la luz.  Enf diverticular  "chipotes" por debilidad de la pared.      Adquirida: son falsos divertículos, están en colon izquierdo, más en sigmoides, están cerca de la vejiga.      Congénita: son verdaderos, solo en colon derecho.    SALUD PÚBLICA  M3:H2  50% en 80 años  30% en 60  5% en 4mm diverticulitis.      Y la USG si la TAC no está disponible.    Complicaciones:      Diverticulitis (es la más frec,10‐25%)    Peforación peritonitis,    Estenosis    Fístulas  zona de fistulización más frecuente es colovesical.  Si tengo un paciente con diverticulitis (dolor), me salto el colon por enema porque el bario lo puede perforar y hago una TAC  directamente.  En resumen si diverticulosis  colon x enema, si diverticulitis TAC    El dato pivote que nos orienta a una diverticulitis es el dolor, si no hay dolor puedo hacer colon por enama.  La colonoscopia la podría hacer en caso de choque.    Clasificación de Hinchey    0  Enf diverticular leve      Enviar a casa no AB  Ia  Inflamación pericólica      Enviar a casa no AB  Ib  Absceso 5cm      AB + drenaje precutáneo [hospitalizar]  III  Peritonitis    purulenta      Qx urgente [colectomía] + Metronidazol + levofloxacino.    IV  Peritonitis fecal      Qx urgente [colectomía] + Metronidazol + levofloxacino    *Ib y II sin mejoría luego de 48 hrs se opera de urgencia. Hospitalario. Investigo si está complicado.    Tx: ambulatorio si tolera la VO, doy líquidos.      Dolor  Mesalazina + rifaximina , paracetamol y metamizol?  AB  Ciproflox + metronidazol    Fiebre, leucocitosis, dolor hospitalario  Hospitalarios III y IV:    nunca morfina mejor meperidina.    Antibiótico gold standar Metronidazol + levofloxacino    Tx Qx electivo si:    a)Inmunodeprimidos,  angiografía y embolización, si fracasa se hace resección.    Debuta cmo hematoquesia, está mezclada con las heces, cesa 80‐90% de forma espontánea.       CONTENIDO ADICIONAL CLAVE  Dolor en:    Epigastrio + FII      = enf diverticular    Epigastrio + FID = apéndice    WEBINAR   

Estudios inciales: placa de abdomen, bhc, es.  Si pienso que es diverticulitis: TAC con doble contraste.    Principal complicación de enf diverticular: diverticulitis  Otras complicaciones: estenosis, sangrado, perforación, fistulización.    Sitio más frecuente de fistulizacion vejiga      Paciente joven de 2‐3 eventos en un año o con comorbildiades (DM) Hinchey I se puede operar.      FUENTES ADICIONALES  1) ALLER DE LA FUENTE, R.. Enfermedad diverticular del colon. Rev. esp. enferm. dig. [online]. 2005, vol.97, n.6 [citado    2016‐04‐21], pp.458‐458. Disponible en: . ISSN 1130‐0108.    RECIBE EN SU CONSULTORIO A PACIENTE FEMENINO DE 65 AÑOS DE EDAD, CON ANTECEDENTE DE PADECER DOLOR TIPO CÓLICO EN  REGIÓN DE FOSA ILIACA IZQUIERDA, DE DOS AÑOS DE EVOLUCIÓN QUE INICLAMENTE SE PRESENTÓ COMO DOLOR CÓLICO LEVE,  DESPARECIENDO DE MANERA ESPONTÁNEA, SIN EMBARGO DESDE HACE 3 MESES, PRESENTE DOS EVENTOS DE DOLOR INTENS, EL  ÚLTIMO LO OBLIGÓ A ADOPAR POSICIÓN ANTIÁLGICA, AGREGÁNDOSE EN ESTE EVENTO FIEBRE NO CUANTIFICADA.    AL MOMENTO DE LA EXPLORACIÓN ENCUENTRA DOLOR DE PREDOMINIO EN FLANCO Y FOSA ILÍACA IZQUIERDA, EL CUAL SE PRESENTA  A LA PRESIÓN MEDIA Y PROFUNDA, SIN DATOS DE IRRITACIÓN PERITONEAL, SIN PLASTRONES NI BORRAMIENTO DE ÁREA HEPÁTICA.    ¿CUÁL ES EL ESTUDIO DE ELECCIÓN?  A) PLACA DE ABDOMEN  B) ANOSCOPIA  C) RECTOSIGMOIDOSCOPIA  D) TC DOBLE CONTRASTE.    Recuerda que el estudio endoscópico si está indicado, pero a modo de hemorragia ya que sería diagnostico y terapéutico.    EN EL ESTUDIO QUE SOLICITA, SE DESCRIBE LA PRESENCIA DE ZONAS DE INFLAMACIÓN SEVERA A NIVEL DE COLON SIGMOIDES CON  PRESENCIA DE UN ABSCESO DE 4CM A NIVEL DE LA ZONA INFLAMATORIA MÁS INTENSA Y OTRO DE MÁS DE 5 CM EN LA ZONA DE  UNIÓN DE SIGMOIDES CON RECTO. ¿Cuál ES LA CONDUCTA A SEGUIR?  A) ANTIBIOTICOTERAPIA CON DOBLE ESQUEMA  B) ANTIBIOTICOTERAPIA CON TRIPLE ESQUEMA  C) ANTIBIOTICOTERAPIA MÁS DRENAJE PERCUTÁNEO  D) CIRUGÍA URGENTE    LA FORMA DE USO DE ANTIBIÓTICO EN LOS PACIENTES CON TAL PATOLOGÍA ES:  A) QUINOLONA Y NITROIMIDAZOL  B) FLUROQUINOLONAS Y METRONIDAZOL  C) AMINUGLUCÓSIDOS Y METRONIDAZOL  D) MONOTERAPIA CON IMIPENEM Y MEROPENEM    RESPUESTA D, C, B    HOMBRE DE 45 AÑOS DE EDAD EL CUAL SERA SOMETIDO A CIRUGÍA DE COLÓN POR DIVERTÍCULOS.  CONSISTE EN EL TRATAMIENTO DE ELECCIÓN PARA LA PREPARACIÓN INTESTINAL PREOPERATORIA EN ESTE PACIENTE:      A) NEOMICINA + ERITROMICINA.  B) METRONIDAZOL + CIPROFLOXACINO.  C) CEFALOSPORINA DE SEGUNDA GENERACIÓN.  D) NEOMICINA + CLINDAMICINA.    RESPUESTA  Nenomicna y eritromicina son de primera elección para la preparación intesitnal.  La opción B podría ser correcta si adicionara neomicina.    La opción C es incorrecta para la preparación.    La opción D es incorrecta porque clindamicina asocia enterocolitis necrotizante.      HOMBRE  DE  65  AÑOS  DE  EDAD.  ACUDE  A  URGENCIAS  POR  UN  CUADRO  DE  4  DÍAS  DE  EVOLUCIÓN,  CARACTRIZADO  POR  DOLOR  ABDOMINAL, ESCALOFRÍOS Y FIEBRE; EL DOLOR INICIÓ EN FOSA ILIACA IZQUIERDA PERO SE HA DISEMINADO. A LA EXPLORACIÓN FÍSICA:  FC90  FR16,  T  38,  TA  110/65,  DOLOR  A  LA  PALPACIÓN  EN  FOSA  ILIACA  IZQUIERDA.  SE  SOLICITAN  ESTUDIOS  DE  LABORATORIO 

DEMOSTRANDO LEUCOCITOS DE 14,000.  LA TAC DE ABDOMEN DEMUESTRA MÚLTIPLES DIVERTÍCULOS EN COLON SIGMOIDES, YA LA  PRESENCIA DE UNA COLECCIÓN SIMPLE BIEN DELIMITADA DE APROXIMADAMENTE 4X6X2CM EN EL ESPACIO SUBFRÉNICO DERECHO.    ¿CUÁL ES EL MANEJO MÁS ADECUADA DE LA COLECCIÓN ENCONTRADA EN LOS ESTUDIOS DE IMAGEN?  A) ANTIBIOTICOTERAPIA  B) RESECCIÓN INTESTINAL  C) DRENAJE PERCUTÁNEO GUIADO POR IMAGEN  D) ANTIBIÓTICOS Y PEDIR CONTROL DE TAC EN 72 HRS    RESPUESTA C  El paciente tiene diverticulitis aguda, con dicho antecedente deducimos que la colección encontrada en la tac es una absceso abdominal.  El drenaje percutáneo guiado por USG o TAC está indocado en el tratamiento.    [Reto enarm]   

      ALTERACIONES ELECTROLÍTICAS Y ÁCIDO BÁSICAS EN EL PACIENTE QUIRÚRGICO  DEFINICIÓN  Pueden ser alteracines de la concentración, composición, volumen o mixtas.    [supertabla de alteraciones eletrolíticas]  Concentración  Hiponatremiua      150mEq/L    Cl:Signos SNC y tisulares, EKg onda T en espiga.Resequedad y viscosisdad mucosa.        Tx: reponer agua.    Hipokalemia*      5mEq/L    Causas: acidosis, trauma quiurgico, IR    Cl: náuseas, vómito, cólico intestinal, diarrea, bloqueo    cardiaco, paro diastólico, EKG T altas en espiga, QRS ancho, ST    deprimido.      Tx: gluconato cálcico I1g IV al 10%, 45mEq HCO3 1000 ml de dextrosa al 10% + 20 UI de insulina    Resinas de intermbio catiónico    Diálisis    [medidas antihiperkalémicas]      Composición      Fisiopato        Causas  Acidosis respiratoria  Retención de CO2 x hipoventilación  Depresión del centro respiratorio, afección pulmonar, EPOC, NAC  Alcalosis respiratoria  Pérdida de CO2x hiperventilación   Emocional, dolor intenso, VMI, encefalitis    Acidosis metabólica  Retención de H+ o pérdida de HCO  Diabetes, hiperazoemia, acum de ác láctico, inanición, diarrea,                fístulasi de intestino delgado  Alcalosis metabólica* Pérdida de H+ o retención de HCO  Vómito, aspiración gástrica, ingesta de HCO, diuréticos.       *Comunes en pacientes quirúrgicos.       

HERNIA UMBILICAL  DEFINICIÓN  Reparación quirúrgica hernia umbilical en niños: >1.5cm    o persistente después de los 2 años.   

Se recominda ténica de Mayo, material absorbible.    Reparación en adultos: técnica de mayo, material absorbible, las mallas si hernias >3cm de diámetro.  AB solo si la tasa hospitalaria de infecciones es superior a 5%.    

HERNIA INGUINAL  DEFINICIÓN 

 

   

  [HERNIA INGUINAL]  MUJER DE 48 AÑOS DE EDAD, CON ANTECEDENTE DE HERNIA INGUINAL BILATERAL. ES VALORADA EN EL SERVICIO DE URGENCIAS POR  PRESENTAR DOLOR INTENSO EN LA REGIÓN INGUINAL DERECHA.  LA POSIBILIDAD DE QUE SE TRATE DE UNA HERNIA ESTRANGULADA SE CONFIRMARA EN CASO DE:    A) QUE HAYAN DATOS DE OCLUSIÓN INTESTINAL.  B) QUE HAYAN DATOS DE PERFORACIÓN INTESTINAL.  C) QUE NO SEA REDUCTIBLE.  D) QUE EXISTA COMPROMISO VASCULAR.    RESPUESTA D  Tanto en hernias incarderadas como estanguladas hay cierto grado de oclusión intestinal.    La perforación intestinal puede ser producto de una isquemia prolongada.    En una hernia estrangulada hay compromiso vascular,    está tensa y muy snesible, la piel que lo cubre tiene color rojizo o azulado. N  existen ruidos intetinales dentro de la hernia. El paciente suele tener leucocitosis con desviación a la izquierda, se encuentra tóxico,  deshidratacio, febril, GASA con acidosis metabólica.    El Tx es Qx de urgencia por riesgo de perforación, el tx de esta última complicación  es la resección del segmento afectado.   

    OCLUSIÓN INTESTINAL  DEFINICIÓN  Detención completa y persistnte del contenido intestinal en algún punto del tubo. Si no es completa y persiste se llama suboclusión  intestinal. El 90% se explica por adherencias, hernias y tumores.    ETIOLOGÍA  Hay que distinguir si es oclusión alta o baja (en función a válvula ileocecal), si es mecánica o no mecánica, parcial o total, complicada o  no complicada.      Diferencia 2 entidades clínicas bien distintas: 

  Íleo paralítico: alt de la motilidad intestinal x parálisis del m. liso, raramente requiere manejo qx.    Íleo mecánico: auténtico obstáculo mecánico del contenido. Puede se intraluminal, extraluminal o parietal.      Obstrucción mecánica estrangulante: obstrucción genera compromiso vascular intestinal.  Oclusión en asa cerrada: onclusión en 2 puntos. Este tipo tiende más a la isquemia y perforación.      Causa más frec de obstrucción intestinal baja: neoplasias de colon y recto.      Ileo mecánico  a) Extraluminal: Adherencias postQx (35‐40%), hernias externas 20‐25% (inguinales, crurales, umbiliales, etc), hernias internas,  torsiones, vólvulos, invaginaciones, efecto masa extraluminal.    b) Parietal: Neoplasias, alt congénitas (atresia, estenosis, duplicaciones), procesos inflamatorios (Crohn, postradiación, etc)  c) Intraluminal: íleo biliar, bezoar, parasitosis, cuerpo extraño, impactación fecal, tumoraciones.      Íleo paralítico    a) Adinámico: postquirúrgic, peritonitis, reflejo (Sx retroperitoneal), compromiso medular, hiperuricemia, hipokalemia, coma diabético,  mixedema, bloqueantes ganglionares, isquemia.    b) Espástico: intoxicación por metales pesados, porfirias.    c) Vascular: embolia arterial, trombo venoso.      PATOGENIA  El gas que se acumula proximal a la obstrucción se debe a 1) aire deglutido (es el principal componente, tiene mucho N+ razón por la  cual no se absorbe en intestino, por eso se pone SNG), 2) CO2 por neutralización del bicarbonato (se absorbe bien en intestino), 3)  gases orgánicos de la fermentación bacteriana.    Hay hipocloremia, hipokalemia, hiponatremia, alcalosis metabólica, oliguria, hiperazoemia, hemoconcentración y deshidratación.    ==>  si persiste==> taquiardia, disminución de PVC y GC==> hipotensión y shock hipovolémico.      Distensión abdominal==>aumento de presión intrabdominal==> disminución de retorno venoso de las piernas==> hipoventilación.  Translocación bacteriana y sepsis.    En obstrucción de colon los cambios electrolíticos son más lentos, si la válvula ileocecal es competente se comporta como asa cerrada,  esto aumenta el riesgo de perforación.      DIAGNÓSTICO  1 ¿Tiene una obstrucción intestinal?  2 ¿Dónde está la obstrucción?  3 ¿Cuál es la naturaleza anatómica y patológica de las lesiones que la provocan?  4 ¿Existe estrangulación?  5 ¿Cuál es el estado general del paciente?    Síntomas cardinales: dolor, distensión, vómito, ausencia de emisión de gases y heces.    Dolor: comienzo gradual, mal localizado, tipo cólico si es mecánica, continuo si es paralítico o isquémico. Intervalo de dolor de 4‐5  minutos. Mientras más baja sea la obstrucción la frecuencia baja.    Distensión abdminal: distendido y timpánico,      Vómitos: primero son de origen reflejo, luego por regurgitación de las asas. Primero alimenticios, luego biliosos (intestinales), más  tarde fecaloides.    Ausencia de emisión de gases y heces: no es un síntomas constante, si la obstrucción es alta puede haber emisión.    Recordar que en obstrucción alta los principales síntoams son náuseas y vómito, en obstrucción baja distensión abdominal.    Exploración física:    Inspección: distendido, ondas perisálticas en delgados,  Palpación: dolor difuso, búsqueda de hernias,  Auscultación: peristaltismo aumentado==> ausencia.    Paraclínicos:      Solicitar: BHC, QSC con CPK, PCR, ES, TP/TTP, GASA (acidosis metabólica?), tele de tórax, ap de abdomen de pie y decúbito..  Deshidratación hemoconcetración alt, electrolticas, leucocitosis.  Rx: Gas intestinal y niveles hidroaéreos, si intesitno delgado u obstrucción alta (imagen en pila de monedas). Si se observa dilatación  intestinal generalizada en tubo digestivo y heces en ampolla o gas, sospecha íleo paralítico.  Aerobilia si íleo biliar, signo de grano de café si vólvulo  Colon x enema: últil si obstrucción de intestino grueso.  TAC con doble contraste: el más sensible si obstrucción sin complicación con material hidrosoluble.    TRATAMIENTO  Si es completa y mecánica cirugía.  Si es parcial mecánica o no mecánica, SNG 48‐72 hs genera remisión en 90% de los casos.   

Datos predictivos: no resolución luego de 72 hrs, SNC con gasto >500ml, >40 años, adherencias complejas, nula presencia de aire en  recto, CPK >130.    El manejo conservador con monitoreo cada 8 hrs de perímetro abdominal, búsqueda de fiebre, taquiardia, irritación peritoneal y gasto  aumentado de SNG.    Gralmente quirúrgico, EXCEPTO adherencias y las incompletas o suboclusiones.    Pseudoobstrucción intestinal e íleos funcionales==> recuperar equilibrio hidroelectlitico.  Adherencias==> SNG, ayuno, control de iones, electrolitos.    Si obstrucción incompleta de intestino grueso==>conservador y qx electiva posteior  Hernia incarcerada==> observación y reducción .    Indicación de Qs: TC con líquido intraperitoneal, edema de mesenterio, signod e las heces en intesitno, isquemia en apred intestinal o  fuga del medio.    Cirugía abdominal 6 semanas previas.    Dolor intenso mayor a 4 en escala análoga + leucos >10mil, PCR >7.5mg/l.  Peritonitis o hernia complicada.  No solución luego de 72 hrs, aun luego de 50‐150ml de medio de contraste.  Laparotomía exploradora es la técnica más recomendada.    EN TU CONSULTORIO REVISAS A UN PACIENTE MASCULINO DE 68 AÑOS DE EDAD, EL CUAL INICIA PADECIMIENTO ACTUAL DE 24 HRS  DE EVOLUCIÓN CON LA PRESENCIA DE DISTENCIÓN ABDOMINAL CON LA IMPOSIBILDIAD PARA CANALIZAR GASES Y DE EVACUAR,  ADEMÁS DE SUMARSE ESTADO NAUSEOSO QUE PROGRESA AL VÓMITO EN VARIAS OCASIONES, AL INICIO FUE ESPORÁDICO PERO  PAULATINAMENTE HA IDO EN AUMENTO, CON ESCALA DE 5 DE 10. APARECIENDO DOLOR DEL TIPO CÓLICO, EL CUAL ES INTENSO, QUE  SE MODIFICABA CON EL VÓMITO AL INCIO, PERO DESPUÉS SE VOLVIÓ CONTINUO E INTENSO, EN LOS ANTECEDENTE DE  INTERROGADOS, REFIERE HABER SIDO OPERADO DE COLECISTECTOMÍA, APENDICECTOMÍA POR APENDICITIS COMPLICADA,  LAPARTOMÍA EXPLORADORA POR LESIÓN DE ARMA DE FUEGO HACE 5 AÑOS, CON RESECCIÓN DE INTESTINO DELGADO.    TU SOSPECHA ES:  A) APENDICITIS AGUDA  B) ISQUEMIA MESENTÉRICA  C) PANCREATITIS AGUDA  D) OCLUSIÓN INTESTINAL    EL GOLD ESTÁNDAR PARA ESTA PATOLOGÍA ES:  A) PLACA SIMPLE DE ABDOMEN  B) TRÁNSITO INTESTINAL  C) TELE DE TÓRAX  D) TAC ABDOMINAL    RESPUESTA D, D    HOMBRE DE 38 AÑOS DE EDAD, CON ANTECEDENTE DE APENDICECTOMÍA HACE 6 MESES Y EN ESTA OCASIÓN INGRESA CON DATOS DE  OBSTRUCCIÓN INTESTINAL.  LA CAUSA MÁS PROBABLE POR LA CUAL EL PACIENTE PRESENTA ESTE CUADRO CLÍNICO ES:      A) ADHERENCIAS INTESTINALES.  B) ABSCESO RESIDUAL.  C) CÁNCER DE COLÓN.  D) ADENITIS MESENTÉRICA.    RESPUESTAS A  El  bloqueo  de  la  luz  intestinal  se  produce  en  su  mayoría  por  factores  extrínsecos  (adherencias,  hernia  inguinal,  hernia  de  pared  estrangulada) y en pocas ocasiones por factores intrínsecos.    Las adherencias intraabdominales por intervenciones quirúrgicas previas  representan el 75% de los casos de obstrucción intestinal.     

PANCREATITIS  PANCREATITIS AGUDA  DEFINICIÓN  15% es considerada grave por presentar necrosis pancreática, 85% leve por presentar daño intersticial.    SALUD PÚBLICA  50% secundaria a colelitiasis, 25% por alcohol y el resto otras causas e idiopática.  FR colelitiasis, alcoholismo, hiperlipidemia, medicamentos, hipercalcemia, trauma, disfunción de Oddi, CPRE, pancreas divisu, Ca  periampular, Ca pánreas, diverticulo periampular, vesculitis, infecciosas, autoinmune (LES, Sjögren)   

PATOGENIA  A) activación de enzimas digestivas  B) secuestro de neutrófilos en páncreas  C) necrosis, edema,    SRIS, IRA y fallo multiorgánico.    DIAGNÓSTICO  2/3 criterios:    Clínica (dolor en epigastrio, irradiación a espalda, intensidad progresiva y súbito, náuseas, vómito, distensión abdominal.    Elevación de amilasa/lipasa al menos al triple.    Alteraciones estructurales en estudios de imagen.      Se debe hacer en las primeras 48 hrs.    Grey Turner's (equimosis en la pared lateral del abdomen)*  Cullen's (equimosis al rededor de la pared abdominal)*  Fox's (cambio de coloración a nivel de lig inguinal)*  *Solo en el 3% de los casos e indican gravedad    APACHE en los primeros 3 días nos define si es leve o grave.  Hematócrito en admisión, 12, 24 hrs también nos permite distinguir.    >55 años, IMC >30, falla orgánica al ingreso, derrame pleural son indicadores de severidad y se deben diagnostiar.      Lipasa es más S y E para diagnóstico. Si la amilasa no llega al triple de la basal decimos que hay hiperamilasemia (causas. Mordidas,  úlceras esofágicas, etc).  Gold standar para valorar pronóstico: proteína C reactiva a las 48 hrs de evolución.  Primer estudio de imagen a pedir: El primer estudio de imagen a pedir es rx de abdmen y torax de pie y decúbito no para diagnóstico,  sino para diagnóstico diferencial, y abocado a pancreatitis==> USG  TAC en las primeras 24 hrs es muy útil ya que nos da pronóstico, se repite a las 72‐120 hrs. El Dr. Sanchez medina dice que hasta las  72hrs! No antes. Nos permite determinar criterios de Baltazar e índice de severidad tomográfica.    Si la TAC demustra algo de necrosis antibiótico para evitar muerte por sepsis (mepropenem, fluoroquinolonas, cefalos  3era o 4ta asociada a metronidazol).    Si >30% de necrosis agregar fluconazol.     Si pancreatitis + sospecha de litiasis biliar==> CPRE dentro de las primeras 72 hrs.    Panceratitis leve:    Falla orgánica mínima, edema intersticial. Falta de respuesta luego de 48‐72 hrs pensar en complicacines.  Pancreatitis grave: falla orgánica importante, necrosis, asbceso y pseudoquiste.    TAS 500cc /24h      CID 30%, o áreas pequeñas con sepsis==>aspiración con aguja fina, tinción, cultivo.    Gram negativo==> imipenem, cirpfloxacino, ofloxacino, pefloxacino.    Gram positico==>vancomicina.    No inicia vía oral según amilasa. Mejor amilasa en orina mínimo de 6hrs, si está disminuida  dieta líquida dieta blanda      Tratamiento quirúrgico  Necrosis pancreática estéril >50% + datos de deterioro.    Ncrosis pnacreática esteril que luego de 2 semana no puede ser alimentado, dolor abdominal aumenta presenta íleo.  Necrosis infectada: necrosectomía y drenaje.  Abscesp pancreático==> drenaje percutáneo o endoscópico.  Pancreatitis aguda con complicaciones, pseudoquiste pancreático==> drenaje.    PRONÓSTICO 

Mortalidad 5% general, 3% si es leve, 17% si es grave.    24‐48 hrs con Ranson  >48 hrs APACHE No aumentar más de 8.  Criterios para UCI:      Inestabilidad hemodinámica, disfunción respiratoria y signos de gravedad (ranson >3, apache II >8, insuf orgánica.    PCR también da pronóstico, incluso procalcitonina.        30‐40% hace pseudoquiste. Esperar 6 semanas, el 60% se reabsorve automáticamente. Si tiene una parede de 6mm y 60cc puede  reabsorverse. El 40% no reabsorve luego de 6 semanas  endoscopia cistogastroanastomosis 

 

 

 

 

  Criterios de Ranson  Al ingreso:  Edad    >55a  Glucosa    >200mg/dl  AST    >250UxL  DHL    >350UxL  Leuc    >16,000    A las 48hrs  Déficit de base      Incremento de nit ureico    Calcio sérico      Disminución de Htc     PaO2        Secuestro de líquidos       

>4mEq  >5mg/dl  10%  6,000 ml 

  INFECCIONES ÓSEAS Y ARTICULARES  OSTEOMIELITIS 

  DEFINICIÓN  Proceso infeccioso que afecta al tejido óseo y su cavidad medular.      ETIOLOGÍA  Bacterias y parásitos (más comúnes) Etiología más frecuente en caso de bacterias S. aureus.  Hongos (inmudeprimidos)  Virus (no son causa)    Niños 1‐4 años: S. aureus, después SBH. SARM en infecciones de tejidos blandos.    Niños > 4años: ¿?    FR: infecciones, cirugía previa, desnutrición, DM descompensada, inmunodeficiencia, adicciones, alt vascualres, traumatismos.    PATOGENIA  Hematógena (más común), inoculación externa (fx espuesta, Qx), extensión de estructura adyacente infectada.    Localización en adultos: cualquier sitio anatómico óseo, 90% miembros pélvicos y solo un hueso.    Localización en niños: nunca la fisis, siempre es metafisaria (invade a médula y cortical).    Aguda  4sem  Residual secuelas      DIAGNÓSTICO 

Fase aguda temprana (24‐48hrs): febrícula mal gral, marcha claudicante, rechazo al caminar, sensibilidad doloroosas.    Fase aguda tardía: (4‐5días): absceso subperióstico. Fiebre, dolor a la movilización tumefacción edema.    Síntomas: [Lactantes] pseudoparálisis de extr afectada, ausencia de fiebre, contractura muscular, irritabilidad, pérdida  de apetito.    [Niño] Limitación de movilidad art, dolor articular, fiebre.    [Adulto] dolor localizado que aumenta tras la mov activa y pasiva de art adyacentes, aumento de vol,  hiperemia, fiebre, escalofríos, celulitis, pérdida de la función.    Paraclínicos: rx simple con técnica para partes blandas (al principio aumento de vol de partes blandas periférico al  hueso), a partir del 7mo día podré pedir técnica para partes óseas donde se verá elevación del periostio, lisis, esclerosis,    reducción de densidad ósea. USG cambio en tejidos blandos, fluidos subperiósticos o yuxtacorticales.    BHC==> leucocitosis [máximo 10,000, neutrófilos totales >6600, segmentados >6%.]  VSG==> elevada entre 1‐5 día[>20 mm/hr]  PCR==> elevada desde 1eras 8 hrs con pico máximo al día 2 [>10mg/l] Recordar que infecciones bacteriana elevan de 4‐ 20 si son leves, graves o quemaduras >20. Virales y emarazo 1‐4.    Hemocultivo Dx en 50%, aspiración de la lesión en 70%.  Luego de 15 días la rx podrá mostrar involucro o absceso subperióstico.  RM: estudio de mayor S y E.    TRATAMIENTO 

 

  IV por 7‐14 dias==> VO 6‐8 sem (hasta que esté asintomático, se normalice VSG)    Si lesión en piel, cubrir con toalla seca y cubrir con plástico. Si fiebre control x medios físicos.  Crioterapia 10‐15min cada 8‐12hrs x 2 días directo en lesión afectada.    MASCULINO DE 4 AÑOS SIN ANTECEDENTES DE IMPORTANCIA, QUE ACUDE A CONSULTA PORQUE LA MAMÁ REFIERE  QUE LLEVA DOS DÍAS COJEANDO DE LA PIERNA IZQUIERDA, ACOMPÑADO DE MALESTAR GENERAL AL ESTAR JUGANDO Y  HABERSE TROPEZADO. EL DÍA DEL INCIDENTE LO LLEVAN A UN HOSPITAL DONDE LO DAN DE ALTA CON DIAGNÓSTICO  DE ESGUINCE DE RODILLA Y LO EGRESAN CON IBUPROFENO.  A SU LLEGADA TA 138/88, FC 95, FR 26 T 37.6, PESO 28, INCAPACIDAD PARA LA MARCHA Y CLAUDICACIÓN A EXPENSAS  DE MPI, DOLOR Y LIMITACIÓN A LOS ARCOS DE MOVIMIENTO DE CADERA Y RODILLA IZQUIERDAS, RESTO DE LA EF SIN  DATOS PATOLÓGICO. RX AP DE PELVIS Y RX DE RODILLAS NORMALES.    CON BASE EN LOS DATOS MENCIONADSO EL SIGUIENTE PASO EN EL ABORDJE DEL PACIENTE SERÍA:  A) RB DE CADERA Y RODILLA  B) BH, PCR Y VSG  C) ASPIRADO DE LÍQUIDO ARTICULAR DE RODILLA Y CADERA IZQUIERDAS  D) AP DE PELVIS EN POSICIÓN DE RANA    ¿CÓMO SE LE LLAMA AL ABSCESO SUBPERIÓSTICO, QUE SE PRODUCE ALREDEDOR DE LA CORTICAL SI SE DEJA LA  EVOLUCIÓN NATURAL DE LA ENFERMEDAD?  A) ABSCESO CORTICAL  B) OSTEOMIELITIS  C) SECUESTRO  D) INVOLUCRO    CON LOS RESULTADO DE LOS LABORATOROS POSITIVOS ¿CUÁL ES EL SIGUIENTE PASO QUE USTED SEGUIRÍA?  A) TX EMPÍRICO VS S. AUREUS  B) TX EMPÍRICO VS S.HEMOLÍTICO  C) TX EMPÍRICO VS H. INFLUENZAE  D) CULTIVO DE LÍQUIDO ARTICULAR E INICIAR TX ESPECÍFICO.    Hueso alrededor de sitio infectado es un involucro, si hay varios segmentos afectados es secuestro.  RESPUESTA B, D, A    ESCOLAR DE 11 AÑOS, CON ANTECEDENTE DE TRAUMATISMO DIRECTO EN MUSLO DERECHO, HACE 12 HORAS PRESENTA DOLOR  AGUDO EN LA ZONA DISTAL DEL MUSLO MOTIVO POR LO QUE ACUDE A LA CONSULTA. A LA EXPLORACIÓN CON TEMPERATURA DE  38°C, INFLAMACIÓN EN EL TERCIO INFERIOR DEL MUSLO DERECHO Y DOLOR A LA MOVILIZACIÓN, LIMITACIÓN FUNCIONAL DE CADERA  Y RODILLA. BIOMETRÍA HEMÁTICA CON LEUCOCITOSIS Y LA VELOCIDAD DE SEDIMENTACIÓN GLOBULAR AUMENTADA. RADIOGRAFÍAS  DE FÉMUR NORMALES.         1 ‐ EL GERMEN PATOGENO MAS PROBABLE DE ESTA PATOLOGIA ES:  STAPHILOCOCUS AUREUS.  ENTEROBACTER.  ESCHERICHIA COLI.  PSEUDOMONAS AERUGINOSA.             2 ‐ EN CASO DE DEJARSE A LIBRE EVOLUCIÓN SE ESPERA ENCONTRAR CAMBIOS RADIOLÓGICOS EN EL SIGUIENTE TIEMPO.  2 DÍAS.  3 DÍAS.  7 DÍAS.  10 DÍAS.          3 ‐ DESPUÉS DE 36 HRS DE INICIADOS LOS ANTIBIÓTICOS EL PACIENTE PERSISTE CON MANIFESTACIONES LOCALES Y SISTÉMICAS, POR  LO QUE SE LE DEBE REALIZAR:  COLOCACIÓN DE SISTEMAS DE IRRIGACIÓN.  TRACCIÓN DE LA PIERNA.  INMOVILIZACIÓN DE LA EXTREMIDAD.  DESBRIDACIÓN DEL FOCO ÓSEO.      Dolor a la movilización activa y pasiva en ARTICULACIONES ADYACENTES al sitio de osteomielitis es un dato pivote, en artritis séptica el 

dolor es agudo MONOARTICULAR (solo 10%‐20% lesiona múltiples articulaciones, generalmente se asocian a Neisseria, gram negativo y  estrepto).  RESPUESTA  1A, 2C, 3D 

    ARTRITIS SÉPTICA  DEFINICIÓN  Generalmente causada por S. aureus. Su tratamiento es una urgencia médico‐quirúrgica  Presentación crónica se debe más a micobacterias u hongos filamentosos.    Generalmente se debe a contaminación hematógena a partir de un foco primario,      SALUD PÚBLICA  75% son cocos gram+ (mas fre S. aureus)  20% son bacilos gram‐  FR: catéteres intavasculares, cirugía de trauma, inmunodeficiencia (DM, LES, alt hepáticas, UDVP), hepatitis C, hipotiroidismo, cirrosis,  EPOC.    DIAGNÓSTICO  Clínico: fiebre, disminución de la función, dolor progresivo, calor, tumefacción.  Paraclínico: pedir BHC, VSG, PCR.    Punción articular 3 tubos  1) 5‐10ml en tubo estéril ==> cultivos y gram  2)5ml con heparina o EDTA ==> citológico y bioquímico (glucosa, proteína y lactato)  3)1ml en tubo seco para observar coagulación ==> sinovial normal no coagula.      Si cultivos normales, sospecha de infección==> biopsia.    Si sospecha de artritis séptica===> 2 hemocultivos    Estudio de imagen inicial ==> rx (normal si reciente inicio)  Estudio de imagen más específico===>USG (pedir si rx normal)    TRATAMIENTO  Si artritis séptica o sospecha: drenaje + AB  Sin FR:        Dicloxacilina    >40kg: 1‐2hr IV c 6h        40kg: 1gr c 24h        40kg: 500mg VO c6h        40kg: 150‐300mg c6‐8h        40kg: 750‐1500mg IV c 8‐12h  +      40kg: 1gr c 24h        40kg: 250‐500mg c 12        40kg: 500mg IV c6h  +      40kg: 1gr IV c 8 hr 

 

 

 

40kg:300mg VO c 12h    TMP/SMX    >40kg: 160/800 c 12 h      Rifampicina   cefuroxima  Con Fr    vanco + cefota ==> rifam+TMP/SMX  Si Neisseria  ceftriaxona    *Si alergia a dicloxa agregar clinda.      Quirúrgico: En todos los casos artrotomía, artroscopia, artroscopia + debridación, aspiración y lavado por punción.  Artritis séptica del hombro, codo, cadera, rodilla==> lavado artroscópico  Artritis séptica multifocal==> drenaje abierto     

TRAUMA DE MIEMBROS  TRAUMA DE CODO  PUNTOS BÁSICOS 

 

         

 

DEFINICIÓN  Consiste en 3 articulaciones: humerocubital, húmero radial y radiocubital.    SALUD PÚBLICA  Trauma de extermidad superior más frec 1) luxación de hombro, 2) luxación de codo.   

    DIAGNÓSTICO  Lesión      Mecanismo          Supracondilea humeral      carga axial con codo en flexión  esteroides inyectados localmente, rehabilitación, si 6 meses y no mejoría Qx    Bursitis  Aumento de    volumen en cara posteior de codo.    Tx aines, inmovilización a 45° sin aplicar presión. si no mejora==> aspiración de bolsa preolecraneana, estudio de tinción, búsqueda de  cristales.    Si infecciosa dicloxacilina 500mg cada    6hrs x 10 días.    2da TMP/SMX 

  ESGUINCE DE TOBILLO  Hueso Ligamento Hueso  Hueso Tendón Músculo  DEFINICIÓN  Ruptura total o parcial de ligamentos en la articulación del tobillo.    Rx AP, lateral de tobillo.          AP en rotación medial de 15‐30 grado en el tobillo lesionado.    Solicitar siempre y cuando cumpla con reglas de Ottawa 

    SALUD PÚBLICA  Mecanismo común (supinación, aducción, inversión en flexión plantar)    DIAGNÓSTICO      TRATAMIENTO  Piroxicam tabletas e 40mg. Tomar 1 tableta cada 12 hrs por 3 días, posteriormente cada 24 hrs 4 días.  Paraccetamol.    Si edema bimaleolar y dolor importante combinar AINE + paracetamol (analgésico)    En las primeras 72hrs.    Hielo local cada 20 min x 8 hrs    Ventaje elástico no compresivo inicial    Movilización activa moderada  Despúes    delas 72 hrs    Vendaje compesivor.    Ejercicios de propiocepión (arrugar toalla con los dedos, rodar botella)  Inmovilización c férul d yeso esguinces grado I y II.    No calor local ni masaje directo.    REHABILITACIÓN  Ameritan envío:  Luxación y fractura asociada.  Esguince grado III y IV   

 

 

FRACTURA INTRACAPSULAR DEL EXTREMO PROXIMAL DEL FÉMUR  DIAGNÓSTICO  Dolor inguinocrural a la movilización, imposibilidad para elevar el talón, rotación extera y acortamiento.  RX Anteroposterior de cadera con rotación medial de 15 grado de la cadera afectada.  Axial de cadera afectada.    TRATAMIENTO  Cefalosporina 1 o 2 da generación, 1 dosis antes, 2 después de cirugía.  Antitrombosis: 12 hrs previas y 12 hrs posterior x 10 días con: enoxaparina o nadrparina    Anaglesia:Metmaizol, ketorolaco, clorhidato de buprenorfina, diclofenaco.      Reducción abierta con fijación interna en adultos mayors con actividad física, si falla==> sección artroplastica o hemiartroplastía.  Fractura no desplazada, impactada que sea ambulatorio=> fijación interna anes de desplazmaiento,  Fractura desplazada: hemiartroplastia primaria si adulto mayor a considerar reemplazo total.        

SÍNDROME DE HOMBRO DOLOROSO  DIAGNÓSTICO  Manguito rotador: dolor en región deltoidea con limitación de abducción, rotaciones.    Test de Neer, Hawkins‐Kennedy, Yocum  Tendinitis bicipítal: dolor en cara anterior que irradia a tendón bicipital hasta antebrazo.      Test de speed, Yergason.  Bursisits: dolor en tercio superior de hombro, limitación de abducción pasiva y activa.    Si sospecha de rotura de manguito rotador==> USG o RMN 

    TRATAMIENTO  1era Anaglésico simple, 2da AINE, 3e    [TENDINITIS DEL MANGUITO ROTADOR][r]  MASCULINO DE 33 AÑOS DE EDAD, TENISTA PROFESIONAL DESDE LOS 15 AÑOS, PRESENTA DOLOR INTENSO EN AMBOS HOMBROS  PREDOMINANDO LADO DERECHO, ASI COMO INCAPACIDAD PARA LA MOVILIZACION. ACUDE A LA CONSULTA Y LE DIAGNOSTICAN  TENDINITIS DEL MANGUITO ROTADOR.    LA CAPSULITIS ADHESIVA QUE ES SECUELA DE LA TENDINITIS DEL MANGUITO ROTADOR SE PUEDE PREVENIR:  A) EVALUANDO LA MOVILIDAD CONTRARESISTIDA.  B) EVITANDO LOS SOBREESFUERZOS.  C) INFILTRANDO ESTEROIDES.  D) INMOVILIZANDO BREVEMENTE EL HOMBRO.    RESPUESTAS D  Manguito de los rotadores: conjunto de músculos y tendones conectados a la cabeza del húmero y dan estabilidad al hombro.    Supraespinoso  Fosa supraespinosa de la escápula ‐ tubérculo mayor del húmero.  Permite primeros 15‐20° de abducción  Inftraespinoso    Fosa infraespinosa de la escápula    ‐    tubérculo mayor del húmero.  Rota brazo lateralmente  Teres minor  Borde lateral de la escápula  ‐ tubéculo mayor del húmero    Rota brazo lateralmente  Subescapular  Fosa subescapular ‐ tubérculo menor del húmero      Rota brazo medialmente    Estos tendones pueden lesionarse con traumatismo o movimientos viciosis.      La capsulitis adhesiva es la adhesión real de la cápsula del hombro a la cabeza del húmero, ocasionando una restrición del movimiento  del hombro.    Fase 1‐Dolorosa: dolor nocturno incidioso. 2‐9 meses  Fase 2‐ Congelada o adhesiva: se agrega incapacidad para moverse a gran amplitud. 3‐9m  Fase 3‐ Descongelación o regresiva: mejora dolor y limitación     

Evitar la inmovilización prolongada en pacientes que pueden estar predispuestos a desarrollar capsulitis adhesiva es importante. La  infiltración de esteroides puede ayudar en el tratamiento pero no es de primera elección.    Deberá inmovilizarse por un periodo no mayor a 15 días y se deberá anexar tratamiento analgésico y antiinflamatorio. Posteriormente  rehabilitación. De hecho más que inmovilización debe preferirse reposo.      CERVICALGIAS, LUMBALGIAS Y TUMORES 

CERVICALGIAS  DEFINICIÓN  Cervicobraquialgia cuando se irradia a ext superiores a través de raiz nerviosa cervical baja (C5‐C8).    DIAGNÓSITCO  AP y lateral de columna cervical    TAC si origen traumático y no se visualiza C7     

LUMBALGIAS  DIAGNÓSTCO  Clasificar en: 1) lumbalgia inespecífica, 2) lumbalgia potencialmente asociada con radiculopatía, 3) lumalgia pontencialmente asocaida  con enfermedad sistémica.    Pensar en neo si >50 años, ant de Ca, pérdida de peso, no mejora en4‐6semanas, dolor continuo o progresivo, dolor en reposo y  nocturno.      95% de casos de lumbalgia aguda es inespecífica, >50 tendrá alteraciones pr Rx.    Pedir Rx AP de columna lumbar y lateral si fiebre x más de 48 hrs, osteoporosis, enf sistémica, déf sensitivio o motor, uso crónico de  esteroides, >50 años, inmunosupresión, sospecha de espondilitis anquilosante.      Prueba de Lassegue si sospecha de radiculopatía 91% sensible, 26% específica. Si duele entre los 30‐70° ==> hernia de disco. (si levanta  la pierna opuesta y produce dolor 29 sensibilidad y 88 especificidad.    Paresia aporta E 91% para compresión 

    Retención urinaria 90% S, 95% E, y anestesia en silla de montar==> cauda equina   inflamatorio    TRATAMIENTO  1era línea paracetamol  2 AINE periodos cortos menor tiempo posible (ibuprofeno tiene menos efectos gastrointestinales)  No se recomienda AINE >4 semanas.  3era opiaceos menores  Relajante muscular no benzodiacepínico, no más de 2 semanas especialmente si espasmo muscular.  Lumbalgia crónica con fracaso a tx convencionales antidepresivos tricíclicos.    NO hay evidencia de que la vitamina B sea eficaz para el tx de dolor lumbar inespecífico.    Masaje disminuye dolor y aumenta capacidad funcional.  Aplicación de calor local.      [LUMBALGIA] [r]  MUJER DE 32 AÑOS DE EDAD, INSTRUCTORA DE UN GIMNASIO, CON ALCOHOLISMO Y TABAQUISMO POSITIVOS DESDE HACE 10 AÑOS.  ACTUALMENTE SIENDO TRATADA POR EL SERVICIO DE PSIQUIATRÍA POR DEPRESIÓN. ES ENVIADA CON USTED PARA DESCARTAR  PATOLOGÍA ORGÁNICA. REFIERE DOLOR EN REGIÓN LUMBAR DE 5 SEMANAS DE EVOLUCIÓN, EL CUAL ES INTENSO, INCAPACITANTE,  AUMENTA CON CAMBIOS DE POSICIÓN Y SE IRRADIA A REGIÓN GLÚTEA DERECHA.    EL SIGUIENTE ES EL PRINCIPAL FACTOR DE RIESGO PARA LUMBALGIA QUE DEBEMOS CONSIDERAR EN LA PACIENTE:     

A) ALCOHOLISMO  B) EJERCICIO EN GIMNASIO  C) DEPRESIÓN  D) TABAQUISMO.     CON BASE AL TIEMPO DE EVOLUCIÓN CLASIFICAREMOS LA LUMBALGIA DE LA PACIENTE COMO:  A) AGUDA  B) SUBAGUDA  C) CRÓNICA  D)RECIDIVANTE    RESPUESTAS A, B  Existe asociación entre malos hábitos alimenticios, sedentarismo,    sobrepeso, obesidad, alcoholismo y    posiciones visciosas con  lumbalgia.    No hay evidencia de que el tabaco sea factor de riesgo para el desarrollo de dolor lumbar.    El ejercicio benéfico y parte del tratamiento.  La depresión favorece la cronicidad, es de decir la persistencia más no la causa.      Respecto al tiempo de evolución:    Aguda si 12 semanas      [r]  MUJER DE 28 AÑOS DE EDAD, DIRECTORA DE UN JARDÍN DE NIÑOS. ACUDE SOLICITANDO ATENCIÓN MÉDICA YA QUE, TRAS CARGAR A  UN NIÑO EL DÍA DE AYER, PRESENTA DOLOR INTENSO EN REGIÓN LUMBAR QUE LE INCAPACITA PARA REALIZAR SUS LABORES. LLAMA  LA ATENCIÓN SU ACTITUD IMPACIENTE Y NERVIOSA, MUESTRA MOVIMIENTOS INVOLUNTARIOS EN PÁRPADO DE OJO DERECHO.  COMO PARTE DE SU TRATAMIENTO ESTARÁ INDICADA EN ESTE MOMENTO LA REHABILITACIÓN MEDIANTE:      A) LA APLICACIÓN DE CRIOTERAPIA  B) EL USO DE SOPORTES LUMBARES  C) LA REALIZACIÓN DE LOS EJERCICIOS DE WILLIAMS  D) LA INDICACIÓN DE TERAPIAS COGNITIVO CONDUCTUALES    RESPUESTA C  La aplicación de calor local en lumbalgia aguda y subaguda,    solo se contraindica en caso de alt sensitivas, escaras, alt de coagulacióon  y procesos neoplásicois. La crioterapia no va al caso.    El uso de soportes lumbares no tienen efectividad comprobada.  Los ejercicios de flexión o ejercicios de Williams están indicados para el tx del sx doloroso lumbar.  La etapa cognitivo conductual no se recomienda para el tratamiento del dolor en etapa aguda, si en subagudo, previene cronicidad y  recurrencias.    [La pregunta claramente dice rehabilitación, eso hace pensar que es tx a largo plazo y posterior ¬¬]   

 

 

OSTEOSARCOMA    DEFINICIÓN  Tumor óseo más frec en la niñez y adolescencia.    SALUD PÚBLICA  10‐20 años. Si población negra o 0‐10 años descartar primero sarcoma de Ewing.    PATOGENIA  45% de todos los sarcomas óseos (neoplasia de cels fusiformes que produce matriz osteoide [hueso nimineralizado] o hueso.    DIAGNÓSTICO  Generalmente afecta metáfisis de huesos largos.  Frec en porción distal del fémur, zona proximal de tibia y húmero.    Síntomas: aumento de volumen en región afectada, puede no haber afección de tej blandos, hay disminución de movimiento y dolor.  (En ostemielitis primero se ve la afección a tejidos blandos y después al hueso, también tiende a haber más fiebre o focos infecciosos a  distancia)  Paciente generalmente acude por dolor e inflamación.      Paraclínicos: rx simple      lesión lítica de aspecto apolillado,      reacción perióstica con espículas (imagen en sol naciente),      triangulo de Codman  TC útil para definir destrucción ósea y calcificación, RMN útil para precisar etrensión intramedular y afección a tejidos blandos. 

Rx y TC para descubrir mets pulmonares.    Gammagrafía para descubrir mets óseas.    Confirmación anatomopatológica: punción biopsia con aguja, y de ser posible con biopsia a cielo abierto.     

    TRATAMIENTO  QT + intervención para conservar de miembro    (>80% de los pacientes).  QT==> doxorrubicina, ifosfamida, cisplatino, metotrexato + ac folínico.      Osteosarcoma es radiorresistnete.      PRONÓSTICO  Principal factor pronóstico es la respuesta a QT  Sobrevida a 5 años, 60‐70%.    CONTENIDO ADICIONAL 

  Diáfisis    Metáfisis    Epífisis       

  Médula ósea  Cartílago de crecimiento  Articulación, médula ósea. 

OSTEOSARCOMA  MASCULINO DE 19 AÑOS, REFIERE DOLOR EN MUSLO IZQUIERDO INCREMENTÁNDOSE POR LAS NOCHES. NO HAY RELACIÓN CON LA  ACTIVIDAD FÍSICA Y SE NIEGA FIEBRE O PÉRDIDA DE PESO. A LA EXPLORACIÓN SE PALPA MASA DURA Y FIJA A PLANOS PROFUNDOS EN  TERCIO DISTAL DE FÉMUR. LA RADIOGRAFÍA PRESENTA LESIÓN PENETRANTE EN FÉMUR CON DESTRUCCIÓN CORTICAL CON ASPECTO DE  "RAYOS SOLARES".    EL DIAGNÓSTICO PROBABLE DE ESTE PACIENTE ES EL DE:  A) OSTEOSARCOMA.  B) QUISTE ÓSEO.  C) OSTEOMIELITIS.  D) OSTEOCONDROMA    LOS HALLAZGOS RADIOLÓGICOS PRESENTES EN EL PACIENTE SON SECUNDARIOS A:  A) INFECCIÓN EN UN SITIO CERCANO AL HUESO.  B) DISPLASIA ÓSEA Y PLEOMORFISMO CELULAR METAFISIARIO.  C) REMANENTE DE LA PLACA DE CRECIMIENTO DEL CARTÍLAGO  D) INFECCIÓN QUE VÍA HEMATOGENA LLEGA A LA ARTICULACIÓN.    RESPUESTAS  A, B,      MUJER  DE  23  AÑOS  DE  EDAD  PRESENTA  RADIOGRAFÍA  CON  DATOS  DE  TUMORACIÓN  MUTILOCULADA  EN  LA  ZONA  EPIFISARIA  DEL  EXTREMO DISTAL DE FÉMUR IZQUIERDO. ¿CUÁL ES EL DIAGNÓSTICO MÁS PROBABLE?  A) SARCOMA OSTEOGÉNICO  B) TUMOR DE CÉLULAS GIGANTES  C)SARCOMA DE EWING  D)MIELOMA MÚLTIPLE     RESPUESTA B  EL ASPECTO MULTILOCULADO O EN POMPAS DE JABÓN ES CARACTERÍSTICO DEL TUMOR DE CÉLULAS GIGANTES, MUY FRECUENTE EN  MUJRES DE 20‐40 AÑOS] CTO TRAUMATLOGÍA PAG 56  El tumor de células gigantes es un tumor raro, constituye el 5% de los tumores óseos primarios. Ocurre en pacientes entre los 20 y 40  años, siendo raro en los menores de 10 y en mayores de 50 años. Tiene un ligero predominio del sexo femenino y más frecuente en países  orientales que occidentales. Habitualmente se loalizan en huesos largos, con afectación de la epífisis y de localización excéntrica, pueden  propagarse a metáfisis, provocar destrucción cortical y extensión eventual a tejidos blando y al especio articular.    Los tres sitios más habituales de localización en orden de frecuencia son: 1 extremo distal del fémur, 2 extremo proximal de tibia, 3  extremo distal del radio.    Clínicamente, el dolor es el signo más freucente de presentación, tanto si se asocia o no a fractura patológica. Localmente la piel puede  estar hiperémica, eritematosa y si el tumoer crece puede aparecer ciruclación colateral con presencia de masa palpable. La clínica es  infrecuente y si el TCG tiene localización raquídea o sacra presenta signos y síntomas neurológicos.    El  diagnóstico  de  TCG  suele  realizarse  por  laws  manifestaciones  clínicas  y  radiográficas,  teniendo  en  cuenta  lo  spisible  sdiagnóstico  diferenciales  Cmo  pruebas  complementarias,  la  RMN  es  el  método  más  aprovechoso  para  determinar  la  extensión  y  l  estadiaje,  la  gammagrafía es uilizada para detectar TCG multicéntricos que son raros ( anestesia en borde lateral del hombro   

DIANGÓSTICO  Clínico, radiográfico (AP y escápula en Y)    TRATAMIENTO  Reducción urgente, maniobra de "tracción contracción" es la menos traumática y lesione smenos, maniobra truamática no se  recomienda.    Cabestrillo + AINES   

LUXACIÓN DE CADERA  DEFINICIÓN  Por mecanismo de alta energía.  Localización más frecuente es posterior.    CLÍNICA  Acortamiento de miembro afectado  Signo del pudor: rotación interna y flexión de cadera.  Dolor intenso  Asocia lesión del nervio ciático.      DIAGNÓSTICO  Clínico y radiográfico: AP de pelvis.      TRATAMIENTO  Reducción urgente bajo antesia por gran riesgo de daño vascular con luxación de cadera y posición de pudor.        MUJER DE 68 AÑOS DIABÉTICA CONTROLADA QUE ACUDE AL SERVICIO DE URGENCIA POR DOLOR INTENSO EN MT DERECHO AL SUFRIR  CAÍDA DE 4 ESCALONES Y AL TRATAR DE SOSTENERSE, PRESENTA DOLOR EN HOMBRO DERECHO E INCAPACIDAD FUNCIONAL.    A LA EF SE OBSERVA CON ASIMETRÍA DEL HOMBRO DERECHO CON RESPECTO AL CONTRALATERAL, PALPACIÓN DE MASA ANTERIOR NO  MÓVIL Y FIJA A PLANOS PROFUNDOS, MOVILIDAD DEL HOMBRO ABOLIDA POR DOLOR Y ANESTESIA EN SUPERFICIE ANTEROLATERAL  DEL HOMBRO, NEUROVASCULAR A NIVEL DE LOS DEDOS CONSERVADA.      POR LA PRESENTACIÓN CLÍNICA USTED PENSARÍA QUE EL PACIENTE SUFRE DE:  A) OSTEOBLASTOMA  B) FRACTURA EN HOMBRO DERECHO  C) LUXACIÓN GLENOHUMERAL  D) LESIÓN DEL PLEXO BRAQUIAL POR DISTENSIÓN    El caso NO nos cuenta traumatismo directo en el MT, sino un mecanismo de rotación externa y abducción forzadas del hombro siendo  este el mecanismo de lesión que ocasiona luxación.      A LA EF CÓMO INTERPRETARÍA LA ASIMETRÍA ENTRE AMBOS HOMBROS:  A) VACIAMIENTO DE LA CAVIDAD GLENOIDEA Y DESPLAZAMIENTO HACA EL PLANO ANTERIOR  B) MASA PROBABLEMENTE DE ORIGEN TUMORAL BENIGNO  C) FRAGMENTOS DESPLAZADOS DE UNA PROBABLE FRACTURA  D) HIPOTROFIA MUSCULAR POR UNA PROBABLE LESIÓN NERVIOSA      ¿CÓMO DIAGNOSTICARÍA USTED UNA LESIÓN DEL NERVIO AXILAR?  A) ANESTESIA ESQUINA POSTEROLATERAL DE LA AXILA  B) ANESTESIA BORDE LATERAL DEL HOMBRO  C) ANTESIA PRIMER ESPACIO INTERDIGITAL  D) ANTESTESIA BORDE CUBITAL PALMAR DE LA MANO    RESPUESTA C, A, B     

HALLUX VALGUS  DEFINICIÓN  Desviación lateral de la falange proximal (puntas hacia afuera de la línea media), y desviación medial del primer metatarsiano (punta de  los metatarsianos acercándose a línea media). más común en mujeres. 

 

  CLÍNICA  Bunion o juanete: prominencia medial dolorosa correspondiente a la cabeza del primer metatarsiano.    Superposición del primer dedo sober el segundo dedo.      PARACLÍNICA  Rx pie  AP      lateral con apoyo      blicua      TRATAMIENTO  No quirúrgico: calzado en horma ancha, plantilla con soporte longitudina interno.  Quirúrgico: la única indicación es el dolor, nunca estética.        PACIENTE MASCULINO DE 58 AÑOS DE EDAD CON ANTECEDENTE DE HIPERURICEMIA CONTROLADA QUE ACUDE A CONSULTA POR  SUFRIR DOLOR DE LARGA EVOLUCIÓN EN EL PRIMER DEDO (DEDO GORDO) DEL PIE IZQUIERDO. EL DLOR ES CONTANTE Y AUMENTA  CON EL USO DE ZAPATOS ANGOSTOS, NO MEJORA CON MEDICAMENTOS ANALGÉSICOS.  A LA EF SE OBSERVA AUMENTO DE VOLUMEN EN SUPERFICIE INTERNA DE LA ARTICULACIÓN METATARSOFALÁNGICA DEL PRIMER  DEDO DEL PIE, LIGERAMENTE HIPERÉMICA Y SIN AUMENTO DE TEMPERATURA, MOVILIDAD CONSERVADA NO DOLOROSA. ÁC. URICO  5.1 MG/DL, GLUCOSA DE AYUNA 148 MG/DL, TA 180/106, FC 80 FR 21    CON LOS DATOS CLÍNICOS OTORGADOS USTED INTEGRA EL DIAGNÓSTICO DE:    A) ATAQUE AGUDO DE GOTA  B) GOTA INTERCRÍTICA  C) JUENETE O HALUX VALGUS  D) HIPERURICEMIA    COMO TRATAMIENTO DE PRIMERA ELECCIÓN EN ESTE PACIENTE USTED INDICARÍA: M  A) COLCHICINA  B) ZAPTOS DE HORMA AMPLIA, AINES Y APARATO ADUCTOR DEL PRIMER DEDO.  C) COLCICINA MÁS INDOMETACINA 50MG BID  D) PROBENECID 500MG DOS VECES AL DÍA.    RESPUESTA C, B     

LESIONES COMUNES EN RODILLA   

Las lesiones más cmunes son ligamentarias y de meniscos.      Lesiones ligamentarias: hay dos ligamentos colaterales, el interno (medial) y el externo (lateral).        Mecanismo de lesión:    valgo forzado ==> lesión de LCI          varo forzado==> lesión de LCE    Clínica: no derrame articular, dolor a la palpación medial o lateral, bostezos lateral y        medial positivos. Flexión y extensión completas con leve dolor.      Rx y RM n indicadas.      Tx: Conservador con inmovilización rígida, hielo, AINES.    Ligamento cruzado anteior    Mecanismo de lesión: flexión, rotación y valgo forzado.      Clínica: derrame articular importante mediato con sangre    No dolor a la flexión y/o extensión    Prueba de Lachman, cajón anterior y pivot shift positivo.      Rx solo para busca fx asociadas.      RM Solo si ya no hay inflamación (luego de 10 días aprox)    Tx inmovilizar con rodillera rígida, AINES, hielo, analgésicos.      Meniscos: menisco lateral y medial.      Mecanismo de lesión: torsión interna o externa con flexión de la rodilla    Clínica: derrame articular moderado, tardío, sin sangre, dolor al palpar línea articular.    Chasquidos y dolor al flexionar y extender.    Rx no indicada    RM de primera elección.      Tx inmovilizar con rodillera rígida, hielo, AINES.    PACIENTE MASCULINO DE 32 AÑOS DE EDAD SIN ANTECEDENTES DE IMPORTANCIA QUE ACUDE A CONSULTA POR DOLOR EN RODILLA  IZQUIERDA DE 20 DÍAS DE EVOLUCIÓN.  AL ESTAR JUGANDO FUTBOL REFIERE HACER UN "QUIEBRE" Y GIRAR HACIA EL LADO EXTERNO CON EL PIE FIJO AL SUELO, EL CUAL  OCASIONÓ UN AUMENTO DE VOLUMEN LIGERO QUE NO LO IMPOSIBILITÓ PARA JUGAR, SIN EMBARGO UNAS HORAS DESPUÉS REFIRIÓ  MÁS AUMENTO DE EVOLUMEN Y DOLOR PARA DOBLAR LA RODILLA.    ACTUALMENTE REFIERE QUE AL BAJAR ESCALERAS O AL BAJAR UNA PENDIENTE LA RODILLA "SE LE VA" A LA EF PRESENTA LIGERO  AUMENTO DE VOLUMEN, PRUEBAS ESPECIALES PARA MENISCO NO VALORABLES POR DOLOR, ÚNICAMENTE CAJÓN POSITIVO.      EL MANEJO INMEDIATO POSTERIOR A LA LESIÓN SERÍA:  A) COLOCACIÓN DE CLORURO DE ETILO, FÉRULA TIPO CALZA A LA RODILLA, AINES Y HIELO EN LAS PRIMERAS 2 4HRS.    B) DEBIDO AL AUMENTO DE VOLUMEN UNA PUNCIÓN DE LÍQUIDO ARTICULAR, INMOVILIZACIÓN RÍGIDA CON RODILLERA Y HIELO.  C)COLOCACIÓN DE RODILELRA RÍGIDA CON VARILLAS LATERALES, AINES, ANALGÉSICOS Y CALOR LOCAL PARA RELAJAR LA  MUSCULATURA.  D) INFILTRACIÓN DE BETAMETASONA Y COLORACIÓN DE RODILLERA RÍGIDA.    POR EL MECANISMO DE ELECCIÓN Y LA SEMIOLOGÍA DEL CASO, USTED PENSARÍA QUE EL PACIENTE TENGA UN DIAGNÓSTICO DE:    A) LIGAMENTO COLATERAL EXTERNO  B) MENISCO EXTERNO  C) FRACTURA CONDRAL  D) LIGAMENTO CRUZADO ANTERIOR    Estos pacientes puede referir datos de inestabilidad con movimientos de flexión, recuerda que la rodilla " se le va".      ¿CUÁL SERÍA EL ESTUDIO DE PRIMERA ELECCIÓN QUE USTED SOLICITARÍA?  A) RADIOGRAFÍA AP, LATERAL Y OBLICUA DE RODILLA  B) RESONANCIA MAGNÉTICA DE RODILLA  C) ARTROCENTESIS DE RODILLA PARA ESTUDIO CITOQUÍMICO  D) RESONANCIA MAGNÉTICA HASTA DESPUÉS DE 2 SEMANAS.      El estudio de primera elección siempre será la radiografía ya que ante cualquier traumatismo siempre se está obligado a descartar  alguna fractura, el estudio gold standard es la RM y se indica cuando la inflamación ya disminuyó.      RESPUESTA A, D, A 

PIE PLANO  DEFINICIÓN  Pie con arco plano, asocia valgo de talón.   

    Flexible: más frecuente es una variación de la psotura normal del pie. Común en niños pequeños, asintomácitos, rengos de mov  normales, al elvar dedos del pie muestran una inversión del talón que corrige la deformiad.      Tx    Mejoran sin tramiento la mayoría.    Fisioterapia.    Rígido: aplanamiento persistente de arco con restricción del movimiento. Asocido a enf neuromusculares.      Rx: AP (desviación del astrágalo)    Lateral, verticalización (flexión plantar) del astrágalo.      Tx: corrección quirúrgic.a      PACIENTE MASCULINO DE 6 AÑOS DE EDAD ASINTOMÁICO SIN ANTECEDENTES DE IMPORTANCIA, QUE ACUDE PORQUE LA MADRE  REFEIRE QUE SU HIJO SE CANSA MUCHO AL CORRER Y QUE METE LOS PIES.    A LA EF CON SV NORMALES, SE OBSERVA UN VALGO DE  RETRO PIE Y LIGERA TENDENCIA AL EQUINO QUE CORRIGE A LOS 90° DE FLEXIÓN DORSAL, AL ELEVAR LOS DETOS DEL PIE EL VALGO DE  RETROPIE SE CORRIGE. LAS RADIOGRAFÍAS NO MUESTRAN DATOS PATOLÓGICOS.      EL DIAGNÓSTICO QUE USTED INTEGRARÍA CON LOS DATOS BRINDADOS ES:    A) PIE PLANO VALGO FLEXIBLE    B) PIE EQUINO VARO ADUCTO CONGÉNITO (PEVAC)  C) PIE PLANO VALGO RÍGIDO  D) COALICIÓN TARSAL.      EL TRATAMIENTO QUE USTED INDICARÍA AL PACIENTE SERÍA:    A) NADA, Y EXPLICA A LOS PADRES QUE DESPUÉS DEL SEGUNDO DECENIO DE LA VIDA ESTE PADECIMIENTO SE CORRIGE.    B) CORRECCIÓN QUIRÚRGICA CON FIJADOR EXTERNO Y ALARGAMIENTO DE TENDÓN DE AQUILES.  C) EJERCICIOS DE ESTIRAMIENTO DEL TENDÓN DE AQUILES Y FORTALECIMIENTO DEL TIBIAL ANTERIOR Y EXPLICAR A LOS PADRES QUE  DESPUÉS DEL SEGUNDO DECENIO DE LA VIDA ESTE PADECIMIENTO SE CORRIGE  D) PLANTALLAS CON SOPORTE LONGITUDINAL INTERNO    RESPUESTA A, C    Halux valgus 

 

   

      Talus valgus 

 

 

Talus varus 

 

 

DISPLASIA DEL DESARROLLO DE LA CADERA    DEFINICIÓN  Anormalidad de la articulación coxofemoral del niño, involucra el borde acetabular y tercio proximal del fémur, ocasionando displasia y  luxación durante el desarollo fetal o infantil.      FR: H1:M3 (principal), primípara, pélvico, historia de hiperlaxitud familiar, oligohidramnios, Sx Down, artrogriposis, menos importante  gemelar.    El lado izquierdo es el más afectado.      CLÍNICA  Diagnóstico clínico se divide en 3 etapas  Del 1er día de nacido a los 10 meses: Maniobra de Ortolani (reducción/Clonk/abducción!) y Barlow    (luxación/aducción) reducción de  ángulo de movimiento abductor del lado afectado, asimetría en pliegues.    10‐12 meses: Galeazzi (acostado, médico en los pies, flexiona las rodillas y una se ve más abajo), signo de Pistón (hace movimiento),  limitación de la abducción, asímetría en miembros pélvicos.    >12 meses: Signo de Trendelemburg (insuficiencia del glúteo medio), si reviso izquierda pido que pierna izquierda la deje abajo y  levanté la derecha= se vascula.    Si cae el lado derecho, está luxado el izquierdo.      PARACLÍNICOS  USG es el método de elección diagnóstica en los primeros 3 meses de vida (muy bueno pero operador dependiente).    Rx gold estándar y primera elección    en términos generales, especialmente en 2 años 30° 

      TRATAMIENTO  Conservador los primeros 6 meses de vida.    Arnés de Pavlik: durante 3‐5 mses y hasta que se alcance abducción completa.    Quirúrgico: solo en caso de falla de método conservador.    >1 año no es candidato a tx quirúrgico    PACIENTE FEMENINO DE 1 SEMANA DE VIDA G1P0C1 POR PRESENTACIÓN PÉLVICA, PRODUCTO DE 39.5SDG, PESO 2.8KG, TALLA 46CM,  APGAR 9 A LOS 5MIN, NO REQUIRIÓ REANIMACIÓN NEONATANAL. CON ANTECEDENTE DE PRECLAMPSIA, QUE ACUDE PARA REVISIÓN.  EF TIENE LIGERO CAPUT SUCCEDANEUM EN PARIETAL DERECHO, ESTÁ DORMIDA PERO RESPONDE A LOS ESTÍMULOS. ESTÁN  PRESENTES TODOS LOS REFLEJOS INCLUYENDO EL DE SUCCIÓN, HAY LANUGO Y PLIEGUES PROFUNDOS EN LAS PALMAS Y PLANTAS. NO  PRESENTA ALTERACIONES EN CABEZA, CUELLO O ABDOMEN. EL CORDÓN UMBILICAL SIN DATOS DE SANGRADO. EL PRIMER REPORTE  QUE SE LE DIO A LA MADRE ES QUE SU NIÑA ESTÁ BIEN.      EN SU EF USTED PERCIBE UN CLUNCK EN LA CADERA MIENTRA REALIZA UN MOVIMIENTO DE ADUCCIÓN Y PRESIÓN SOBRE LA  SUPERFICIE INTERNA DEL MUSLO. ¿CUÁL ES EL NOMBRE DE LA MANIOBRA?  A) BARLOW    B) ORTOLANI  C) GALEAZZI  D) TRENDELENBURG    Aducción luxa.    Abducción reduce.      EN SU EXPLORACIÓN USTED PERCIBE UN CLUNCK EN LA CADERA IZQUIERDA MIENTRAS REALIZA UN MOVIMIENTO DE ABDUCCIÓN Y  PRESIÓN SOBRE LA SUPERFICIE DEL TROCÁNCER MAYOR. ¿CUÁL ES EL NOMBRE DE ESTA MANIOBRA?  A) BARLOW    B) ORTOLANI  C) GALEAZZI  D) TRENDELENBURG    LA PRUEBA DE TRENDELENBURG POSITIVA TRADUCE:  A) INSUFICIENCIA GLÚTEO MAYOR  B) INSUFICIENCIA GLÚTEO MEDIO  C) INSUFICIENCIA GLÚTEO MENOR  D) INSUFICIENCIA DEL TENSOR DE LA FASCIA LATA    SON SIGNOS POSITIVOS EN PACIENTES CON LUXACIÓN DE CADERA DIAGNOSTICADA A LOS 5 MESES DE EDAD:  A) BARLOW, ASIMETRÍA DE PLIEGUES, ORTOLANI, TRENDELENBURG. 

B) BARLOW, ASIMETRÍA DE PLIEGUES, ORTOLANI, PISTÓN  C) BARLOW, ASIMETRÍA DE PLIEGUES, ORTOLANI, GALEAZZI.    D) BARLOW,    ASIMETRÍA DE PLIEGIUES, ORTOLANI, LIMITACIÓN DE LA ABDUCCIÓN.      ESTE SIGNO SE CARACTERIZA POR UNA ALTURA DISTINTA A LA QUE SE ENCUENTRAN LAS RODILLAS CUANDO SE JUNTAN LOS PIES DEL  NIÑO EN DECÚBITO SUPINO, ESTANDO FLEXIONADAS LAS CADERAS Y RODILLAS.    A) BALOW    B) ORTOLANI  C) GALEAZZI  D) TRENDELENBURG    EL SIGUIENTE NO ES UN FACTOR DE RIESGO PARA DDC:  A) VARÓN  B) OLIGOHIDRAMNIOS  C) ECLAMSPIA  D) PARTO GEMELAR    EL ESTUDIO DE ELECCIÓN PARA VALORAR LA CADERA NEONATAL ES:  A) RADIOGRAFÍA DE PELVIS EN POSICIÓN NEUTRA  B) USG  C) RADIOGRAFÍA AP DE CADERA  D) RADIOGRAFÍA LATERAL DE CADERA    EL ESTUDIO DE ELECCIÓN PARA VALORAR LA CADERA A PARTIR DE LOS 3 MESES ES:  A) RADIOGRAFÍA DE PELVIS EN POSICIÓN NEUTRA  B) US  C) RADIOGRAFÍA EN AP DE CADERA  D) RADIOGAFÍA LATERAL DE CADERA    RESPUESTAS A, B, B, D, C, C?, B, A       

PATOLOGÍA DEL OÍDO    Respecto al tatamiento amoxi es de primera elección cefalosporina de 2da generación es segunda eleccion, y cloranfenicol es de  tercera. En otits media S.pneumonae tiene 50% de resistencia. TMP/SMX no ha    mostrado ser mejor que amoxicilina,    OTITIS EXTERNA  ETIOLOGÍA  Etiología bacteriana en el 90% de los casos, 10% por hongos.    Patógens más frecuentes: Pseudomonas aeruginosa, S. aureus, estreptococos, bacilos gram negativos. Hongos: Aspergilluis, Candida  albicans y Candida krusei.      FR: pérdida de la integridad de la piel, alt de la barrera protectora de cerumen, nadadores, conducto estrecho.      DIAGNÓTICO  Prurito en CAE, rascado, otalgia moderada‐intensa, edema, oclusión del lumen, plenitud ótica, hipoacusia conductiva superficial. Si los  restos dérmicos==> secreción purulenta. Si persiste pude afectarse la región preauricular, auricular y cervical.      TRATAMIENTO  a) Limpieza médica  b) AB con o sin esteroides aplicado cada 3‐4 gts cada 6 hrs en oido afectado.    De elección son las fluoroquinolonas (ciproflox, ofloxa). También buenos resultados con neumocina ótica + dexametasna + ác. acético.    Si más infección más allá del CAE cambiar a AB oral (fluoroquinolonas, cefalosporinas)      HOMBRE DE 27 AÑOS DE EDAD QUE ACUDE A CONSULTA POR PRESENTAR OTALGIA DERECHA SEVERA. EL PACIENTE REFIERE QUE EL  DOLOR SE IRRADIA HACIA LA REGIÓN TPM Y TAMBIÉN TIEN PRURITO. NIEGA ANTECEDENTES DE IMPORTNAICA. REFIERE REALIZAR  NATACIÓN 3 VECES POR SEMANA. FC 67, FR 13, T37.5 TA 114/76. TIENE ERITEMA Y EDEMA DIFUSO EN CONDUCTO AUTITIVO  DERECHO.    ¿CUÁL ES EL TRATAMIENTO MÁS ADEUCADO PARA ESTE PACIENTE?  A) AMOXICILINA/CLAGVULANATO ORAL + GOTAS ÓTICAS + AC. AC´TICO  B) NEOMICINA ÓTICA + DEXAMETASONA + AC ACÉTICO  C) CIPROFLOXACINO ORAL + AINES  D) GOTAS ÓTICAS CON DEXAMETASONA Y ÁC. AC´TICO 

  RESPUESTA B  Ciertos factores se asocian con la presencia de otitis externa, que es el diangóstico de este caso: la alteración del pH del conducto  (secundario a exposición a agua como por ejemplo antación), trauma local (cotonetes), limpieza agresiva entre otros. La manifestación  clínica de esta patología es la hiperestesia locorregional a la manipulación de pabellón auricular (o también conocido como signo del  trago). La exploración física por medio de otoscopia demuesta un entorno inflamatorio (ritema, edema). De acuero a las guias del  CENETEC, el tratamiento es a base de gotas óticas que combinan esteroide o antibiótico + ac acético. El uso de ác acético sólo es menos  efectivo que la combinación previamente mencionada. recordar que los agentes etiológicos más frecuentes son: peusomonas  aeruginosa, S. aureus y el tratmiento tópico es de elección.   

OSTEOMIELITIS DE LA BASE DE CRÁNEO  DEFINICIÓN  También llamada ostitis externa maligna o necrosante. Es una infección bacterian de la base del cráneo, más frec en diabético e  inmunodeficientes.    Puede probocar septicemia y muerte.      PATOGENIA  Progresa a celulitis==> condritis==> osteítis==> osteomielitis    Conforme progresa a lo largo de la base del cráneo afecta pares craneales: V, VI, VII, IX, X, XI, XII.    Microorganismo más frecuente P. aeruginosa y en segunda Aspergillus.      CLÍNICA  Otalgia intensa, otorrea, tejido de granulación en piso de CAE, edema, linfadenopatía preauricular, Trismus. Nervio facial es el más  afectado. Progresión a trombosis del seno venso==> meningitis==> septicemia==> muerte.      DIAGNÓSTICO  Clínicos, rastreo con TC‐99.    Para seguimiento Galio 67, indio 111.    TRATAMIENTO  Tx de elección: AB parenterales a largo plazo. Aminoglucósidos, betalactámicos antipseudomonas, ceftazidima.    PACIENTE MASCULINO DE 54 AÑOS DE EDAD, DIABÉTICO MAL CONTROLADO, INCIA SU PADECIMIENTO ACTUAL HACE  APROXIMADAMENTE 4 SEMANAS CON OTALGIA DERECHA INTENSA Y OTORREA, HACE 2 DÍAS PRESENTA PARÁLISIS FACIAL IPSILATEAL.  A LA EF ENCUENTRAS DOLOR A LA MOVILIZACIÓN DEL PABELLÓN AURICULAR, ASÍ COMO SIGNO DE TRAGO POSITIVO. LA OSTOSCOPIA  REVELA EDEMA, HIPEREMIA DEL CONDUCTO AUDITIVO EXTERNO PRÁCTICAMENTE IMPIDE LA VISUALIZACIÓN DE LA MEMBRANA  TIMPÁNICA, ADEMÁS OBSERVAS TEJIDO DE GRANULACIÓN EN EL PISO DEL CONDUCTO.      EL DIAGNÓSTICO MÁS PROBABLE EN ESTE PACIENTE SERÁ:  A) OTITIS EXTERNA MICÓTICA  B) OTITIS EXTERNA AGUDA  C) OTITIS EXTERNA CRÓNICA  D) OTITIS EXTERNA MALIGNA U OSTEOMIELITIS DE BASE DE CRÁNEO.      EL TRATAMIENTE DE ELECCIÓN SERÁ:  A) ANTIBIÓTICOS TÓPICOS  B) TRATAMIENTO PARA PSORIASIS  C) ANTIBIÓTICOS ANTIPSEUDOMONAS IV Y CONTROL METABÓLICO  D) ANTIMICÓTICOS TÓPICOS.    LA CAUSA MÁS PROBABLE DEL PADECIMIENTO ACTUAL SERÁ:  A) ALTERACIÓN DE LAS BARRERAS DE DEFNSA DE LA PIEL DEL CONDUCTO AUDITIVO EXTERNO  B) DESCONTROL METABÓLICO  C) USO CRÓNICO DE ANTIBIÓTICOS TÓPICOS Y DESCONTROL METABÓLICO  D) PSORIASIS, ECCESMA U OTRA PATOLOGÍA DE BASE.    El descontrola metabólico provoca microangipatía en el oído, aumentando con ello este tipo de infecciones.      RESPUESTA D, C, B     

OTITIS MEDIA AGUDA  DEFINICIÓN  Infección del oido medio  amoxi clav 80mg/kg/d div c 8 x5‐10d    Si > 2 años sin factores de mal pronóstico, analgésico y revalorar en 48 hrs.  Afección intensa (>39° o  timpanocentesis  Posible OMA    Si leve o moderado expectante    Si intenso==> valorar amoxi    Fracaso de tx amoxi==> amoxiclav==> ceftriaxona IM 50mgkgd x3==> trimpanocentesis, cultivo, gram, antibiograma.  Si alergia no anafiláctica a peni: cefpodoxima proxetilo o cefuroxima axetilo x    5‐10d  Si alergia anafilactica (tipo 1): azitro o claritromicina.    Si alergia a peni, macrólidos  Miringotomía si otalgia severa en pacietes tóxicos.    PRONÓSTICO  Mastoiditis aguda simple puede ser asintomática, dx solo x imagen.  Mastoiditis aguda con osteitis o periostitis: edema retroauricular (dx x imagen, especialmente TAC) si osteitis quirúrgica  Parálisis facial: 2da compliación más frec. Buen px, evoluciona con el Tx de la OMA.  Laberintitis: excepcional.  meningitis: excepcional, vía hematógena    o por proximidad (rara).  Absceso cerebral: por extensión local.               [OTITIS MEDIA]  MUJER DE 17 AÑOS QUE INICIA SU PADECIMIENTO HACE 7 DÍAS CON LA PRESENCIA DE CEFALEA Y OTALGIA IZQUIERDA. EL DÍA DE AYER  SE AGREGA FIEBRE DE 38.5°C, HIPOACUSIA. A LA EXPLORACIÓN SE ENCUENTRA MEMBRANA TIMPÁNICA IZQUIERDA INTEGRA,  HIPERÉMICA, ABOMBADA E INMÓVIL.  EL DIAGNÓSTICO CLÍNICO MÁS PROBABLE ES:      A) OTITIS MEDIA AGUDA.  B) OTITIS MEDIA CRÓNICA.  C) OTITIS SEROSA. 

D) OTITIS NECROTIZANTE.    EL FACTOR DESENCADENANTE MÁS COMÚN EN ESTE TIPO DE PACIENTES ES:  A) NADAR SIN PROTECCIÓN.  B) LA ACUMULACIÓN DE CERUMEN.  C) LAS INFECCIONES DE VÍAS RESPIRATORIAS SUPERIORES.  D) LA PERFORACIÓN DE LA MEMBRANA TIMPÁNICA.      ADEMÁS DE ADMINISTRAR ANALGÉSICOS Y ANTIPIRÉTICOS SE DEBERÁ AGREGAR:  A) TRIMETOPRIMA MÁS SULFAMETOXAZOL.  B) CIPROFLOXACINA.  C) AMOXICILINA.  D) PENICILINA PROCAÍNICA.    RESPUESTA A,C,C  Otitis media aguda: principal agente etiológico 1) S. pneumoniae, 2)H. influenzae no tipable. Es una enfermedad íntimente relacionada  con resfraido común del que es complicación habitual. La infección vírica de vías respiratorias altas provoca una toxicidad sobre el  epitelio respiratorios, causante d discinesia ciliar en la trompa de Eustaquio, con alteración subsecuente del aclaramiento de moco.  La otitis serosa es muy frecuente en la infancia es una condición muy frecuente, se caracteriza por la pesencia de líquido detrás del  tímpano. Generalmente se asocia a resfraido o a otitis aguda previa, aunque no significa que el oido esté infectado. Mejora  espontáneamente en unos meses.  La osteomilitis de la base del cráneo también conocida como otitis externa maligna o necrotizante. Comineza como infección del  conducto auditivo externo, lugo del temporal, meningitis, septicemia,    muerte.    Se observa en inmunocomprometidos (diabéticos de  edad avanzada). Etiología: pseudomonas aeruginosa. Clínica: a parte de los síntomas óticos puede haner hipoacusia, trismus, parálisis  facial o datos de septicemia. (en sus inicios hay síntomas de otitis externa aguda). 

  OTITIS MEDIA CRÓNICA    DEFINICIÓN  Exudado persistente o intermitente de una membrana timpánica perforada o con tubo de ventilación por más de 6 semanas.    Se divide en:    OMC colesteatomatosa      OMC no colesteatomatosa.      La OMC colesteatomatosa: son quistes de inclusión epidérmica en el oido medio o mastoides que contienen productos de descamación  del epitelio. Un colesteatoma puede ser congénito o adquirido. Dx de colesteatoma por otoscopia, endoscopia o microscopia.    ETIOLOGÍA  Las bacterias más frecuentes en la OMC son P. aeruginosa, S. aureus, Klebsiella y bacterias anaerobias com Bacteroides sp.    TRAMIENTO    OMC Colesteatoma. El manejo de colesteatoma congénito o adquirido debe ser mediante resección quirúrgica (mastoidectomía).  OMC no colesteatomatosa: antibióticos tópicos. Si más de 3 meses ligres de enfermedad sin colesteatma==> timpanoplastia curativa.      OMC no colestomatomatosa con otorra recurrente; timpanomastoidectmía.        PACIENTE MASCULINO DE 40 AÑOS DE EDAD QUIEN ACUDE A CONSULTA POR PRESENTAR EPISODIOS DE OTORREA IZQUIERDA  INTERMITENTE DESDE LA INFANCIA, ADMEÁS REFIERE PÉRDIDA DE LA AUDICIÓN Y ACÚFENO IPSILATERAL.    A LA EF OBSERVAMOS CAE PERMEABLE, MEMBRANA TIMPÁNICA IZQUIERDA CON RETRACCIÓN DE LA PARS FLÁCIDA Y PRESENCIA DE  MASA BLANQUECINA DE ASPECTO GRUMOSO EN PARED POSTERIOR QUE EROSIONA CONDUCTO Y AL PARECER TAMBIÉN LOS  HUESECILLOS.    EL DIAGNÓSTICO MÁS PROBABLE EN ESTE PACIENTE SERÁ:  A) OTITIS MEDIA CRÓNICA NO COLESTEATOMATOSA  B) OTITIS MEDIA CRÓNICA COLESTEATOMATOSA  C) CARCINOMA EPIDERMIOID DE CONDUCTO AUDITIVO EXTERNO  D) GRANULOMA DE COLESTEROL    DENTRO DE TU PROTOCOLO DIANGÓSTICO DECIDES SOLICITAR UN ESTUDIO DE GABINTE. EL MÁS INDICADA PARA ESTE PADECIMIENTO  SERÍA:  A) RESONANCIA MAGNÉTICA  B) TOMOGRAFÍA SIMPLE DE OIDO  C) TOMOGRAFÍA CONTRASTADA DE OIDO  D) GRAMMAGRAFÍA CON TC 99   

En caso de infecciones crónicas de oido el estudio de elección sería la tomografía simple, cuando sospechamos de una etilogía tumoral  entonces está indicado el contraste.      EL TRATAMIENTO INDICADO EN ESTE PACIENTE SERÁ:  A) RESECCIÓN EN BLOQUE DE HUESO TEMPORAL  B) MASTOIDECTOMÍA  C) ANTIBIÓTICOS TÓPICOS Y VIGILANCIA  D) COLOCAGIÓN DE TUBO DE VENTILACIÓN TIMPÁNICA      El tratamiento de otitis media crónica colesteatomatosa es la resección del colesteatoma, el procedimiento para realizarlo es la  mastoidectomía en cualquiera de sus variedades.    RESPUESTAS B, B, B 

  HEMATOMA AURICULAR  Acumulación de sangre en espacio subpericóndrico, por lo general por traumatismo contuso.    El cartílago auricular carece de riego sanguíneo, se alimenta del pericontrio. Cuando un traumatismo produce un hematoma dificulta la  nutrición del cartílago por obstruir la difusión desde el pericondrio.    Necrosis==> infecciones.      PACINETE FEMENINO DE 6 AÑOS DE EDAD, QUE INICIA SU PADECIMIENTO ACTUAL POSTERIOR A CAÍDA DESDE SU BASE DE  SUSTENTACIÓN, PRESENTANDO UN TRAUMATISMO EN HEMICARA DERECHA, PRESENTA AUMENTO DE VOLUMEN EN PABELLÓN  AURICULAR DERECHO, LA PIEL SE OBSERVA EDEMATOSA Y CON COLOR ROJO, AL TACTO PRESENTA FLUCTUACIÓN. LA MADRE NIEGA  FIEBRE, OTORREA U OTORRAGIA.      EL DIAGNÓSTICO MÁS PROBABLE DE LA PACIENTE SERPA:  A) HEMATOMA  B) PERICONDRITIS  C) PERICONDRITIS SUPUADA  D) ERISIPELA    LA ETIOLOGÍA MÁS PROBABLE EN ESTE CASO CLÍNICO SERÁ_  A) INFECCIÓN POR ESTREPTOCOCO BETA HEMOLÍTICO DEL GRUPO A  B) INFECCIÓN POR PSEUDOMONAS AEURIGINOSA  C) TRAUMATIMOS CON ACÚMULO SANGUINOLENTO EN ESPACIO SUBMUCOPERICNDRICO  D) INFECCIÓN POS PSEUDOMONAS MAL TRATADA    LA TERAPÉUTICA INDICADA PARA ESTA PACIENTE ES:  A) OBSERVACIÓN  B) DRENAJE DEL HEMATOMA Y FERULIZACIÓN  C) QUINOLONAS VÍA ORAL  D) BETALACTÁMICOS    RESPUESTA A, C,B         

HIPOACUSIA  DEFINICION  El ser humano puede oir desde la 27 sdg,  ETIOLOGÍA  Hipoacusia congénita: 50% adquirida, 50% genética.    Infeciosas: TORCH, parotiditis, meningitis.  Hipoxia neonatal: insuf placentaria y compresión del cordón producen lesión de cóclea y núcleos del tronco.  Parto prematuro: produce hemorragia y atrofia de la estría vascular  Hiperbilirrubinemia: se deposita bilirrubina en cóclea y ganglios basales.    Hereditaria: lo más frecuente (70%)es que se presente de forma aislada.    Autoinmune:    anticuerpos y linfocitos vs células ciliadas y estría vascular. Puede formar parte de granulomatosis de Wegener, LES, AR,  hipotiroidismo.    Trauma acústico  Tumores (Schwanoma vestibular)  Fármacos (aminoglucósidos, diurétcos de asa, quimioterápicos.      SALUD PÚBLICA 

Discapacidad auditiva es la pérdida de audición mayor a 25 dB.    25% es genético, 25% no genético, 50% indeterminado.    En México la hipoacusia es la anormalidad congénita más común en el RN. 3‐1000 NV.  Hipoacusia congénita: FR: UCI >5 días, ventilación asistida, ototóxicos, diruéticos de ASA, hiperbilirrubinemia, infección intauterina  (CMV, herpes, rubeola, sífilis, toxo) Malf cráneofaciales, neurofibromatosis, osteopetrosis, Usher, Waardenburg, Alport, Pendrer y  Jervell y Lange‐Nielson    PATOGENIA  Investigar prematurez, consanguinidad, hipoxia, suf fetal, hiperbilirrubinemia, historia familiar de niños con pérdida auditiva  neurosensorial, infevviones in úter (rubeola, CMV, sífilis, toxoplasmosis, herpes). APGAR al minuto de 1‐4, a los ,min de 0‐6, ventilación  mecánia x más de 5 días, meningitis bacteriana, anomalías cránefaciales, Sx TORCH, ototóxicos.      DIAGNÓSTICO  Tamiz auditivo.  Emisiones otoacústicas: identifica la pérdida más no la severidad. Si sale alterado se repite en dos meses (el paciente podría tener  líquido en oido medio. Si nuvamente sale alterado realizar PAETC  Potenciales evocados auditivos de tallo cerebal: registra actividad neural en la cóclea. Si se corrobora hipoacusia neurosensorial se  indican auxiliares bilaterales. A los 6 meses de edad deberá estar adaptado con auxiliares auditivos si lo requiere.    Si al año no logra  oir, en candidato a implante coclear.    ¿Al año del auxiliar o al año de edad?   

    TRATAMIENTO  Se recomienda implante coclear multicanal bilateral en aquellos que no se beneficien de auxiliar auditivo convencional debido a su  severidad.  Candidatos a implante coclear:    Pérdida auditiva >90 dB medidos por PPATC en frec de 2000‐4000.    Umbrales superiores 65 dB SPL (sound pressure level) en campo libre con uso de audífonos    Reconocimiento del lenguaje >40% en condiciones de amplificación.    Función y anatomía de oído medio normal    Ausencia de cirugía previas en oído medio    No alt etrococleares en la vía auditiva    Insatisfacción comprobada a audífonos al menos durante 6m.    +    Prelingüísticos entre 1‐3 años    Antecedente de educación oralista y uso de auxiliar auditivo x 3meses.    >3 años y hasta preadolescencia solo si ya se sometió a educación oralista y adquisiciones lingüísticas.  No se recomienda implantar si:    >6 años prelingüísticos    Descartar con estudios de imagen: anomalias del VII par, dilatación del acueducto coclear, monitoreo del VII par transoperatorio.    CONTENIDO ADICIONAL CLAVE  EO si no pasa ==> 2da EO si no pasa===> 2 y 3er nivel==>ORL/audiólogo==> PPTAC    PACIENTE MASCULINO DE 1 AÑO 6 MESES DE EDAD, LOS PADRES ACUDEN A CITA PORQUE REFIEREN QUE EL NIÑO SOLO DICE  ALGUNAS PALABAS, PAPA, MAMA, AGUA, LA    MADRE ADEMÁS, MENCINA QUE PARECE QUE EL NIÑO TODO EL TIEMPO ESTÁ  IRRITABLE Y NO LE HACE CASO. CUANDO INTERROGAS A LOS PAPÁS ELLOS TE COMENTAN QUE EL PACINTE NACIÓ A LOS 7 MESES, CON  SUFRIMIENTO FETAL Y ESTUVO EN LA TERAPIA INTENSIVA DURANTE 15 DÍAS. SOSPECHAS QUE EL PACIENTE TIENE HIPOACUSIA Y POR  ELLO LA FALTA DE DESARROLLO DEL LENGUAJE.      PRIMER ESTUDIO QUE SOLICITAMOS EN ESTE PACIENTE SERÍA:  A) ELECTROCOCLEOGRAFÍA  B) POTENCIALES AUDITIVO EVOCADOS DE TALLO CEREBRAL  C) AUDIOMETRÍA TONAL  D) TAMIZ NEONATAL EXTENDIDO    EL ESTUDIO TE REVELA UNA HIPOACUSIA SENSORIAL SEVERA BILATERAL. LO MÁS RECOMENDADO PARA ESTE PACIENTE SERÁ:  A) OBSERVAR Y ESPERAR MEDURACIÓN DEL SISTEMA AUDITIVO 

B) ADAPTACIÓN DE AUXILIARES AUDITIVOS  C) ADAPTACIÓN DE AUXILIARES AUDITIVOS Y TERAPIA DE LENGUAJE  D) COLCACIÓN DE IMPLANTE COCLEAR    Por la edad requiere adaptación inmediata de auxiliar auditivo además de terapia de lenguaje. Si no logra oir==> implante coclear    LA ETIOLOGÍA MÁS PROBABLE DE LA HIPOACUSIA EN ESTE PACIENTE SERÁ:  A) CONGÉNITA SINDRÓMICA  B) ENFERMEDAD DE MNDINI  C) MULTIFACTORIAL  D) OTOTOXICIDAD    RESPUESTA B, C, C      [HIPOACUSIA][r]    MUJER DE 48 AÑOS DE EDAD, QUE ACUDE A LA CONSULTA EXTERNA AL PERCIBIR DISMINUCIÓN DE LA CAPACIDAD AUDITIVA.  PARA SOLICITAR E INTERPRETAR UNA AUDIOMETRÍA, ES NECESARIO SABER QUE LA ESTRUCTURA QUE INTERVIENE EN EL AJUSTE DE LA  IMPEDANCIA DEL SONIDO ES:  A) LA MEMBRANA TENTORIA.  B) LOS HUESECILLOS.  C) EL MÚSCULO ESTAPEDIO.  D) EL ÓRGANO DE CORTI.    RESPUESTA B  Las huesecillos tienen la función de ajustar la impedancia aire‐liquido para transmisión del impulso auditivo.    El músculo estapedio se inserta en el cuello del estribo y jala de él, no contribuye por si solo a la impedancia.    Una vez que el impulso sonoro llega a la ventana oval en el oido interno se produce movimiento de la perilinfa  La lámina tectoria esta por encima del órgano de Corti, las células pilosas externas está embebida en el lámina tectoria, constituye la  formación del estímulo acústic cuando se produce deflexión en ella  El órgano de corti está en la cóclea y transforma vibraciones en impulsos nerviosos.  .     

     

 

   

 

 

PRESBIACUSIA  DEFINICIÓN  Pérdida auditiva por cambios degenerativos de la edad. Tiene un compente periférico y central.    Es un diagnóstico de exclusión.    CLÍNICA  Edad 60‐70 años,  Pérdida de audición y regresión fonémica.  Bilateral, lentamente progresiva.    Alteraciones del equilibrio (presbiestasia)    DIAGNÓSTICO  Audiometría tonal.  Logo audiometría: es la prueba más importante donde se obsrva una grave deterioro del umbral de inteligibilidad    y del umbal de  máxima discriminaci´nde la palabra.      TRATAMIENTO  No existe médico ni quirúrgico efctivo.    Protésico en caso necesario.      CONTENIDO ADICIONAL CLAVE            Weber      Rinne    Schawabach  Hipoacusia neurosensorial  Lateraliza al normal  Positiva    Acortado  Hipoacusia de conducción  Lateraliza al enfermo  Negativa  Normal‐Prolongada    PACIENTE MASCULINO DE 65 AÑOS DE EDAD, QUIEN INICIA SU PADECIMIENTO ACTUAL DESDE HACE 5 AÑOS CON HIPOACUSIA  BILATERAL, MENCIONA QUE LE CUESTA TRABAJO ESCUCHAR EN AMBIENTES RUIDOSOS, TAMBIÉN PRESENTA ZUMBIDO DE OÍDOS.  REFIERE LEVE EXPOSICIÓN A RUIDO LABORAL Y QUE ES INGENIERO, NO PRESENTA ANTECEDENTES FAMILIARES DE PÉRDIDA DE  AUDICIÓN, NI INFECCIONES OTOLÓGIAS DE REPETICIÓN.      DIAGNÓSTICO MÁS PROBABLES:  A) OTOSCLEROSIS  B) TRAUMA ACÚSTICO  C) PRESBIACUSIA  D) ACÚFENO IDIOPÁTICO    ¿CÓMO ESPERARÍAS ENCONTRAR LA ACUAMETRÍA EN ESTE PACIENTE?  A) WEBER CENTRAL, RINNE NEGATIVO BILATERAL  B) WEBER CENTRAL, RINNE POSITIVO BILATERAL  C) WEBER CENTRAL, RINNE SIN CAPTAR BILATERAL  D) WEBER SIN CAPTAR, RINNE SIN CAPTAR.    Es simétrica y neurosensorial.      EL ESTUDIO DE ELECCIÓN PARA CONFIRMAR EL DIAGNÓSTICO SERÁ:  A) AUDIOMETÍA TONAL  B) AUDIOMETRÍA TONAL CON LOGOAUDIOMETRÍA  C) AUDIOMETRÍA DE BEKESY  D) ÚNICAMENTE LOGOAUDIOMETRÍA     RESPUESTA C, B, B   

OTOSCLEROSIS  DEFINICIÓN  Enfermedad metabólica ósea primaria de la cápsula ótica y la cadena osicular que causa fijación de los huesecillos con la resultante  hipoacusia. Enf de transmisión genética AD con penetrancia incompleta y expresión variable.    EPIDEMIOLOGÍA  M2:H1 esta relación se explica porque el embarazo acelera el proceso de otosesclerosis. 2/3 de los pacientes tienen historia familiar de  hipoacusia.    Presentación clínica común: “Mujer en la segunda o tercera década de la vida, con hipoacusia conductiva lentamente progresiva,  bilateral (80%) y asimétrica, así como con acufeno (75%)    CLÍNICA  Hipoacusia progresiva, algunos refieren mejoría de la audición en ambiente ruidoso, fenómeno conocido como Paracusia de Willis. 

  PARACLÍNICA  Timpanometría: pico bajo en el oído medio con rango de presión de aire normal se denomina patrón tipo AS, es característico de  otoesclerosis.  Reflejos acústicos: reflejo estapedial disminuye progresivamente.  Audiometrías con tonos puros: nicho de Carhart que se caracteriza por disminución del umbral de conducción ósea de  aproximadamente 5Db A 500Hz, 10Db A 1000HZ, 15Db A 2000 Hz y 5db a 4000Hz?    TRATAMIENTO  No quirúrgico: auxiliar auditivo, la ventaja es que se evita el riesgo de hipoacusia sensorioneural postquirúrgica. Los pacientes que se  somenten a tratamiento quirúrgico tienen significativamente mayor satisfacción de audición.  Quirúrgico: candidato aquel con buen edo de salud, audiometría socialmente aceptable, prueba de rinne negativa, buen  discriminación, deseo de operarse. Estapedectomía.        PACIENTE FEMEINO DE 35 AÑOS DE EDAD, QUIEN INCIA POSTERIOR A EMBARAZO CON HIPOACUSIA PROGRESIVA DERECHA,  ACOMPAÑADA DE ACUFENO, REFIERE QUE OCASIONALMENTE PRESENTA INESTABILIDAD Y MAREO. MENCIONA QUE CUANDO ESTÁ EN  LUGARES RUIDOS EN OCASIONES ESCUCHA MEJOR QUE EN LUGARES SIN RUIDO. UNA DE SUS HERMANAS TAMBIÉN PADECE  HIPOCUSIA. LA PACIENTE REFIERE QUE EN ALGUNA OCASIÓN EN LA INFANCIA PRESENTÓ OTITIS.      CON ESTE CUADRO CLÍNICO EL DIAGNÓSTICO MÁS PROBABLE SERÁ:  A) OTITIS MEDIA AGUDA  B) ENFERMEDAD DE MENIERE  C) OTOSCLEROSIS  D) HIPOACUSIA CONGÉNITA DE PRESENTACIÓN TARDÍA.    ¿QUÉ ESPERARÍAS ENCONTRAR A LA EXPLORACIÓN FÍSICA?  A) OTOSCOPIA NORMAL, WEBER DERECHO, RINE NEGATIVO DERECHO Y POSITIVO IZQUIERDO.  B) OTOSCOPIA CON PERFORACIÓN DE MEMBRANA TIMPÁNICA, WEBER DERECHO RINNE NEGATIVO DERECHO Y POSITIVO IZQUIERDO  C) OTOSCOPIA NORMAL, WEBER IZQUIERDO, RINE POSITIVO BILATERAL  D) OTOSCOPIA NORMAL, WEBER IZQUIERDO, RINE NO CAPTANTE    LOS ESTUDIOS AUDIOLÓGICOS QUE CONFIRMAN TU SOSPECHA DIAGNÓSTICA SON:  A) EMISIONES OTOACÚSTICAS  B) AUDIOMETRÍA TOTAL CON LOGOAUDIOMETRÍA  C) AUDIOMETÍA TONAL, TIMPANOMETRÍA Y REFLEJOS ESTAPEDIALES  D) AUDIOMETRÍA AUTOMÁTICA    RESPUESTA C, A, C       

PATOLOGÍA DE LA FARINGE Y LARINGE  FARINGOAMIGDALITIS AGUDA   

VÉRTIGO GENERALIDADES  Vértigo periférico: fatigable, latencia, se agota, horizontal o rotatorio  Vértigo central: Vertical, cambiante, asocia síntomas neurológicos    Segundos: VPPB, persistencia de canal semicircular superior y fístula perilinfática.  Síntomas otológicos: plenitud, acufeno e hipoacusia.  VPPB no presenta síntomas otológicos, persistencia de canal semicircular y fístula perilinfática sí.    Persistencia de canal semicircular presenta hipoacusia conductiva y la fístula perilinfática neurosensorial.    Minutos a horas: Enf de Meniere    Horas a días: neuritis vestibular (presenta solo vértigo), laberintitis (presente vértigo + hipoacusia, infección del oído medio oído  interno)    VÉRTIGO POSTURAL PAROXÍSTICO BENIGNO  Vértigo objetivo: se mueven los objetos  Vértigo subjetivo: se mueve el sujeto  ETILOGÍA  Puede deberse a lesión cefálica, laberintitis viral u oclusión vacular. Más comunmente idiopático.   

  DIAGNÓSTICO  Maniobra de Dix‐Hall‐Pike. Una posición provoca el vértigo más fuertemente, lo que indica que la lesión está en el lado al que está  girada la cabeza. El nistagmo rotatorio puede observarse durante la maniobra después de un periodo de latencia de 2‐10 seg. Por lo  gral el nistagmo cambia de dirección cuando el paciente se sienta.    La enfermedad es de resolución espontánea y el paciente se  recupera por completo en plazo de tres meses.      TRATAMIENTO  Maniobras de reposicionamiento (Epley) son el tx de elección.     

NEURONITIS VESTICULAR  EPIDEMIOLOGÍA  50% tiene antecedente de IVRS semanas previas.  Puede estar presente en epidemias.      CLÍNICA    Se presenta con vértigo grave y súbito (puede durar varios días). Náuseas y vómito. Puede persistir incluso pOr meses y de modo  especial en ancianos, inestabilidad sin vértigo.    TRATAMIENTO    EsteriOdes y frenadores laberíntis en fase aguda, en fase de recuperación ejercicios de rehabilitación vestibular.   

SX DE MENIERE  PATOGENIA    CLÍNICA  Planitud aural, hipoacusia neurosensorial fluctuante, plenitud aural, tinitus y vértigo. Alcanza máxima intensidad a los pcos minutos,  cede lento en varias hrs.  En la fase de extinción origina hipoacusia permanente grave pero cese del vértigo.      TRATAMIENTO  Dieta hiposódica, vasodilatadores, diuréticos.    PACIENTE FEMENINO DE 30 AÑOS DE EDAD QUIEN INICIA SU PADECIMIENTO ACTUAL EL DÍA DE HOY DE MANERA SÚBITA PRESNTA  SENSACIÓN DE GIROS, TE MENCIONA QUE DESDE LA MÑANA DESPERTÓ Y N SE HA TENIDO LA SENSACIÓN DE VÉRTIGO, PRESENTA  NÁUSEA Y VÓMITO, NIEGA OTORREA, OTALGIA, ACÚFENO, HIPOACUSIA. A LA EF REALIZAS DIAPASONES OBTENIEDO WEBER CENTRAL,  RINNE POSITIVO BILATERAL, LA PACINET PRESENTA NISTAGMUS ESPONTÁNEO Y EVOCADO CON MIRATA HORIZONTAL CON FASE  RÁPIDA A LA DERECHA, NO PUEDES REALIZAR MANIOBRA DE DIX HALLPIKE POR LA CONDICIÓN DE LA PACINETE. ROMBERG POSITIVO,  SIN ASIMETRÍAS NI DISDIADOCOCINESIS. FUDUKA A LA IZQUIERDA, HALMAGYI POSITIVO A LA IZQUIERDA. TE RIFERE QUE TUVO UNA  GRIPE MALTRATADA HACE 2 SEMANAS.      EL DIAGNÓSTICO MÁS PROBABLE EN ESTA PACIENTE SERÁ DE:  A) CEREBELITIS  B) NEURONITIS VESTIBULAR  C) ENFERMEDAD DE MINERE  D)TUMOR DE ÁNGUL PONTOCEREBELOSO    El equilibrio está dado por visión, audición y propiocepción. Por eso en este paciente al hacer la prueba de Romberg da positivo (se  suprime la visión al cerrar los ojos, la audición por tener vértigo y la propiocepción que está indemne no es suficiente para dar un  Romberg negativo. En un paciente con cerebelitis se anula la visión, la propiosepcion (por daño cerebeloso) y la audición queda  indemne pero no es suficiente para hacer un Romber negativo)    Fukuda y Halmagyi traducen paresia vestibular.    EL TRATAMIENTO DE ELECCIÓN PARA ESTE PADECIMIENTO ES:  A) DIURÉTICOS  B) ESTEROIDES Y FRENADORES LABERÍNTICOS  C) CIRUGÍA  D) MANIOBRAS DE REPOSICIONAMINETO OTOLÍTICO    No usar frenadores laberínticos fuera de la agudización porque retrasamos la compensación.    LA ETIOLOGÍA MÁS COMÚN EN ESTE PADECIMIENTO ES:  A) VIRAL  B) HIDROPS ENDOLINFÁTICO  C) COMPRESIÓN DEL NERVIO VESTIBULAR 

D) VASCULAR    Principal etiología es viral por Herpes virus  RESPUESTA    B, B, A       

PATOLOGÍA DE LA FARINGE Y LARINGE  FARINGOAMIGDALITIS AGUDA  DEFINICIÓN  Infección de faringoamigdalar caracterizada por garganta roja de más de 5 días.    SALUD PÚBLICA  FR: tabaquismo, RGE, SAOS, inmunodepresión, clima artificial, uso de la voz.  Causa frecuente EBHGA    DIAGNÓSTICO  Criterios  Puntos  Criterios de Centor modificados:    Temperatura >38  1  0 pts  1‐2.5%  Exudado amigdalar  1  2 pts  11‐17  Ausencia de tos  1  3 pts  28‐35  Adenopatías  1  >4  51‐53  laterocervicales doloras    3‐14 años  1    15‐44  0    >45  1      Faringitis 85% por virus, 15 por EBHGA. Habitualmente 1) exudado blanquecino en amígdalas, 2) adenomatía cervical, ausencia de  rinorrea, 3)tos, 4) fiebre >38°C. Dx clínico con 3 de esos datos, tiene S 75%.    Rinorrea, tos húmeda, disfonía, vesículas, conjuntivitis sugiere origen viral.  Gold stándar: cultivo de exudado faringeo    TRATAMIENTO  Aceptable dar tx empírico si 4 datos y aspecto tóxico.    Bencilpenicilina benzatínica compuesta de 1,200,000 UI, aplicar intramuscular cada 12 hrs por 2 dosis. Seguida de 3 dosis de  Bencilpenicilina procaínica 800,000 UI cada 12 hrs IM.  Si alergia: eritromicina 500mg. Tomar 1 tableta cada 6 hrs por 10 días o TMP/SMX 160/600. Tomar 1 tableta cada 12 hrs por 10 días.    Tx erradicación inicia 9 dias después de la enf aguda. Se aconseja cultivo 7 dias después del tratamiento en agudo.  Dado que S. pyogenes no es productor de beta lactamasas no amerita añadir clavulanato, además este aumenta la hepatotoxicidad.    ¿Cuándo demonios iniciar tx de erradicación? a todos a solo a cultivos positivos post AB inicial?    REHABILITACIÓN  Si exudado purulento, bicarbonato.  Abundantes liquidos, dieta sin irritantes.    PRONÓSTICO  Síntomas de obstrucción de vía aérea superior: estridor, disnea y cianosis, relacionados con epiglotitis, en relación con asbceso  periamigdalino, y retrofaringeo.    COMPLICACIONES  Supurativas: Otiitis media, isnusitis, absceso periamigdalino, mastoiditis, absceso retofaringeo, síndrome de shock tóxico  estreptocócico.  No supurativas: fiebre reuma´tica, glomerulonefritis postestretocócica, artritis reactivas, escarlatina.      CONTENIDO ADICIONAL CLAVE  Los cultivos positivos a Staphylococcus aureus, se encuentran en    personas sanas asintomáticas. Los intentos de erradicación son a  menudo inútiles e innecesarios, excepto entre el personal que trabaja con inmunodeprimidos y pacientes con heridas abiertas.    Criterios de amigdalectomía:    Faringoamigdalitis de repetición: 7 x 1 año, 5 x2 años, 3 x 3 años.      Antecendente de abscesos periamigdalinos    Amígdalas tumorales    Amigdalitis por corynebacterium   

  PACIENTE MASCULINO DE 17 AÑOS DE EDAD, QUE INCIA SU APDECIMIENTOA CTUAL HACE 48 HRS CON ODINOFAGIA INTENSA, BOCIO  EN PAPA CALIENTE, FIEBRE DE 40° CON ATAQUE AL ESTADO GENERAL. A LA EF OBSERVAMOS AMÍGDALAS GRADO 2 CON EXUDADO  PURULENTO Y ABOMBAMIENTO EN PALADAR BLANDO Y DESPLAZAMIENTO DE AMÍGDALAS IZQUIERDA HACIA LÍNEA MEDIA.      EL CUADRO CLÍNICO CORRESPONDE A:  A) MONONUCLEOSIS INFECCIOSA  B) DIFTERIAE  C) ABSCESO PERIAMIGDALINO  D) FARINGOAMIGDALITIS VIRAL    EL AGENTE CAUSAL DE ESTA ENFERMEDAD ES:  A) S. PNEUMONIAE  B) S. AUREUS  C) CORYNEBACTERIUM DIFTERIAE  D) S. PYOGENES    EL TRATAMIENTO INDICADO PARA ESTA PACIENTE ES:  A) SINTOMÁTICO (PARACETAMOL, ANTIHISTAMÍNICO, ANTIINFLAMATORIO)  B) ANTIBIÓTICOS IV  C) DRENAJE POR PUNCIÓN Y ANTIBIÓTICOS  D) AMIGDALECTOMÍA DE URGENCIA    RESPUESTAS C, D, C   

    ABSCESO PERIAMIGDALINO  DEFINICIÓN  Lesión difusa (celulitis o flemón) o localizada (absceso propiamente dicho).    SALUD PÚBLICA  30‐100 mil habitantes.    Morbilidad grave: fascitis necrotizante, mediastinitis, mortalidad.    PATOGENIA  2 teorías: Infección de glándulas de Weber, celulitis y absceso.    DIAGNÓSTICO  Dolor severo, disfagia, otalgia ipsilateral referida, trismus, edema de paladar blando,    desplazamient de amígdalas y úvula.    Cultivo: generalmente polimicrobiano.    USG intraoral S 89%, E 100%, TAC S 100%, E 75%, clínica S 75%, E 75%    TRATAMIENTO  La tendencia es realizar drenaje y continuar manejo ambulatorio.  Peni procaínica si no precisa hospitalización,    AMOXI CLAV para casos de resistencia.    ABSCESO PERIAMIGDALINO  MASCULINO DE 28 AÑOS CON ANTECEDENTE DE RINITIS ALÉRGICA, INICIA SU PADECIMIENTO HACE 11 DÍAS CON ODINOFAGIA, FIEBRE,  DISFAGIA, TRISMUS Y OTALGIA IZQUIERDA. RECIBIÓ TRATAMIENTO CON AMOXICILINA POR 7 DÍAS SIN PRESENTAR MEJORÍA. A LA  EXPLORACIÓN PRESENTA AMÍGDALAS HIPEREMICAS CON DESPLAZAMIENTO MEDIAL DE LA AMÍGDALA IZQUIERDA.    EL DIAGNÓSTICO CLÍNICO MÁS PROBABLE ES:  A)  AMIGDALITIS ESTREPTOCOCICA.  B)  ABSCESO PERIAMIGDALINO.  C)  FARINGITIS CRÓNICA.  D)  MONONUCLEOSIS INFECCIOSA.    EL PROCESO FISIOPATOLÓGICO QUÉ MAS PROBABLEMENTE DIO ORIGEN A ESTA ENFERMEDAD ES:  A)  ACUMULACIÓN DE CASEUM EN CRIPTAS AMIGDALINAS.  B)  INFLAMACIÓN REACTIVA DE TEJIDO LINFOIDEO.  C)  EDEMA FARÍNGEO.  D)  COLECCIÓN PURULENTA PERIAMIGDALINA.    EL TRATAMIENTO INDICADO SERÍA:  A)  ANTIBIÓTICO INTRAVENOSO. 

B)  C)  D) 

DRENAJE QUIRÚRGICO.  COLUTORIOS CON ANTISÉPTICO.  ACICLOVIR. 

  RESPUESTA B,D, B   

LARINGITIS SIMPLE O CATARRAL  GENERALIDADES  Las laringitis generalmente son víricas.    CLÍNICA  Prutito faringolaríngeo, tos irritativas, disfonia progresiva en horas. Evoluciona a curación espontánea en pocos días.      DIAGNÓSTICO  Clínico + laringoscopía indirecta (mucosa congestiva, brillantes y edematosa).    TRATAMIENTO  Aliviar los síntomas con fluidificantes de la secreción laringotraqueal, humidificación con nebulización, antitusígenos, analgésicos y  antitérmicos.     

LARINGOTRAQUEOBRONQUITIS  GENERALIDADES  Suele inciar como una laringits subglótica, posteriormente se agrega sintomatología traqueobronquial, tiraje intercostal, roncus y  sibilancias.      DIAGNÓSTICO  Laringoscopia directa (rodete subglótico y pseudomembranas fibrinosas gran adherencia que aumentan la obstrucción respiratoria.    TRATAMIENTO  Fluidificación, aspiración de secreciones y pseudomembranas, mediante laringotraqueobroncoscopia, AB y traqueotomía para faciliar  las aspiraciones.    PACIENTE FEMENINO DE 50 AÑOS DE EDAD, REFIERE INICIAR SU PADECIMIENTO ACTUAL HACE TRES MESES CON SENSACIÓN DE  GLOBUS FARINGEO, DISFONIA MATUTINA QUE MEJORA AL PASO DEL DÍA, REFIERE QUE SU SINTOMATOLOGÍA MEJORA AL ACLARAR  LAS SECRECIONES DE LA GARGANTA. OCASIONALMENTE PRESENTA PIROSIS. REFIERE ANTECEDENTE DE TABAQUISMO OCASIONAL.      LO QUE ESPERAS ENCONTRAR A LA EXPLORACIÓN FÍSICA DE ESTA PACIENTE ES:  A) ADEMA INTERARITENOIDEA Y DE BANDAS VENTRICULARES  B) MASA LARÍNGEA  C) LEUCOPLAQUEA  D) PLACAS BLANQUECINA COMPATIBLE CON INFECCIÓN MICÓTICA    EL DIAGNÓSTICO MÁS PROBABLE SERÁ:  A) CÁNCER LARINGEO  B) LARINGITIS CRÓNICA VIRAL  C) LARINGITIS ALÉRGICA  D) LARINGITIS CRÓNICA POR REFLUJO    EL TRATAMIENTO RECOMENDADO PARA ESTE PACIENTE ES:  A) RESECCIÓN QUIRÚRGICA  B) ANTIHISTAMÍNICOS  C) ESTEROIDES VÍA ORAL  D) MEDIDA ANTIREFLUJO E INHIBIDORES DE BOMBA DE PROTONES    RESPUESTA A, D, D 

  TUMORES LARINGEOS BENIGNOS  GENERALIDADES  Puedes ser pólipos, nódulos y granulomas.    Los pólipos se dan por uso y mal uso de voz. Frec entre 20‐40 años. Síntoma principal: disfonía. Este es agudo. Reabsorción  espontánea Quirúrgico  Los nódulos se dan en el tercio medio de la cuerda vocal, gritan o hablan mucho. Principal síntoma es la disfonia. Dx por laringoscopia  indirecta o fibrolaaringoscopia.  terapia foniátrica si no  microcirugia laringea.  Papilomas laringeos: por VPH. Masa vegetante blancuzca. Gris rosácea y sésil. Tx es quirúrgico. 

  TUMORES LARINGEOS MALIGNOS 

GENIA  Los más comúnes son epidermoides.  FR: tabaquismo, alcoholismo, ERGE, exposición a sustancias, VPH    Puede ser supraglóticos, glóticos o subglóticos.  Temprano si no hay fijación de cuerda vocal. Se trata con cordectomía o RT  Avanzados si hay fijación. Se trata con laringectomía, disección de cuello, QT y RT como ayuda.   

  QUISTE DE CONDUCTO TIROGLOSO  GENIA 

  QUISTES FARINGEOS  GENIA 

 

  PATOLOGÍA DE LA NARIZ Y SENOS PARANASALES  EPISTAXIS  PATOGENIA  Origen traumático (digital frecuentemente)  Fármacos tópicos y cocaína  Otras: Rinitis, rinosinutisis crónicas    DIAGNÓSTICO  Exploración armada define si anterior o posterior, si se sospecha neoplasia ameritará endoscopia.      TRATAMIENTO  1 Presión nasal con vasocontrictor local (oximetazolina, felilefrina 0.25%, nafazolina) detiene sangrado en 70%.    2 Taponamiento anterior 1‐5 d c lubricación  3 Taponamiento nasal con gasa lubricada, material expansible (Merocel o espojas Kennedy) y sondas inflables (Foley, Epistat, Rush)  resuelven en 60‐80% de casos díficiles.    Sellador de fibrina sustituye de forma efectiva taponamiento nasal y cauterización quimica o eléctica y con menos efectos secundarios.      Quirúrgico  Ligadura microscópica transnasal de arteria esfenopalatina          93%  Ligadura carótida externa        93  Ligdura Maxina interna transnasal      91  Embolización percutánea de arteria maxilar interna    88  Cauterización endoscópica          83    Embolizaciones tienen mayor frecuencia de complicaciones que ligaduras.     

SINUSITIS AGUDA Y CRÓNICA  DIAGNÓSTICO  Dos mayores, un mayor y dos menoes.    Mayores: rinorrea purulenta, obstrucción nasal, hiposmia, anosmia, fiebre.  Menores: cefalea, tos, halitosis, fatiga, otalgia, plenitud ótica, dolor dental.      Estudios de imagen rutinarios no si clínica franca.  Si cefalea frontal donde quede la duda de si hay sinusitis: Caldwell y lateral de cráneo    TRATAMIENTO  Amoxi 500c 8 x 10‐14 d.  Si alergia TMPSMX 160/800 c 12 x 10‐14 d.  Oximetazolina  Paracetamol 500c 6  Naproxeno 250 c 12 hrs  Sol salina    CATARATAS  CATARATAS  DEFINICIÓN  Opacificación del cristalino   

SALUD PÚBLICA  Catarata senil, causa número 1 de ceguera reversibl.    FR: DM, esteroides crónicos, >50 años, inmunosuprsión, agonisa alfa 1a    DIAGNÓSTICO  a)Disminución progresiva de agudeza visual    b)deslumbramiento (fotofobia?) en condiciones de baja luminosisdad    Oftalmoscopio directo: opacidad en el reflejo rojo, si hay duda hacemos midriasis en no hipertensos con fenilefrinas.      TRATAMIENTO  Quirúrgico    ¿Cuál es la técnica de elecicón?  No se debe esperar a que madure la catarata, si hay déficil visual que puede mejorar con cirugía y no hay contraindicaciones, se opera.    PRONÓSTICO  Si se deja evolucionar aumentar riesgo de glaucoma facomórfico.      CONTENIDO ADICIONAL  Agonista α1    Vasoconstricción, midriasis  Fenilefrina, Oximetazolina  Estimulan actividad de fosfolipasa C  Agonista α2  Vasodilatador, sedantes  Clonidina      Inhibe adenil ciclasa, inhubie SNS  Agonista β1  Estimulación cadíaca    Dobutamina, isprotenerol  Estimulan adenil ciclasas y abren canales de Ca  Agonista β2  Broncodilaación    Salbutamol, fenoterol isoprot  Estimulan adenilciclasa, abre canales de Ca   

  ENFERMEDADES DE LA RETINA  RETINOPATÍA HIPERTENSIVA  DEFINICIÓN  Hipertensión arterial==>estrechamiento vascular de arterias retinianas (a)contricción, b)extravasación, c) arteriosclerosis.  Dichas 3 lesiones determinarán estadios de enfermedad retiniana.    Cruces arteriovenosos    Exudados duros y algodonosos    Trombosis y embolias    Hemorragias parenquimatosas retininana    Desprendimiento seroso de retina    Edema de papila    Neuropatía óptica isquémica    PATOGENIA  Ateroesclerosis: arterias grandes con depósitos de gasa en íntima y fibrosis.  Arterioesclerosis: arterias pequeñas con fibrosis.    Arterioesclerosis moderada===> alambre de cobre, grave==>plaa.    DIAGNÓSTICO  Retinopatía hipertensiva (daño a órgano blanco) en el contexto de una crisis:    Edema de papila    Exudados    Hemorragias retinianas    Hemorragias retinianas, microaneurismas, exudados algodonosos, asocian 2‐4 veces riesgo de EVC.    Retinopatía hipertensiva de larga evolución:    TRATAMIENTO  control presor    CONTENIDO ADICIONAL   

 

       

RETINOPATÍA DIABÉTICA  DEFINICIÓN  Daño microvascular del diabgético:      Arrosariamiento venoso    Alt microvasculares    Manchas algodonosas    Microaneurismas    Neovasos    Exudados duros      Tej fibroso.    SALUD PÚBLICA  Principal causa de ceguera en México.  Prevalencia 31.5%  Las formas de RD que afectan potencialmente la visión la EMCS (edema macular clínicamente significativo) y la RDP (retinopatía  diabética proliferativa). Ambas se pueden tratar con fotocoagulación.    21‐39% tiene retinopatía al diagnóstico, casi 100% luego de 20 años.  Primera causa de ceguera prevenible.    FR: alcoholismo, mal control metabólico, >5 años de ser diabético, embarazo, disliidemia LDL >100, TA 130/80, >30 IMC,  USG    [Microaneurismas, microhemorragias y microexudados en un solo cuadrante es leve, en todos los cuadranes es moderada]  Es clínico en no proliferativa. Flurangiografía en proliferativa.        1. Retinopatía diabética NP leve:      Al menos un microaneurisma.  2. Retinopatía diabética NP moderada:      Hemorragias y/o microaneurismas >2A foto estándar; y/o    EB, RV o AMIR.   

3. Retinopatía diabética NP severa:    EB, RV y AMIR, todos presentes en al menos 2 campos de las 4 a las 7 ó    2 de 3 lesiones, en al menos 2 de los campos a través de 4‐7 y hemorragias con microneurismas en los 4 campos >2A ó    AMIR en todos los campos de las 4 a las 7 y >8A foto estándar    en al menos 2 de ellos.      4. Retinopatía diabética proliferativa de bajo riesgo.      Neovasos  5. Retinopatía diabética proliferativa de alto riesgo    Neovasos    Hemorragia en vítreo    Hemorragia subhialoidea    Proliferación neovascular 

  Proliferativa con AR los neovasos están dentro del disco óptico.      EB Exudados blandos  RV Rosarios venosos  AMIR Anormalidades microvasculares intrarretinaninas    TRATAMIENTO  RDNP leve o moderada    NO son candidatos a láser. Control glucémico.    RDNP severa:    Fotocoagulación si: 1)Mal control metabólico, 2) no cumple con revisiones de retina habituales        3)Cursa con catarata que pude limitar la aplicación láser a futuro.        4)Presenta isquemia generalizada, 5)Embarazada    Tratamietno con fotocoagulación de retina periférica o transretiniana(en breve):  RDP sin AR  RDP con AR  NV en iris    Aplicacion fotocoagulación con láser focal o en rejilla si:  RDNP moderada y EMCS  RDNP severa y EMCS  RDP sin AR y EMCS  RDP con AR y EM    REHABILITACIÓN  Envío a 3er nivel: hemorragia vítrea, desprendimienot de retina traccional o regmatógeno.    CONTENIDO ADICIONAL  Exudados duros son un signo de edema macular actual y antiguo.    El edema macular diabético se define como engrosamiento de la retina: esto require de una evalcuación 3D que se realiza bajo  dilatación pupilar por examen de biomicroscopia con lámpara de hendidura y fotografía estereoscópica de fondo.      [RETINOPATÍA DIABÉTICA][r]  HOMBRE  DE  63  AÑOS  DE  EDAD  CON  DIAGNÓSTICO  DE  DIABETES  MELLITUS  DESDE  18  AÑOS.  ACUDE  A  CITA  DE  CONTROL  OFTALMOLÓGICA. A LA EXPLORACIÓN DEL FONDO DE OJO SE OBSERVA LA SIGUIENTE IMAGEN. 

      CON BASE EN LA IMAGEN DE FONDO DE OJO MOSTRADA SE PUEDE CONCLUIR QUE EL PACIENTE PRESENTA:      A) RETINOPATÍA NO PROLIFERATIVA.  B) UNA HEMORRAGIA VÍTREA.  C) RETINOPATÍA PROLIFERATIVA.  D) EDEMA DE PAPILA.    EL TRATAMIENTO DE ELECCIÓN EN ESTE CASO INCLUYE:      A) CONTROL METABÓLICO + APLICACIÓN DE LÁSER.  B) UNICAMENTE CONTROL METABÓLICO.  C) CONTROL METABÓLICO + DERIVACIÓN DE LÍQUIDO CEFALORRAQUÍDEO.  D) CONTROL METABÓLICO + REPOSO EN SEMIFOWLER    RESPUESTA A, B  En la retinopatía diabética podemos encontrar varias lesiones: microaneurismas, hemorragias, exudados duros, manchas algodonosas,  alteraciones microvasculares, arrosaraiamiento venoso, neovasos, tejido fibroso.    Se clasfica en RDP y RDNP.    La RDNP se caracteriza por aneurismas vasculares retinianos, manchas hemorrágicas, dilatación venosa y exudados algonosos.    RDNP mínima: microanerusismas  RDNP moderada: microaneurismas + exudados duros + engrosamiento de capilares  RDNP avanzada: Más de 20 hermorragias retinianas x cuadrante o Arrosaramiento venoso en 2 o más cuadrantes o AMIR en un cuadrante.    MUJER DE 48 AÑOS DE EDAD, QUE ACUDE A CONSULTA DE CONTROL Y QUE CUENTA CON EL DIAGNÓSTICO DESDE HACE UNA SEMANA  DE HERPES ZÓSTER OFTÁLMICO. REFIERE, NO HABER TOMADO EL MEDICAMENTO INDICADO (ACICLOVIR) AL NO PODERLO COMPRAR.    USTED LA REVISA, Y LE COMENTA QUE EN ESTE CASO SU RIESGO DE PERDER LA VISTA ES:    A) DEL 80 AL 100%.  B) DEL 50 AL 70%.  C) DEL 30 AL 40%.  D) DEL 10 AL 20%.    Sin terapia antiviral 50‐70% puede desarrolar enfermedad crónica, 20% desarrollar uveítis y perder la vista.    RESPUESTA D   

      RETINOPATÍA HIPERTENSIVA 

Keith Wagener y Barker  Hilos de cobre estadio III  Hilos de plata: estadio IV 

 

 

GLAUCOMA  GLAUCOMA DE ÁNGULO ABIERTO    DEFINICIÓN  Neuropatía óptica crónica y progresiva    (se excava el nervio óptico) no siempre relaciodado a hipertensión ocular (de ángulo abierto=    PATOGENIA  GPAA: NO está más sensible a los cambios de PIO  Procesos ciliares==> humor acuoso==> camara posterior==> pupila==> cámara anterior==> malla trabecular*==>canal de Schlemm==>  plaexo coroides===> sistema venoso.  *Es el que más influye en la PIO   

    DIAGNÓSTICO  Criterios para GPAA:    PIO sin manejo >21mmHg con aunsencia de causa secundaria    Ángulo abierto    Paquimetria    Cabeza del nervio óptico      Anillo neuroretiniano con pérdida de la relación SINT      Excavación de la papila siendo superior el eje vetical    Campo visual      Campo visual pre‐perimétrico      Escotoma de Bjerrum      Escotoma paracentral profundo      Escalón nasales      Depersiones concéntricas    Hemorragias en astilla del nervio óptico  Atrofia peripapilar con presencia de anillos alfa y beta.    TRATAMIENTO  1)análogos de prostaglandinas[Latanaprost, bimatoprost] (si fracasa adicionar 2))  Si contraindicación beta bloqueador [timolol hemihidrato, metilpranolol]  2)inhibidores de anhidrasa carbónica tópica, alfa agonistas o pilocarpina [acetazolamida, dorzolamida]    Indicaciones de tx quirúrgico:      PIO no alcanzada    Etapa de claucoma: avanzado  Indicaciones de implante valvular:    Glaucoma neovascular en etapa de ángulo abierto    Glaucoma secundario a silicón en cámara anterior  Catarata más GPAA:    Inicial y controlado con 1‐2 medicamentos==> facoemulsificación + LIO    Moderado, avazado o descontrolado==>famoemulsificación + LIO + trabeculectomía    REHABILITACIÓN  Objetivo si daño leve: PIO 20‐30% de basal.    si daño avanzado: PIO 40% o más de basal.    CONTENIDO ADICIONAL CLAVE  Ángulo de 1‐2 está cerrado  Ángulo de 3‐4 está abierto   

GLAUCOMA DE ÁNGULO CERRADO?  DEFINICIÓN  Un ataque agudo conlleva pérdida d ela visión sino se maneja a tiempo. 

 

  PATOGENIA  Grado 0  [0°]    Ocluible  Grado I  [10°]  Alto riesgo  Grado II  [20°]  Cierre poco proba ble  Grado III  [25‐35°]  Abierto  Grado IV  [35%‐45%] Cuerpo ciliar se visualiza con facilidad    Cierre angular primario agudo: el iris contacta con malla trabecular, el ángulo se cierra, PIO aumenta mucho, edema corneal (visión  borosa, halos multicolores), congesión vascular, dolor ocular o cefalea, náuseas, vómito,    Cierre angular primario crónico: puede no haber síntomas, hay sinequias.      TRATAMIENTO    

 

 

    UNA MUJER DE 61 AÑOS, HIPERMÉTROPE Y CON CATARATAS, ACUDE POR DOLOR INTENSO EN OJO IZQUIERDO, DE UNAS HORAS DE  EVOLUCIÓN. PRESENTA UNA TENSIÓN OCULAR DE 40 MMHG, REACCIÓN HIPERÉMICA, MIDRIASIS Y EDEMA CORNEAL. ¿CUÁL SERÍA LA  ACTITUD INMEDIATA MÁS CORRECTA?   



1. 

 MANITOL INTRAVENOSO ASOCIADO A TRATAMIENTO MIÓTICO Y CORTICOIDES TÓPICOS 



2. 

 CORTICOIDES TÓPICOS ASOCIADOS A TRATAMIENTO MIDRIÁTICO 



3. 

 DILATAR ADECUADAMENTE EL OJO PARA EXPLORAR LA RETINA Y DESCARTAR TUMOR INTRAOCULAR 

 4.    RESPUESTA 1   

 OPERACIÓN DE URGENCIA LA CATARADA DE ESE OJO, DESENCADENANTE DEL CUADRO 

 

                 

  GLAUCOMA    FR    PATOGENIA    SÍNTOMAS   

EXPLORACIÓN   

TRATAMIENTO 

GLAUCOMA CRÓNICO SIMPLE  GLAUCOMA PRIMARIO DE ÁNGULO ESTRECHO    Miopía levada, antecedentedes familiares, DM.      Cribado >40 años o con FR.              Hipermetropía, ángulo iridocorneal estrecho, cámara anterior pequeña, ojos pequeños, >50 años.  Aumento de la presión daña la papila óptica.                                                                  Bloqueo pupilar==>    protrusión anterior del iris==> cierre angular==> elevación dela PIO        Se produce en midriasis media. (Oscuridad, ansiosos, uso de simpaticommiméticos [atropina])                Escotoma pericentral (área de Bjerrum) y escotoma priférico nasal d Ronne.  Si >30mmHg: edema corneal, pérdida de la transparencia corneal, dificultad para ver el iris.  (excavación 0.4‐0.5)  Halos de colores alrededor de las luces  Isla de visión central o isla temporal residual (excavación 0.8‐0.9)    Si 40‐50mmHg: dolor intenso, puede irradiarse al territorio del trigémino, blefaroespasmo,  Agudeza se pierde en estadíos avanzados.  lagrimeo, blefaroespasmo, epífora, bradicardia, hipotensión arterial, náuseas, vómito.  Papila >5/10 sospechar glaucoma  Si 80‐90mmHg: colapso de arteria central de la retina, dolor más intenso, disminución de la agudeza    visual, dureza del globo ocular al tacto. Si no se resuelve pronto==> atrofia retiniana irreversible.    Excavación de la papila (E/P 0.7patológica o  Dureza pétrea, midriasis media arreactiva, hiperemia mixta o ciliar, nubosidad corneal.  asimetría >0.2  Tonometría PIO (25 patológica)  Campimetría (primer estudio diagnóstico)  Tomografía de coherencia ópitca  Permanente  Diuréticos osmóticos [manitol, acetazolamida]  Análogos de prostaglandinas F2: aumentan la salida de humor acuoso por la vía  Corticoides tópicos: limitan componente inflamatorio  úveo‐escleral. [latanaprost]  Mióticos: rompen bloqueo pupilar [pilocarpina]  B bloqueantes: disminuyen producción. Cuidado en IC descompensada y bloqueo  B bloquentes, a2 agonistas (solo selectivos)  AV, broncópatas y asmáticos. [timolol]  Iridotomía quirúrgica o con láser.  Agonistas a2 adrenérgicos: disminuyen producción. [apraclonidina, brimonidina]    Inhibidores de anhidrasa carbónica: disminuyen producción. [dorzolamina]  No hay uno de primera elección, generalmente da todos. Pero nunca es de primera instancia  Estimuladores adrenérgicos: aumentan la salida. Producen midriasis.  boqueante a2.  Contraindicados en cámara anterior estrecha, hipermetropes ya que    podríanpercipitar glaucoma agudo.    Prohibido dar análogos de prostaglandinas.    Pilocarpina: aumenta salida de humor acuoso, produce miosis. Poco usado prque  ocasiona miopización del ojo y catarata.  Trabeculoplastía con láser  Trabeculectomia quirúrgica (más eficaz). 

  Glaucoma neovascular  Causa No 1 : DM  Clínica: mala visión, rubeosis (neovasos en iris) + PIO elevada  Tratamiento: panfotocoagulación + válvula de Ahmed  Pronóstico: malo para la función. 

MUJER DE 75 AÑOS DE EDAD, QUE ACUDE A LA CONSULTA CON SINTOMATOLOGÍA COMPATIBLE CON GLAUCOMA.  PARA ENTENDER LA FISIOPATOLOGÍA DEL GLAUCOMA, ES IMPORTANTE SABER QUE EL DAÑO GLACOMATOSO SE DA EN LA SIGUIENTE  PORCIÓN DEL NERVIO ÓPTICO:      A) INTRAOCULAR.  B) ORBITARIA.  C) INTRACANALICULAR.  D) INTRACRANEAL.    RESPUESTA A  El glaucoma es una neuropatía óptica crónica y progresiva, hay pérdida de fibras nerviosas de la retina, excavación y palidez progresivas  de la papila y defectos del campo visual. Va asociado a hipertensión intraocular pero NO en todos los casos. De hecho hay glaucoma  normotenso, y hay hipertensión intraocular (>21mmHg) sin glaucoma.    La papila corresponde a la parte intraocular del nervio óptico.  El  glaucoma  de  ángulo  abierto  (GPAA)  es  el  más  freucente  con  un  60%.  El  ángulo  esta  abierto  y  suele  ser  simétrico.    Etiología  desconocida.    Pero el principal defecto fisiopatológico es la resistencia al paso del humor por la malla trabecular.    Factores de reisgo: 1) PIC, 2) antecdenetes familiares (4‐16% de riesgo de padercerla si son de primer grado, 3)>40 años.    Epitelio  de  cuerpo  ciliar===>  humor  acuoso  en  cámara  posterior==>  pupila==>  cámara  anterior  ==>ándulo  iridocorneal==>  trabeculum===> canal de Schlemm==>vaso espiesclerales==> circulación sanguínea general (eliminación del 90% del humor acuoso)    Solo 10% se eliminará vía uveoescleral atravesando directamente el cuerpo ciliar.    En el glaucoma de ángulo cerrado el origen está en un bloqueo pupilar relativo a una mayor aposición del iris y el cristalino que dificutla  el paso del humor acuoso de la cámara posterior a la anterior a través de la pupila (empujando a la periferia el iris).      Independientemente de la causa, habrá un pérdida progresiva de la capa de fibras nerviosas ya sea de forma mecánica directa o por  compromiso vasculaar.    Tratatamiento farmacológico:  1)betabloqueadores: disminuyen la producción del humor acuoso al actuar sobre los receptores beta de los procesos ciliares [timolol,  carteolol, betaxolol (selectivo B1)]  2)Mióticos parasimpaticomiméticos    y    anticolinesterásicos: disminuyen la resistencia de la malla trabecular. [pilocarpina y aceclidina]  3) Simpaticomiméticos A y B adrenérgicos:    [adrenalina] aumentan el flujo de salida del humor acuoso y disminuyendo la producción en  el cuerpo ciliar. [NO en ángulo cerrado]  4) Agonistas alfa 2 adrenérigcos: disminución de la prodccuón de humor acuoso  5)  Prostaglandinas:  [latanaprost  al  0.005%]  aumenta  la  salida  de  humor  acuoso  a  través  de  vía  uveoescleral.  [NO  en  glaucoma  inflamatorio]  6) Inhibidores de anhidrasa carbónica    Tratamiento láser: trabeculoplastia con lárser de argón en GPAA y trabeculectomía, iridotomía periférica con láser argón o lsa Ng‐YAG en  GPAC.  Tratamiento quirúrgico: trabeculectomía simple o combinada cirugía de catarata (facoemulsificación)    ¿CUÁL ES LA CAUSA MÁS FRECUENTE DE UNA EXCAVACIÓN PAPILAR CON RECHAZO NASAL DE LOS VASOS EMERGENTES DE LA MISMA?  A) HIPERTENSIÓN INTRACRANEAL  B) UVEÍTIS POSTERIOR  C) CONJUNTIVITIS CRÓNICA  D) GLAUCOMA CRÓNICO SIMPLE    RESPUESTA D  No entendí la pregunta :(    UNA  MUJER  DE  64  AÑOS,  HIPERMÉTROPE  Y  CON  CATARATAS  EN  AMBOS  OJOS,  ACUDE  A  CONSULTA  CON  DOLOR  INTENSO  EN  OJO  IZQUIERDO,    DE UNAS HORAS DE EVOLUCIÓN. LA EXPLORACIÓN DE ES OJO PONE DE MANIFIESTO UNA TENSIÓN OCULAR DE 40MMHG,  REACCIÓN HIPERÉMICA CILIO‐CONJUNTIVAL, MIDRIASIS Y EDEMA CORNEAL. ¿CUÁL SERÍA LA ACTITUD INMEDIATA MÁS CORRECTA DE  LAS QUE SE ENUMERAN A CONTINUACIÓN?  A) MANITOL INTRAVENOSO, ASOCIADO A TRATAMIENTO MIÓTICO Y CORTICOIDES SISTÉMICOS  B) PRACTICAR TRABECULECTOMÍA  C) CORTICOIDES TÓPICOS, ASOCIADOS A TRATAMIENTO MIDRIÁTICO  D) OPERAR CON URGENCIA LA CATARATA DE ESE OJO, DESENCADENANTE EL CUADRO.    RESPUESTA A    ACUDE ACONSULTA UN PACIENTE PORQUE DICE QUE EN LOS ÚLTIMOS MESES LE HA CAMBIADO EL COLOR DEL OJO IZQUIERDO. EN LA  ANAMENSIS DEBE INSISTIR EN POSIBLE OCNSUMO DE ALGIUNA DE LAS SIGUIENTES MEDICAMENONES POR VÍA TÓPICA:  A) COLIRIOS PARA GLAUCOMA  B) AINE 

C) CORTICOIDES  D)  ACICLOVIR,  PUES  ESTÁ  DEMOSTRADO  QUE  PUEDE  PRODUCIR  HIPERPIGMENTACIÓN  EN  EL  IRIS  CUANDO  SE  MANTIENE  EL  TRATAMIENTO DURANTE UN PERIODO MUY LARGO DE TIEMPO    RESPUESTA A  Hay que recordar que los análogos de prostaglandinas F2 pueden producir como efectos secundarios cambios en la coloración del iris  (hiperpigmentación) e hipermia y aumento del tamaño de las pestañas.    A pesar de ello son los medicamentos mejor tolerados y eficacez  en el manejo de glaucoma de ángulo abierto.    A 93‐YEAR OLD WOMAN, COMES TO THE EMERGENCY ROOM COMPAINING ABOUT ACUTE PAIN AND REDNESS IN HER RIGHT EYE. TEN  YEARS AGO SHE WAS SUBMITTED TO CATARACT SURGERY OF HER LEFT EYE. HOWEVER THEY "DIDN'T OPERATE MY RIGHT EYE, BECAUSE  IT IS A LAZY EYE". VISUAL ACUITY IS COUNTING FINGERS IN HER RIGHT    EYE AND 8/20 IN HER LEFT EYE. IOP IS 48MMHG IN HER RIGHT  EYE  AND  15MMHG  IN  HER  LEFT  EYE.  SLIT  LAMP  EXAMINATION  SHOWS  CORNEAL  EDEMA,  HYPERMATURE  CATARACT  AND  A  DEEP  ANTERIOR  CHAMBER  WITH  INTENSE  FLARE,  AND  GROSS  TYNDALL.  AGGREGATES  OF  WHIT  MATERIAL  ARE  PRESENT  OVER  THE  ENDOTHELIUM AND THE ANTERIOR FACE OF THE LENS. FUNDUS EXAMINATION REVEALS THE PRESENCE OF DRUSEN IN HER LEF EYE, BUT  IT IUS NOT POSSIBLE IT IN HER RIGHT EYE, DUE TO THE DENSITY OF THE CATARACT. THE MOST PROBABLE DIAGNOSIS IS:  A) ACUDE ANGLE‐CLOSURE GLAUCOMA  B) PHACOLYTIC GLAUCOMA  C) PHACOMORPHIC GLAUCOMA  D) CHOROIDAL MELANOMA    THE MOST APROPIATE TREATMENT FOR THIS PATTIEN IS:    A) OCULAR HYPOTENSIVE DRUGS, TOPICAL STEROIDS AND THE CATARACT SURGERY ON HER RIGHT EYE.  B) OCULAR HYPOTENSIVE DRUGS, AND TOPICAL STEROIDS ON HER RIGHT EYE  C) ENUCLEATION OF HER RIGHT EYE  D) EVISCERATION OF HER RIGHT EYE    ANSWERS  B, A    A 45‐YEAR‐OLD HEALTHY MAN HAS STARTED TREATMENT WITH LATANAPROST AND TIMOLOL 0.5% FOR A RECENTLY DIAGNOSED OPEN  ANGLE GLAUCOMA. HE TELLS YOU THAT HE CAN'T KEEP ON WITH HIS FRIENDS WHEN HE IS PLAYING PADDLE. WICH OF THE FOLLOWING  WOULD BE THE WISER ATTITUDE?  A) CHANGE LATANAPROST FOR A DIFFERENT DRUG  B) CHANGE TIMOLOL FOR A DIFFERENT DRUG  C) CHANGE BOTH DRUGS  D) SEND HIM T THE CARDIOLOGIST    ANSWER B  No entendí la pregunta pero es poco factible que latanaprost tenga que suspenderse. Timolol debe usarse con cuidado en cardiopatas y  broncópatas. Probablemente este paciente tenga alguna contraindicación.     

                                         

    PATOLOGIA DE LAS VÍAS VISUALES      DEFINICIÓN  La vía visual puede interrumpirse dando lugar a tres síndromes: 1)Sx del nervio óptico, 2)Sx quiasmático, 3)Sx retroquiasmático.    PATOGENIA  El conocimiento de estos tres síndromes permite inferir el sitio del proceso patológico (tiumor, traumatismo, EVC, aneurisma), en  función de los datos clínicos.  La vía óptica se inicia en la retina (ahí se encuentran cels bipolares [1eras] y cels ganglionares [2das]), los axones de las cels ganglinares  se unen y forman el nervio óptico que penetra el agujero craneal. La unión de ambos nervios ópticos constituye el quiasma óptico. Las  fibras retinianas nasales se decusan pero las temporales continuan su trayecto. Posterior al quiasma hay cintillas hasta llegar al cuerpo  geniculado lateral. En el cuerpo geniculado se desprenden las 3eras neuronas. Estas 3era neuronas constituyen las radiacione ópticas y  parten a la corteza visual.  [Corregir cuadrito, el "b" es bitemporal no binasal]  a) Hemianopsia derecha, b)hemianopsia bitemporal [heterónima] c)hemianopsia nasal unilateral derecha, d)hemianopsia homónima  izquierda, e)cuadrantopsia homónima inferior izquierda. f) hemianopsia homónima izquierda. 

   

 

 

      DIAGNÓSTICO  Papiledema es la "hinchazón" de la papila secundaria a hipertensión intracraneal (por hemorragia IC o aumento de LCR. Una vez que se  leva el LCR entre 1‐5 días habrá edema de papila. Si es por hemorragia IC tardará 2‐8hrs.    Papiledema desarrollad tarda de 6‐8 semanas en recuperarse.  Síntomas oscurecimiento con conservación de agudeza (en papilitis la agudeza disminuye desde el comienzo). Episodios de 10‐30 seg  frecmente provocados por cambios posturales. Diplopia si parálisis uni o bilateral del VI par.    Cefalea que empeora en la mañana o con  Valsalva. Náuseas y vómitos.  Signos: reflejos pupilar normal, FO con afectación bilateral gralmente...    Precoz. Papila borrosa, rimero superior e inferior, luego nasal y al final temporal.      Desarrollado: hemorragia retinianas, papilares, peripapilares, focos blancos algodonosos, pérdida de plsación benosa  espontánea.    Crónico: tarda en desarrollarse entre 6‐8 sems, atrofia ópica.    Neuritis óptica o papilitis es la tumefacción papilar asociada a pérdida visual debida a desmilinización del NO o inflamación.  Etiología: esclerosis múltiple, infecciones virales, inflamaciones de vecindad (senos, meninges, órbitas), inflamación intraocular.  Síntomas: dismiunución progresiva de agudeza visual 2‐5 días. Adultos 75% unilateral, dolor con movimientos oculares, fénomeno de  Pulfrich (alt en la prcepción de objetos en movmiento), Signo de Uhtoff (empeoramiento con ejericcio o fiebre). Algunos puedne tener  síntomas neurológicos.    Signos: Reflejo pupila alterado, FO no se diferencia del papiledema si existe papilitis.    Sx quiasmático generalmente se deben a tumores extrínsecos (adenoma hipofisario, meningiomas, craneofaringiomas.  Defecto campimétrico típico==> hemianopsia bitemporal.                          Sx retroquiasmático produce alteraciones campimétricas homónimas.    Cintillas ópticas y cuerpo geniculado externo: edema cerebral o alt vascualres la pueden causar.    Radiaciones ópticas y córtex visual: tumores, inflamación, trauma, vascular.                          

  TRATAMIENTO  Papiledema: tratar la causa de HIC, si esclerosis múltiple, sino esclerosis múltiple...    Hacer RMN, si no tiene alteraciones, tiene bajo riesgo de esclerosis múltiple, se pueden dar esteroides..      Si áreas de desmilinización: bolos de esteroides para disminuir recurrencia de neuritis ópitca.    Sx quiasmáticos: dependerá de la causa.      CONTENIDO ADICIONAL CLAVE  Corteza visual primaria [Área 17] recibe la radiación óptica del núcleo geniculado lateral del tálamo. La lesión del área 17 produce  ceguera completa. Su estimulación produce destellos alucinacines visuales a manera de destellos brillantes.      Corteza visual secundaria [área 18, 19] su estimulación produce alucinaciones visuales realistas, su daño produce incpacidad para  recnocer rostros familiares (prosopagnosia), pérdida del color en ciertas partes del campo visual.  ´   

    ENFERMEDADES DE LOS PÁRPADOS, APARATO LAGRIMAL Y CONJUNTIVA  CONJUNTIVITIS GENERALIDADES  DEFINICIÓN  Inflamación de conjuntiva tarsal y bulbar atribuida a distantas causas y que ocasionea escozor ocular, sensación de cuerpo extraño,  presencia de folículos o papilas, hiperemia, epífora, fotofobia, secreción serosa, fibrinosa o purulenta.      SALUD PÚBLICA  FR: disfunción de glándula de Meibomio, deficiencia de película lagrimal ==> conjuntivitis papilar gigante  FR: trauma, def severa de    película lagrijmal, mal posición palpebral==> conjuntivitis mucopurulenta  FR lente de contacto, medicamentos tópicos, laxitud palpebral===> conjuntivitis mecánica irritativa  Fr: pediátrico con sinusitis, OMA, faringitis, obstrucción de conducto nasolagrimal==> conj. mucopurulenta    DIAGNÓSTICO  Mucosa==> alérgico  Mucopurulenta==>bacterina  Acuosa==> irritativa o def. de película lagrimal.    Aguda: bacteriana, UV, por lente o trauma  Recurrente: atópica  Crónica: def de plícula lagrila, laxitud palpebral.    Unilateral: mecánica o químia  Bilateral: bacteriana, UV def de película lagrima, alérgica.      TRATAMIENTO  Conjuntivitis alérgica: [PATANOL 0.1% gotas ofálmicas] hidrocloridrato de olopatadina 0.1% Aplicar 1 gota cada 12 hrs, terminar un  frasco y continuar con [ALERCROM 4% gotas oftálmicas] Cromoglicato de sodio durante período de recurrencia.  Prednisolona 5mg/ml. Aplicar 1 gta cada 8hrs por 5 días máximo (riesgo de cataratas e hipertensión ocular secundaria=  Conjuntivitis mecánica irritativa: Hipromelosa 0.5% gotas oftálmicas. Aplicar 1 gta cada 2 hrs por 5 días. Si disfunción de película  lagrimal de por vida.    Conjuntivitis mucopurulenta: Cloranfenicol 5mg/ml gotas ofálmicas . Aplicar 1 gta cada 4 hrs pr 7 días. Si alergia o hipersensibilidad  Neomicina, polimixina, gramicidina 1 gota cada 4 hrs por 7 días.      CONJUNTIVITIS BACTERIANA DEL ADULTO    DEFINICIÓN  Enfermedad común y autoilimitada    ETIOLOGÍA 

Grampositivos: S. aureus, S. epidermidis, H.influenzae, Moraxella.  N. gonorrhoeae: secreción mucopurulenta, invasión de epitelio sano.      CLÍNICA  Comienzo agudo, bilateral, ojo rojo, secreción purulenta, sensación de cuerpo extraño, prurito, lagrimeo, fotofobia, disminución de la  agudeza visual, edema palpebral, papilas, folículos, secreción en fondo de saco, hiperemia conjuntival, infiltrados cornales.    PARACLÍNICOS  Si la secreción es abundante: cultivo agar chocolate o medio Thayer Martin, por sospechar de N.honorrhoeae    DIAGNÓSTICO DIFERENCIALES  Conjuntivitis viral  Dacriocistitis  Obstrucción del conducto nasolagrimal    TRATAMIENTO  Cloranfenicol, polimixina B,    bacitracina.  Aminoglucósidos: gentamicina, neomicina, netilmicina.  Quinolonas: ciprofloxacino, gatifloxacino y moxifloxacino.  Suspender uso de LDC  Medidas de higiene      PACIENTE MASCULINO DE 18 AÑOS, QUIEN ACUDE A CONSULTA POR REFERIR SENSACIÓN DE CUERPO EXTRAÑO, DOLOR OCULAR, Y  SALIDA DE SECRECIÓN MUCOPURULENTA EN AMBOS JOS DE 5 DÍAS DE EVOLUCIÓN.    A LA EF SE OSBERVA AGUDEZA VISUAL  OD:20/80, OI 20/30 (DIFÍCIL EVALUAR POR DOLOR Y SECRECIÓN). PÁRPADOS EDEMATOSOS, CONJUNTIVA HIPERÉMICA,FONDOS DE  SACO CON SECRECIÓN PURULENTA ABUNDANTES, CÓRNEA TRANSPARENTE Y POLO POSTERIOR NORMAL.   

    ¿CUÁL ES LA PRINCIPAL SOSPECHA DIAGNÓSTICA?  A) CONJUNTIVITIS VIRAL  B) CONJUNTIVITIS BACTERIANA POR S. AUREUS  C) CONJUNTIVITIS BACTERIANA POR N. GONORRHOEAE  D) CONJUNTIVITIS ALÉRGICA COMPLICADA    Siempre que veas un caso de hipermia conjuntivitis    importante con abundante exudado hay que pensar en Neisseria gonorhoea. Las  conjuntivitis virales muchas veces se presentaran como unilaterales.  Cuando es S. aureus tiene poca secreción y es bilateral.    Una conjuntivitis alérgica presentará prurito y tendrá la caracterísita de presentar remisión‐exacerbación    CONDUCTA A SEGUIR EN ESTE PACIENTE:  A) INICIAR TRATAMIENTO ANTIBIÓTICO EMPÍRICO  B) TOMA DE CULTIVO Y POSTERIOR INICIO DE MANEJO ANTIMICROBIANO  C) LAVADOS PROFUSOS DE LA SECRECIÓN 

D) EXPECTANTE YA QUE ES AUTOLIMITADA    Recuerda que si la sospecha es de conjuntivitis vírica o simple bacteriana es diagnóstico es clínico, pero si se sospecha de gonorrea se  indica cultivo antes de dar tratamiento.      MÉTODO DIAGNÓSTICO MÁS ESPECÍFICO PARA ESTE PADECIMIENTO:  A) CULTIVO THAYER MARTIN  B) GIEMSA  C) GRAM  D) CULTIVO AGAR SANGRE    TRATAMIENTO INDICADO EN ESTE PACIENTE:  A) CEFTRIAXONA IV O IM 25‐50MG/KG 1 SOLA DOSIS.  B) ERITROMICINA (12.5MG/KG VO O IV CADA 8 HRS POR 14 DÍAS)  C) MEDIDAS DE HIGIENE SIN SUSPENDER EL USO DE LENTE DE CONTACTO  D) CLORANFENICOL 1 GOTA CADA 4 HRS POR 10 DÍAS.      No olvidar que generalmente gonococo y clamidia van juntas, así que habrá que valorar manejo conjunto para este microorgansmo.  Podriamos dar como coadyunvante eritromicina (para clamidia)    PRINCIPAL RIESGO EN ESTE PACIENTE:  A) DOLOR AGUDO INCONTROLABLE  B) TRATAMIENTO PROLONGADO  C) AUMENTO DE PRESIÓN INTRAOCULAR  D) PERFORACIÓN CORNEAL    N. gonorrhoeae tiene la capacidad de penetrar el tejido epitelial sano, por lo tanto el principal riesgo y la importancia de la premara y  agresividad del tratamiento es evitar una perforación corneal con endoftalmitis secundaria.      RESPUESTA C, B, A, D    A parte de la cantidad de exudado, ¿existen diferencias adicionales entre conjuntivitis bacteriana simple y gonocócica?  La única diferencia será el exceso de secreción. No hay forma de confundirla.    CONTENIDO ADICIONAL CLAVE  Unilateral viral  Foliculos viral  Papilas: bacteriana y alérgica  Infiltrado numular: viral  Úlcera en escudo: alérgica  Ulcera corneal: bacteriana    Los principales en bacteriana: eritromicina, ciprol cloranfenicol. En la práctica se puede dar esteroide, para fines de ENARM la respuesta  más adecuada es solo antibiótico. Para fines de ENARM cloranfenicol está bien, pero en la práctica tiene mucha resistencia y ya no se  utiliza. Mejo tobra, eritro, cipro.          CONJUNTIVITIS NEONATAL                                   

                     

 

 

                             

TOXOPLASMOSIS  Causa No 1 de uceítis posterior  Transmisión: ingesta de carne contaminada, leche no pasteurizada, heces de gato, transplacentario.  Clínica: uveítis granulomatosa, retininas, viritis.  Típico: imagen en faro de niebla (reactivación).  Dx: anticuerpos IgG (memoria), IgM (actividad) vs toxoplasma  Tx: clindamicina, TMP/SMX      TRACOMA  Serotipo A, B, C son los más frecuentes.              ORZUELO Y CHALAZIÓN   

ETIOLOGÍA  Obstrucción e infección bacteriana aguda de las glándulas sebáceas palpebrales, con mayor frecuencia a partir del glándulas de  meibomio (internas) y Zeiis (externas)  Causa No1: S. aureus    CLÍNICA  Tumoración palpebral rojiza, caliente, dolorosa, puede haner punto de drenaje visible, solitario, unilateral, único o múltiple.    DIAGNÓSTICO  Clínico    DIAGNÓSTICO DIFERENCIAL  Celulitis preseptal, carcinoma de células sebáceas, granuloma piógeno, chalazión.    TRATAMIENTO  Compresas calientes + masaje + ungüento antibiótico (eritromicina, ciprofloacno) en fondo de saco.    Antibiótico + corticoesteroide  Crticoide intralesional    PRONÓSTICO  bueno, pero si se hace crónico ==> chalazión   

  PACIENTE FEMENINO DE 1 AÑO QUIEN INICIA SU PADECIMIENTO CON AUMENTO DE VOLUMEN EN PÁRPADO SUPEROMEDIAL,  DOLOROSO A LA PALPACIÓN, DE 1 CM. BLANDO HIPERÉMICO DE 4 DÍAS DE EVOLUCIÓN.   

 

    SU PRINCIPAL SOSPECHA DIAGNÓSTICA ES:    A) CHALAZIÓN  B) ORZUELO  C) CELULITIS PRESEPTAL  D) BLEFARITIS    Piensa en orzuelo si es agudo, localizado, blando, doloroso. Piensa en chalazión si >3 semanas, indurado, no doloroso.      LA CAUSA DE DICHO PADECIMIENTO ES:  A) INFECCIOSO  B) OBSTRUCTIVO  C) INFLAMATORIO  D) METABÓLICA    EL TRATAMIENTO INDICADO POR EL TIEMPO DE EVOLUCIÓN ES:    A) CURETAJE  B) EXPECTANTE  C) COMPRESAS TIBIAS + ANTIBIÓTICO  D) REFERIR AL OFTALMÓLOGO POR GRAVEDAD DEL CUADRO    El curetaje está inicado en chalazión.      RESPUESTA B, B, C      CELULITIS PRESEPTAL Y ORBITARIA    DEFINICIÓN  Inflamación de tejido subcutáneo (los párpados) que no se extiende por detrás del tabique orbitario, no existe afección ocular ni  orbitaria.    Habitualmente no existen datos de respuesta inflamatoria sistémica (fiebre, taquicardia, afección del estado general).    ETIOLOGÍA  Agentes: S. aureus, antecedente de traumatismo periorbitario y cutáneo.    Diseminación por infección local: sinusitis, dacriocistitis, infección dérmica.    A partir de infección a distancia: IVRA    CLÍNICA  Tumefacción palpebral  Enrojecimiento  Ptosis  Dolor  Fiebre baja  Eritema palpebral doloroso  Edema palpebral    Agudeza Visual normal  Motilidad ocular normal e indolora 

No hay proptosis  Conjuntiva y esclera sin inflamación    ABORDAJE  HC oftalmológica  TC orbitaria y de senos par*anasales  BH, hemocultivo y cultivo.*  No se piden de forma sistemática.    DIAGNÓSTICO DIFERENCIAL  Celulitis orbitaria: disminución de agudeza visual + dolor intenso ocular    + limitacióon de mov extraoculares + fiebre + ataque a edo  gral + leucocitosis  Inflamaciójn orbitaria idiopática  Dacriocistitis y conjuntivitis.    Traumatismo.    TRATAMIENTO  Amoxicilina + ác. clavulánico  Cefaclor    Antibióticos tópicos: eritromicina  Drenar absceso palpebral en caso de estar presente.      PACIENTE MASCULINO DE 9 AÑOS, QUIEN ACUDE AL HOSPITAL INQUIETO, CON AUMENTO DE VOLUMEN PALPEBRAL, ERITEMA  PALPEBRAL DERECHO DE 5 DÍAS DE EVOLUCIÓN. A LA EF SE OBSERVA AGUDEZA VISUAL AO: 20/40, MOTILIDAD OCULAR NORMAL Y  CONSERVADA, PÁRPADOS ERITEMATOSOS, DOLOROSOS A LA PALPACIÓN Y CONJUNTIVA HIPERÉMICA.   

  SU PRINCIPAL SOSPECHA DIAGNÓSTICA ES:  A) CELULITIS ORBITARIA  B) CELULITIS PRESEPTAL  C) DACRIOCISTITIS CRÓNICA AGUDIZADA  D) CONJUNTIVITIS BACTERIANA SEVERA    EL ESTUDIO DIAGNÓSTICO A SOLICITAR DE FORMA INICIAL ES:    A) TAC  B) BHC  C) RM  D) ECOGRAFÍA OCULAR    EL MANEJO INICIAL EN ESTE PACIENTE ES:  A) INICIO DE ANTIBIÓTICO TÓPICO EN UNGÚENTO  B) INICIO ANTIBIÓTICO INTRAVENOSO  C) REFERIR AL OFTALMÓLOGO  D) HOSPITALIZACIÓN    Por ser pediátrico, el manejo no puede hacerse de forma ambulatoria y debe haerse seguimiento por un especialista en oftalmología,  por las posibles complicaciones.   

  LA COMPLIACIÓN MÁS FRECUENTE DE DICHA PATOLOGÍA ES:  A) CELULITIS ORBITARIA  B) CELULITIS PRESEPTAL  C) TROMBOSIS DEL SENO CARVERNOSO  D) DISMINUCIÓN PERMANENTE DE LA AGUDEZA VISUAL    La compliación más frecuente es la celulitis orbitaria, y la más grave es la trombosis del seno cavenoso.      Si el diagnóstico del paciente fuere celulitis orbitaria, la complicación más frecuente sería la tombosis del seno cavernoso.      RESPUESTA B, B, C,        PERLAS  Niño, masa aguda, dolorosa          Orzuelo  Niño, masa crónica, no dolorosa          Chalazión  Aumento de vol + visión normal + mov normales      Celulitis preseptal  Aumento de vol + agudeza visual disminuidas + limitación de mov oculares  Celulitis orbitaria    Principal causa de obstrucción de vía lagrimal baja en niños: obstrucción de válvula de Hasner  Tratamiento de obstrucción    de vía lagrimal baja en niños: masaje, no cirugía.    Tumor benigno más común en niños: quiste dermoide  Tumor maligno más común en niños: retinoblastoma    Tumor más freuente de la órbita en niños: hemangioma capilar  Tumor benigno más común en adulto: hemangioma cavernoso  Principal causa de proptosis en adulto: orbitopatía distiroidea (no pongan ningún tumor)     

PATOLOGÍA DE COLON ANO Y RECTO  FISURA ANAL  [La Salle]      GENERALIDADES  Ocurren en línea media, 90% son posteriores, 10% son anteriores, 1% mixtas. Nunca deben ser en otro sitio, de ser así considerar  enfermedad inflamatoria o cáncer.  La división entre úlcera anal aguda y crónica es de 3 semanas.      La úlcera crónica hace borde fibrosos y tríada patognomónica: úlcera + papila centinela + papila hipertrófica    ETIOPATOGENIA  Resultan de la dilatación busca del canal anal  Ruptura del anodeermo  Exposición del esfínter anal interno  Espasmo muscular  Ciclo vicioso, el pacinete n quiere defectar, con almacenamiento de más heces y endurecidas, rasgarán nuevamente.    Isquemia del anodermo y no cura.    CLÍNICA  Dolor desgarrante o quemante.  Sangrado: GOTEO, no mezclado con las heces.    Estreñimiento, raras veces son indoloras.      PARACLÍNICOS  BH, QS, ES, TP/TTP en caso de que sea candidato a cirugía.  Anoscopia o sigmoidoscopia: no forman parte del examen inicial, descarta malignidad o enferemdad inflamatoria intestinal, biopsia en  cualquier fisura en línea media que no sane, sospecha de Crohn, malignidad, manometría anal para corroborar al aumento de la  presión.    Diagnóstico diferencial: sospechar si no está sobre la línea media. (Crohn, TB, malignidad, absceso, fístula, CMV; hepes, Chlamydiosis,  sífilis, SIDA, actinomicosis)      TRATAMIENTO MÉDICO 

Ablandar heces (32 grs diarios de fibra), anaglésicos e inflamatorios locales, baño de asiento (solución templada 2 cucharadas de  bicarbonato, 2 cucharadas de árnica líquida c 12 hrs por 10 días), pomada con 1 tableta molida de isosorbide (10mg/kg) aplicar cada 12  hrs después de cada baño de asiento,    90% curan, toxina botulínica, nitroglicerina 0.2%      TRATAMIENTO QUIRÚRGICO  Úlceras crónicas o recurrentes.    Fistulectomía + Esfinterotomía lateral parcial      SE TRATA DE PACIENTE DE LA TERCERA DÉCADA DE LA VIDA, CON PRESENCIA DE DOLOR INTENSO AL MOMENTO DE DEFECTAR, REFIERE  ADEMÁS DE SANGRADO, QUE SE HACE EVIDENTE AL EVACUAR CON    PRESENCIA DE GOTEO, TAMBIÉN LA INTENSIDAD DEL DOLOR 9  DE 10, CON TIEMPO DE EVOLUCIÓN DE MÁS DE 6 SEMANAS.      A LA EXPLORACIÓN FÍSICA USTED BUSCA:  A) TRÍADA DE PAPILA HIPERTRÓFIA, CENTINELA Y ÚLCERA.  B) ÚLCERA AGUDA CON FONDO SANGRANTE  C) ORIFICIO EXTERNO EN REGIÓN GLÚTEA  D) NINGUNA DE LAS ANTERIORES    La tríada clásica de prentación de un fisura crónica es la presencia de úlcera, así como de papila hipertrófica y centinela.    El orificio extrno en región glútea es la presentación clínica de una fístula.    Úlcera aguda con fondo sangrante, es la presentación de fisura aguda.      LA CIRUGÍA INDICADA PARA EL CASO EXPUESTO ES:  A) HERROIDECTOMÍA  B) FISTULECTOMÍA  C) DRENAJE  D) FISURECTOMÍA MÁS ESFINTEROTOMÍA LATERAL PARCIAL.      La fístula no produce dolor.    Drenaje es el tx de absceso.    Es necesario hacer estinterotomía lateral parcial para tomer la hipertensión del esfínter y el cículo vicioso.      RESPUESTAS A, D            [FISURA ANAL][r]  MUJER DE 33 AÑOS CON ANTECEDENTE DE MIGRAÑA, ACUDE A CONSULTA POR FISURA ANAL.  EL  SIGUIENTE  MEDICAMENTO  PARA  EL  TRATAMIENTO  DE  LA  FISURA  ANAL  ESTARÁ  ESPECIALMENTE  CONTRAINDICADO  EN  ESTA  PACIENTE:      A) TRINITRATO DE GLICERILO TÓPICO.  B) DILTIAZEM TÓPICO.  C) TOXINA BOTULÍNICA LOCAL.  D) LIDOCAÍNA CON HIDROCORTISONA TÓPICA.    RESPUESTA A  El trinitratato de glicerilo tópico cura las fisuras anales mejor que un placebo independientemente de sus dosis. Ocurre curación en un  60% de los pacientes. Se asocia a cefalea en el 25% de los pcaintes por lo que está especialmente controindicado en paciente con migraña.  Diltiazem es tan eficaz como el glicerilo pero con menos efectos secundarios razón por la cuál es de primera elección en el manejo de  fisura anal.    La toxina botulínica es tan efectiva que el glicerilo pero con costos más elevados, esta indicada en fisuras resistenten a tratamiento con  trinitrato de glicero y ditiazem.  El tratamiento con anestésicos y corticoides de forma asilada o complementaria están indicados en la enfermedad hemorroidal.        ABSCESO Y FÍSTULA ANO‐RECTAL   

[La Salle]      GENERALIDADES  Cuando no está relacionado con una enfermedad sistémica, el absceso entonces se considera criptoglandular.    Son más difíciles de tratar los abscesos que están proximales o circunferenciales al plano interesfinteriano.    Pueden ser: 

       

Perianales  Isquiorrectales  Interesfinterianos  Supraelevadores 

  La exploración del absceso es difícil y debe hacerse bajo antestesia.      Las fístulas son compliaciones frecuentes (50%), estas son:      Interesfinterianas        transesfinterianas (menos frecuente)        extraesfinterianas.  SALUD PÚBLICA  Abscesos: H2:M1,    20‐60 años.      CLÍNICA  Absceso anorectal    Dolor intenso, punzante que empora al deambular y con esfuerzos.      Fiebre    Retención urinaria    Sepsis  Fístula      Dolor intenso    Descarga hematopurulenta que disminuye el dolor    Descarga mucopurulenta crónica    EF: tumoración blanda perianal o rectal, la fístula se logra palpa el conducto.      LABORATORIO Y GABINETE  BHC, QS, ES, EGO.  No estudios de imagen en enfermedad no complicada      Enferemedad compliada: la Tomografía con doble contraste es la más adecuda para determinar su extensión. Otros estudios:  USG transrectal, RMI en enfermedad recurrente o compleja.      DIANGÓSTICO DIFERENCIAL  Complicaciones de Crohn, hidrosadeinitis supurativa, TB, actinomycosis, trauma, fisuras, carcinoma, lesión por radiación.    En pacinetes con DM o inmunocompromiso hay que prestar especial atención.    TRATAMIENTO    Absceso: 

Antibióticos solo alcanzan el 50% de la resolución.    Drenaje es el principal tratamiento, curación diaria, cierre secundario.  Baños de asiento si absceso en fase de celulitis.    AB sistémicos solo si celulitis circundante, bacteriemia o sisgnos de sistemico, inmunodeprimidos. (amoxi clav o dicloxa)  Fístula:    Interesfinterianas ==> resección del trayecto  transesfinterianas ==> setón  extraesfinterianas==> resección del trayecto    CASOS CLÍNICOS  SE TRATA DE PACIENTE MASCULINO DE 58 AÑOS, EL CUAL ES DIABÉTICO DE LARGA EVOLUCIÓN, CONTROLADO CON SITAGLIPTINA,  QUIEN REFIERE PRESENCIA DE DOLOR DE 5 EN ESCALA DE 10 A NIVEL DEL ANO, CON SENSACIÓN DE CALOR LOCAL, SIN PRESENCIA DE  TUMORACIÓN O INDURACIÓN, PERO QUE AL ESTAR INCLUSO SENTADO CONDICIONA MAYOR DOLOR. A LA EF NO ENCUENTRA ZONAS  DE INDURACIÓN SOLO AUMENTO DE LA TEMPERATURA LOCAL.    DECIDE REALIZAR:  A) TACTO RECTAL  B) ANOSCOPIA  C) PROCTOSIGMOIDISCOPIA  D) NINGUNA DE LAS ANTERIORES    El cuadro clínico sugiere la presencia de absceso anal de la región del supraelevador, por lo cual es completamente necesario,  incialmente realizar tacto rectal y posteriormente anoscopia. Característicamente cuando tenemos absceso del supralevador no  encuentramos áreas de induración o fluctuación perianales ni glúteas, es por es que precisa tacto rectal o incluso anoscopia.      LA PRINCIPAL COMPLICACIÓN EN ESTE TIPO DE ABSCESO ES:    A) SEPSIS PERINEAL  B) SEPSIS PÉLVICA  C) FOURNIER  D) NINGUNA DE LAS ANTERIORES    El absceso supraelevador se encuentra relacionado    con mayor frecuencia a apertura hacia hueco pélvico, por lo que su rápido  diagnóstico y drenaje es imperioso, ya que además se presenta con más frecuencia en inmunodeprimidos. Este absceso es el más  riesgoso.      La sepsis perineal se presenta como complicación de absceso perianal,Isquierrectal o Interesfinteriano.    RESPUESTA A, B     MUJER DE 33 AÑOS DE EDAD, DE OCUPACIÓN SECRETARIA, POR LO QUE LA MAYOR PARTE DE SU DÍA LABORAL LA PASA EN POSICIÓN  SENTADA. USTED CONFIRMA EL DIAGNÓSTICO DE ABSCESO PERIANAL AGUDO  EL MECANISMO FISIOPATOLÓGICO QUE MÁS PROBABLEMENTE ANTECEDÍO A LA FORMACIÓN DEL ABSCESO PERIANAL DE LA PACIENTE  FUE:      A) UNA FISURA ANAL.  B) UNA FÍSTULA PERIANAL.  C) INFLAMACIÓN CRIPTOGLANDULAR.  D) COLITIS ISQUÉMICA.      LA COMPLICACIÓN ESPERADA DEL ABSCESO PERIANAL DE LA PACIENTE EN CASO DE NO TENER UNA RESOLUCIÓN PRONTA SERÍA:    A) FISURA ANAL  B) FÍSTULA PERIANAL  C) SEPSIS ABDOINAL  D) PROLAPSO RECTA.      RESPUESTAS C, B    El Absceso anal    frecuente entre 20‐60 años, en ambos sexos. Causas específicas:    cuerpos extraños, traumatismos, enfermedades  inflamatorias intestinales (Crohn) procesos infecciosos específicos, tumores, secuelas de tratamientos radioterápico y otras enf anales  como la fisura anal. Causas inespecíficas: como la enf criptoglandular.    La teoría criptoglandular es la base fisiopatológica más aceptada  del absceso perianal.    La Fisura anal    es una úlcera longitudina por debajo de la línea dentada. Generalmente posterior, menos habitual anterior y en 3%  pueden  coexistir  las  dos.    Incidencia  igual  en  ambos  sexos,  frec  en  edad  media  de  la  vida.  Generalmente  idiopáticas.  Etiología  más  probable traumatismo agudo del conducto anal durante la defecación.    Si fístulas múltiples pensar en tuberculosis, sífilis, Crohn o sx de  inmunodeficiencia.    Si es crónica se verá como úlcera con bordes indurados, acompañada de pliegue cutáneo indurado en el extremo 

distal (hemorroide  centinela)  y  una  papila hipertrófica  en  el  borde  proximal  (pólipo  de  Lane).  La  físura puede  acompañar  al  absceso  perianal agudo e incluso favorecerlo, pero no forma parte de la fisiopatología.      La Fístula perianal es un conducto que comunica una cripta anal o el lumen del recto con la piel. Las fístulas simples son normalmente  interesfinterianas  [B]  o  transesfinterianas  bajas  [C],  generalmente  de  trayecto  único.    La  fístula  compleja  tiene  un  trayecto  que  comprende más del 30‐50% del esfínter externo (transesfintérica alta [C] , supraesfintérica [D] o extraesfintérica[E]), es de localización  anterior en mujeres.    Se presentan en 50%    de los pacientes con absceso anorrectal desarrollan una fístula.    Los antecedentes de  importancia  en  este  caso  son:    patología  intestinal  asociada,  eventos  que  comprometan  el  esfínter,  cirugía  anorrectal,  trauma  obstétrico,  procesos  infecciosos  anorrectales.  Presentan  como  descarga  a  través  del  orificio  externo,  en  ocasiones  dolor.  EF  orificio  externo con descarga y se palpa trayecto con fibrosis. La fístula perianal es más bien una complicación del absceso.      La Colitis isquémica es la forma de presentación más frecuente de isquemia intestinal (70‐75%). Afecta princilamente a adultos mayores,  y en jovenes asocia DM2, LES o crisis de células falciformes. La pancreatitis se ha asociado como factor al ocluir en ocasiones los vaso  mesocólicos.    La  fisiopatlogía  de  la  colitis  isquémica  yace  en  una  insuficiencia  venosa  o  arterial  previa.  La  colitis  isquémica  derecha  aislada de relaciona con    cardiopatía crónica, en especial estenosis aórtica. Se manifiesta como un dolor abdominal agudo, hemorragia  gastro intesitnal y diarrea aguda. No se relaciona con abscesos perianales, patología distinta.        Causas de sepsis abdominal:      Colecistopancreatitis      27.5%    Perforación de colon    17.2    Heridas penetrantes abd.  13.7    Apendicitis aguda    13.7    Politraumtizado    10..3    Abscesos peripancráticos  6.9    Trombosis mesentéricas  3.4    El prolapso rectal se asocia a: demensia senil, parasitosis (esquistosomiasis, tricocefalosis, amebiasis), enf neurológias (espina bífida),  multiparidad, histectomía, estreñimiento de larga evolución, hipermotilidad sigmdea.     

 

              

FÍSTULA ANAL  DEFINICIÓN 

Fístula simple solo un conducto entre orificio primario y secundario.  Fístula compleja aquella que precisa sección de aparato esfinteriano.    SALUD PÚBLICA  50% de los abscesos fistulizan.    95% de los abscesos son de origen criptoglandular o inespecífico.  5% de los abscesos son específicos o secundarios (ej Crohn, tb, carcinoma rectal, caricnoma anal, actinomicosis)    DIAGNÓSTICO  Clínica. Dolor cíclico x acumulación de material en el trayecto.    USG endoanal estudio inicial.  RM estándar de oro para fístulas completas y recidivantes.  Fistulografía solo útil si extraesintéricas  TAC solo si secundaria a Crohn.    TRATAMIENTO  Fístula:    Interesfinterianas ==> resección del trayecto  transesfinterianas ==> setón  extraesfinterianas==> resección del trayecto    Fístula anal simple: fistulotomía, marsupialización.    Fístula anal completa: determinación por Parks, no existe manejo estándar.    Con riesgo de incontinencia: legrado, cierre de orificio primario, colocar sello de fibrina y colgajo de mucosa.    Sin riesgo de incontinencia setón y fistulotomía  Fístula anal por Crohn: si asintomático no manejo, con síntomas fistulotomía, setón no cortante, si compleja setón para drenaje.     

     [FÍSTULA ANAL]    MUJER  DE  38  AÑOS  QUE  ACUDE  A  URGENCIAS  AL  PRESENTAR  SALIDA  DE  MATERIAL  PURULENTO  POR  EL  ANO.  HACE  APROXIMADAMENTE 25 DÍAS PRESENTÓ UN ABSCESO GLÚTEO QUE DRENÓ Y CURÓ ESPONTÁNEAMENTE.    EL DIAGNÓSTICO CLÍNICO MÁS PROBABLE EN ESTE PACIENTE ES:  A) ENFERMEDAD DE CROHN.  B) COLITIS ULCERATIVA CRÓNICA.  C) DIVERTICULITIS COMPLICADA.  D) FÍSTULA PERIANAL    EL TRATAMIENTO INDICADO EN ESTE PACIENTE SERÁ CON:      A) FISTULOTOMÍA CON MARSUPIALIZACIÓN.  B) ANTIBIÓTICOS Y COLOSTOMÍA.  C) INMUNOSUPRESORES Y ANTIBIÓTICOS.  D) DRENAJE ABIERTO. 

  RESPUESTA D, A  Las fistulas anorrectales se pueden explicar por las cuatro patologías mencionadas, pero el antecedente de absceso 25 días antes soporta  la fisiopatología de la fístula perianal y no de las demás condiciones.    La enfermedad de Crohn    es una trastorno inflamatorio crónico transmural que frecuenemnte afecta íleon, ciego y colon. Se caracteriza  por diarrea crónica, dolor abdominal, pérdida de peso, fiebre y sangrado rectal.    Una de las características de esta enfermedad es la  presencia de fístulas que derivan a colon, vejiga, vagina o piel.    Una característica primordial de Crohn es su capacidad para fistulizar.  1/3 parte tiene afección perianal formando fisuras anales, abscesos perianales y fístulas.    La colitis ulcerativa se caracteriza por inflamación de la mucosa. La mayor parte de los casos se limita al recto o rectosignmoides, casos  poco frecuentes afectarían al colon o íleon. Recordar que por lo general CUCI no produce fístulas.    La enfermedad diverticular tiene clínica variada, como dolor abdominal crónico en cuadrante inferior izquierdo o sangrado diverticular,  trastorno de la motilidad. En le diverticulitis se presenta inflamación de los divertículos, es un cuadro agudo caracterizado por dolor,  sensibilidad,  masa  en  cuadrante  inferior  izquierdo,  fiebre  y  leucocitosis.  La  diverticulitis  complicada  se  define  como  aquella  que  se  acompaña de absceso, fístula, obstrucción, perforación.    La fístula anal se presenta en 50% de los casos que ha desarrollado absceso anorrectal. Todo absceso no resuelto tiende a fistulizarse ya  que el material purulento que contiene busca un trayecto de salida. El manejo Qx incluye fistulotmía, fistulectomia con o sin reparación  de esfínteresm sedal, colocación de setón de corte o de drenaje, avance de colgado de mucosa retal y aplicación de fibrina.    El setón de  corte es una técnica quirúrgica para quellas fístulas perianales en las que se encuentra comprometido alguno o los dos esfrínteres anales.  Ciertamente el setón es un drenaje abierto de la fístula pero se reserva a quellos casos en que las fístulas son complicada o el abortaje pr  fistulostomía no ha sido suficiente.        Diverticulitis complicada==> antibióticos y colostomía?  Fístula perianal==> fistolotomía con marsupialización  Diverticulitis complicada==>    Colitis ulcerativa crónica==> inmunosiupresores y antibióticos

 

  Definición  Epidemiología 

Clínica 

ENFERMEDAD DE CROHN [EC]  Es un trastorno autoinmunitario que genera inflamación persistente en íleon, sin embargo puede afectar boca,  intestino delgado, grueso y recto.    Afección más grave: fístulas.    Adultos 15‐35 años.    FR: AHF, judío, tabaco, apendicectomía,    5.6 por cada 100,000 habitantes, mayor afectación en mujeres.  Formas de presentación: 1) inflamatoria, 2) fistulizante, 3) fibroestenótica (quirúrgica)    Dolor abdominal tipo cólico (oclusión / suboclusión)  Diarrea acuosa persistente, Fiebre, Pérdida de peso, Anemia y/o hemorragia digestiva  Inapetencia, fatiga, dolor con las deposiciones    Otros: Úlceras bucales, gingivtis, dolor articular, úlceras en piel, hepatitis, estreñimiento, fisuras, fístulas y abscesos  perianales. (Fístulas gralmente en área rectal), pasa esto porque afecta de la mucosa a la serosa. Rectorragia.   

Paraclínica 

BHC (anemia), Coproparasitoscópico/ coprocultivo (descarta infección)  VSG y PCR, PFH (causa hepatitis), VIH (por diarrea crónica), Sangre oculta en heces (causa sangrados), Perfil tiroideo  (hipotiroidismo autoinmune puede asociar EC). ANCAs: presentes en 20‐25%% ASCAS 47‐76%  Tránsito intestinal estudio inicial. Empedrado.(valora extensión, búsqueda de estenosis?)  Panendoscopia o colonoscopia (según dia el tránsito)    Gold estándar (puede haber afección alta)  Ileocolonoscopia    (úlceras aftosas, úlceras profundas, pérdida de patrón vascular, lesiones en parche, imagen en  empedrado, granularidad, eritema, friabilidad, pseudopólipos, estenosis) Generalmente respeta recto.  Toma de biopsias (granulomas no caseificantes 15‐60%, afección transmural metaplasia foveolar, fisuras, ausencia  de infección o neoplasias, inflamación crónica [linfocitos y cels plasmáticas], abscesos en criptas.    Anticuerpos antisarcomises: positivos en 41‐76% 

Diagnóstico 

No existe gold standard para diagnóstico de EC, se usan métodos clínicos, endoscópicos, histológicos y radiológicos.  Para diagnóstico de EC es necesaria: ileocolonoscopia (entra por recto), endoscopia alta (entra por boca), y examen  histopatológico.    En la presentación aguda son útiles: Tránsito intestinal, USG abdominal, TAC  Inducción: prednisona, antiinflmatorios 5‐ASA (mesalazina y sulfazalasina) si enfermedad leve que no desea  glucocorticoide.  Mantenimiento: azatrioprina  CDAI si buena respuesta continuar azatioprina y valorar suspensión en 4 años. Si mala respuesta incrementar  dosis de azatioprina.  CDAI en 2 meses si buena respuesta continur azatioprina y valorar suspensión en 4 años. Si mala respuesta  realizar TORCH, PPD, iniciar antiTNF.  Inducción: infliximab o adalimumab  Mantenimiento: infliximab o adalimumab.  CDAI c 8 sem si buena respuesta continuar, si mala respuesta infliximab + azatioprina  En agudo si no remite con esteroides también puedo dar metotrexate /anti‐tnf  Enfermedad ileal estenosante.  Estenosis  rectosigmoidiscopia o colonoscopia completa.    PROEDUMED dice que anoscopia confirma el diagnóstico.  DIAGNÓSTICO DIFERENCIAL  Sangrado indoloro: cáncer, enfermedad inflamatoria intestinal, enf diverticular, pólipos adenomatosos.    Sangrado doloroso: úlcra rectal o fisura anal, maceración perianal, condiloma acuminado.     

TRATAMIENTO MÉDICO  Hemorroides grado 1, 2: dieta, líquidos, ejercicio, menos tiempo sentado, baños de asiento cada 12 hrs (bicarbonato y árnica)  PROEDUMED, corticoides y anestésicos tópicos (no más de 5‐7 días).    TRATAMIENTO QUIRÚRGICO  Falla del tratamiento médico, ligadura con banda elástica, escleroterpia, hemorroidetomía.    Si trmbosadas: trombectomia NO hemorroidectomía.    Externas se envía a 2 do nivel    CASO CLÍNICO  SE TRATA DE PACIENTE FEMENINO DE 41 AÑOS DE EDAD, LA CUAL CUENTA CON ANTECEDENTE DE 3 GESTACIONES, IMC 34, CON  PRESENCIA DE MOLESTIAS A NIVEL ANAL, CON LEVE DOLOR ADEMÁS MOLESTIAS, PRURITO, DESCARGA ANAL DE TIPO MUCOSO,  SENSACIÓN DE CUERPO EXTRAÑO Y AL REALIZAR HIGIENE DE REGIÓN PERSENTA HEMRRAGIA QUE NO SE MEZCLA CON LAS HECES, A LA  EXPLORACIÓN FÍSICA ENCUENTRA ABULTAMIENT DE LA ZONA CON PRESENCIA DE TUMOR VIOLÁCEO, INTENSAMENTE DOLOROSO,  QUE IMPOSIBILIDAD LA REALIZACIÓN DE EXPLORACIÓN MANUAL.    SU PRINCIPAL SOSPECHA CLÍNICA ES:    A) HEMORROIDEES MIXTAS  B) HEMORROIDES INTERNAS  C) HEMORROIDES EXTERNAS  D) FISURA ANAL    EN EL CASO DE ESTA PATOLOGÍA LO MÁS CONVENIENTE ES:  A) HEMORROIDECTOMÍA  B) FISTULETOMÍA  C) FISURECTMÍA  D) TROMBECTOMÍA    La hemorroidectomía está contraindicada en caso de trombosis    RESPUESTA A, D               HOMBRE DE 43 AÑOS, QUE SE PRESENTA A LA CONSULTA EXTERNA REFIRIENDO SANGRADO RUTILANTE AL FINAL DE CADA EVACUACIÓN.  EL  SANGRADO  SE  PRESENTA  ALGUNAS  VECES  EN  GOTEO  Y  OTRAS  VECES  ABUNDANTE,  ACOMPAÑANDOSE  DE  LA  PROTRUSIÓN  INTERMITENTE A TRAVÉS DEL ANO DE UNA MASA INDOLORA DE 1.5 CM DE DIÁMETRO QUE SE REDUCE ESPONTÁNEAMENTE.    EL DIAGNÓSTICO CLINICO MÁS PROBABLE ES:  A) HEMORROIDES GRADO I.  B) HEMORROIDES GRADO II.  C) HEMORROIDES GRADO III.  D) HEMORROIDES GRADO IV.    EN CASO DE NO RESPONDER AL MANEJO CONSERVADOR EL TRATAMIENTO INDICADO SERÍA:  A) HEMORROIDECTOMÍA.  B) LIGADURA CON BANDA DE HULE.  C) HEMORROIDOPEXIA CON ENGRAPADORA.  D) FLEBOTÓNICOS POR VÍA ORAL.    Ligadura con banda de hule    Hemorroides I, II y III      Hemorroidectomía      Hemorroides IV    Hemorroidopexia con engrapadora  Grado III que requiera incorporación a la vida diaria rápido (tiene más recidiva)  RESPUESTA B, B                       

CÁNCER DE COLON Y RECTO  DEFINICIÓN  Son adenocarcinomas, del ciego al recto, no incluye intestino delgado ni ano.    Los adenomas son lesines premalignas que acorde al grado de displasia o de patrones histológicos (tubular, tubulovelloso) asocian un  aumento de potencial maligno.    SALUD PÚBLICA  Colon derecho más frec en mujeres, recto más frec en hombres.    FR: Colitis ulcerativas (Croh, por radiación, esquistosómica), ureterocolostomía, tabaco, obesidad, poca fibra, alcohol.    FP: ejercicio, AINES, vitamina C, consumo de ac de pescad dietético ric en omega 3 (inhibe la sx de PG a partir del ac araquinónico)    PATOGENIA  A más grasas dietética, más sx de colesterol y ác biliares===> intestino ==> bacterias convierten esto en ac biliares secundarios==>  carcinogénesis.    Vías de diseminación.     Extensión directa:    colon descendente tiene un lumen más pequeño razón por la cual es más factible que se debute como  obstrucción intestinal. El tumor crece de forma radial y le lleva aproximadamente un año en cubrir 66% de la luz intestinal. Penetra las  capas del intestino y se extiende por contiguidad a hígado, curvatura mayor del estómago, duoden, intestino delgado, páncreas, bazo,  vejigad, riñones, ureteresl,pared abdominal.    Metástasis  hematógnas:  cels  viajan  por  el  sistema  porta  hepático  y  hace  mets  hepáticas,  vertebras  lumbares,  pulmones,  ovarios,    DIAGNÓSTICO  Dolor abdominal, ambio hábitos intestinales, pérdida de peso, sangrado rectal (rectorragia, hematoquezia, melena), anemia microcítica  hipocrómica.      Gold standar: para ca de colorrectal y pólipos adenomatosos es la colonoscopia con toma de biopsia.  Indicado como estudio inicial en pacientes con alto riesgo.      Sigmoidoscopia flexible detecta Ca colo rectal y los politos adenomatosos a nivel de la inserción del endoscopio (40 y 60cm), solo  examina el colon izquierdo. Se debe realizar cada 5 años independiente de la sangre oculta en heces.    Colon por enema está inciado cuando no se puede realizar colonoscopia cada 5 años.  Colonoscopia virtual cuando los procedimiento anteriores no son factibles.    Colonoscopia cada 10 años.  Sigmoidoscopia flexible cada 5 años.    Colon por enema con doble contraste cada 5 años.    Colonografía virtual TC variable.    Si hay bajo riesgo para el método de cribado es guayaco o inmunoquímica para buscar sangre.     

 

   

DIVERTICULOSIS  DEFINICIÓN  Son adenocarcinomas, del ciego al recto, no incluye intestino delgado ni ano.      PATOGENIA  ds   

TROMBOSIS MESENTÉRICA    GENERALIDADES  Interrupción aguda o crónica circulación arterial o del drenaje venoso,    la más frecuente es por afección de la vía arterial, en la que los  pacientes con alteraciones del ritmo en el corazón son los afectados en primer lugar, (50% de frecuencia), siendo la arteria mesentérica  superior la más afectada en 50% de los casos.      FISIOPATOLOGÍA    Venosa: cualquier alteración de la tríada de Virchow. Puede tener estasis venosa, depleción del volumen intravascular, lesión en el  endotelio de vasos venosos. Ej. Deshidratación quemados. 

Arterial: ateroesclerosis, alteraciones del ritmo.      CLÍNICA  Arterial:    desarrollo agudo, dolor, distensión abdominal, diaforesis, imposibilidad para evacuaciones, canalización de gases.  Venosa: desarrollo progresivo. dolor, distensión, diaforesis, evacuaciones diarreicas (en grosella), alt electrolíticas, afecciones salteadas,  más específicas, menos extensa.    PARACLÍNICOS  En la aguda los estudios iniciales: BHC, QS, Es, TP/TTP, GA (a más acidosis mayor isquemia, riesgo de perforación), PCR ultrasensible,  CPK, tele de tórax, placa de abdomen de pie y decúbito, TAC si obstrucción venosa, arteriografía si es arterial.    La arteriografía es la prueba diagnóstica y terapéutica más sensible ya que además de darnos diagnóstico, permite administración de  vasodilatadores, trombolítico y anticoagulantes, además de iniciar con antibiótico de amplio espectro por la posible fuga intestinal.      TRATAMIENTO  Teniéndose como límitacion máxima para efectuar trombectomía con catéter de Fogarty 6 hrs, después de esas 6 hrs instalar un catéter  e iniciar trombolíticos, vasodilatadores y anticoagulantes.      Luego de 6 hrs está indicado tratamiento contra el síndrome de reperfusión. Luego de 24 hrs se deberá realizar arteriografía y observar  la recanalización del vaso afectado. Si recanaliza quito catéter y cambio medicamentos a vía oral. Negativo el estudio deberá proceder a  intervención quirúrgica, se usará fluoresceína y luz negra para visualizar áreas mal perfundidas y cortar selectivamente.      EN EL SERVICIO DE URGENCIA, RECIBE A UNA PACIENTE FEMENINO DE 68 AÑOS DE EDAD, LA CUAL, INICIA PAEDCIMIENTO ACTUAL  DESDE HACE 10 DÍAS, CON PRESENCIA DE EVACUACIONES DISMINUIDAS EN CONSISTENCIA DE GRAN CANTIDAD, SIN MOCO, SIN  ANGRA, DOS DÍAS DE EVACUACIONES EN NÚMERO DE 10‐11, LÍQUIDAS COMPLEMTAMENTE, CON TRATAMIENTO ANTIDIARREICO Y  ANTIBIÓTICO, SIN MEJORÍA, PERO QUE CEDE AL 4TO 5TO DÍA, PERSISTIENDO CON DISTENSIÓN ABDOMINAL, MAL ESTADO GENERAL,  POLIDIPSIA, SE AGREGA NÁUSEA, VÓMITO SOMNOLENCIA HASTA HACE 24 HRS, CON PRESENCIA DE DOLOR ABDOMINAL QUE SE  TORNÓ SÚBITO, INTENSO, LOCALIZADO EN MESOGASTRIO, EL DOLOR SE HA PRESENTAOD MÁS INTENSO.    CON PRESENCIA DE DOLOR HASTA PARA MOVERSE, PROVOCANDO QUE TENGA QUE ADOPTAR POSICIÓN ANTIÁLGICA.      SI DIAGNÓSTICO DE SOSPECHA ES:  A) SÍNDROME DE INTESTINO IRRITABLE  B) GASTROENTERITIS PROBABLEMENTE INFECCIOSA  C) ENFERMEDAD ÁCIDO PÉPTICA COMPLIADA  D) ISQUEMIA MESENTÉRICA    ¿QUÉ ESTUDIO INICIAL DE LABORATORIO DE ACUERDO A SU DIAGN´SOTICO FINAL SE DEBE REALIZAR?  A) BHC, QS, ES  B) QS, ES, GASA  C) TOXINA A Y B  D) REACCIONES FEBRILES    DE CORROBORARSE SU DIAGNÓSTICO EN EL ESTUDIO SOLICITADO, DÓNDE SE EVIDENCIA LA DISMINUCIÓN SEVERA DE PH, APARTE DE  ELEVACIÓN DE LEUCOCITOS Y DATOS DE ÍLEO,    ¿CUÁL ES EL ESTUDIO SIGUIENTE A REALIZAR?  A) TAC ABDOMINAL  B) RM  C) USG ABDOMINAL  D) PLACA DE TORAX Y ABDMEN    EL PACIENTE ES MANEJADO CON RESTITUCIÓN DE VOLUMEN INTRAVASCULAR, ASÍ COMO CORRECCIÓN DE ESTADO  HIDROELECTROLÍTICO, SIN EMBARGO NO PRESENTA MEJORÍA, POR LO TANTO, ¿QUÉ ESTUDIO DEBERÁ REALZIARSE ANTE LA  SITUACIÓN?  A) TAC CON CONTASTE INTRAVENOSO  B) TMOGRAFÍA CON CONTRASTE ORAL  C) RESONANCIA MAGNÉTICA  D) ULTRASONIDO DOPPLER.    EL ESTUDIO SOLICITADO MUESTRA DE ACUERDO A SU CERTEZA DIAGNÓSTICA: UNA ZONA DE INTERRUPCIÓN EN LA IRRIGACIÓN  VASCULAR, ¿CUÁL SERÁ EL PASO A SEGUIR?  A) ARTEROGRAFÍA CON CATETERISMO  B) VENOGRAFÍA CON CATETERISMO  C) CIRUGÍA URGENTE  D) ENDARTERECTOMÍA    La deshidratación provoca aumenta de la viscosidad sanguínea (hipercoagulabilidad) 

La GASA nos reportará acidosis metabólica con un anión GAP elevado por desviación de las rutas metabólicas.      Imagen en ileo (liquido libre o imagen en vidrio despulido), si hay aire libre en abdomen o espacio subdiafragmático. Trombosis  necrosis perforación.      Si es venosa TAC  Si es arterial RM  generalmente es crónica.      Recordar que es más frecuente la arterial.    La arteriografía es buena opción terapéutica apero si fuera arterial, en este caso sabemos que el paciente tuvo depleción del volumen  intravascular, estamos y trombosis venosa. La respuesta sería venografía.    RESPUESTA D, B, D, A, B      MASCULINO  DE  65  AÑOS  DE  EDAD  CON  DIAGNÓSTICO  DE  POSTOPERADO  DE  RESECCIÓN  INTESTINAL  SECUNDARIO  A  TROMBOSIS  MESENTÉRICA. LA MAYOR PARTE DEL INTESTINO RESECADO FUE ILEON.    A PARTIR DE AHORA EL PACIENTE TENDRÁ PRINCIPALMENTE DIFICULTAD PARA LA ABSORCIÓN DE:      A) VITAMINA B1.  B) EL COLESTEROL.  C) LA GLUCOSA.  D) HIERRO.    RESPUESTA B  En el íleon se absorben las grasas, vitamina B12 y sales biliares.  La mayor parte de glucosa se absorbe en duodeno y yeuno terminal.  El hierro se absorbe principalmente en duodeno y yeyuno.    El hierro se absorbe principalmente en duodeno y yeyuno.  La mayor parte de glucosa se absorbe en duodeno y yeuno terminal.  En el íleon se absorben las grasas, vitamina B12 y sales biliares.       

CÁLCULOS URETERALES Y UROPATÍA OBSTRUCTIVA  CÁLCULOS URINARIOS    DIAGNÓSTICO  Generalmente debutan como dolor lumbar que irradia a la fosa ilíaca ipsilateral (a labio de vulva o escroto).  sintomagolotía irritativa urinaria baja (urgencia, disuria, hematurina). Expulsión del lito  ==> alivio del dolor.        Urografía excretora ha sido el    estándar de oro para el diagnóstico==> TC helicoidal NO contrastada ha demostrado se supeiror.    USG renal ureteral y vesical junto con rx simple de abdomen ofrecen evaluación aleranativa.  Pielografía retrógrada en situaciones especiales.      TRATAMIENTO  Manejo del dolor: 1era. Diclofenaco, indometacina, ibuprofeno,  2da Hidromorfina, hidrocloruro de atrpina, metamizol, pentazocina, tramadol.      Bloqueadores alfa 1 facilia el paso de cálculos.     0.15, velocidada >0.75ng/ml año y tiempo    de  duplicación 3m    c) APE >10    d)hallazgo anormal al TR aislado.    TAC: para estadificación. No útil si el paciente está asintomático    y PSA 10 años.      PRONÓSTICO  T1  Tumor clínicamente no aparente  T2  Tumor locacalizado a próstata  T3  Tumor con extensión etracapsular  T4  Tumor invade estructurae sdistintas a las v. seminales, cuello de vejiga, esfínter externo, recto, músculos elevadores, pared    pélvica.    Agresividad la definite la escala de Gleason.  Grado 1 o bien diferenciado [2‐4]  Grado 2 o moderadamente diferenciado [5‐6] 

Grado 3 o pobremente diferenciado [7‐10] 

     

 

    HOLA PROEDUMED. Me encuentro leyendo Tema Patología de la próstata, Subtema Cáncer de Próstata y tengo una duda respecto al  contenido,    en el subtítulo Diagnóstico dice      "No se recomienda la realización de tamizaje de cáncer de próstata a población masculina asintomática entre 50 a 69 años de edad  empleando la prueba de APE y el TR, debido a que se tiene evidencia de que no es efectivo en la reducción de la mortalidad por cáncer  de próstata; a que su implantación representaría un elevado impacto presupuestal y a que se expondría a los pacientes a riesgos  innecesarios."    Y en el diagrama de flujo del final dice “que está indicado realizar TR y APE en pacientes ASINTOMÁTICOS sin factores de riesgo a partir  de los 50 años y 40 con factores de riesgo”    Mi duda es, ¿se recomienda o no se recomienda realizar el tamiz?  Saludos cordiales, gracias por atender mi mensaje.      EPIDEMIOLOGÍA  Riesgo de cáncer a lo largo de la vida es de 16%.  Menos de 2% se diagnostica antes de los 50.    Escritidio 40‐50 años: APE + tacto rectal.      FR: historia familiar (familiar = más de un familiar afectado, hereditario= más de tres o más de dos antes de los 55 años), raza negra,  casos esporádicos 85%, edad.      CLÍNICA 

Asintomático en estadios temprano  Si avanzado: dolor óseo, fracturas patol´gicas, crecimiento local, obstrucción urinaria o ureteral.      DIAGNÓSTICO  Biopsia prostática (transrectal o perineal).  Zonas hipoecoicas mayor riesgo de cáncer.    Técnica: sextantes extendidos ( al menos 10, usualmente 12)de la zona periférica.  Histología Gleason (2‐10) según patrón primario y secundario.    Riesgo bajo 8.    La mayoría adenocarcinoma acinar + o ductal y neuroendocrinos (indifernciados).    LABORATORIOS  Antígeno prostático específico.      APE >4ng/ml. Sospecha    Zona gris entre 4‐10ng/ml (20‐30% con cáncer.    APE >100ng/ml prácticamente todos tienen mets.    Elevación de azoados (creatinina), fosfatasa alcalina y calcio (enf ósea).    Extensión:  TAC de abdomen y pelvis con contraste, RMN (enf extracapsular)  Gammagrama óseo si APE >20 (lesiones blásticas)    TRATAMIENTO  Tratamiento curativo: prostatectomía radical y radioterapia.    El bloqueo androgénico se hace con: ketoconazol, dietilestilbestrol (riesgo trombótico), agonistas de LHRH que son los más  utilizados(leuprolide, goserelina), antagonistas (degarelix), antiandrogénicos (bicalutamida, flutamida). Orquiectomía simple bilateral  (estándar).    Caazitaxel, abirateron, enzalutamida, docetaxel: Ca refractario a castración.    Ac. zolendrónica, denosumab, radio‐223: enfermedad mestásica ósea, prevención de fracturas.    Para no metastásicos dar tratamiento curativo con prostatectomía o radioterapia, si queda Ca residual luego de cirugía hay que radiar  el lecho prostático.  Hormonterapia es para casos metastásicos.    MASCULINO DE 65 AÑOS DE EDA, SIN COMORBILIDADES, NIEGA ALÉRGICOS Y QUIRÚRGICOS PREVIOS, ACUDIÓ HACE 6 MESES PARA  ESTUDIOS GENERALES Y REPORTARON ANTÍGENO PROSTÁTICO DE 10NG/ML, ACUDIÓ CON MÉDICO PARTICULAR Y RECIBIÓ  LEVOFLOXACINO POR UN MES. HACE UN MES, NUEVAMENTE SE REALIZA LOS ESTUDIOS Y PRESENTA UN ANTÍGENO PROSÁTICO DE 12  NG/ML. AL INTERROGATORIO SÓLO REFIERE CHORRO DÉBIL OCASIONALMENTE. TACTO RECTAL CON PRÓSTATA AUMENTADA DE  TAMAÑO, SIN NÓDULOS PALPABLES. GENITALES SIN ALTERACIONES. EGO SIN ALTERACIONES. UROCULTIVO NEGATIVO.    MANDA A BIOPSIA Y REPORTAN: ADENOCARCINOMA DE PRÓSTATA GLEASON 4+4 EN 3 DE 12 FRAGMENTOS.      ¿QUÉ ESTUDIOS MANDA PARA TERMINAR ABORDAJE?  A) TAC DE ABDOMEN Y PELVIS CONTRASTADA, RX DE TÓRAX, GAMMAGRAMA ÓSEO Y PRUEBAS DE FUNCIÓN HEPÁTICA.  B) USG HEPÁTICO, FOSFATASA ÁCIDA, SERIE ÓSEA COMPLETA, TAC SIMPLE DE TÓRAX  C) GAMMAGRAMA ÓSEO, TAC DE ABDOMEN SUPERIORES, RX DE TÓRAX.  D) RESONANCIA MAGNÉTICA DE PELVIS, SERIE ÓSEA    SI LA ENFEREMDAD SE ENCUENTRA LOCALIZADA DESPUÉ SDE COMPLETAR SUS ESTUDIOS, LA CONDUCTA A SEGUIR ES:  A) RESECCIÓN TRANSURETRAL DE PRÓSTATA.  B) PROSTATECTOMÍA RADICAL  C) BLOQUEO HORMONAL  D) RADIO 123    EL APE NADIR DE 0.5NG/ML NOS HACE SOSPECHAR DE PERSISTENCIA DEL CÁNCER, QUE DECIR HACER:    A) BLOQUEO ANDROGÉNICO  B) RADIOTERAPIA ADYUVANTE  C) PROSTATECTOMÍA RADICAL  D) VIGILAR HASTA ELEVACIÓN DEL ANTÍGENO    EN CASO DE HANER DEMOSTRADO ENFERMEDAD METASTÁSICA:  A) QUIMIOTERAPIA  B) BLOQUEO ANDROGÉNICO  C) RADIOTERAPIA DIRIGIDA A SITIOS METASTÁSICOS  D) CUIDADOS PALIATIVOS. 

  EN CASO DE TENER METÁSTASIS ÓSEAS, ¿CUÁLES SON LAS MÁS COMUNES?  A) OSTEOBLÁSTICAS  B) OSTEOCLÁSTICAS  C) NO SON FRECUENTES EN CÁNCER DE PRÓSTATA  D) B Y D SON CORRECTAS.      RESPUESTA A, B, B, B, A        [CÁNCER DE PRÓSTATA]  HOMBRE DE 64 AÑOS DE EDAD QUE ACUDE A LA CONSULTA PORQUE DESDE HACE 3 MESES PRESENTA DISMINUCIÓN DEL CALIBRE DEL  CHORRO URINARIO, GOTEO TERMINAL, POLAQUIURIA Y DISURIA. A LA EXPLORACIÓN SE REALIZA TACTO RECTAL ENCONTRANDOSE LA  PRÓSTATA CON UN NÓDULO PÉTREO EN LA PARTE POSTERIOR DE ESTA GLÁNDULA.  PARA CONFIRMAR EL DIAGNÓSTICO SE DEBERÁ INDICAR EL SIGUIENTE ESTUDIO:  A) ULTRASONIDO PROSTÁTICO.  B) FOSFATASA ALCALINA EN SUERO.  C) NIVELES DE ANTÍGENO PROSTÁTICO ESPECÍFICO.  D) BIOPSIAS TRANSRECTALES DE PRÓSTATA.    RESPUESTA D  No se recomienda tamizaje para próstata a población asintomática entre 50‐69 años    de edad empleando la prueba de APE y TR (ya que  no asocia reducción de mortalidad).  La detección basada en APE demostró reducción del 20% pero se relaciona con importante riesgo de sobrediagnóstico.    La mayor parte de cánceres se presentan en la parte posterior de la glándula en forma de nódulos indoloros y pétreos.    La biopsia de próstata guida por USG es la única prueba confirmatoria, idelmente tres de la derecha, tres de la izquieda y si procede  clínicamente 1 de la zona transicional. No se recomienda si prostatitis, a menos que haya recibido un ciclo de antibioticoterapia.    TR VPP 21%. 17% de pacientes con PSA 2.5‐4ng/ml y datos normales en TR tiene cáncer.        [r]  HOMBRE  DE  60  AÑOS  DE  EDAD  QUE  ACUDE  A  CONSULTA  CON  SINTOMATOLOGÍA  URINARIA  IMPORTANTE.  USTED  SOSPECHA  LA  POSIBILIDAD DE CÁNCER DE PROSTATA.  EN ESTE CASO DEBERÁ IDENTIFICAR EL SIGUIENTE FACTOR DE RIESGO:      A) CONSUMO DE DIETA CON ELEVADO CONTENIDO DE GRASAS.  B) CONSUMO OLIGOELEMENTOS.  C) EL CONTACTO PERMANENTE CON SUBSTANCIAS DERIVADAS DE LA ACTIVIDAD INDUSTRIAL.  D) LA RAZA O GRUPO ÉTNICO    RESPUESTA D  Factores ambientales: zona geográfica, dieta alta en grasa, exposición a humo de automóviles, polución del aire, cadmio, fertilizantes,  sustnacias de la industria de la goma, imprenta y pintura.    Factores genéticos: gen HPC 1.  Factores infecciosos: ITS.  Factores hormonales: andrógenos (en eunucos no aparecen).    El consumo elevado de grasa si es FR pero no el más importante, los oligolementos no tienen nada que ver.    La sustnacias de la industria de pueden generar riesgo pero la respuesta es vaga, no especifica que sea de pintura y la goma.    La raza negra es el mayor factor de riesgo de neoplasia prostática intraepilial multifocal y con gran inestabilidad. Quizás por altos niveles  de testosterona.  En términos generales son los factores de riesgo más imprtantes para Ca prostático: Edad avanzada, antc familiares, raza negra.        HOMBRE DE 35 AÑOS DE EDAD ACUDE A CONSULTA CON MÉDICO GENERAL. EL PACIENTE SE MUESTRA CONSTERNADO DEBIDO A QUE  DOS DE SUS TÍOS LES DETECTARON CÁNCER DE PRÓSTATA A LOS 50 Y EL OTRO A LOS 65 AÑOS, NIEGA SINTOMATOLOGÍA ASOCIADA Y  NO CUENTA CON ANTECEDENTS DE IMPORTANCIA. AL TACTO RETAL SIN PRSENCIA DE NÓDULOS O ALTERACINES. EL APE ES DE 2.3NG/DL.    ¿CUÁL ES LA RECOMENDACIÓN MÁS APROPIADA PARA EL PACIENTE?  A) DETERMINACIONES BIENALES DE APE A PARTIR DE LOS 65‐85 AÑOS  B) DETERMINAICONES ANUALES Y BIENALES DE APE DE LOS 35‐75 AÑOS  C) DETERMINACIONES ANUALES DE APE DE LOS 40‐75 AÑOS  D) DETERMINACIONES ANUALES DE APE DE LOS 50‐75 AÑOS    RESPUESTA C 

Ca de próstata tiene fenómeno de anticipación de 7 años. El paciente en cuestión tiene factores de riesgo y por lo tanto el cribado se  inicia con APE a los 40 años, de no tenerlos sería    a los 50.    [info del proedumed]    El antígeno prostático específico es el examen más importante según las guías de la Secretaría de Salud, a pesar de que hay varios países  que lo han retirado de sus guías de práctica clínica, porque aunque detecta    el cánr no modifica la motalidad.  un gram de cáncer produce 3ng/ml de antígeno.    los valores normales van de 0‐4ng/ml. La velocidad de crecimiento anual de más de 1ng/ml por año se considera anormal. El porcentaje  de antígeno libre también se puede detectar, y si este es menor a un 10% en general es debido a áncer y es más de 25% probablemnte  se trate de un adenoma. El estándar de oro para el diag nóstico es la biopsia. Se sugiere realizar determinaciones deanuales de APE, pero  denbido al crecimiento lento del cáncer, se podría realizar también en intervalos de 2 años iniciando desde los 40‐45 años hasta los 75‐ 65 añls si el pacient presentó determinaciones bajas persistentes entre 0.5‐1ng/dl. [reto enarm]      HOMBRE DE 67 AÑOS DE EDAD QUE ACUDE A CONSULTA PORQUE, HACE ALGUNOS MESES, DEBE PUJAR CON ÁS FUERZA PARA PODER  ORINAR Y AUN ASÍ, EL CHORRO URINARIO HA DISMINUIDO DE CALIBRE. AL INTERROGATORIO DIRIGIDO REFIERE QUE SE PARA A ORINAR  MÁS DE DOS VECES POR LA NOCHE. NIEGA DISURIA, HEMTURIA O ALGUNA CONDICIÓN ASOCIADA. TIENE DIAGNÓSTICO DE DM2 HACE  10 AÑOS, EN TRATAMIENTO REGULRA. FC 77, FR 29 T 36 TA 115/67. A LA EXPLORACIÓN FÍSICA SE PALPA PRÓSTATA SIN NÓDULOS O  INDURACIONES PERO AUMENTADA DE TAMAÑO. LOS ESTUDIOS DE LABORATORIO REPORTAN APE 4‐5NG/DL. SE REALZÓ UN USG QUE  DEMOSTRÓ UNA PRÓSTATA DE 45 CC. SIN NÓDULOS NI CALCIFICACIONES.  ¿CUÁL ES EL MANEJO INICIAL DE ESTE PACIENTE?  A) CLONIDINA + FINASTERIDE  B) DANAZOL + DOXAZOSINA  C) PRAZOSINA + DANAZOL  D) DOXAZOSINA + FINASTERIDE    RESPUESTA D  La hiperplasia prostática benigna se reifere a una detección microscópica de la hiperplasia (una proloferación de estroma y epitelio),  crecimiento de próstata detectado por examen rectal digital o por medio de ultrasonido además de presentar síntomas asociados a esta  hiperplasia. Se recuerda que el tamaño de la próstata no siempre correlaciona con los síntomas. La prevalencia aumenta en forma lineal  con la edad, teniendo una prevalencia de 90% en pacientes de 85 años o más. La clásica sintomatología obstructiva es pujo miccional,  dismiinución del calibre de la orina, nicturia, etre otros. Entre los métodos diagnósticos está la USG con medici´n de orina residual. Hay  una relación directa entre lso niveles de APE y el volumen prostático pero no tienen mayor riesgo de desarrollar cáncer de próstata. El  tratamiento farmacológico evita la morbilidad asociada a la cirugía. En pórstata de más de 40cc el tratamiento combiada (alfablqoueador  e inhibidor de la 5 alfa redutasa) es la mejor opciójn. La clonidina es una gonista alfa 2.        HIPERPLASIA PROSTÁTICA BENIGNA  DEFINICIÓN  El diagnóstico y el término es histológico exclusivamente.  Multifactorial, edad (>40 años), dihidrotestosterona.      Testosterona    dihidrotestosterona    5 alfa reductasa tipo 2    SALUD PÚBLICA  Afecta a hombres >45 años, clínica por lo regular 60‐65.  Obesidad aumenta >10% el riesgo de HPB clínica.      DIAGNÓSTICO  Almacenamiento (urgencia, frecuencia, nicturia), vaciamiento (pujo, intermitencia, disminución del chorro, tenesmo, otros: doble  micción, goteo terminal).    Clínica:    >50 años    Pujo    Intermitencia    Nicturia    DIsminución del calibre    DIsminución de la fuerza    I‐PSS Leve 0‐7, Moderado 8‐19, severo 20‐35.    Datos que predicen retención aguda de orina, necesidad de cirugía y que además ameritan tratamiento.    Vol prostático >30ml    Flujo urinario débil    APE >1.4ng/ml   

‐Si clínica solicitar EGO, y glucosa/creatinina en caso selectos.    ‐TODO PACINETE que ingresa a protocolo de tratamiento de HPB deberá tener USG vesical y prostático con medición de orina residual.    ‐Si >50 % de orina residual postmiccional, elevación de creatinina, ant de litiasis, IVUs de repetición, hematuria macro, incontinencia x  rebosamiento==> USG renal.      HPB no aumenta el riesgo de Ca de prostata.      TRATAMIENTO  1)Medidas generales: Disminuir cafeína, alcohol y líquido libre por la tarde noche.    2)Alfabloqueadores relajan próstata y cuello de vejiga, son los mas efectivos para los STUI su eficacia se mantiene luego de 6‐12 meses  de tratamiento. NO REDUCEN TAMAÑO DE PRÓSTATA NI MODIFICAN EVOLUCIÓN DE LA ENFERMEDAD.  Alfuzosina 10mg c 24  Tamsulosina 0.4mg c 24  Terazosina 2‐5mg c 24  Doxazosina 2‐4mg c 24  3)Inhibidores de 5 AR, reduce el tamaño prostático, reduce riesgo de RAO en 50% y el riesgo de cirugía indicada si STUI y >40cc de vol  prost, en estos casos es mejor tx combinado    Finasterinde 5mg c 24  Dutasteride 0.5mg c 24  4) Inhibidores de la fosfodiesterasa. Se indica si hiperplasia prostática + disfunción eréctil.  5) Quirúrgico: indicado si complicaciones (hidronefrosis, IR, RAOR, IVUR, hematuria persistente, síntomas que afecten su calidad de  vida, fracaso a manejo farmacológico, litiasis vesical,  a)RTUP (resección transuretral del próstata) es el tx más efectivo pero no el de primera elección.      Complicaicones:    eyaculación retrógrada  65‐70%        disfunción eréctil    6.5        Transfusión    2‐5        estenosis uret, contractur ves  3.8        incontinencia urinria de esf  1.8        Mortalidad    0.25        Sangrado  Sx postRTUP (hiponatremia)  alt de edo de consciencia  Ameritarán monitoreo anual con TR y APE  b) Prostatectomía abierta si >80cc o >80‐100gr, complicaciones asociadas, litiasis vesical. Tiene más complicaciones.    Otras: resección transuretral con láser, vaporicazión con láser verde. Puede usarse en anticoagulados.   

     

 

 

  PACIENTE MASCULINO DE 55 AÑOS DE EDAD, TABAQUIMSO NEGADO, SIN CIRUGÍA PREVIAS, PREVIAMENTE SANO, CON CUADRO DE 2  AÑOS DE EVOLUCIÓN CARACTERIZADO POR FRECUENCIA URINARIA, NICTURIA, DISMINUCIÓN DEL CHORRO URINARIO, NIEGA DISURIA.    A LA EF AFEBRIL, ABDOMEN CON PERISTALSIS PRESENTE, SIN DOLOR A LA PALPACIÓN SUPERFICIAL NI PROFUNDA, GIORDANO  NEGATIVO, PRÓSTATA NO DOLOROSA Y SIN NÓDULOS, CREAT 1.1MG/DL, APE 3 NG/ML EGO 1 LEUCOCITOS, ERITROCITOS 1,  BACGERIAS 30, NITRITOS NEGATIVOS.      EL DIAGNÓSTICO PARA ESTE PACIENTE ES:  A) ESTENOSIS DE URETRA  B) INFECCIÓN DE VÍAS URINARIAS  C) CRECIMIENTO PROSTÁTICO OBSTRUCTIVO  D) PROSTATITIS CRÓNICA    ESTUDIOS QUE COMPLEMENTAN SU ABORDAJE:    A) USG VESICAL Y PROSTÁTICO  B) CISTOURETROGRAFÍA  C) UROCULTIVO  D) CULTIVO DE EXUDADO PROSTÁTICO    EL TRATAMIENTO QUE USTED ELEGIRÍA ES:  A) ALFA BLOQUEADORES  B) FLUOROQUINOLONAS  C) DILATACIÓN URETRAL  D) TMP/SMX    RESPUESTA C, A, A 

   

PROSTATITIS  DEFINICIÓN  No aumentan riesgo de Ca de próstata.    PATOGENIA, DIAGNÓSTICO, TRATAMIENTO.  Prostatitis aguda bacteriana: generalmente gramnegativas (E coli, Klebsiella p,    Proteus mirabilis, Pseudomona aeroginosa, Stap h  aureus, enterococos, y anerobios (bacteroides spp). ITS com gonorrea, clamidia, trichomonas. Disuria, polaquiuria, urgencia, dolor  lumbosacro, perineal, en pene, en recto,fiebre, mialgias, malestra gral. Tx AB, AINEs, liquidos.    Prostatitis crónica: clínica similar pero gralmente sin fiebre, mismos gérmenes además Clamidia tachomatis.    Prostatitis no bacterianas: es la mas frec, se sospecha sea por bacterias no comunes,    (Clamidia, Ureaplasma urealyticum,  Micoplasma hominis) o sea    inmunológica.    en orina y fluidos prostátitos cno hay evidencia de células inflamatorias.    Prostatodinia: igual que la no bacteriana pero sin céls infalmatorias en líquidos. Tx alfa1bloqueadores.  Las prostatitis elevan    PSA     

PROSTATITIS AGUDA    PATOGENIA  Ascendente, reflujo de orina a conductos prostáticos. Vía linfática y hematógena.      CLÍNICA  Fiebre, síntomas irritativos, obstructivos, dolor suprapúbico o perineal, dolor intenso al tacto rectal.    Tacto rectal vigoroso o masaje prostático pueden ocasionar septicemia, no deben hacerse.      PARACLÍNICOS  Leucocitosis, piuria, bacteriuria, hematuria.    Urocultivo: gramnegativos (E. coli y Pseudomonas), menos frec. gram positivos (Enterococcus)    DIAGNÓSTICO DIFERENCIAL  Pielonefritis, epididimitis, diverticulitis.      CLASIFICACIÓN DE PROSTATITIS  Categoría I: prostatitis bacteriana aguda  Categoría II: prostatitis bactriana crónia  Categoría III: prostatitis no bactriana/ síndome pélvico doloroso    IIIA‐ Inflamatorio    IIIB‐ No inflamatorio  Categoía IV: Prostatitis asintomática (diagnóstico histológico o andrológico)    TRATAMIENTO  Hospitalización vía parenteral (ampicilina y aminoglucósidos) hasta obtener sensibilidades. Luego de 24‐48 hrs afebril, cambiar    vía  oral y completar ciclo de 4‐6 semanas.    Si retención aguda de orina: cistostomía suprapúbica o sondeo uretral.      Complicaciones: absceso prostático, prostatitis crónica bacteriana, retención urinaria.     

PROSTATITIS CRÓNICA BACTERIANA  CLÍNICA  Muchos asintomáticos, antecedente de IVU.    Próstata    blanda, indurada, molestia o dolor sordo mal localizado en periné suprapúbico, síntomas irritativos.      PARACLÍNICOS  EGO normal, a excepción de cistitis secundaria. Secreción prostática con aumento en número de leucocitos (>10/HPF), especialmente  macrófagos.        El cultivo de secreciones prostáticas u orina post masaje es necesario para el diagnóstico.      DIAGNÓSTICOS DIFERENCIALES  Uretritis crónica, cistitis, enf anales.    TRATAMIENTO  AB de adecuada penetración a próstata (TMP/SMX, quinolonas o efalexina, eritromicina) por 6‐12 semanas, se puede dar tx sintomático  con antiinflamatorios y baños de asiento.   

 

PROSTATITIS NO BACTERIANA  ETIOLOGÍA  Es la más común de los sx prostáticos, etiología desconocida.    Pudiera obedecer a Clamydia, micoplasmas, ureaplasmas, virus. Enf autoinmune, inflamación no infecciosa. Diagnóstico    de exclusión.      No antecedente de IVU.    CLÍNICA  Se usa NIH‐CPSI para valorar los síntomas  Síntomas irritativos, molestia perineal o suprapúbica.    PARACLÍNICOS  Aumento de número de leucocitos en secreciones prostáticas, cultivos negativos.      DIFERENCIALES  Prostatitis crónica bacteriana    TRATAMIENTO  Ensayo de tx para atípicos con eritromicina.    Tratamiento sintomático y baños de asiento.     

PROSTATODINIA  Dolor prostático sin inflamación, jóvenes y mediana edad, etiología diversa como disfuncción miccional y disfunción de los músculos  pélvicos.    Síntomas igual que prostatitis crónica. Puede haber intermitentecia o dificultad para orinar.      EF normal o molesto al examen  EGO nrmal, secreciones prostáticas normales.    Tratamiento: alfabloqueadores, ejercicios del piso pélvico, baños de asiento.      MASCULINO DE 60 AÑOS DE EDAD, ANTECEDENTE DE DIABETES MELLITUS DESDE HACE    5 AÑOS EN TRATAMIENTO CON  METFORMINA; HIPERPLASIA PROSTÁTICA BENIGNA EN TRATAMIENTO CON TAMSULOSINA DESDE HACE 2 AÑOS. HACE 4 MESES TUVO  INFECCIÓN DE VÍAS URINARIAS TRATADA EMPÍRICAMNETE CON ÁCIDO NILIDÍXICO/FENAZOPIRIDINA CON LO CUAL CEDIERON LAS  MOLESTIAS.    INICIÓN HACE 3 DÍAS, CON DISURIA LEVE Y EXACERBACIÓN DE LOS SÍNTOMAS COMO PUJO, TENEMOS, FRECUENCIA E INTERMITENCIA  URINARIAS.    AL DÍA SIGUIENTE PRESENTÓ UN PICO DE FIEBRE DE 39° Y AYER DE FORMA    PERSISTENTE QUE CEDÍA TEMPORALMENTE  A LA ADMINISTRACIÓN DE PARECETAMOL.    A LA EXPLORACIÓN: FC 96, FR 20, T 38.5, TA 110/70, DIAFORESIS, PALIDEZ DE TEGUMENTOS, ABDOMEN DOLORSO A LA PALPACIÓN  PROFUNDA EN HIPOGASTRIO, TESTÍCULOS Y PENE SIN ALTERACIONES, DOLOR INTENSO A LA REALIZACIÓN DEL TACTO RECTAL, NO SE  PALPAN SONAS DE FLUCTUACIÓN, HIPERTÉRMICO. BIOMETRÍA HEMÁTICA LEUC 18MIL, NEU 80%, HB 15, HTO 45, PLAQ 300, CREAT 1.1,  GLUC 105, EGO 20, ERI30 BACT ABUNDANTES, NITRITOS POSITIVOS.      POR EL CUADRO CLÍNICO ACTUAL, LA SOSPECHA DIAGNÓSTICA ES:    A) PROSTATITIS AGUDA  B) PROSTATITIS CRÓNICA BACTERIANA  C) CISTITIS AGUDA COMPLICADA  D) URETRITIS    EL TRATAMIENTO A ELEGIR ES:  A) COLOACIÓN DE SONDA FOLEY A DERIVACIÓN  B) OBTENER CULTIVO DE ORINA E INICIAR TRATAMIENTO DE AMPLIO ESPECTRO INTRAVENOSO  C) RESECCIÓN TRANSURETRAL DE PRÓSTATA  D) OBTENER PRUEBA DE STAMEY‐MEARES (4MUESTRAS)    La prueba de Stamey es solo para localizar la infección en caso de sospecha de prostatitis crónica.      TRES DÍAS DESPUÉS NO SE OBSERVA MEJORÍA CON EL TRATAMIENTO PREVIO, USTED DECIDE:  A) TOMOGRAFÍA COMPUTADA DE ABDOMEN Y PELVIS  B) CULTIVO DE SECRECIONES PROSTÁTICAS  C) CULTIVOS DE MICRORGANISMOS ATÍPICOS EN SANGRE Y ORINA  D) CISTOSTOMÍA SUPRAPÚBICA    Se realiza ante la sospecha de abseso prostático, otro estudio adecuado es el USG transrectal.     

SEGÚN LO ANTERIOR, USTED ESPERA ENCONTRAR:  A) DIVERTÍCULO VESICAL GIGANTE CON AUMENTO DE ECOGENICIDAD EN SU INTERIOR  B) ASILAMIENTO DE MICOORGANISMOS ATÍPICOS  C) GRANGRENA DE FOURNIER  D) ABSCESO PROSÁTICO    EL TRAMIENTO A ELEGIR ES:    A) DIVERTICULECTOMÍA TRANSVESIAL  B) ANTIBIÓTICO DIRIGIDO A MICROORGANISMO AISLADOS  C) DRENAJE POR PUNCIÓN TRANSRECTAL  D) DEBRIDACIÓN EXTENSA DE TEJIDO NECÓTICO Y ANTIBIÓTICOS.    El drenaje del absceso se puede transuretral o transrectal guido por USG  RESPUESTA A, B, A, D, C 

  EPIDIDIMITIS AGUDA  ETIOLOGÍA  Infecciosa vs no infeccionsa  >80% es bacteriana  6 semanas)    Menores de 40 años==> Chlamydia o N. honorrhoeae  Adultos mayores ==> IVU y prostatitis (gram negativos).  Epidimitis crónica infecciosa: Micobacteriua, Brucella.      PATOGENIA  Flujo retrógrado de orina a conductos eyaculatorios. Desde vejiga, uretra o próstata.    FR: instrumentación de vía urinaria, obstrucción al flujo de salida, anormalidades genitourinarias.    En niños pensar anomalías  genitourinarias o etiolgía viral.      CLÍNICA  Dolor y aumento de tamañ del epidídimo, disminuye con la elevación escrotal (Prehn positivo).  Acompaña dolor en conducto deferente.  Fiebre, síntomas de almacenamiento (cistitis) o de uretritis son comunes.    Si avanza, la infección puede afectar al testículo (orquiepididimitis).    PARACLÍNICOS  Leucocitosis, EGO (piuria, bacteriuria, hematuria)., cultivo de secreción uretral y cultivo de orina.      DIAGNÓSTICOS DIFERENCIALES  Tumos testicular, torsión testicular (USG con duda diagnóstica).    TRATAMIENTO  Antibióticos 2‐4 semanas (en caso de ITS tratar a la pareja sexual)  Ceftriaxona, doxiciclina y levofloxacino.  Sintomático: analgésicos, antiinflamatorios, reposo y elevación escrotal.    Complicaciones: absceso, orquiepididimitis    GPC recomienda ceftria o azitro + doxi si tienen prácticas sexuales, 2 da opción levo.    Si no tienen prácticas sexuales:    TMP/SMX o levo.  Si cateterismo o instrumentación: TMP/SMX o ciproflox.      Respecto a epididimitis. ¿Cuál es el manejo de las complicaciones (orquiepididimitis y absceso)?    ¿Que tan frecuentes son?    Epididimitis >6 semanas puede hacer abscesos, casi no vemos epididimitis aisladas.      MASCULINO DE 25 AÑOS DE EDAD, IVSA 18 AÑOS DE EDAD, 8 PS, USO IRRGULAR DE MÉTODO DE BARRERA, VIDA SEXUAL ACTIVA.  QUIRÚRGICOS: ORQUIDOPEXIA DERECHA AL AÑO DE EDAD POR CRIPTORQUIDIA. REALIZA ACTIVIDAD FÍSICA 3 VECES A LA SEMANA.  HACE 5 DÍAS INICIÓN CON DOLOR CON TESTÍCULO DERECHO, DESDE HACE DOS DÍAS NOTÓ AUMENTO DE VOLUMEN IPSILATERAL, SE  OBSERVA CON COLORACIÓN ERITEMATOSA. TESTÍCULO CONTRALATERAL NORMAL. TEMPERATURA 37.5, DOLOR A LA PALPACIÓN DEL  TESTÍCULO DERECHO, AUMENTADO DOS A TRES VECES EL VOLUMEN DEL TESTÍCULO IZQUIRDO, AUMNTA AL ESTAR DE PIE, CEDE  LEVEMENTE AL SOSTENER EL TESTÍCULO.   

EL DIAGN´STICO DE SU ELECCIÓN ES:  A) TORSIÓN TESTICULAR,  B) HIDROCELE  C) TUMOR TESTICULAR  D) ORQUIEPIDIDIMITIS    LA MANIOBRA DE ELEVAR EL TESTÍCULO Y DISMINUCIÓN SUBSECUENTE DEL DOLOR SUGIERE:  A) ORQUIEPIDIDIMITIS  B) TORSIÓN TESTICULAR  C) HIDROCELE  D) TUMOR TESTICULAR    AL CONFIRMAR SU DIAGNÓSTICO, USTED DECIDE:    A) DESTORSIÓN TESTICULAR QUIRÚRGICA Y ORQUIDOPEXIA  B) TRATAMIENTO ANTIBIÓTICO  C) HIDROCELECTOMÍA  D) ORQUIECTOMÍA RADICAL    LA ETIOLOGÍA MÁS PROBABLE DE ESTA PATOLOGÍA ES:  A) INFECCIÓN RETRÓGRADA DE EPIDÍDIMO Y TESTÍCULO POR BACTERIAS, PROBABLEMENTE DE TRANSMISIÓN SEXUAL.  B) ROTACIÓN DEL TESTÍCULO SOBRE SU EJE LONGITUDINAL POR IMPLANTACIÓN ANÓMALA DE LA TÚNICA VAGINALIS  C) AUMENTO PROGRESIVO DE LÍQUIDO ENTRE LAS CAPAS DE LA VAGINALIS.  D) TESTÍCULO NO DESCENDIDO FUE EL FACTOR PREDISPONENTE DE NEOPLASIA INTRATUBULAR Y DESARROLLO DE SEMINOMA.      RESPUESTA D, A, B, A    Webinars  Ca de próstata  afección periférica (por eso casi no da síntomas)  HPB zona de transición (está cerca de la uretra por eso da síntomas)  La velocidad de crecimiento del antígeno no es indicativo de biopsia, tiene mayor peso si eleva >4.    Energía monopolar no usas NaCl, de hecho sería un factor de riesgo para Sx postRTUP  Sx postRTUP : bardiardia, convulsiones, náuseas, hiponatremia.  ____  Ca de próstata RT si hay mets a hueso y compresión medular.    Si ya se salió de la próstata no se opera.  La próstata no se radia.  ________________  12 años orquiectomía  Si antes menor riesgo malignidad o hipofertilidad.    ______________________________________‐    Mejor studio para litiasis TAC simple.  Litos renales >2cm cirugía (nefrolitotomía percutánea), 7mm casi imposible que pase por uretera, romperá la piedra o catéter doble J.      Cuando no hacerle algo a la piedra? Dolor controlable, función renal normal, lito unilateral 60  17% 

>70  75%    Cirugía prostática puede generar lesión de nervios cavernosos, mala oxigenación de cuerpos cavernosos, insuficiencia vascular.    DIAGNÓSTICO  Prueba de tumescencia y rigidez peneana nocturna mediante Rigiscan, estudios vasculares (inyección intracavrnsa de fármacos  vasoactivos, eco doppler de arterias cavernosas, carvernosografía/carvenosometría con infusión, arteriografia de art pudencia interna),  estudios neurológicos (latencia del reflejo bulbocavernoso, estudios de conducción nerviosa), estudios endocrinológicos.    Prueba de tumescencia normal: 60% de rigidez, >10 min.  USG doppler normal: flujo sistólico pico >30cm/s y un índice de resistencia >0.8 (si es normal no son necesarios más estudios).    Si USG anormal==>arteriografia y prueba de cavernosografia/cavernosmetria si candidato a qx vascular.    TRATAMIENTO  Pequeños cambios de vida pueden solucionar el problema.  Si causa hormonal aplicar testosterona, contraindicada si antecedente de Ca de próstata o síntomas de prostatismo.    Tx de primera línea: inhibidores de 5‐PDE aumentan los efects de NO.  Sildenafil (viagra): comienza entre 30‐60min, comida con abundante grasa puede reducir absorción.    La eficacia puede durar hasta 12  hrs.    Vardenafil (levitra): hace efecto a los 30min, grasas reduce su efecto, dosis de 5‐10‐20mg.    Tadalafil (cialis): efecto a partir de los 30min, eficacia máxima a las 2hrs, eficacia mantenida durante máximo 36hrs. Los alimentos NO  afectan su eficacia, dosis de 10 ‐20mg.    Vardenafil y sildenafil asociarn visión borrosa.  Nitratos contraindicados con todos. Si un paciente desarrolla angina mientras usa PDE5 puede usar otros antianginosos o o esperar 24  hrs si tomó sildenafilo o vardenafilo, 48 con tadalafi. El perfil de efectos adversos no empeora nisiquiera si toma múltiples  anithirtensivos.      Vardenafilo + alfabloqueante NO.  Sildenafil + alfabloqueante: solo 4 hs despúes del alfa  Tadalafilo + alfabloqueante NO, solo si el alfa es tamsulosina.      Dispositivo de constricción al vacío: Adecuado en ancianos.    Tx de segunda línea: Alprostadil intracavernoso  Tx de tercera línea: prótesis peneana.      Testosteronaa, pedir LH, FSH, proalctina, estróngenos.    MASCULINO DE 55 AÑOS DE EDAD, DM E HIPERCOLESTEROLEMIA DE RECIENTE DIAGNÓSTICO. TABAQUISMO POSITIVO DESDE LOS 18  AÑOS (2 CIGARRILLOS/DÍA), CIRUGÍAS NEGADAS. TATAMIENTOS: METFORMINA Y PRAVASTATINA (APEGO IRREGULAR)  ACUDE POR DISMINUCIÓN DE FIRMEZA DE LAS ERECCIONES Y DETUMESCENCIA TEMPRANA, DOS AÑOS DE EVOLUCIÓN, AUMENTANDO  DE FORMA PROGRESIVA, EN MÁS DE LA MITAD DE LAS OCASIONES NO PERMITE LA PENETRACIÓN. INTERROGATORIO DIRIGIDO  TAMBIÉN NOTA PÉRDIDA DE ERECCIONES NOCTURNA Y DEL LIBIDO.    EXPLORACIÓN GENITAL NORMAL. PRÓSTATA DE 25 GR, SIN NÓDULOS.      ESTUDIO INICIAL DE ABORDAJE:  A) PERFI HORMONAL, LÍPIDOS, GLUCOSA.  B) USG DOPPLER PENEANO  C) RMN PELVIS  D) TESTOSTERONA, HORMONAS TIROIDEAS.    EL ESTUDIO QUE PIDIÓ NO ENCONTROL ALTERACIONES Y DECIDE INICIAR CON TRATAMIENTO MÉDICO, LA PRIMERA ELECCIÓN ES:  A) TESTOSTERONA  B) INHIBIDORES DE 5 ALFA REDUCTASA  C) INHIBIDORES DE FOSFODIESTERASA 5  D) ALFA BLOQUEADORES    EJEMPLOS DE FÁRMACOS:  A) VARDENAFIL, ALPROSTADIL  B) TADALAFIL, VARDENAFIL  C) ALPROSTADIL, PAPAVERINA 

D) FENTOLAMINA, DOXAZOSINA    MECANISMO ACCIÓN:  A) BLOQUEA DEGRADACIÓN DE GMPc Y RELAJACIÓN DE MÚSCULO LISO CAVERNOSO  B) BLOQUEO DE RECEPTORES ALFA, VASODILATACIÓN Y AUMENTO DE FLUJO SANGUÍNEO CAVERNOSO.  C) ACTIVA LOS RECEPTORES ALFA, CONSTRICCIÓN DE LOS VASOS SANGUÍNEOS D ELOS CUERPOS CAVERNOSOS.    D) INHIBE LA DEGRADACIÓN DE LA DIHIDROTESTOSTERONA.    CONTRAINDICACIONES  A) USO SIMULTÁNEO DE BETA BLOQUEADORES, CRISIS ASMÁTICA  B) CÁNCER DE PRÓSTATA  C) HIPOTENSIÓN ORTOSTÁTICA, BETABLOQUEADORES, DM INSULINO DEPENDIENTE  D) USO DE NITRATOS, ANGINA INTETABLE, INFARTO AGUDO AL MIOCARDIO RECIENTE.    RESPUESTAS A, C, B, A, D   

TORSION TESTICULAR  DEFINICIÓN  Torisón de cordón espermático y pérdida de flujo sanguíneo. Más frecuente en adolescentes y neonatos.    DIAGNÓSTICO  Duración menos a 6 hrs hace posible el salvamiento del testículo.    Torsión extravaginal constituye 5% (70% es prenatal), asocia alto peso al nacimiento. La torsion intravaginal se observa normalemente  en menores de 30 años.      Tiempo de evolución  Salvamiento de testículo  24hrs    0‐10%    Complicaciones: infarto testicular, pérdida de testículo, infeccion, malignidad.      CLÍNICA  Comienzo súbito (ej después del ejercicio, trauma o espontáneo=).  33% náusea y vómito.    Ausencia de reflejo cremastérico, testículo en posición anormal,  En neonatos el testículo duro, no doloroso, fijo y con cambios de coloración en la piel.    Puede haber hdirocele reactivo o edema escrotal. Testículo horizontal, Prehn negativo‐, eritema es escroto.    DIAGNÓSICOS DIFERENCIALES  Torsión de apéndice testicular o de epidídimo, epididitimisi, orquitis, idrocele, turmos, edema escrotal idiopático, truama.      PARACLINICOS  USG cuando existen una sospecha baja de torsión.  USG Doppler: ausensi de flujo sanguineo, disminución de la velocidad de flujo en arterias intratesticulares, aumento en los índices de  resistencia.    S y E 90‐100%    TRATAMIENTO  Exploración quirúrgica, está indicada de forma inmadiata. Si fracaso orquiectomía.   Destorsión quirúrgica manual, orquidopexia.    MASCULINO DE 25 AÑOS DE EDAD, PREVIAMENTE SANO, INICIÓ CON DOLOR TESTICULAR IZQUIERDO INTENSO HACE 4 HRS DE FORMA  SÚBITA, SIN ASOCIACION A TRAUMATISMO, NO CEDIÓ CON ANALGÉSICOS NI DISMINUE CON LOS CAMBIOS DE POSICIÓNM  IRRADIACIÓN A REGIÓN SUPRAPÚBICA, NO TIENE SÍNTOMAS URINARIOS, NO HAY SECRECIÓN URETRAL.  TEMP 37, FC 80, FR 18, IMC 20, ABDOMEN SIN DATOS PATOLÓGICOS, NO HAY ALTERACIONES EN PENE, TESTÍCULO IZQUIERDO  ELEVADO, SIN AUMENTO DE VOLUMENM NO SE PALPAL ADENOPATÍAS INGUINALES, SIGNO DE PREHN NEGATIVO, REFLEJO  CREMASTÉRICO ABOLIDO.    SOSPECHA DIAGNÓSTICA:  A) TORSIÓN TESTICULAR  B) ORQUIEPIDIDIMITIS  C) VARICOCELE IZQUIERDO  D) TUMOR TESTICULAR    LE SOLICITA UN ULTRASONIDO TESTICULAR PARA CORROBORAR SU DIAGNÓSTICO, USTED ESPERA ENCONTRAR:   

A) B) C) D)

AUSENCIA DE FLUJO SANGUÍNEO TESTICULAR  AUMENTO DE FLUJO SANGUÍNEO TESTICULAR, EPIDÍDIMO ENGROSADO  PLEXO PAMPINIFORME DIALTADO >3MM  TUMORACIÓN INTRATESTICULAR CON AUMENTO DE VASCULARIDAD Y ÁRES QUÍSTICAS. 

  TRATAMIENTO DE ELECCIÓN:  A) DESTORSIÓN MANUAL INMEDIATA  B) DESTORSIÓN QUIRÚRGICA INMEDIATA Y ORQUIDOPEXIA BILATERAL  C) ANALGÉSICOS Y VARICOCELECTOMÍA IZQUIERDA  D) ORQUIECTOMÍA    La orquidopexia bilateral se hace para prevenir futuros cuadros.    FISIOPATOLOGÍA DEL TRASTORNO:  A) TORSIÓN INTRAVAGINAL DEL TESTÍCULO POR IMPLANTACIÓN ALTA DE LA VAGINALIS, EL EJE DEL CORDÓN ESPERMÁTICO SE  TUERCE E ISQUEMIA SUBSECUENTE.    B) INFECCIÓN DEL EPIDÍDIMO Y POSTERIOMRENTE DEL TESTÍCULO DE FORMA RETRÓGRADA Y PROBABLMENTE POR  GÉRMENES DE TRANSMISIÓN SEXUAL.  C) FLUJO DE VENA GONADAL IZQUIERDA HACIA LA VENA RENAL IZQUIERDA EN ÁNGULO RECTO, MALFUNCIONAMIENTO DE  VÁLVULAS VENOSAS Y CONGESTIÓN VASCULAR  D) DISPLASIA INTRATUBULAR QUE PROGRESIÓ AL DESARROLLO DE TUMOR TESTICULAR    RESPUESTA A, A, B, A   

CRIPTORQUIDIA  DEFINICIÓN  Ausencia de testículos en escroto por alteración en el descenso. Alrededor de 85% es unilateral, frecuentemente es el derecho. Por lo  general se encuentra en el canal inguinal    ETIOLOGÍA  Multifactorial. Fr prematuros, bajo peso, talla baja, exposición intrauterina a estrógenos. Peso al ancer es el principal determinante  asociado.    En niños a término el descenso de puede completar durante los primeros 6 meses de vida, en prematuros este descenso se peude  retrasar hasta los 12 meses.    Puede ser espontánea y aislada, asociar AHG o otras anomalías (tumor de Willms, Prader Willi, Beckwith‐Wiederman, Klinefelter, alt de  cromosoma Y).      PARACLÍNICOS  Solo en caso de criptorquidia bialteral, ambigüedad genital.  cariotipo, gonadotropinas y testosterona basales.  Laparoscopia útil para visualizar testículo no palpable, sirve para Dx y Tx.      TRATAMIENTO  Hormonal: Hcg (éxito de 6‐21), GnRH (éxito 6‐38)  Cirugía: éxito 70‐90%  Orquidopexia es la de primera elección.        Si el paciente tiene >32 con testículo no descendido observamos porque ya pasó lo crítico.    Si tiene 65 años,historia de hemorragia, alt de la coagulación, uso de anticoagulantes.  Mecanismos de TCE peligroso: caída >1metro o cinco escalones, carga axial, atropellamiento por vehículo de motor en movimiento,  colisión entre vehículos a >100km/h, accidente automovilístico con vuelco, salir despedido de un vehículo.    Signos de alarma prehospitalaria:    [MAS CAAFI VIP]  Múltiples fracturas  Alt pupilares,    Sat O2  UCI  Rx de cráneo: contusión o laceración de piel cabelluda, profundida de la herida hasta hueso, longitud de la herida >5cm.    TCE + dolor o rigidez cervical + >65 años o mecanismo traumático peligroso: AP, lateral y trans‐oral.  Si FR + signo de alarma: TAC  Si alt de edo alerta: glucosa sérica    Criterios de alta TCE no complicado: ausencia de datos de alarma luego de 24 hrs de observación, TAC normal, familiar que puede  acompañar al pacinete.      TRATAMIENTO  Si hiperventila y hace PCO2 de 25 tiene mal pronóstico a 3‐6 meses en comparación con el que tiene 35mmHg.    Indicación para intubación antes del traslado:      deterioro consciencia,    fractura bilateral mandibular,      convulsiones,      signos de fractura de base del cráneo.    Inmovilización cervical en TCE:      Glasgow 65    Hiprcapnia >45    Hiperventilación espontánea (PCO2 30cc, grosor >15mm, desviación de línea media >5mm, localización temporal, compresión  de cisternas mesencefálicas    B) Hematoma subdural: grosor >10mm, desviación de línea media >5mm, diferencia de grosor y desviación >5mm, compresión de  cisternas mesencefálicas    No se ha observado ningún efecto benéfico de esteroides en el control de la PIC en pacientes con TCE grave.      Motora      Verbal      Ocular        Orden    6  Normal    5  Espontánea  4    Localiza dolor  5  Desorientada  4  Orden    3  Retira al dolor  4  Cambiar palabras  3  Dolor    2  Decorticación  3  Incompresible  2  No los abre  1  Descerebración  2  No habla    1  Sin repsuesta  1    Se considera que está en coma cuando la sume es 8 o menos.    TCE leve  14‐15  TCE mod  9‐13  TCE sev  8 o menos   Hay hipoxemia requiere O2 rápido y corregir hipotensión,        HOMBRE DE 33 AÑOS DE EDAD, QUE ES ASALTADO AFUERA DE SU DOMICILIO, RECIBIENDO GOLPE CONTUSO CON TUBO EN CRÁNEO. A  SU LLEGADA AL HOSPITAL USTED LO ENCUENTRA DESPIERTO, CON GLASGOW QUE FLUCTÚA ENTRE 14 Y 15. DURANTE LA  EXPLORACIÓN USTED PALPA UNA FRACTURA HUNDIDA DE CRÁNEO.    EL SIGUIENTE HALLAZGO DURANTE EL SEGUIMIENTO Y EVALUACIÓN DEL PACIENTE, JUSTIFICARÍA LA INDICACIÓN DE  HIPERVENTILACIÓN:      A) APARICIÓN DE PARESIAS Y PARESTESIAS.  B) DETERIORO NEUROLÓGICO AGUDO.  C) INTEGRACIÓN DE LA TRIADA DE CUSHING.  D) PREVENCIÓN DE CRISIS CONVULSIVAS SECUNDARIAS.    RESPUESTA  La presencia de parestesias o hemiparesia, junto con dilatación pupilar o pérdida del estado de alerta forman parte del deterioro  neruológico agudo.   

Triada de Cushing es hipertesión, bradicardia y respiración irregular. Es una respuesta del SNC a la hipertensión intracraneana. Son  indicadores de deterioro neurológico.    La HIC puede manifestarse por: cefalea, vómito, edema de papila, paresia del recto externo por compesión del VI par craneal o  abducens    Se prefiere mantener normocarbia en la mayoría de los pacientes (35‐45mmHg).    La hiperventilación deberá ser utilizada solo en "deterioro neurológico agudo" puesto que se hay que recordar que la reducción de  PaCO2 genera vasoconstricción y puede producir isquemia especialmente si está debajo de 30mmHg.  Es aceptable una PaCO2 ebtre 25‐30mmHg por periodos breves para tratar un deterioro neuroógico agudo. El efecto de la  hiperventilación es transitorio (menos de 6 hrs)      [r] HOMBRE DE 18 AÑOS DE EDAD, QUE ES LLEVADO POR SU MADRE AL SERVICIO DE URGENCIAS AL PRESENTAR HERIDA SANGRANTE  EN REGIÓN PARIETAL DERECHA, SECUNDARIA A TRAUMATISMO CRANEOENCEFÁLICO CON UN PALO DURANTE UNA RIÑA. DURANTE LA  EXPLORACIÓN NEUROLÓGICA RÁPIDA USTED LO ENCUENTRA CON GLASGOW DE 15.  LA INDICACIÓN DE UNA TAC DE CRÁNEO EN ESTE PACIENTE SE JUSTIFICARÁ CON LA IDENTIFICACIÓN DEL SIGUIENTE HALLAZGO:      A) HERIDA MAYOR DE 5CM.  B) POLITRAUMATIZADO.  C) HIPOTENSIÓN SEVERA.  D) AMNESIA.    RESPUESTA  Criterios para TAC en TCE  Pérdida del edo de alerta > 5min  Amnesia  Cefalea grave  Glasgow 18mm,  contusión cerebral, lesiones extracraneantes agregadas.  DLM  Rx de columna  cervical.  2) Buscar factores de bajo riesgo: simple colisión, estar sentado mucho tiempo, caminar todo el tiempo, el dolor se retrasó, estado  ambulatorio, aunsencia de relajación de columan cervical. ==> si puede rotar 45% el cuello a los costados no precisa rx    TRATAMIENTO  Fase I: Reducir dolor, disminución de cargas   1‐4d  Fase II: Incrementar actividad, aumentar cargas  4‐21d  Fase III: Aumentar actividad, más carga    3s‐6s  Fase IV: Capacidad funcional total    6s‐3m    GradoI/II:    naproxeno 250 c 12 + Paracetamol 500 1‐2 c/8  Piroxicam 20mg cada 24 + paracetamol 500 1‐2 c/8  Diclofenaco tab.100mg c 24 + paracemol 1‐2 c /8  Ranitidiina 150mg c 12 si enf ác péptica.    Frío local primeras 72hrs  Calor superficial después de 72hrs  Evitar uso de collarin en Grado I  Evitar reposo absoluto  Evitar cirugía salvo compicaciones grado IV  Evitar almohadas "cervicales"  Evitar inyecciones intratecales    Grado III o IV:    Referencia   

   

HOMBRE DE 53 AÑOS DE EDAD, DEDICADO ACTUALMENTE AL CUIDADO DE ANCIANOS Y NIÑOS ENFERMOS. DURANTE SUS VACACIONES  Y  TRAS  PRACTICAR  BUCEO  PRESENTA  DOLOR  EN  CUELLO  DE  TIPO  INCAPACITANTE.  A  SU  LLEGADA  AL  SERVICIO  DE  URGENCIAS  SE  SOLICITAN RADIOGRAFÍAS EN 2 PROYECCIONES DE CUELLO.    EL FACTOR DE RIESGO MÁS IMPORTANTE PARA PRESENTAR LESIÓN CERVICAL EN ESTE PACIENTE ES:  A) SER HOMBRE.  B) BUCEAR.  C) LA EDAD.  D) SER CUIDADOR DE PERSONAS.    RESPUESTAS    B           

TRAUMA DE TÓRAX  TRAUMA DE TÓRAX    DIAGNÓSTICO  Rx torax AP y lateral, cervical lateral, pelvis.  TAC si dudas y estabilidad hemodinámica.    Ecocardiograma trasnesofágico: para dx de lesión aórtica  Ecocardiograma transtorácico: taponamiento cardiaco    Neumotórax abierto:  Herida abierta hace efecto válvula y provoca neumo a tensión si es progresivo, simple si no progresa.  Clínica: dolor, disnea, hiperresonancia a la percusión, hipoventilación. Si supera 20 del volumen pulmonar==> clapso  Tx. cerrar orificio de entrada con 3/4 bordes (deja salir aire pero no entrar). Tubo de drenaje en sitio diferente. Revisión y reperación  qx.    Neumotórax a tensión:    Fisiopatología: esplazamiento de mediastino a contralateral (compromiso bipulmonar), compromiso de retorno venoso (IRA y shock).    Clínica: distención de venas en cuello ( salvo que haya hipovolemia), hipoventilación uni o bilateral, hiperresonancia a la percusión del  lado afectado.    Tx: inserción de cateter 14 en 2do pescio intercostal, línea MC. Rx de control si todo bien==> tubo de tórax en 4‐5 espacio IC línea AM.    Hemotórax masivo:    >1500ml en espacio pleural, frec en trauma abierto  Clínica: Shock, insuf respiratoria, hipoventilación, matidez a percusión, distensión de venas de cuello pese a hipovolemia.    Tx: Reposiciion de volumenes (cristalides, coloides, sangre), drenaje torácico con tubo 32F en 4‐5 espacio IC LMA.  Toracotomía si >1500ml en los primeros momentos, especiamente si paciente inestable o si 200‐400ml/h en las primeras 4 hrs.    PRONÓSTICO    Mortalidad 25‐30%    CONTENIDO ADICIONAL  Cristaloides: sol fisiológica (salino al 0.9%), ringer‐lactato, sol salina hiptónica, sol salina hipertónica, sol glucosalina, solución dextrosa o  glucosa a 5‐10%.  Coloides: albúmina, gelatina y almidones.        TAPONAMIENTO  NEUMOTÓRAX  NEMOTÓRAX A  HEMOTÓRAX  CARDIACO  ABIERTO  TENSIÓN  SÍNTOMAS  Triada: hipotensión + pulso  Dolor, disnea  Disnea  Insuficiencia respiratoria  paradógico + ingurgitación  yugular.  ¿?  Triada de Beck:  hipotensión + ingurgitación  yugular + velamiento de  ruidos cardiacos.  SIGNOS  Ruidos cardiacos  Hiperresonancia,  Ingurgitación yugular,  Shock, hipoventilación,  apagados, rocepericárdico.  hipoventilación (si  hiperresonancia (si  matidez, ingurgitación yugular  grave  grave colapso),  pese a hipovolemia.  colapso),enfisema  hipoventilación, 

TRATAMIENTO 

Punción subxifoidea  Intubar empeora gasto  cardiaco   

subcutáneo?  Parche 3/4 + tubo de  drenaje + revisión  reparación. 

enfisema subcutráneo  Cateter #14 en 2do  espacio IC LMC. + Rx +  tubo en 4‐5 espacio IC  LMA 

Reposición de volumen +  drenaje torácico tubo 32F en 4‐ 5 espacio IC LMA  Si >1500ml en primer  momento o 200‐00ml/hr en  primeras 4 hrs==>  toracotomía. 

    *el aire del enfisema subcutáneo pude ir del medio ambiente a los tejidos, o de los pulmones a los tejidos.        [PONER ALGORITMO DE GPC TRAUMA DE TÓRAX]    Claro pulmonar o Resonancia normal. Normalmente es un sonido de intensidad fuerte, de tono bajo y de duración prolongada.    Timpanismo. Sonido que se obtiene cuando se percute sobre un órgano que contiene aire como el estómago e intestino.    Es más  resonante que el sonido claro pulmonar, y su intensidad, tono y duración son variables, su característica acústica es la de ser musical y  rico en sobretonos.    Hiperresonancia o hiperclaridad.Se obtiene cuando se percuten zonas cuyo contenido de aire está aumentado, por ejemplo cuando se  aumenta la entrada del aire al pulmón en el caso patológico del neumotórax; el sonido se caracteriza por ser más fuerte, más grave y  más prolongado que el sonido claro pulmonar, pero desprovisto de carácter musical.    Sonido  mate.Se  obtiene  cuando  se  percuten  órganos  sin  aire,  músculos  y  vísceras  macizas  como  hígado  y  bazo.  Es  un  sonido  débil,  apagado, de tonalidad alta y duración breve. El sonido mate es la completa y absoluta ausencia de resonancia y se oye con dificultad y  sólo a cierta distancia.    Sonido submate. Es el sonido que se obtiene cuando se percute un órgano macizo que se encuentra cubierto parcialmente con aire,  como sucede con el hígado cubierto por el borde inferior del pulmón —se escucha al golpear aproximadamente entre el quinto y sexto  espacio intercostal derecho—. Su tono es menos elevado que la matidez.      [TRAUMA TORÁCICO]  MUJER DE 24 AÑOS DE EDAD ES LLEVADA AL SERVICIO DE URGENCIAS POR SUFRIR ACCIDENTE AUTOMOIVILÍSTICO EN CARRETERA, NO  TRAÍA CINTURÓN DE SEGURIDAD, SALE EXPULSADA DEL AUTOMÓVIL. CON PÉRDIDA MOMENTÁNEA DEL CONOCIMIENTO, A SU LLEGADA.  REFIERE DOLOR TORÁCICO Y ESTÁ DISNEICA. SE COLOCAN PUNTAS NASALES Y SE ESTABILIZA LA COLUMNA CERVICAL. RESPIRACIÓN Y  VENTILACIÓN, CON AUSENCIA DE RUIDOS RESPIRATORIOS EN HEMITÓRAX IZQUIERDO, HIPERRESONANCIA A LA PERCUSIÓN DEL MISMO.  NO HAY HEMORRAGFIAS EVIDENTES. SE ENCUENTRA CONFUSA. SE EXPONE POR COMPLETO A LA PACIENTE. TA 60/40, FC138, FR 29.  SIMULTÁNEMANETE A LA VALORACIÓN PRIMARIA . ¿CUÁL ES EL PROCEDIMIENTO DE MAYOR IMPORTANCIA QUE SE LE DEBE REALIZAR  A ESTA PACIENTE?  A) RX PÓRTATIL DE TÓRAX  B) TORACOTOMÍA ABIERTA  C)    PUNCIÓN CON AGUJA GRUESA EN SEGUNDO ESPACIO INTERCOSTAL, LÍNEA MEDIA CLAVUCULAR.  D) PUNCIÓN CON AGUJA FINA EN SEGUNDA ESPACIO, A NIVEL COSTOVERTEBRAL    RESPUESTA C  Lesiones  torácicas  graves  que  deben  ser  reconocidas  y  tratadas  durante  la  valoración  primaria.  Incluyen  neumotórax  a  tensión,    neumotórax abierto, tórax intestable y neumotórax masivo.    El neumotórax a tensión va a colapsar el pulmón afectado, desplaza al  mediastino hacia el lado opuesto, reduciendo el retorno venoso y comprimiento el pulmón contralateral. La situación de choque se debe  a disminucion del retorno venoso. Este es un diagnóstico clínico, no radiológico. Por lo tanto no debe demorarse su tratamiento a la  confirmación  radiológica.  este  se  basa  en  descomprimir  con  punción  de  aguja  gruesa  en  el  segundo  espacoi  intercostal,  línea  media  clavicular en el hemitórax afectado.    No olvidar datos pivote de hiperresonancia, ausencia de ruidos respiratorios y TA baja eso es igual a neumotórax a tensión.      MASCULINO DE 27 AÑOS DE EDAD, SUFRE ACCIDENTE AL CHOCAR SU MOTO CONTRA EL MURO DE CONTENCIÓN, A SU LLEGADA AL  HOSPITAL  SE  ENCUENTRA  CON  TEMPERATURA  DE  38°c,  TA  90/40,  FC  130,  FR  24.  SE  ENCUENTRA  DESORIENTADO  CON  PALIDEZ  DE  TEGUMENTOS, AUSENCIA DE RUIDOS RESPIRATORIOS, MATIDEZ A LA PERCUSIÓN, CREPITACIÓN DEL CUARTO AL OCTAVO ARCO COSTAL  EN HEMOtÓRAX DERECHO.  EL DIAGNÓSTICO PRINCIPAL DEL PACIENTE ES:  A) TAPONAMIENTO CARDIACO  B) ROTURA TRUAMÁTICA DE DIAFRAGMA  C) TÓRAX INESTABLE  D) HEMOTÓRAX MASIVO   

EL TRATAMIENTO QUE REQUIERE DE INMEDIATO EL PACIENTE ES CON:  A) TORACOSTOMÍA CON SONDA  B) PLASTIA DIAFRAGMÁTICA CON ABORDAJE ABDOMINAL  C) PERICARDIOCENTESIS  D) SONDA ENDOPLEURAL      RESPUESTA D, A  El neumotórax masivo se pdocuce por la rápida acumulaciónde sangre en el espacio pleural, como ya sabemos su principal causa es la  lesión del hilio pulmonar o de los vasos sistémicas de la reja costa. el paciente se puede presentar con intestabildiad hemodinámica y  ausencia de ventilación en un hemitórax, el cuadro clínico es secundario    a la hipovelamia y a la alteración de V/Q que lleva a shock y  dificultad respiratoria severa. Clíniamente estos pacientes presentan "datos de bajo gasto", con hipotensión, taquicardia, palidez y pulsos  disminuidos, a la EF ese se tacracteriza por la ausencia de murmullo vesicular y matidez a la percusión del tórax yal y como presenta  nuestro apciente. La ausencia de ruidos respiratorios y matdezen torax sustentan el diagnóstico.    El tratamiento del hemotórax consiste en el manejo del shock, descompresión del espacio pleural con tubo de toracotomía, luego de una  rápida estabilización se lleva a cirugía y no solo con sonda endopleural.          FEMENINA DE 32 AÑOS DE EDAD, QUE SUFRE ACCIDENTE AUTOMOVILÍSTICO, RECIBE GOLPE DIRECTO DEL VOLANTE SOBRE EL TÓRAX,  PRESENTA TAQUICARDIA, HIPOTENSIÓN, ESTERTORES BILATERALES Y CIANOSIS IMPORTANTE DE FORMA AGUDA.  EN BASE AL CUADRO CLÍNICO EL DIAGNOSTICO PROBABLE ES:      A) TAMPONADE CARDIACO.  B) CHOQUE HEMORRÁGICO.  C) NEUMÓTORAX A TENSIÓN.  D) CHOQUE NEUROGÉNICO    RESPUESTA D        MASCULINO  DE  24  AÑOS  DE  EDAD,  SUFRE  CAÍDA  DE  UNA  MOTOCICLETA  A  GRAN  VELOCIDAD,  ES  LLEVADO  A  URGENCIAS.  A  LA  EXPLORACIÓN CON TA 90/60, FRECUENCIA CARDIACA 95 POR MINUTO, FRECUENCIA RESPIRATORIA DE 26 POR MINUTO, TEMPERATURA  DE  36.4  °C.  CONCIENTE,  ORIENTADO,  CON  DISNEA  INTENSA  DE  PEQUEÑOS  ESFUERZOS,  INGURGITACIÓN  YUGULAR,  ENFISEMA  SUBCUTÁNEO EN CARA ANTERIOR DEL TÓRAX QUE SE EXTIENDE AL CUELLO Y ABDOMEN DEL LADO DERECHO, AUSENCIA DE RUIDOS  RESPIRATORIOS EN HEMITÓRAX DERECHO E HIPERCLARIDAD A LA PERCUSIÓN.        CON LOS DATOS CLÍNICOS QUE PRESENTA EL PACIENTE, EL DIAGNOSTICO PROBABLE ES:  A) NEUMOTÓRAX A TENSIÓN.  B) HEMOTÓRAX MASIVO.  C) CONTUSIÓN PULMONAR.  D) TAPONAMIENTO CARDIACO.          DE ACUERDO AL DIAGNOSTICO QUE TIENE EL PACIENTE, SE DEBE REALIZAR INMEDIATAMENTE UNA:  A) PUNCIÓN CON AGUJA DE GRUESO CALIBRE.  B) INTUBACIÓN ENDOTRAQUEAL.  C) PERICARDIOCENTESIS.  D) TORACOSTOMÍA CON SONDA.    RESPUESTAS    A, A      A 27‐YEAR‐OLD MALE PRESENTS TO THE EMERGENCY ROOM WITH A STAB WOUNG TO THE LEFT SIDE OF HIS CHEST. HE IS DYSPNEIC AND  COMBATIVE, AND HAS A BLOOD PRESSURE THAT CHANGES WITH RESPIRATION. EXAMINATION REVEALS A DISTENDED JUGULAR VEIN  ON HIS NECK AND DISTANT, MUFFLED HEART SOUNDS. BASED ON THESE CLINICAL FINDINGS, YOU SUSPECT THE PATIENT MAY HAVE  CARDIAC TAMPONADE. WHICH OF THE FOLLOWING IS THE STRONGEST INDICADOR OF CARDIAC TAMPONADE?  A)  HYPOTENSION  B)  PULSUS PARADOXUS  C)  JUGULAR VENOUS DISTENSION  D)  DYSPNEA    RESPUESTA B  Recordar que la triada de Beck de tamponamiento cardíaco es Hiipotensión + ingurgitación yugular + disminución de los ruidos cardiacos.  Sin embargo en la medicina y en el trauma de tórax mismo hay muchas causas de hipotensión y disnea razón por la cual esas respuestas  son  fácilmente  descartables.  Neumotorax  abierto,  hemotórax  también  son  presentaciones  de  trauma  de  tórax  y  ambas  hacen 

ingurgitación yugular. El pulso paradójico si bien no es exclusivo de tamponamiento cardiaco si es más el datos más característico. Otras  causas de pulso paradójico son la embolia pulmonar masiva, choque hemorrágico grave, enfermedad EPOC.      MASCULINO DE 37 AÑOS DE EDAD, CHOFER DE TAXI, SUFRE IMPACTO EN SU AUTO, RECIBIENDO TRAUMATISMO CONTUSO EN TÓRAX.  ES LLEVADO A URGENCIAS. A LA EXPLORACIÓN FÍSICA CON TEMPERATURA DE 36.2°C, TENSIÓN ARTERIAL 90/50, FRECUENCIA CARDIACA  110  X  MINUTO,  FRECUENCIA  RESPIRATORIA  18  X  MINUTO.  SE  ENCUENTRA  CONSCIENTE,  CON  PALIDEZ  DE  TEGUMENTOS,  DOLOR  INTENSO EN CARA ANTERIOR DEL TÓRAX, DIAFORÉTICO, CON INGURGITACIÓN YUGULAR, LACERACIONES EN HEMITÓRAX DERECHO Y  ESTERNÓN. RUIDOS CARDIACOS DISMINUIDOS EN INTENSIDAD    EL DIAGNÓSTICO PROBABLE DEL PACIENTE ES DE  A) DERRAME PERICÁRICO  B) HEMOTÓRAX  C) NEUMOTÓRAX  D) TAPONAMIENTO CARDIACO    EL TRATAMIENTO QUE DEBE RECIBIR EL PACIENTE INMEDIATAMENTE ES:    A) TORACOTOMIA  B) COLOCACIÓN DE SONDA ENDOPLEURAL    C) PERICARDIOCENTESIS  D) URESIS FORZADA    RESPUESTAS D, C       FEMENINA DE 40 AÑOS DE EDAD, SUFRE CAÍDA DESDE UNA ESCALERA DE 2 METROS RECIBIENDO GOLPE DIRECTO EN LA REGIÓN COSTAL,  POR LO  QUE ES LLEVADA A URGENCIAS. A LA EXPLORACIÓN CON TENSIÓN ARTERIAL DE 110/70, FRECUENCIA CARDIACA DE  90 POR  MINUTO,  FRECUENCIA  RESPIRATORIA  DE  40  POR  MIN.  SE  ENCUENTRA  AGITADO,  RESPIRACIONES  RÁPIDAS  Y  SUPERFICIALES,  A  LA  PERCUSIÓN  CLARO  PULMONAR,  RUIDOS  RESPIRATORIOS  PRESENTES  EN  AMBOS  HEMITÓRAX,  TRAQUEA  CENTRAL,  PRESENTA  CREPITACIÓN  EN  REGIÓN  COSTAL  DERECHA.  ASIMETRÍA  DE  LOS  MOVIMIENTOS  RESPIRATORIOS.  SE  LE  ADMINISTRA  OXÍGENO  SUPLEMENTARIO A 10 LTS/MIN. Y SOLUCIONES CRISTALOIDES INTRAVENOSAS. RADIOGRAFÍA DE TÓRAX NO CONCLUYENTE.    ESTA PACIENTE CURSA CON EL DIAGNOSTICO DE:  A) TÓRAX INESTABLE.  B) CONTUSIÓN PULMONAR.  C) HEMOTÓRAX.  D) NEUMOTÓRAX A TENSIÓN.    LA PACIENTE SE DETERIORA A PESAR DEL TRATAMIENTO MÉDICO IMPLEMENTADO; SE TOMA GASOMETRÍA ARTERIAL PRESENTANDO  HIPOXEMIA E HIPERCAPNEA, POR LO QUE LA MEDIDA TERAPÉUTICA DE URGENCIA ES REALIZAR:  A) TORACOSTOMÍA CON SONDA.  B) AUMENTAR EL APORTE DE O2 Y ANALGESIA.  C) DESCOMPRESIÓN CON AGUJA DE GRUESO CALIBRE.  D) INTUBACIÓN ENDOTRAQUEAL CON VENTILACIÓN ASISTIDA.    El tórax inestable se define como la fractura de 3 o 4 costillas en más de dos segmentos de su longitud casionando un fenómen paradójico  en la caja torácica colapsándolo en inspiración y expandiéndolo en espiración.    Es importante reconocer que pude asociarse a otras  condiciones como neumotórax, hemotórax, contusión pulmonar.  El estudio de imagen inicial es Rx portátil de tórax, si hay compromiso multiorgánico ocn gran severidad del trauma se sugiere realizar  tomografía torácica y abdominal simple y con contraste. Aunque realmente el diagnóstico es clínico.    Cuando se lesionan las dos primeras costillas se debe sospehcar lesión de grandes vasos,    y cuando se lesionan las últimas se  debe  descartar lesión diafragmática o intrabdominal..    La paciente presenta hipoemia, hipercapnea como datos de insfueciencia respiratoria.    Los critrios de ventilación mecánico son:    PO2 23    >65 años      PAFI 15% 

  Diferencia alveolo arterial >350      El inicio de ventilación mecánica disminuye 68% la mortalidad hasta el 7%.  RESPUESTA A, D      A 56‐YEAR‐OLD WOMAN WITH A 6‐YEAR HISTORY OF STAGE II CANCER OF THE LEFT BREAST PRESENTED WITH PROGRESSIVE  SHORTNESS OF BREATH AND FATIGUE. PHYSICAL EXAMINATION REVEALED HYPOTENSION, TACHYCARDIA, JUGULAR VENOUS  DISTENTION, PULSUS PARADOXUS, AND DISTANT HEART SOUNDS; THE BLOOD PRESSURE WAS 63/44 MM HG, AND THE HEART RATE  110 BEATS PER MINUTE. RADIOGRAPHY OF THE CHEST PERFORMED WITH THE USE OF PORTABLE EQUIPMENT REVEALED THE WATER‐ BOTTLE SIGN (IN WHICH THE ROUNDED AND ENLARGED CARDIAC SILHOUETTE ON FRONTAL CHEST RADIOGRAPHY MIMICS THE  APPEARANCE OF AN OLD‐FASHIONED WATER BOTTLE), WHICH IS SUGGESTIVE OF PERICARDIAL EFFUSION (PANEL A). TRANSTHORACIC  ECHOCARDIOGRAPHY REVEALED A LARGE PERICARDIAL EFFUSION WITH SWINGING OF THE HEART FREELY IN AN ANTERIOR– POSTERIOR FASHION (SEE VIDEO) AND COLLAPSE OF THE RIGHT AND LEFT ATRIA IN END DIASTOLE, WHICH IS CONSISTENT WITH  CARDIAC TAMPONADE. LEFT VENTRICULAR SYSTOLIC FUNCTION WAS MODERATELY IMPAIRED. SUCH CHANGES IN THE POSITION OF  THE HEART RESULT IN BEAT‐TO‐BEAT VARIATION IN BOTH THE AMPLITUDE AND THE AXIS OF THE QRS COMPLEX (ELECTRICAL  ALTERNANS) SEEN ON ELECTROCARDIOGRAPHY (PANEL B, ARROWS). PERICARDIOCENTESIS RESULTED IN DRAINAGE OF 1.3 LITERS OF  BLOODY FLUID, WHICH WAS SUBSEQUENTLY ATTRIBUTED TO A MALIGNANT EFFUSION. REPEAT ECHOCARDIOGRAPHY OVER THE NEXT  6 HOURS REVEALED REACCUMULATION OF EFFUSION. THE PATIENT SUBSEQUENTLY HAD CARDIAC ARREST AND DIED.     

 

         

TAPONAMIENTO CARDIACO    PATOGENIA  Aumento de presión intrapericárdica, bajo gasto cardiaco, restricción de llenado. Compensación pr SRAA. (aumento de resistencias  vasculares periféricas) ¿?    DIAGNÓSTICO  Tríada clásica: hipotensión arterial + pulso paradójico + ingurgitación yugular.      Hipotensión arterial: Si shock. Si hay respuesta adrenérgica compensadora ==> hipertensión  Pulso paradóiico: Caída >10mmgHg con inspiración.  Ingurgitación yugular: signos de presión venosa elevada con datos de IC derecha.      Oliguria (IR por bajo GC).  Frialdad, cianosis acra (aumento de RVP)  Taquicardia (compensa bajo GC)  Fiebre (si sepsis)  Ruidos apagados, roce pericárdico.    EKG:    Alternancia eléctrica QRS (2:1) [PONER IMAGEN QUE ENVIÉ A GIGI]    Bajo voltaje de QRS      Taquicardia    Disociación eletromecánica    Tele de tórax:    cardiomegalia cuando >250ml, con silueta en garrafa 

ECOCardioTranstorácico    Derrame pericárdico    Colapso de cavidades (más las derechas)    Variaciones respiratorias exageradas de flijos AV y venas suprahepáticas (seno Y)    Vena cava inferior dilatada con abolición del colapso  ECOCardioTrasnesofágico    útiles en ocasines, derrames ocultos, mets, coágulos.      TRATAMIENTO  Drenaje pericárdico, PEEP en intubados.     

TRAUMA ABDOMINAL  TRAUMA ABDOMINAL CERRADO Y ABIERTO  PATOGENIA  Trauma es el intermbio de energía entre un objeto externo y un organismo. Daño tisular proporcional a la energía intercambiada.    Trauma cerrado==> vísceras sólidas  Trauma penetrante==> visceras huecas    DIAGNÓSTICO  A    Vía aérea  B    ventilación  C    circulación  D  Déficit neurológico  E  Examen completo desnudo    Indicación de laparotomía:  ‐Inestabilidad hemodinámica: paciente hipotenso que no responde a volumen o que respondión y se inestabilizó nuevamente.    Laparotomía==> hemostasia.  ‐Hematemesis o enterorragia==> lesión intestinal  ‐Herida transabdominal x arma de fuego==>95% tendrá lesión visceral  ‐Signos de irritación peritoneal  ‐Evisceración (excepto epiplón)    ‐Trauma abdominal cerrado con hipotensión y evidencia de hemorragia intraperitoneal  ‐Trauma abdominal cerrado con LPD o ECO positivos.  ‐Hipotensión con herida penetrante  ‐Hemorragia de esómago, recto, aparato GU x trauma penetrante  ‐Neumoperitoneo, aire en retroperitoneo, ruptura de diafragma.      Trauma cerrado: impacto lateral del chofer (trauma hepático), lado del acompañante (trauma esplénico).  Equimosis periumbilical (Signo de Cullen), en flanco (signo de Turner) ==> hemorragia retroperitoneal.    Se debe hacer examen rectal ¿Próstata alta o no palpable? ¿sangre?   

 

      Rx de pelvis: solo en paciente politraumatizado o glasgow  2)maniobra de Pringle o pinzamiento de pedículo  hepático).  RESPUESTA A      HOMBRE DE 24 AÑOS DE EDAD, QUE DURANTE UNA RIÑA RESULTA HERIDO POR PROYECTIL DE ARMA DE FUEGO EN EL ABDOMEN. SE  DESCONOCE CALIBRE Y TIPO DE ARMA. ES LLEVADO INMEDIATAMENTE AL SERVICIO DE URGENCIAS DEL HOSPITAL MÁS CERCANO, EL  CUAL SE ENCUENTRA A 30 MINUTOS DEL LUGAR. A SU LLEGADA SE INGRESA DE INMEDIATO A QUIRÓFANO PARA CIRUGÍA DE CONTROL  DE DAÑOS.      CONFORMAN LA “TRIADA LETAL” EN PACIENTES CON ESTE TIPO DE HERIDAS:  A) INESTABILIDAD HEMODINÁMICA, HIPOTERMIA Y ACIDOSIS  B) SEPSIS, COAGULOPATÍA Y FIEBRE  C) ACIDOSIS, HIPOTERMIA Y COAGULOPATÍA  D)HEMORRAGIA, INESTABILIDAD HEMODINÁMICA E HIPOTENSIÓN      LO MÁS PROBABLE ES QUE LA CIRUGÍA DE CONTROL DE DAÑOS FUE INDICADA DE INMEDIATO PORQUE A SU INGRESO EL PACIENTE  PRESENTÓ:  A) FRACTURA INCOMPLETA DEL ANILLO PELVIANO  B) ESTABILIDAD HEMODINÁMICA  C) ORIFICIO EN EL CUADRANTE SUPERIOR DERECHO  D) SÍNDROME COMPARTIMENTAL ABDOMINAL      La tríada de la muerte es: acidosis, hipotermia y coagulopatía.   

Criterios para someter a un paciente a cirugía de control de daños:    Déficit de base 2.5mEq(L    Trayecyto transabdominal del misil    Ubicación del orificio de entrada o de salida en cuadrante superior derecho    RESPUEESTA C, C      HOMBRE DE 32 AÑOS DE EDAD. ACUDE A URGENCIAS TRAS SE IMPACTADO CON AUTOMÓVIL; SE ENCUENTRA ESTABLE, AL COLOCAR  LA SONDA VESICAL SE OBSERVA FRANCA HEMATURIA.    DE ACUERDO A LO VALORADO, ¿QUÉ ESTRUCTURA ÓSEA ES LA MÁS PROBABLEMENTE AFECTADA?  A) RAMA ISQUIPÚBICA  B) SACRO ILIACA  C) ACETÁBULO  D) CABEZA DE FÉMUR    LA RADIOGRAFÍA MUESTRA FRACTURA NO DESPLAZADAS DE LA LESIÓN DIAGNOSTICADA. EL TRATAMIENTO SIGUIENTE ES:    A) COHIBIR HEMORRAGIA  B) OSTEOSÍNTESIS CON FIJADOR EXTERNO  C) PRÓTESIS DE RAMA ISQUIPÚBICA  D) CONSERVADOR, REPOSO Y REHABILITACIÓN    El paciente tiene fractura isquipúbica. En pacientes con accidentes en automóvil frontales las estructuras más frecuentementeafectas  son cabeza femoral y el acetábulo. En este caso clínico por la hematuria franca hay que pensar en la porción que cause daño a órganos  intrapélvicos (en este    caso es lesión en uretra), por lo tanto tiene una fractura isquiopúbica. En las fracturas de cadera, lo primero a  evaluar es la estabildiad hemodinámica, de no existir compromiso y ser una fractura no desplazada el tatamiento de elcción es  conservador; dejando los demás para los casos complicados (ancianos, lesión de otros órganos, hemoragia activa).    ¿Por que no    cohibir la hemorragia?     

CHOQUE HIPOVOLÉMICO TRAUMÁTICO    [CHOQUE] [r]  MUJER INDÍGENA DE 50 AÑOS DE EDAD, HIPERTENSA Y DIABÉTICA CONTROLADA. AL SALIR DE SU CASA SUFRE CAÍDA AL IR  CAMINANDO. POSTERIOR A LA CUAL PRESENTA DOLOR INTENSO EN PELVIS Y EXTREMIDAD INFERIOR DERECHA. EL HOSPITAL MÁS  CERCANO QUEDA A DOS HORAS DE SU CASA, POR LO QUE A SU LLEGADA AL HOSPITAL SE REPORTAN LOS SIGUIENTES DATOS AL  MOMENTO DE SU VALORACIÓN: FC DE 130X´, PRESIÓN ARTERIAL 90/50 MMHG, FR DE 35X´, GASTO URINARIO DURANTE EL TRASLADO  APARENTEMENTE DISMINUIDO Y NEUROLÓGICAMENTE ANSIOSA CON TENDENCIA A LA CONFUSIÓN.      CON BASE EN LOS DATOS CLÍNICOS USTED CLASIFICARÍA EL CHOQUE DE LA PACIENTE COMO:  A) GRADO I.  B) GRADO II.  C) GRADO III.  D)GRADO IV.    CON BASE EN LOS DATOS CLÍNICOS USTED CALCULA QUE LA PÉRDIDA DE SANGRE ES DE:  A) HASTA 750 ML.  B) DE 750 A 1500 ML.  C) DE 1500 A 2000 ML.  D) MÁS DE 2000 ML    CON BASE EN LOS DATOS CLÍNICOS USTED ESPERARÍA QUE EL GASTO URINARIO DE LA PACIENTE SEA DE:  A) > DE 30ML/HR.  B) 20 A 30 ML/HR.  C) 5 A 15 ML/HR.  D) DESPRECIABLE.    SU TRATAMIENTO DE REPOSICIÓN DE LÍQUIDOS BASADOS EN LA CLASIFICACIÓN DEL GRADO DE CHOQUE DEBERÁ CONSISTIR EN:  A) NO REQUIERE REPOSICIÓN.  B) EXCLUSIVAMENTE CRISTALOIDES.  C) CRISTALOIDES MÁS TRANSFUSIÓN DE SANGRE.  D) TRANSFUSIÓN DE SANGRE.   

RESPUESTA C,C,C,C  En fracturas de fémur la principal causa de muerte es la hemorragia incontrolable.    Grado  I 

ml pérdida  2000 

>40 

>140 

>35 

Muy  baja 

Disminuida 

5‐10veces de su límite  superior se corrobra el diagnóstico. Pueden encontrase arritmias cardíacas (TV, FA) en el EKG.    TRATAMIENTO  Se han experimentado con    múltiples alternativas desde barredores de radicales libres (ej manitol), inhibidores de la xantina oxidasa  (alopurinol), antioxidantes (vitamina E y C), sin embargo, hasta el momento no se encuentra con un grado de evidencia que suporte su  utilidad. Por lo que se limita el manejo de soporte y la corrección de las alteraciones (falla renal: hidratación con cristaloides,  rabdomiólisis: cristaloides, diurético, alcalinización de la orina y en casos severos hemodiafiltración, SDRA: oxígeno suplementario o  ventilación mecánica).      El alopurinol y el manitol tienen un nivel de evidencia III (Recomendación por expertos)    PRONÓSTICO  Está ligado a la causa inicial de la isquemia, así como, el órgano afectado, puede tener mortalidad >50% en caso de miembros pélvicos.      MASCULINO DE 72 AÑOS, POSTOPERADO EN SU DÍA UNO DE EMBOLECTOMÍA DERECHA SECUNDARIA A INSFUCIENCIA ARTERIAL  AGUDA QUE SE LLEVA A CABO SIN COMPLICACIONES NI INCIDENTES. AL MOMENTO DEL PASE DE VISITA USTED ENCUENTRA QUE EL  PACIENTE PRESENTA EDEMA CON GODETTE +++ DEL MIEMBRO PÉLVICO AFECTADO, ASÍ COMO DOLOR INTENSO A LA PALPACIÓN, LOS  PULSOS PERIFÉRICOS SE ENCUENTRAN PRESENTES; CALCULA UNA URESIS HORARIA DE 0.25ML/KG/H Y OBSERVA VENTILACIONES  RÁPIDAS Y SUPERFICIALES.    DE ACUERDO A LOS DATOS CLÍNICOS PROPORCIONADOS, ¿CUÁL SERÍA SU IMPRESIÓN DIAGNÓSTICA EN ESTE CASO?  A) TROMBOSIS VENOSA PROFUNDA  B) SÍNDROME DE REPERFUSIÓN  C) INFECCIÓN DE TEJIDOS BLANDOS  D) NUEVO EVENTO DE EMBOLIZACIÓN ARTERIAL    ¿QUÉ EXÁMENES DE LABORATORIO Y/O GABINETE SOLICITARÍA EN ESTE CASO PARA INCIAR SU PROTOCOLO DIAGNÓSTICO?  A) ARTERIOGRAFÍA DE MEIBMRO PÉLVICO DERECHO  B) USG DOPPLER VENOSO MPD  C) QS, ES, EGO, GA  D) BH, VSG, PCR    ¿CUÁL ES EL FACTOR QUE ESPERARÍA ENCONTRAR PARA CORROBORAR SU SOSPECHA DIAGNÓSTICA?  A) INCREMENTO DE LA CREATININA SÉRICA >0.3MG/DL EN LAS SIGUIENTES 48 HRS  B) ACIDOSIS METABÓLICA COMENTASADA CON ALCALOSIS RESPIRATORIA  C) FIBRILACIÓN AURTICULAR DE RECIENTE INICIO  D) ENZIMAS MUSCULARES ELEVADAS >5‐10 VEECES DE SU LÍMITE SUPERIOR   

EL TRATAMIENTO DE ELECCIÓN EN ESTE CASO SERÍA:  A) NUEVA EMBOLECTOMÍA ARTERIAL DE MIEMBRO PÉLVICO CERECHO  B) ANTICOAGULACIÓN ORAL  C) ANTIBIOTICOTERAPIA DE AMPLIO ESPECTRO  D) MEDIDAS DE SOSTÉN COMO SOLUCIONES CRISTALOIDES PARA MANTENER UN BALANCE HÍDRICO POSITIVO Y  OXIGENOTERAPIA.      RESPUESTA B, C, D, D 

  SÍNDROME COMPARTIMENTAL  [Resumen la Salle]    DEFINICIÓN  Conjunto de signos y síntomas ocasionados por la alteración en la perfusión debido al aumento en la presión en un compartimento del  cuerpo.    FISIOPATOLOGÍA  La presión de perfusión en las extremidades está determinada por la diferencia en la presión hidrostática capilar y la presión  hidrostática en las vénulas poscapilares; al presentarse un incremento en la presión intracompartimental (normal 10‐12mmHg), ya sea  por el aumento de volumen de su contenido o reducción del volumen del compartimento,    se compromete    inicialmente el flujo en  las vénulas y vasos linfáticos ocasionando edema, posteirormente se compromete la perfusión arteriolar y con ello la perfusión de los  tejidos dentro del compartimiento, presentándose isquemia. Por últmo, la presión comprime las arterias de quepeño calibres y medio,  conasionado isquemia completa del segmento.    ETIOLOGÍA  Existen múltiples causas del síndrome compartimental: vasculares (síndrome de isquemia‐reperfusión, trauma arterial y venoso,  flegmasía cerúlea dolens). Hemorrágicas no vasculares (fracturas, lesiones por aplastamiento por extravasación de fluidos,  inmovilización (aparatos de yeso), en paciente crítico con SIRS y reanimación hídrica masiva.      CUADRO CLÍNICO  Dolor desproporcionado de la magnitud de la lesión (generalmente a la movilización pasiva o compresión), aumento de volumen de la  extremidad, aumento de la tensión del compartimento, hipoestesias, parestesias y ausencia de pulsos.    Es importante destacar que la sintomatología puede ser inicialmente insidiosa y posteriormente rápidamente progresiva, se debe tener  en cuenta que en pacientes con alteraciones del estado de conciencia o con alteraciones neurológicas (sensitivas o motoras) es difícil  que nos refieren síntomas tempranos.    DIAGNÓSTICO  Para el diagnóstico se debe tener un alto índice de sospecha por antecedente y que a la exploración física tiene una senbilidad de 13‐ 19% con VPP 11‐15% y especificidad de 97, VPN 98%, es más útil para descartar que para confirmar el diagnóstico. Sin embargo la  presencia de 3 signos clínicos puede incrementar la probabilidad de diagnóstico hasta 93%. En caso de pacientes pediátricos con  alteraciones neurológicas es de utilidad la medición de la presión intracompartimental la cual en caso de encontrarse >30mGh es  diagnóstica.    TRATAMIENTO  Es necesario llevar a cabo fasciotomías para descompresión del compartimento afectado de manera pronta para evitar la pregresión y  las secuelas.    PACIENTE MASCULINO DE 36 AÑOS, SIN ANTECEDENTE DE IMPORTANCIA QUE PRESENTA HACE 6 HRS HERIDA POR PROYECTIL DE  ARMA DE FUEGO EN CARA ANTEROMEDIAL DE MUSLO DERECHO, POR LO QUE ES LLEVDO A URGENCIAS DONDE SE DIAGNOSTICA  TRUAMA VASCULAR A NIVEL DE ARTERIA Y VENO FEMORAL SUPERFICIALES, SE REALIZA TRATAMIENTO QUIRÚRGICO CON REPARACIÓN  DE AMBOS VASOS, SIN COMPLICACIONES. A LAS 3 HRS INICIA CON EDEMA ++++ DE PIERNA, DOLOR INTERNSO, 10/10 COMPARABLE AL  DOLOR DE INICIO ACOMPAÑADO DE PARESTESIAS TIPO HORMOGUEO.  A LA EXPLORACIÓN FÍSICA SE ENCUENTRA CON TA 130/85MMHG, FC 98, FR 22, T 37, MIEMBRO PÉLVICO DERECHO CON PRESENCIA DE  HERIDA QUIRÚRGICA LIMPIA, SIN DATOS DE SANGRADO, A NIVEL DE PIERNA AUMENTO DE VOLUMEN, EDEMA ++++, AUMENTO DE  TENSIÓN DE MASAS, DOLOR INTESO A LA PALPACIÓN SUPERFICIA EN CARA ANTERIOR DE PIERNA Y A NIVEL VASCULAR CON PULSO  FEMORAL, POPLÍTEO, TIBIAL ANTERIOR, POSTERIOR Y PEDIO 2/2, LLENADO CAPILAR 4S, FUERZA MUSCULAR 4/5 POR DOLOR,  SENSIBILDIAD SUPERFICIAL CON PARESTESIAS EN L5‐S1.  SE SOLICITAN EXÁMENES DE LABORATORIO ENCONTRANDO COMO RELEVANTE CPK EN 5348.    ¿CUÁL ES EL DIAGNÓSTICO MÁS PROBABLE EN ESTE PACIENTE?  A) TROMBOSIS VENOSA PROFUNDA  B) SÍNDROME COMPARTIMENTAL  C) CÉLULITIS DE MIEMBRO PÉLVICO DERECHO  D) LINFEDEMA DE MIEMBRO PÉLVICO DERECHO   

Respecto al diagnóstico estamos ante un cuadro agudo con dolor desproporcionado a la lesión. No hay datos de respuesta inflamatoria  propiamente por lo que descartamos celulitis, descartamos linfedema por ser crónico. Descartamos TVP por que el dolor es más  característico de Sx compartimental así como la presencia de tensión de masas.    LA ETIOLOGÍA DEL SÍNDROME COMPARTIMENTAL QUE RPESENTA ESTE PACIENTE ES:  A) FLEGMASÍA CERÚLEA DOLENS  B) SÍNDROME DE ISQUEMIA REPERFUSIÓN  C) INMOVILIZACIÓN PROLONGADA CON APARATO ORTOPÉDICO  D) REANIMACIÓN HÍDRICA MASIVA    Todas son causas de Sx compartimental pero el antecedente de cirugía previa de revascularización y CPK elevada, nos hace pensar que  hay un síndrome de isquemia‐reperfusión    EL DIAGNÓSTICO DE ESTE PACIENTE SE REALIZA POR MEDIO DE:  A) USG  B) ARTERIOGRAFÍA    C) CLÍNICA  D) TAC DE MEIBMRO PÉLVICO DERECHO    EL TRATAMIENTO DE ELECCIÓN EN EL CASO PRESENTADO SERÁ:  A) FASCIOTOMÍA INMEDIATA  B) TERAPIA DESOCMPRESIVA COMPLEJA  C) ANTCOAGULACIÓN CON HEPARINA Y CUMARÍNICO  D) ANTIBIOTICOTERAPIA DE AMPLIO ESPECTRO      Recordar que el linfedema generalmente es indoloro. Respecto a los estudios diagnósticos, la terapia descompresiva compleja es  tratamiento pero del mismo linfedema.  RESPUESTA B,B 

CRITERIOS DE AMPUTACIÓN  [Resumen la Salle]    EPIDEMIOLOGÍA  En E.U se realizan 60 000 amputaciones mayores (por arriba del tobillo) al año.    México 75 000 amputaciones mayores y menores de miembrosl pélvicos.  FR DM y enferemdad arterial periférica.    Los objetivos de amputar: retirar todo el tejido infectado, necrosado o isquémico preservado la mayor longitud funcional del miembro  afectado; es importante evitar las amputaciones repetidas y buscar cicatrización no complicada del muñón.    INDICACIONES DE AMPUTACIÓN:    Isquemia aguda:      Presente cuando hay cambios irreversibles (anestesia, parálisis, rigidez)    Cuando no hay opciones de revascularización    Posterior a intento fallido de revascularización    Isquemia crónica crítica    Posterior a múltiples intentos fallidos de revascularización    Cuando no hay opciones de revascularización    Cuando se presentan comorbilidades seberas que ponen en riesgo la vida.    Estatus funcoinal paurpérrimo.    Expectativa de vida limitada    Gangrena extensa o infección en las cuales no es posible el salvamiento de la extremidad.    Infección ascociada a DM2    Infección extensa qu eimpide salvamiento    Infección extensa asociada a sepsis    Lesiones traumáticas extensas      Tumores malignos óseos o de tejidos blandos    Escala MESS (Mangled Extremity Severity Score) >7 indica amputación      Determinación del nivel de amputación  Siempre se debe considerar preservar la mayor longitud del miembro que sea funcional.    Si la causa es por isquemia deberá realizars a el nivel hsta donde se presenten pulsos o temperatura de la extremidad, es decir si  presenta pulso femoral puede realziar a nivel supracondíleo, si presenta pulso poplíteo a nivel infracondíleo, si presenta pulsos distales  puede hacerse amputaciones menores (dedos, mtransmetatarsiana, , tarso‐metatarsiana [Lisfranc], transtarsal [Chopart]). En ocasiones  pueden realziars eáun en ausencia de pulsos cuando se tiene una determinación de oxígeno transcutánea >80% con lo que la  probabilidad de cierre del muñón es mayor.   

  Si la causa es infecciosa deberá retirarse todo el tejido infetado, es importante considerar que por la respuesta a la infección los tejidos  circundantes pueden presentarse edematosos, lo cual dificulta el cierre primario. Puede decidirse el manejo en 2 tiempos para 3evitar  absceso de muñón, esto es, realziar amputación retirando, todo el tejido afectado y diferir el cierre para un segundo momento cuando  la infección se encuentre controlada.        PACIENTE MASCULINO DE 69 AÑOS DE EDAD, CON ANTECEDENTE DE HAS, DM2 DE MÁS DE 20 AÑOS DE DIAGNÓSTICO Y TABAQUISMO  DE LARGA EVOLUCIÓN, QUIEN ACUDE AL SERVICIO DE URGENCIAS POR PRESENTAR CUADRO CLÍNICO QUE INICIÓ HACE 24 HRS CON  DOLOR INTENSO, INCAPACITANTE, EN MIEMBRO PÉLVICO DERECHO A PARTIR DEL TERCIO INFERIOR DEL MUSLO, ASÍ COMO  HIPOTERMINA, PALIDEZ EN REGIÓN MENCIONADA, AUSENCIA DE SENSIBILIDAD Y MOVILIDAD.  A LA EF MIEMBRO PÉLVICO DERECHO, USTED CORROBORAR QUE EL PACIENTE CURSA CON POIQUILOTERMIA, PULSO FEMORAL 2/2,  POPLITEO, TIBIAL ANTERIOR, POSTERIOR Y PEDIO AUSENTES, COLORACIÓN MARMOREA DE LA PIERNA, ANTESTESIA A NIVEL DE L4 Y S1  Y FUERZA MUSCULAR 0/5 EN ESCALA DE DANIELS, ASÍ COMO LLEGADO CAPILAR DE >6S      EL CUADRO CLÍNICO PRESENTADO CORRESPONDE A:  A) INSUFICIENCIA ARTERIAL AGUDA CON EXTREMIDAD VIABLE  B) INSUFICIENCIA ARTERIAL AGUDA CON ISQUEMIA IRREVERSIBLE  C) INSUFICIENCIA ARTERIAL CRÓNICA  D) SEPSIS POR INFECCIÓN DE TEJIDOS BLANDOS    Hay que recordar las 6P de insuficiencia arterial aguda, los datos de irreversibildiad son parestesias (o anestesia) y parálisis. También el  tiempo de ventana para revascularizar es de 6‐8 hrs, razón por la cual ya no es viable.      DE ACUERDO AL CUADRO CLÍNICO MENCIONADO Y UNA VEZ CORROBORADA LA EXISTENCIA DE OCLUSIÓN A NIVEL DEL CANAL DE LOS  ADUCTORES (CANAL DE HUNTER) DERECHA, SU CONDUCTA TERAPÉUTICA SERÁ:  A) ANTICOAGULACIÓN ORAL  B) ANTIBIOTICOTERAPIA DE AMPLIO ESPECTRO  C) EMBOLECTOMÍA FEMOROPOPLÍTEA  D) AMPUTACIÓN    Si no está viable se somete a amputación. Si se intentar revascularizar a parte de no ser exitosa, aumentaría el riesgo de complicaciones  como Sx de reperfusión severo y sepsis.      A QUÉ NIVEL SE REALIZARÁ LA AMPUTACIÓN EN ESTE CASO PARTICULAR:  A) SUPRACONDILEA  B) INFRACONDILEA  C) DESARTICULACIÓN DE LA EXTREMIDAD  D) AMPUTACIÓN DE SYME    Si el pulso más distal que se palpa es el femoral se hace amputación supracondilea, si el más distal en palparse es poplíteo es  infracondíela.      RESPUESTA B, D, A     

ENFERMEDAD CAROTÍDEA  [Resumen la Salle]    DEFINICIÓN  Es una manifestación de la enferemdad ateroesclerosa que afecta a las carótidas, que puede progresar a la complicación (trombosis o  hemorragia) dando lugar a eventos isquémicos cerebrales transitorios o irreversibles.    EPIDEMIOLOGÍA  EVC es la 2da causa    a nivel mundial, primera de discapacidad en EU. Aumenta con si >60 años, se duplica el riesgo por cada década  después de los 55a.      FISIOPATOLOGÍA  Al igual que otras enferemdadesde vaculares la lesión endotelial es el principal mecanismo. Se generan células espumosas, que  fagocitan LDL oxidasas, producen estado inflamatorio y placa fibrolipídica. La placa prolifera y obstruye.    Es frecuente encontrar dichas  placas en la bifurcación de la carótida interna y la externa.    Si la placa se rompe se acumulan depóstios plaquetarios.      CUADRO CLÍNICO  Es responsable del 16‐20% de todos los EVC.  Es responsable del 50% de todos los EVC isquémicos secundarios a embolismos.   

Asíntomática en varias ocasiones y solo diagnosticada con USG doppler en >65 años.    Se puede presentar como AIT u EVC.  Cuando ocurre un EVC por enferemdad carotíeda se afecta la arteria cerebral media.    Oclusión de ACM: alt motoras (hemiplegia, hemiparesia), alt sensitivas del himicuepo contralateral al afectado.    Alteraciones del lenguaje (disartria, disfasia, afasia) en caso de afección del hemisferio dominante (80% es el derecho).    Alteraciones visuales (amaurosis fugax)    DIAGNÓSTICO  Soplo en triángulo carotídeo.  Estudio inicial si clínica compatible o si >65a+>2FR CV ==> USG dúplex  Estudios confirmatorios y prequirúrgicos son: angioTAC, angioRM, angiografía.a    TRATAMIENTO  Prevención secundaria: aas, estatinas, control de comorbilidades, cese de tabaquismo.      Tratamiento Qs: endarterectomía o angioplastia endovascular + colocación de stet en caso de estenosis críticas >0% en sintomáticos,  >60% en asintomáticos.      MASCULINO DE 75 AÑOS, QUIEN CUENTA CON ANTECEDENTE DE DIABETES MELLITUS TIPO 2, HAS, DISLIPIDEMIA MIXTA DEMÁS DE 20  AÑOS DE DIAGNÓSTICO EN TRATAMIENTO IRREGULAR CON METFORMINA, INSULINA NPH Y AMLODIPINO.    INICIA SU PADECIMIENTO  ACTUAL HACE 4 HRS DE FORMA SÚBITA CON DIARTRIA, POSTERIORMENTE SE AGREGA HEMIPARESIA FACIOCORPORAL DERECHA Y  PARESTESIAS EN LA MISMA LOCALIZACIÓN, POR LO QUE ES LLEVADO A URGENCIAS.    A LA EF DIRIGIDA ENCUENTRA SOPLO CAROTÍDEO IZQUIERDO, RUIDOS CARDÍACOS RÍTMICOS DE ADECUADA INTESIDAD SIN  AGREGADOS, A LA EXPLORACIÓN NEUROLÓGICA DISARTRIA AL HABLA, RESTO DE LAS FUNCIONES MENTALES SIN ALTERACIONES,  NERVIOS DEL CRÁNEO CON DESVIACIÓN DE LA COMISURA LABAL A LA IZQUIERDA, TONO Y TROFISMO CONSERVADO, FUERZA  MUSCULAR 3/5 EN HEMICUERPO DERECHO, REMS 2/4 GENERALIZADOS, SIN DATOS DE LIBERACIÓN PIRAMIDAL, SENSIBILIDAD CON  NIVEL HIPOESTÉSICO C5‐C6, L4‐S1, CEREBELO SIN ALTERACIONES Y SIN DATOS DE IRRITACIÓN MENÍNGEA.      EL RIESGO DE EVENTO VASCULAR CEREBRAL POR ENFERMEDAD CAROTÍDEA ES MAYOR EN QUÉ TIPO DE PACIENTES:  A) HOMBRES  B)  55 años, especialmente 60 años.  ¿Qué relación guarda la enfermedad carotídea con la fibrilación auricular?    LOCALIZACIÓN MÁS FRECUENTE DE LAS PLACAS DE ATEROMA A NIVEL CAROTÍDEO:  A) CARAÓTIDA EXTERNA  B) CARÓTIDA COMÚN  C) BIFURCACIÓN DE LA CARÓTIDA INTERNA Y EXTERNA  D) CARÓTIDA INTERNA    MECANISMO FISIOPATOLÓGICO MÁS FREUCNETE EN EL DESARROLLO DE EV POR ENFERMEDAD CAROTÍDEA:  A) TROMBOSIS IN SITU  B) MICROEMBOLISMO  C) VASOCONSTRICCIÓN  D) PLACA DE ATEROMA ESTABLE    ESTUDIO INICIAL A REALIZAR EN PACINTES CON SOSPECHA DE ENFERMEDAD CAROTÍDEA:  A) ANGIOGRAFÍA  B) ANGIOTAC  C) ANGIORM  D) USG DÚPLEX    RESPUESTA A, C, B, D   

INSUFICIENCIA VENOSA  [Resumen la Salle]  DEFINICIÓN  Condición donde el sistema venoso es incapacidad de retornar la sangre, debido a anormalidades de la pared venosa y valvular que  lleva a una obstrucción o reflujo sanguíneo en las venas.      EPIDEMIOLOGÍA  80% de la pob gral presenta algún grado de enfermedad venosa, de 25‐50% várices, 1‐2% úlceras flebo estáticas. Más frec en mujeres.     

FISIOLOGÍA  Hay un sistema superficial que maneja 15‐20% ,uno profundo 70‐80%    y las venas perforantes que dirigen del superficial al profundo      FISIOPATOLOGÍA  Al dañarse las válvulas del sistema venoso o al presentar obstrucción al flujo (ej. trombosis venosa previa), el sistema de bomba  muscular no resulta eficiente, por lo cual existe estasis y se genera hipertensión venosa, ocasionando cambios endolteliales y  favoreciendo extravasación de líquido a nivel capilar (edema), extravasación de células y hematíes, inflamación, degradación de  pigmentos derivados de hemoglobina (hemosiderina), acumulación de pigmentos en el TCSC    (dando color ocre a nivel del tobillo),  estos factores producen lesion cutánea que deriva en lipodermatoesclerosis o atrofia, y por último ulceración.      FACTORES DE RIESGO  Edad, sexo femenino, gestación, antecedente familiar de IVC, ortostatismo prolongado, obesidad, sedentarismo, trabajos que  impliquen actividad de pie o sedestación prolongada (más de 5hrs)    CUADRO CLÍNICO  Inicio incidioso, dolor, pesantez, prurito, cansancio, edema y calambres de predominio vespertino que se exacerba con el ortostatismo  y con el calor, mejoran con decúbito y frío.    A la EF telangiectasia 3mm, edema,  pigmenación ocre, lipodermatoeslerosis, eccema o úlceras; es importante destacar que los cambios cutáneos inicialmente se presentan  a nivel del maléolo medial.    Se utiliza CEAP para su clasificación.   

    Clase 1: telangiectasias y venas reticulares  Clase 2: varices    Clase 3: edema  Clase 4: lipodermatoesclerosis + pigmentación ocre en maléolo medial  Clase 5: úlcera cicatrizada  Clase 6: úlcera activa    DIAGNÓSTICO  Exploración física  Schwartz: percusión del sistema venoso superficial con transmisión distal de la onda  Trendelemburg: evalúa reflujo superficial y competencia de perforantes. [si se llenan al levantarse fallan el superficial y las perforantes]  Perthes: evalúa perforante y sistema venoso profundo. [la paciente debe caminar, si se vacían habla de que perforante y profundo  funcionan bien]    USG dúplex: el más usado, acorde a GPC se recomienda en pacientes candidatos a tratamiento quirúrgico. Reflujo > 1seg en venas  superficiales de la ingle es patológico, a nivel de las perforantes >1seg.  Pletismografía: para cambios cutáneos si várices si no se cuenta con doppler. Se usa en investigación.    Medición invasiva de presión venosa del dorso del pie: es el estudio de mayor desempeño diagnóstico, es el ideal según GPC; pero casi  no se usa por que los no invasivos arrojan datos suficientes generalmente.    Flebografía: info anatómica precisa en pacientes con malformaciones anatómicas, candidatos a Qx con várices recidivantes.          TRATAMIENTO  Conservador: control del peso, evitar ortostatismo prolongado, realizar medidas físico‐posturales, uso de ropa cómica y calzadocómici,  tacones 50% se considera masiva y existen alteracioens hemodinámicas, hasta choque cardiogénico y la muerte. Se pueden presentar disnea,  dolor torácico, palpitaciones, tos, hemoptisis, ortopnea, síncope, arritmias, taquicardia, hipotensión, hioxemia, sin embargo debido a  los datos inespecíficos no se puede fundamenter dx con la EF.    DIAGNÓSTICO  Cuadro clínico con criterios de Wells para TEP no son diagnósticos pero establecen probabilidad.  Rx de tórax: signo de Westermark (disminución de trama vascular), joroba de Hampton (radioopacidad en forma de domo con base en  la pleura), mayor tulidad para descartar otras patologías pulmonares.   

 

   

 

  EKG: es noramal hasta en 70%, SIQIIITIII es patognomónico pero raro, pueden encontrarse datos de sobrecarga de ventrículo derecho.    Ondas S profundas en DI  Ondas Q en DIII  Onda T negativa en DIII  GA: hipoxemia  Dímero D, sirve para excluir.  Ecocardiograma, gamagrama V/Q puden ser confirmatorios  TAC helicocoidal contrastada: es estándar de oro  Se clasifica en aguda, masiva, submasiva y no masiva y crónica.      TRATAMIENTO  Depende de la severidad: medidas de soporte (VM, aminas).  Trombolisis o trombectomía en pacientes con TEP masiva y choque cardiogénico debido a su alta mortalidad es >50%.    Para  submasiva y no masiva se recomienda anticoagulación iniciando con heparina convencional o HBPM y continuar con anticoagulación  oral con INR 2‐3. En los pacientes que no pueden recibir anticoagulación que presentan TEP a pensar de la decuada anticoagulacion  está inciado la colocación de un filtro de vena cava para prevenir nuevo evento.      PACIENTE FEMENINO DE 32 AÑOS, SE ENCUENTRA EN PUIERPERIO FISIOLÓGICO MEDIATO. INICIA CON EDEMA DE MIEMBRO PÉLVICO  DERECHO A NIVEL DE GASTROCNEMIOS, ERITEMA, AUMENTO DE TEMPERATURA EN DICHA LOCALIZACIÓN, RED VENOSA SUPERFICIAL Y  DOLOR A LA DORSIFLEXIÓN Y A LA COMPRESIÓN DE LA PIERNA. SE DIAGNOSTICA TVP Y SE INCIA MANEJO CON HEPARINA DE BAJO  PESO MOLECULAR A DOSIS PLENAS.    ES EFRESADA DEL SERVICIO A DOMICILIO SIN RECIBIR TRATAMIENTO CON ANTICOAGULACIÓN ORAL. TRES DÍAS DESPUÉS LA PACIENTE  INGRESA A URGENCIAS POR PRESENTAR DISENA DE PEQUEÑOS ESFUERZOS, DOLOR TORÁCICO DE PREDOMINIO A LA INSPIRACIÓN,  CIANOSIS PERIBUCAL Y PAPITACIONESL.  A LA EF SE ENCUENTRA CON SIGNOS VITALES TA 100/60, FC 109LPM, FR 24, TEMP 37, SAT O2 A AIRE AMBIENTE 75%, TORÁX ANTERIOR  CON RUIDOS CARDÍACOS RÍTMICOS DE ADECUADA INTENSIDAD, CON FC 1110, SIN AGREGADOS, TORAX POSTERIOR CO MURMULLO  VESICULA RPRESNETE, LIGERAMNETE DISMINUIDO BASAL DERECHO, SIN OTRAS ALTERACIONES DE IMPORTANCIA. MIEMBRO PÉLVICO    DERECHO CON EDEMA +++, AUMENTO DE TEMPERATURA Y DOLOR A LA PALPACIÓN. SE INICIA MANEJO CON CON OXÍGENO  SUPLEMENTARIO POR MASCARILLA,    Y SE SOLICITA DD CUYO RESULTADO REPORTA 2563.    DE ACUERDO A LOS DATOS CLÍNICOS PROPORCIONADOS, ¿CUÁL SERÍA SU IMPRESIÓN DAIGNÓSTICA EN ESTE CASO?  A) TVP 

B) TEP  C) IAM  D) NEUMONÍA NOSOCOMIAL  ¿CUÁL ES EL ESTUDIO DE GABINETE ESTÁNDAR ORO PARA EL DIAGNÓSTICO DE LA PATOLOGÍA PRESENTADA POR LA PACIENTE?  A) USG DOPPLER  B) EKG  C) RX TÓRAX  D) TOMOGRAFÍA HELICOIDAL CONTRASTADA.    ¿CUÁL ES EL SIGNO ELECTROCARDIOGRÁFICO PATOGNOMÓNICO DE LA PATOLOGÍA PRESENTADA EN ESTE CASO?  A) ELEVACIÓN DEL SEGMENTO ST  B) TAQUICARDIA SINUSAL  C) SIQIIITIII  D) INVERSIÓN DE ONDA T    SE REALIZA TOMOGRAFÍA HELICOIDAL CONTRASTADA Y SE ENCUENTRA UNA TEP SUBMASIVA ¿CUÁL ES EL TRATAMIENTO DE ELECCIÓN  PARA ESTA PACIENTE?  A) ANTICOAGULACIÓN CON HEPARINA E INCIAR ANTICOAGULACIÓN ORAL  B) ANTIAGREGANTE PLAQUETARIO  C) COLOCACIÓN DE FILTRO DE VENA CADA  D) ANTICOAGULACIÓN CON HEPARINA    RESPUESTA B, D, C, B      MUJER DE 45 AÑOS    DE EDAD CON DIAGNÓTICO DE SX ANTIFOSFOLÍPIDO, Y ANTECEDENTE DE CIRUGÍA DE MIEMBRO INFERIR DERECHO  POR FRACTURA DE TIBIA HACE 2 SEMANAS. ACUDE A URGENCIAS POR UN CUADRO DE 2 HRS DE EVOLUCIÓN, CARACTERIZADO POR  DISNEA SÚBITA Y DOLOR TORÁCICO. A LA EXPLORACIÓN TA 100/70 MMHG, FC 120LPM, FR 24, DISNEICA Y CON USO DE MÚSCULOS  ACCESOREIOS DE LA RESPIRACIÓN. EN LOS EXAMENES: HIPOXEMIA E HIPOCAPNIA EN LA GASOMETRÍA, Y TAQUICARDIA SINUAL EN EKG,  SIGNO DE WESTERMARCK CON LA RX DE TÓRAX Y ELEVACIÓN DE HEMIDIAFRAGMA DERECHO.  ¿CUÁL ES EL ESTUDIO MÁS APROPIADO EN ESTA PACIENTE?    A) DÍMERO D  B) TAC PULMONAR HELICOIDAL CON CONTRASTE  C) ECO CARDIOGRAMA  D) ANGIOGRAFÍA PULMONAR    Clasificar al paciente con sistema Wells.    Probabilidad alta  TAC pulmonar helicoidal con contraste, si contraindicación grammagrafía ventilación perfusión.        Si ninguna de las dos son factibles: USG de extremidades    No se debe usar gammagrafía si: derrame pleural o atelectasis  Angiografía teóricamente es el gold stándar pero por ser invasiva no deberá utilizarse a menos que no se llegue al diagnóstico con los  métodos anteriores.    RESPUESTA COMENTADA B    MUJER DE 50 AÑOS DE EDAD CON ANTECEDENTE DE TROMBOEMBOLIA PULMONAR HACE 2 AÑOS. ACUDE A URGENCIAS POR UN  CUADRO QUE INICIÓ HAE 2 HORAS, CARACTERIZADO POR DISNEA SÚBITA, DOLOR TORÁCICO Y TOS. SIGNOS VITALES: TA  100/70MMHG, FC 130LPM, FR 25 RPM, T 36.5 A LA EXPLORACIÓN FÍSICA DOLOR A LA PALPACIÓN EN PANTORRILLA IZQUIERDA. LA  RADIOGRAFÍA DE TÓRAX MUESTRA CONDENSACIÓN PARENQUIMATOSA TRINAGULAR DE BASE PLEURAL DERECHA.    ¿CUÁL ES LA ALTERACIÓN ELECTROCARDIOGRÁFICA MÁS FREUCENTE EN PACIENTES CON ESTA ENFERMEDAD?  A) TAQUICARDIA SINUAL  B) SI, QIII, TIII  C) DESVIACIÓN DEL EJE A LA DERECHA  D) BLOQUEO DE RAMA DERECHA    ¿CUÁL ES EL ESTUDIO DE ELECCIÓN PARA REALIZAR EL DIAGNÓSTICO EN ESTA PACIENTE?  A) ECOCARDIOGRAMA  B) TAC HELICOIDAL CON CONTRASTE  C) GAMMAGRAFÍA VENTILACIÓN‐PERFUSIÓN (V/Q)  D) ANGIOGRAFÍA    ¿CUÁL ES EL TRATAMIENTO MÁS ADECUADO EN ESTE CASO?  A) FIBRINÓLISIS 

B) ANTICOAGULACIÓN  C) FILTRO DE VENA CAVA INFERIOR  D) EMBOLECTOMÍA    La pacinet tiene tromboembolia pulmonar. El diagnóstico se apoya en: antecedente de tromboembolia pulmonar, disnea súbita  (síntomas más característicos), dolor torácico, dolor en pantorrilla (signo de Hommans), taquicardia sinusal e imagen radiográfica  característica de joroba de Hampton. El diagnóstico diferencial se deberá realizar con: pericarditis, pleuritis, derrame pleural,  hipertensión arterial pulmonar y neumonía; principalmente. En la pericarditis existe dolo precordial pero no es súbito y da imagen  electrocarddiográfica característica con supradesnivel del segmento S‐T (en sonrisa); en la radiografía de tórax solo si va acompañda de  derrame pericárido se ve aumento de la silueta cardiaca. En la pleuritis, el dolor no es subito y se reproduce a la inspiración profunda;  en la radiografía de tórax si se acmpaña de derrame pericárdico, habrá borramiento de ángulos. En la hipertensión arterial pulmonar la  disena es progresiva y no se acompaña de dolor precordial, sin embargo hay que considerar que la tromboembolia pulmonar es causa  de hipetensión pulmonar aguda. El principal factor de riesgo para la tromboembolia pulmonar es el antecedente    enfermedad  tromboembólica venosa previa; otros factores de riesgo son: cáncer, inmovilización, cirugía en los últimos 3 meses, evento  cerebrovascular reciente,    obesidad, tabaquismo, anticonceptivos orales, síndrome antifosfolípido, resistencia a la proteína C activada  (trombofilia hereditaria más frecuente). El 20% de los TEP son idiopáticos (sin factores de riesgo). La radiografía en una tromboembolia  pulmonar es normal en la mayoría de los casos, aunque ocasionalmente se pueden observar infartos pulmonares, joroba de Hampton  (condensación parenquimatosa trinagular de base peural ), signo de Westermarck (oligoemia focal que produce hiertransparencia  pulmonar). La alteración electrocardiográfica más frecuente, es la taquicardia sinusal, aunque puede presentarse otros datos que  sugieran hipertrofia ventricular derecha como desviación del eje hacia la derecha, bloqueo de rama derecha, SI, QIII, TIII. El estudio de  elección para realizar el diagnóstico es la TAC helicoidal con contraste, a excepción de alergia al contraste o presencia de insuficiencia  renal, ese caso está indicada la grammagrafía ventilación‐perfusión.    El ecocardiograma está indicado para evaluar la gravedad de la hipetensión pulmonar, en búsqueda de disfunción      ventricula  derecha, y en casos donde el diangóstico no quede claro o no se pueda confirmar el diagnóstico por otro estudio por contraindicacines.  La angiografía    es el estándar de oro de la TEP, sin embargo no es el primer eestudio indicado en este caso. En en caso sde reisgo lee o  intermedio (sin choque o hipotensión), el tratamiento más adecuado es la anticoagulación. En casos de riesgo alto se indica fibrinólsiis.  La embolectomía se realizaen pacintes com compromiso hemodinámico grave, que no responden a fibrinólisis. El filtro de vena cada  inferior se utiliza en pacientes con enfermed tromboemólica venosa o con alto riesgo de la misma, TEP recurent y que tengan  contraindicación par anticoagulaicón.  RESPUESTA A, B, B           

TROMBOSOSIS VENOSA PROFUNDA  [Resumen la Salle]    DEFINICIÓN  Es parte del espectro de la enfemedad tromboembólica venosa que afecta al sistema venoso profundo de los miembros pélvicos.    EPIDEMIOLOGÍA Y RELEVANCIA  Se presenta en 1:10 000 adultos jóvenes y hasta en 1:100 adultos mayores al año. Junto con la tromboembolia pulmonar es la 3era  causa de muerte hospitalaria y la 1era prevenible.      FACTORES DE RIESGO  Edad >40 años, hospitalizado, cirugía reciente o trauma son los de mayor riesgo. Neoplasias, inmovilización, embarazo, puerperio, AO e  inyectados, procedimientos invasivos vaculares (catéteres), antecedentes de TVP. Trombofilia: La más frecuente en la polbación es la  deficiencia del factor V de Leiden.    FISIOPATOLOGÍA  Tríada de Virchow (estasis, hipercoagulabilidad y lesión endotelial), no es necesario tener los 3.    La leisón endotelial es parte angular del desarrollo de la trombosis. La estasis promueve plaquetas y los factores de la coagulación se  acumulen y puedan ponerse en contaco con las peredes del vaso, la hipercoagulabilidad se presenta en pacientes con enfermedades  agudas, traumatismos, cirugía reciente (como parte de la respuesta metabólica al trauma) o bien alteracioens de la coagulación.    La  lesión o disfunción endotelial puede ser por traumatismo directo, la mayoría de las veces se presenta por hipoxia y citocinas  proinflamatorias que hacen que se pierdan las características del endotelio.    CUADRO CLÍNICO  Son asintomáticas en 50% de los casos. Se puede presentar con dolor edema, eritema, hipertermia, sensibilidad, fiebre, presentcia de  red venosa superficial, así como, dolor a la dorsiflexión o compresión de la pierna. Es de inicio súbito y progresivo.      El edema es un dato PIVOTE.  Signos clínicos: son poco frecuentes aprox en 10%    Homans (dolor a la compresión en gastroctnemios,  Ollow (dolor en gastrocnemios a la dorsiflexión de pie).    Prats (presencia de red venosa colateral superficial),   

  DIAGNÓSTICO  Escala de Wells. Solicitar dímero D (en ámbito hospitalario) valor predictivo negativo 98%.    Se confirma el Dx si Wells ≥2 + DD+. Realizar USG duplex en todos los paciente con sspecha de TVP con DD+ como confirmatorio    TRATAMIENTO  Anticoagulación con HBPM, HNF o fondaparinux por 5 días al mismo tiempo que anticoagulación oral. Objetivo INR de 2‐3.  Duración mínimo 3 meses, puede extenderse hasta 1 año en 1er evento con múltiples factores de riesgo y de forma indefinida en  recurrencia. Medias de higiene venosa y deambulación temprana.      Debido al alto riesgo de TVP en pacientes quirúrgicos (los de mayor eisgo son la cirgía de cadera y rodilla) así como pacinetes  hospitalziados o postrados se sugiere tromboprofilaxis de actuero a riesgo de TVP, desde calcetines de compresión y deambulación  temprana hasta combinación de medidas físicas y psoturales y uso de anticoagulanes (HBPM, IVK, heparina convencional,  fondaparinux, rivaroxaban, dabigatran) previa y posterior al tratamiento. Todas las pacientes a los que se les realizará cirugía mayor  ginecológica, urológica o bariátrica, deben recibir profilaxis farmacológica.      COMPLICACIONES  Secuela post trombótica o secuela posflebítica: es un cuadro de insfuciencia venosa crónica secundaria, flegmasia cerúlea dolens. TEP  en 10‐50% de los casos.    PRONÓSTICO  Recurrencia: 25% a 5 años y 30% a 10 años. Mortalidad hasta 19% sin tratamiento. 7%), tabaquismo, sedentarismo, >5 años de DM, enfermedad arterial periférica, neuropatía periférica,  ceguera, pérdida visual, enf renal crónica, >40 años.  Deformidad estructural del piel, trauma, calzado inadecuado, hiperqueratosis, historia previa de úlceras o amputación, ortostatismo  prolongado, movilidad articular limitada.      CLÍNICA  Alteraciones sensitivas, afección inicial a parestesias.  Alteraciones motoras: formación de puntos de hiperqueratosis por alteraciones de puntos de apoyo (mal perforante plantar‐ formación  de úlceras en puntos de hiperqueratosis), deformidad estructural con pérdida de los arcos plantares, puntos de apoyo anómalos y  dedos en martillo (pie de Charcot).  Alteraciones autonómicas: anhidrosis, edema, hiperemia.    Alteraciones vasculares: pérdida de anexos cutáneos, hiptermia, ausencia de pulsos distales, hiperemia reactiva. Úlceras: su localización  en puntos de apoyo o sitios de traumatismo, pueden o no presentar signos de infección.      DIAGNÓSTICO  HC y EF completa, valoración de la sensibilidad con monofilamente 10g (10 puntos) o diapsón (palestesia). Enviar a valoración a 2 nivel  de forma anual, si hay FR enviar cada 3‐6 meses.    Se establece diagnóstico al encontrar alteraciones físicas (desde hiperqueratosis, deformidad hasta ulceras o necrosis) con datos de  disminución de sensibilidad.  Solicitar Rx de pie para determinar compromiso óseo.    TRATAMIENTO 

Referir al 2nivel a cualquier pacientes con una nueva úlceras.    En caso de presentar sigos de infección inciar AB empírica con cefalosporina VO, amoxiclav, macrólidos, peniclina y fluoroquinolonas.    En una extremidad sin riesgo inminente cubrir para gram +, una extremidad en riesgo deberá cubririse con doble esquema y si pone en  riesgo la vida valorar triple esquema con amplio espectro.  En caso de determinarse isquemia, valorar la posibilidad de revascularización.    En caso de lesiones extensas (W4 o W5, Texas III) se requerirá desbridación extensa, amputación menor y hasta amputación mayor  para control del cuadro.   

  PREVENCIÓN  Control metabólico, lubricación, no cortar callos, actividad física >30 min diarios o por lo menos 3 dias de la semnaa. Inspección diaria  asistida, valoración de especialidad periódica.      PACIENTE MASCULINO DE 65 AÑOS, CUENTA CON ANTECEDENTE DE DM TIPO 2 DE 10 AÑOS DE DIAGNÓSTICO EN TRATAMIENTO  IRREGULAR CON METFORMINA 850MG CADA 24 HRS, HAS DE 5 AÑOS DE DX EN TX CON CAPTOPRIL CADA 24 HRS, TABAQUISMO DE  MAS DE 20 AÑOS DE EVOLUCIÓN A RAZÓN DE 10 CIGARRILLOS AL DÍA.    ACUDE A CONSULTA MENSUAL DE UMF REFIRIENDO PRESENCIA DE LESIÓN EN PLANTA DE PIE DERECHO, DE APROXIMADAMENTE  4X5CM DE DIÁMETRO, FÉTIDA Y CON EXUDADO BLANQUECINO MODERADO, REFIERE QUE AHCE 6 MESES INICIÓN CON DOLOR  ARDOROSO EN LA MISMA EXTREMIDAD DISTAL.  A LA EF SE OBSERVA MIEMBRO PÉLVICO DERECHO CON PRESENCIA DE ÚLCERA PLANTAR DE 4X5 CM DE DIÁMETRO A NIVEL DE LA  CABEZA DE TERCER METATARSIANO, CON EXUDADO PURULENTO, FÉTIDO, QUE SE EXTIENDE HASTA TEJIDO CELULAR SUBCUTÁNEO SIN  PENETRAR LA FASCIA, RESTO DE LA EXTREMIDAD HIPOTRÓFICA CON TONO CONSERVADO, AUSENCIA DE VELLO EN PATE INFERIOR DE  LA PIERNA, PULSOS FEMORAL 2/2, POPLÍTEO 2/2, TIBIAL ANTEIOR, POSTERIOR Y PEDIO 0/2. FUERZA MUSCULAR 4/5. REMS 2/4,  SENSIBILIDAD SUPERFICIAL CON HIPOESTESIA A NIVEL DE S1, SENSIBILIDAD PROFUNDA CON DISMINUCIÓN DE PALESTESIA, LLENADO  CAPILAR 4S.    ¿CUÁL ES SU DIAGNÓSTICO PRESUNTIVO EN ESTE CASO?  A) PIE DIABÉTICO CON ULCERACIÓN SIN INFECCIÓN NI ISQUEMIA  B) PIE DIABÉTICO INFECCIOSO  C) PIE DIEBÉTICO ISQUÉMICO  D) PIE DIABÉTICO ISQUÉMICO E INFECCIOSO    DE ACUERDO A LA CLASIFICACIÓN DE WAGNER ¿CUÁL ES EL GRADO DE LA LESIÓN DE ESTE PACIENTE? 

 

A) 0  B) 1  C) 2  D) 3    ¿Es en serio? Parece más un Wagener 2 por que llega a TCSC y tiene datos de infección.    DE ACUERDO A LOS DATOS CLÍNICOS PRESENTADOS USTED DECIDE INICIAR TRATAMIENTO ANTIBIÓTICO, ¿CUÁL SERÍA SU PRIMERA  ELECCIÓN?  A) TMP/SMX  B) CLINDAMICINA  C) AMOXI CLAV  D) FOSFOMICINA    TODAS SON MEDIDAS DE PREVENCIÓN PARA EL PACIENTE CON PIE DIABÉTICO, EXCEPTO:  A) CORTE DE UÑAS CON CORTAUÑAS  B) CONTROL GLUCÉMICO ADECUADO  C) CALZADO CÓMOCO  D) ACTIVIDAD FÍSICA 30 MINUTOS DIARIOS    RESPUESTA D, B, C, A 

SECUELA POSTROMBÓTICA  [Resumen la Salle]    DEFINICIÓN  Es el conjunto de signos y síntomas tardíos secundarios a la obstrucción y lesión valvular posteriores a una trombosis venosa profunda  que dan como resultado a una insuficiencia venosa secundaria.      FISIOPATOLOGÍA  Durante la formación de un trombo se da inicialmente la adhesión plaquetaria y activación de la cascada de coagulación, dentro de este  proceso se liberan citocinas proinflamatorias, se presenta acumulación de leucocitos (PMN), los cuales van a liberar enzimas  proteolíticas como las metaloproteinasas (colagensas, elastasas) con la finalidad de en conjunto con las plasmina lograr la disolución del  trombo. Sin embargo estas enzimas también lesionan las paredes del vaso ocasionado pérdida de la elasticidad y lesión de las válvulas  venosas.      “En una TVP se despliegan mecanismo antitrombóticos que se comen las válvulas”.    En la historia natural de la trombisis existen 3 finales distinso: recanalización (85‐100% a las 6 semanas), la mayoría de los segmentos  ocluidos inicialmente se recanalizan dejando como secuela la destrucción valvular. En los casos restante puede existir recanalizacion  parcial o bien organización de trombo con estenosis venosa. Recanalizació parcial y organización del trombo con estenosis venosa.      Se presenta en 29‐79% de los pacientes con TVP, aumentando la probabilidad de TVP proximal (ilio‐femoral) así como retrombosis,  hasta 23% puede ser severa (C4), y en 4‐6% desarrolar úlceras flebo estáticas (C6).    CLÍNICA  Es el de insfuciencia venosa con: dolor, pesantez, prutito, cansancio, edema y calambre en los miembros pélvico, los cuales son de  predominio vesperino, se exacerban con en orstostatismo y con el calor, y mejoran con el decúbito y el frío.    A la EF se verán cambios  como telangiectasis, venas reticualres y várices, edema, pigemnetación ocre, lipodermatoesclerosis, eccema o úlceras; es importante  destar que los cambios cutáneos inicialmente se presentan a nivel del maléolo medial.      Importante destacar que el miembro afectado en la mayorías de las ocasiones presenta aumento de perimetría >1cm, respecto al  contralateral.      TRATAMIENTO  Medidas de higiene venosa con elásticos de 30‐40mmHg.  Destarcar que las várices que se encuentran en contexto de una secuela postrombótia deberán ser valoradas por angiólogo para  determinar otra posibildiad de tratamiento.      PACIENTE FEMENINO DE 36 AÑOS, QUIEN AHCE 6 MESES PRESENTA EVENTO DE TORMBOSIS VENOSA PROFUNDA DE SEGMENTO  ILIOFEMORAL DE MIEMBRO PÉLVICO DERECHO DURANTE PUEPRERIO MEDIATO, FUE TRATADA CON ANTICOAGULACIÓN ORAL CON  CUMARÍNICOS, ACTUALMENTE YA SUSPENDIDOS.    ACUDE AL CENTRO DE SLUD YA QUE AHCE TRES MESES PRESENTA VÁRICES DE MIEMBRO PÉLVICO DERECHO, DOLOR PUNZANTE‐ URENTE, ACOMPAÑADO DE PRURITO, PENSANTEZ Y PRESENCIA DE CALAMBRES ESPECIALMENTE POR LA TARDE POSTERIOR A  PERMANECER MUCHO TIEMPO SENTADA, ASÍ MISMO, REFIERE QUE SU PIERNA DERECHA SE ENCUENTRA “MÁS HINCHADA” QUE LA  IZQUIERDA.    A LA EXPLORACIÓN FÍSICA DIRIGIDA A DICHA EXTREMIDAD USTED ENCUENTRA EDEMA +++, PRESENCIA DE PAQUETES VARICOSOS EN  CARA INTERNA DE PIERNA Y MUSLO, AUMENTO DE PERIMETRÍA MÁS DE UNA CM. RESPECTO AL MIEMBRO CONTRALATERAL, VENAS  RETICUALRES Y DISCRETA PIGMENTACIÓN OCRE A NIVEL DEL MALÉOLO MEDIAL.   

DE ACUERDO AL CASO CLÍNICO PRESENTADO¿CUÁL ES SU IMPRESIÓN DIAGNÓSTICA?  A) RETROMBOSIS VENOSA PROFUNDA  B) INSUFICIENCIA VENOSA CRÓNICA PRIMARIA  C) SECUELA POSTTROMBÓTICA  D) CELULITIS DEL MIEMBRO PÉLVICO DERECHO    ¿QUÉ ESTRUCTURA SE ENCUENTRA AFECTADA A NIVEL VASCULAR PARA EL DESARROLLO DE UNA SECUELA POSTRÓMBÓTICA?  A) VÉNULAS  B) CAPILARES  C) PARED ENDOTELIAL  D) VÁLVULAS    ¿DÓNDE SE PRESENTAN INICIALMENTE LOS CAMBIOS CUTÁNEOS ASOCIADOS A ESTA PATOLGÍA?  A) MALEOLO LATERAL  B) MALEO MEDIAL  C) SEGEMENTO INFERIOR DE LA PIERNA  D) A NIVEL DE GASTROCNEMIOS    ¿CUÁNTA ES LA COMPRESIÓN QUE DEBE APLICAR EL SOPORTE ELÁSTICO AL MIEMBRO AFECTADA DE ESTA PACIENTE?  A) 15‐20MMHG  B) 8‐10  C) 30‐40  D) 50‐60    RESPUESTA C, D, B       

VÁRICES  [Resumen la Salle]    DEFINICIÓN  Dilataciones venosas del sistema superficial >3mm.    Dentro de la insfuciencai venosa crónica se puede encontrar en C2 dentro de la clasificación de CEAP (C3 es edema, C4 cambios  dermatológicos, C5 úlcera cicatrizada y C6 úlcera activa).  Es importante destacar que las várices pueden presentarse en diferentes territorios: esófagno‐estógmado, recto (hemorroides),  testículos (varicocele), las cuales van más allá del abordaje del angiólogo y son vistas por otros especialistas.    FISIOPATOLOGÍA  Misma que enfermedad venosa crónica.    Hay factores genéticos como mutaciones en gen FOXC2 asociadas a falla valvular primaria. En la destrucción de las válvulas se ha  determinado el papel de enzimas como metaloproinasas que generand egracion de fibras de colágeno.  FR: edad, sexo femenino, gestación, antecedente familiar de IVC, ostostatismo prolonado, obesidad, sendentarismo.      CLÍNICA  Lo mismo que insuficiencia venosa crónica más dialtaciones benosas >3mm.      TRATAMIENTO  Medidas de higiene venosa con soporte elástico 18‐21 mmHg, en caso de no presenta rmejoría en 6 meses o presentar complicaciones  (tromboflebitis, varicotrombosis, varicorragia) se considera cnadidato a tx quirúrgico con safeno excéresis.    PACIENTE FEMENINO DE 48 AÑOS DE EAD, CON ANTECEDENTES DE IMPORTANCIA: OCUPACIÓN SECRETARIA, SEDENTARIA, GESTA 2,  CESÁREA 2, SIN OTROS DE RELEVANCIA, QUIEN NOTA QUE DESDE HACE 6 MESES PRESENTA DILATACIONES VENOSAS EN CARA INTERNA  DE LA PIERNA DERECHA, DOLOROSAS A LA PALPACÓN, ASOCIADAS A EDEMA DE PREDOMINIO VESPERTINO, PESANTES DE LA  EXTREMIDAD Y PRURITO OCASIONAL QUE MEJORA CUANDO EN DECÚBITO; ADEMÁS REFIERE PRESENCIA DE TELANGIECTASIS EN  MALEOLO INTERNO DERECHO DESDE HACE 10AÑOS.    A LA EF Y SOMATOMETRÍA TA 120/80, FC 80, FR 18 T 3, GB, IMG 24, A LA EXPLORACIÓN DEL MIEMBRO PÉLVICO HALLA DICHA  DIALTACIÓN DE PAQUETES VASCUALRES DESDE TERCIO INFERIOR DEL MUSLO, HASTA AL TERCIO MEDIO DE LA PIERNA, DE 4MM DE  DIÁMETRO, DOLOR A LA PALPACIÓN Y EDEMA +++ A NIVEL DEL TOBILLO DERECHO.    DE ACUERDO AL CASO CLÍNICO PRESENTADO ¿CUÁL ES SU IMPRESIÓN DIAGNÓSTICA?  A) INSUFICIENCIA VENOSA CRÓNICA SECUNDARIA  B) LINFEDEMA PRIMARIO  C) VÁRICES PRIMARIAS  D) VARICES SECUNDARIAS   

LA PACIENTE CUENTA CON TODOS LOS FACTORES DE RIESGO PARA DESARROLLO DE SU PATOLOGÍA, EXCEPTO:  A) OBESIDAD  B) SEXO  C) SEDENTARIAMOS  D) SEDESTACIÓN PROLONGADA.    CARACTERÍSTICA DEL EDEMA QUE PRESENTA ASOCIADO A VÁRIDES:  A) ASOCIADO AL DECÚBITO  B) PREDOMINIO VESPERTINO  C) LA CONSISTENCIA ES DURA  D) MEJORA CON EL CALOR    ¿CUÁL ES EL TRATAMIENTO QUE USTED OFRECE INICIALMENTE A ESTA PACIENTE?  A) SAFENO EXÉRESIS  B) LIGADURA DE PERFORANTES  C) MEDIDIAS DE HIGIENE VENOSA  D) LIGADURA DE PAQUETES VASCULARES.    Recordar que si solot eine várices las medias deben ser de 18‐21mmHg (equivale a un C2).  Si fracasa luego de 3‐6 meses se refeire y se valora safeno exéresis.    RESPUESTA C, A, B, C                                           

                            CASOS CLÍNICOS ANALIZADOS POR CLASIFICAR      [r]HOMBRE DE 72 AÑOS DE EDAD, SE ENCUENTRA HOSPITALIZADO HACE 7 DÍAS POSTERIOR A UNA COLECTOMÍA SECUNDARIA A LA  PERSENCIA  DE  DIVERTICULITIS  QUE  NO  CEDIÓ  A  TRATAMIENTO  ATNIMICROBIANO  PREVIAMENTE.  EL  PACIENTE  PRESENTA  DOLOR  INTENSO EN ABDOMEN QUE NO CEDE ANTE LA INGESTA DE MEDICAMENTOS. A LA EXPLORACIÓN FÍSICA: FC 90 FR 12, T38 TA 100/60. SE  REALIZAN ESTUDIOS DE IMAGEN MEDIANTE LOS QUE S ECORROBORA FUGA DE ANASTOMOSIS COLÓNICA, POR LO QUE EL PACIENT EES  REINTE4RVENIDO QUIR´RUGICAMENTE.    ¿CUÁL ES TRATAMIENTO ANTIMICROBIANO MÁS ADECUADO PARA ESTE PACIENTE  A) AMOXICILINA ‐ VANCOMICINA  B) LINEZOLID‐ FLUCONAZOL  C) PIPERACILINA‐TAZOBACTAM  D)ANFOTERICINA‐ METONIDAZOL    RESPUESTA C  El  paciente  tiene  fuga  anastomótica.  La  fuga  anastomótica,  desencadena  sepsis  abdominal  o  en  absceso  intaperitoneales.  Las  manifstaciones que presenta son dolor abdominal, fiebre que no tarda en vlucionar a peritonitis generalizada, y choque séptico pr lo que  el tratamiento oportuno es lo ideal . El paciente debe tener cbertura poara gramnegativas, gram positivas y anerobios demás de cubrir  pseudomonas spp.  Piperacilina‐Tazobactam, debido a que el paciente fue tratad previamente con antibióticos y por su estancia hospitalaria.             [QUISTE PILONIDAL]  [r]HOMBRE  DE  38  AÑOS  DE  EDAD,  QUE  ACUDE  A  SU  CONSULTA  REFERIDO  DEL  CENTRO  DE  SALUD  CON  DIAGNÓSTICO  DE  QUISTE  PILONIDAL. A LA EXPLORACIÓN LO ENCUENTRA EN FASE AGUDA, INFECTADO Y ABSCEDADO.  EN ESTE PACIENTE EL TRATAMIENTO INDICADO ES:        A) PUNCIÓN Y ASPIRACIÓN DE CONTENIDO.  B) ADMINISTRACIÓN DE ANTIBIÓTICOS SISTÉMICOS.  C) LAVADO CON ANTISÉPTICOS Y EMPAQUETAMIENTO CON GASAS CON ISODINE.  D) DRENAJE ABIERTO Y MARSUPIALIZACIÓN.    RESPUESTA  El tratamiento del quiste pilonidal en fase aguda es el drenaje del absceso y marsupialización de los bores, se puede dar antibiótico de ser  necesario.      Una vez resuelta la fase aguda se puede vigilar la recidiva, o se puede operar directamente (excisión del seno pilonidal completo hasta  fascia persacra y legrado del tej de granulación, herida puede quedar abierta y esperar cierre por segunda intención).    [ÚLCERA PÉPTICA]  HOMBRE DE APROXIMADAMENTE 30 AÑOS DE EDAD, ES ENCONTRADO EN LA CALLE CON PÉRDIDA DEL ESTADO DE ALERTA E INTENSO  ALIENTO ALCOHÓLICO. ES LLEVADO AL SERVICIO DE URGENCIAS DEL HOSPITAL. A LA EXPLORACIÓN FÍSICA CON MALA HIGIENE, RESTOS  DE  VÓMITO  EN  SU  ROPA,  GLASGOW  DE  13‐14,  TA  90/55,  FC  115X´,  FR  26X´,  TEMPERATURA  38.9°C,  EL  ABDOMEN  SE  ENCUENTRA  DISTENDIDO,  REFLEJO  DOLOROSO  A  LA  PALPACIÓN  MEDIA  Y  PROFUNDA  PRINCIPALMENTE  EN  EPIGASTRIO,  REBOTE  GENERALIZADO,  PÉRDIDA DE LA MATIDEZ HEPÁTICA Y TIMPANISMO GENERALIZADO.          PARA APOYAR SU DIAGNÓSTICO DEBERÁ SOLICITAR DE MANERA INICIAL:  A) ULTRASONIDO ABDOMINAL.  B) ESTUDIOS CON MEDIO DE CONTRASTE.  C) TOMOGRAFÍA COMPUTADA. 

D) RADIOGRAFÍA DE TÓRAX Y ABDOMEN.       EL DIAGNÓSTICO CLÍNICO MÁS PROBABLE ES:  A) APENDICITIS AGUDA COMPLICADA.  B) VÓLVULO INTESTINAL.  C) ULCERA PÉPTICA COMPLICADA.  D) PANCREATITIS AGUDA.          EL TRATAMIENTO INDICADO EN ESTE CASO ES:    A) RESECCIÓN INTESTINAL Y COLOSTOMÍA EN BOLSA DE HARTMANN.  B) CIERRE CON PARCHE DE GRAHAM.  C) APENDICECTOMÍA.  D) COLOCACIÓN DE SONDA, ANALGÉSICOS Y ANTIBIÓTICOS.    RESPUESTAS D, C, B        [HERNIA HIATAL POR DESLIZAMIENTO]     CONTENIDO  Principal objetivo de procedimiento antirreflijo: recuperar el ángulo de His, recuperar la presión del EEI.      MUJER  DE  60  AÑOS  DE  EDAD  CON  OBESIDAD  MÓRBIDA.  PRESENTA  DESDE  HACE  6  MESES  CUADROS  INTERMITENTES  DE  PIROSIS,  REGURGITACIÓN  Y  DISFAGIA  OCASIONAL,  QUE  CEDEN  CON  INHIBIDORES  DE  LA  BOMBA  DE  PROTONES.  SE  LE  INDICA  UNA  SERIE  ESÓFAGOGASTRODUODENAL, PARA COMPLEMENTAR SU ESTUDIO, OBTENIÉNDOSE LA SIGUIENTE IMAGEN: 

  EL DIAGNÓSTICO RADIOLÓGICO MÁS PROBABLE ES EL DE:  A) HERNIA HIATAL POR DESLIZAMIENTO.  B) REFLUJO GRASTROESOFÁGICO.  C) ACALASIA.  D) ANILLO DE SCHATZKI.    RESPUESTA A  Las hernias hiatales se dividen en 4 tipos principales:  TipoI/deslizante: 95%, secundaria    debilidad y elongación de las fibras frenoesofágicas.  Tipo II/paraesofágicas: esta es una hernia real donde el fundus se torna paralelo al esófago secundario a la debilidad de la membrana  peluroperitoneal.  Tipo III/mixtas: componentes de las antriores  Tipo IV/complejas: migración intratorácica de cualquier otro intrabdominal    Síntomas clásicos: regurgitación, pirosis y disfagia. La disfagia puede ser secundaria a edema de mucosa, anillo de Schatzki, estenosis    o  incapacidad para organizar la actividad peristáltica en el cuerpo del esófago. Estudio de elección: SEGD 

 

ACALASIA  [r]HOMBRE DE 30 AÑOS DE EDAD, ACUDE POR PRESENTAR DISFAGIA Y REGURGITACIÓN PROGRESIVA. USTED SOSPECHA LA POSIBILIDAD  DE ACALASIA.  EL ESTUDIO QUE DEBERÁ SOLICITAR PARA CONFIRMAR SU SOSPECHA DIAGNÓSTICA ES:  A) UN ESOFAGOGRAMA CON BARIO.  B) UNA ENDOSCOPÍA.  C) UNA PH METRÍA DE 24 HRS.  D) UNA MANOMETRÍA.    EN CASO DE CONFIRMAR SU DIAGNÓSTICO EL PACIENTE DEBERÁ SER TRATADO CON  A) ANTAGONISTAS DE CALCIO.  B) TOXINA BOTULINICA.  D) DILATACIONES NEUMÁTICAS.  E) MIOTOMÍA.    RESPUESTAS D, E  Para el ENARM es muy importante comprender qué es lo que te están preguntando.    El estudio inicial en paciente con acalasia es el esofagograma  con bario, el cual demustra la imagen clásica en "punta de lápiz". Este  estudio orienta al diagnóstico pero no lo confirma. La manometría es el estudio ideal para confirmarlo y demostrar ausencia de relajación  del esfinter esofágico inferior y aperistalsis del cuerpo esofágico. Hay que recordar que en estadios iniciales aun no es posible encontrar  la clásica imagen en el esofagograma.  La endoscopia y la TAC son estudios útiles para diferenciar de acalasia de pseudoacalasia.  La pHmetría en 24 hrs mide el reflujo ácido y es gold standar de ERGE, sin utilidad diagnóstica en acalasia.  La miotomía y las dialtaciones son los únicos tratamiento duraderos en acalasia, y son más efectivos en pacientes vírgenes a tratamientos  médicos. La miotomía de Heller con funduplicatura parcial tiene menor riesgo y es más eficaz que las dilatacines, por lo que en este  paciente es gold standar.        UN HOMBRE DE 65 AÑOS DE EDAD SE PRESENTA A SU MÉDICO QUEJÁNDOSE DE DIFICULTAD PARA DEGLUTIR, DOLOR EN EL PECHO DE  VEZ EN CUANDO, Y REGURGITACIÓN DE ALIMENTOS. EN LOS ÚLTIMOS 2 MESES HA PERDIDO ALREDEDOR DE 7 KG. LOS RESULTADOS DE  UN ESTUDIO DE DEGLUCIÓN DE BARIO SE MUESTRAN EN LA IMAGEN. ¿QUÉ PRUEBA SE DEBE REALIZAR PARA BUSCAR POSIBLES CAUSAS  DE SU CONDICIÓN? 

 

  A. 

PHMETRÍA DE 24 HORAS  B. MANOMETRÍA ESOFÁGICA  C. MEDICIÓN DEL NIVEL DE GASTRINA SÉRICA  D. ENDOSCOPIA SUPERIOR  E. PRUEBA DEL ALIENTO CON UREA  RESPUESTA D  (YO)       

COLECISTITIS  MUJER DE 35 AÑOS CON ANTECEDENTE DE CIRUGÍA BARIÁTRICA POR OBESIDAD MÓRBIDA HACE 6 MESES. ACUDE A CONSULTA DEBIDO  A CUADRO CARACTERIZADO POR DOLOR INTERMITENTE EN CUADRNATE SUPERIOR ERECHO SOBRE TODO RELACIOADO A COSUMO DE  ALIMENTOS GRASOSOS, REFIERE QUE HACE UN MES TUVO UN EPISODIO    SIMILAR. SE REALIZA USG DE HÍGADO Y VÍAS BILIARES CON  REPORTE DE DOS LITS VESICULARES DE 10‐14MM, RESTO SIN ALTERACIONES.  ¿CUÁL ES LA CONDUCTA MÁS APROPIADA PARA ESTA PACIENTE?    A)TRATAMIENTO SINTOMÁTICO  B)COLECISTECTOMÍA  C)ÁCIDOS BILIARES  D)LITOTRICIA   

RESPUESTA    A  El USG de la paciente solo muestra lito, más no "agudización" o inflamación de la vesícula.    Criterios  para  colecistectomía  laparoscópica  profiláctica.  Vesícula  biliar  calcificada,  cálculos  mayores  de  3cm  o  paciente  candidato  a  cirugía bariátrica o trasplante cardiaco.    Criterios para litotricia: litiasis única, no calcificada con diámetro de 20‐30mm  Acidos bilaires: se reservan para pacientes con alto riesgo quirúrgico, pero debido a su costo y efectos secundarios son poco utilizados.     

PANCREATITIS AGUDA  HOMBRE DE 56 AÑOS DE EDAD, ACUDE AL SERVICIO DE URGENCIAS POR PRESENCIA DE DOLOR ABDOMINAL INTENSO,  INCAPACITANTE  Y  QUE  IMPIDE  LA  DEAMBULACIÓN.  A  LA  EXPLORACIÓN  PRESENTA  ABDOMEN  DISTENDIDO,  EXTREMADAMENTE  DOLOROSO  A  LA  PALPACIÓN  Y  LA  PERISTALSIS  SE  ENCUENTRA  DISMINUIDA.  SE  SOLICITA  RX  ABDOMINAL ENCONTRÁNDOSE LA SIGUIENTE IMAGEN   

  1)LA IMAGEN RADIOLÓGICA OBTENIDA APOYA EL DIAGNÓSTICO DE:    A) APENDICITIS AGUDA  B) ÚLCERA PÉPTICA PERFORADA  C) PANCREATITIS  D) COLECISTITIS AGUDA    2)UNA VEZ ESTABLECIDO EL DIAGNÓSTICO, DEBERÁ INDICARSE EL SIGUIENTE TRATAMIENTO:    A) APENDICECTOMÍA ABIERTA  B) REPOSO PANCREÁTICO  C) HEMOSTASIA ENDOSCÓPICA  D)COLECISTECTOMÍA LAPAROSCÓPICA    RESPUESTAS C, B  La Rx de tórax de un paciente con pancreatitis puede mostrar elevación de hemidiafragmas, derrame pleural, atelectasia  o neumonía, (la presencia de derrame pleural asocia pobre desenlace). Rx de abdomen puede msotrar cálculos biliares,  asa centinela (segmento de intestino lleno de aire generalmente en cuadrante superior izquierdo), signo del colon cortado  (se interrumpe de forma súbita donde hay inflamación pancrática), o aumento del espacio gastrocólico.    Se recomienda reposo pancreático, junto con alimentación enteral temprana con sonda nasoyeyunal, transpilórica y en  casos de cirugía yeyunostomía. Se realiza cirugía inmediata si se complica con pancreatisis necrótica o colangitis aguda.        FEMENINA  DE  39  AÑOS,  QUE  POSTERIOR  A  PERMANECER  HOSPITALIZADA  POR  3  DÍAS  CON  DIAGNÓSTICO  DE  PANCREATITIS AGUDA DE ORIGEN BILIAR, SE ENCUENTRA CON DOLOR ABDOMINAL PERSISTENTE, CON INCREMENTO DEL  PERÍMETRO ABDOMINAL, SE MANTIENE AFEBRIL Y SIN ICTERICIA. SE SOLICITA BIOMETRÍA QUE REPORTA LEUCOCITOS DE  18,000/MM3. 

  EL PROCEDIMIENTO QUE SE DEBERÁ SOLICITAR PARA ESTABLECER EL DIAGNÓSTICO EN ESTE MOMENTO ES:  A)LAVADO PERITONEAL DIAGNÓSTICO.  B)TOMOGRAFÍA ABDOMINAL.  C)ULTRASONOGRAFÍA ABDOMINAL.  D)RADIOGRAFÍA DE TÓRAX.    EN ESTE MOMENTO SE DEBE INDICAR:  A) DEBRIDACIÓN PANCREÁTICA INMEDIATA.  B) COLANGIOPANCREATOGRAFIA RETROGRADA ENDOSCOPICA(CPRE).  C) COLECISTECTOMÍA ABIERTA URGENTE.  D) SONDA NASOGÁSTRICA, LIQUÍDOS Y ELECTROLITOS, ANTIBIOTICOS, ANALGÉSICOS Y EL APOYO NUTRICIONAL.      RESPUESTAS      B, D  ¿NO DEBERÍA SER SONSA NASOYEYUNAL?    REFLUJOGASTROESOFÁGICO  [r]HOMBRE DE 51 AÑOS DE EDAD QUIEN PADECE DESDE HACE 10 AÑOS ENFERMEDAD POR REFLUJO GASTROESOFÁGICO  TRATADA SÓLO DE MANERA PARCIAL, ES SOMETIDO A ENDOSCOPÍA, REPORTÁNDOSE EPITELIO ROSA SALMÓN ARRIBA  DE LA UNIÓN GASTROESOFÁGICA.    CON BASE EN EL HALLAZGO ENDOSCÓPICO LO MÁS PROBABLE ES QUE EL PACIENTE PRESENTA EN ESTE MOMENTO:  A) ESOFAGITIS.  B) CARCINOMA ESCAMOCELULAR.  C) METAPLASIA.  D) ACALASIA.    CORRESPONDE AL TRATAMIENTO DE ELECCIÓN EN ESTE CASO:  A) INHIBIDORES DE LA BOMBA DE PROTONES.  B) ANTIHISTAMÍNICOS H2.  C) ESOFAGECTOMÍA.  D) FUNDUPLICATURA    RESPUESTA C, A    Se trata de paciente con ERGE es somentido a endoscopia y se muestras el hallazgo de epitelio color salmón.    Hay que recordar que una metaplasia es el cambio de una células por otra similar, se dice que que hay displasia cuando  hay pérdida de la arquitectura habitual de la célula. El esófago de Barret es una metaplasia que endoscópicamente luce  como  una  sona  de  epitelio  color  salmón,  razón  por  la  cual  el  apciente  presenta  una  metaplasia.  Y  el  tratamiento  conducente es con IBP.      Por medio de endoscopia la esofagitis se puede clasificar en grados:    Grado I‐  Erosiones pequeñas, circulares, no confluyen.    Grado II‐  Erosiones lineales cubiertas por tejido de granulación que sangran facil si se toca.      Grado III‐ Erosiones lineales o circulares que confluyen para formar área circular sin epitelio    Grado IV‐ Estenosis (obligada la toma de biopsias)      Respecto al tratamiento hay que recordar que de primera elección están los IBP.    Displasia leve: IBP x 1‐2meses y repetir endoscopia.    Displasia de alto grado: vigilancia estricta endoscópica c/ 3 meses.  Los inhibidores H2 solo están indicados junto con funduplicatura en pacientes con displasia leve y después del tratamiento  inicial con IBP.  Esofagectomía inciada en jovenes que no están dispuestos a control endoscópico trimestral.    La funduplicatura está inciada en quienes no teleran los IBP o no responden a ellos.        OSTEOSARCOMA  MASCULINO DE 19 AÑOS, REFIERE DOLOR EN MUSLO IZQUIERDO INCREMENTÁNDOSE POR LAS NOCHES. NO HAY RELACIÓN CON LA 

ACTIVIDAD FÍSICA Y SE NIEGA FIEBRE O PÉRDIDA DE PESO. A LA EXPLORACIÓN SE PALPA MASA DURA Y FIJA A PLANOS PROFUNDOS EN  TERCIO DISTAL DE FÉMUR. LA RADIOGRAFÍA PRESENTA LESIÓN PENETRANTE EN FÉMUR CON DESTRUCCIÓN CORTICAL CON ASPECTO DE  "RAYOS SOLARES".    EL DIAGNÓSTICO PROBABLE DE ESTE PACIENTE ES EL DE:  A) OSTEOSARCOMA.  B) QUISTE ÓSEO.  C) OSTEOMIELITIS.  D) OSTEOCONDROMA    LOS HALLAZGOS RADIOLÓGICOS PRESENTES EN EL PACIENTE SON SECUNDARIOS A:  A) INFECCIÓN EN UN SITIO CERCANO AL HUESO.  B) DISPLASIA ÓSEA Y PLEOMORFISMO CELULAR METAFISIARIO.  C) REMANENTE DE LA PLACA DE CRECIMIENTO DEL CARTÍLAGO  D) INFECCIÓN QUE VÍA HEMATOGENA LLEGA A LA ARTICULACIÓN.    RESPUESTAS  A, B,      MUJER  DE  23  AÑOS  DE  EDAD  PRESENTA  RADIOGRAFÍA  CON  DATOS  DE  TUMORACIÓN  MUTILOCULADA  EN  LA  ZONA  EPIFISARIA  DEL  EXTREMO DISTAL DE FÉMUR IZQUIERDO. ¿CUÁL ES EL DIAGNÓSTICO MÁS PROBABLE?  A) SARCOMA OSTEOGÉNICO  B) TUMOR DE CÉLULAS GIGANTES  C)SARCOMA DE EWING  D)MIELOMA MÚLTIPLE     RESPUESTA B  EL ASPECTO MULTILOCULADO O EN POMPAS DE JABÓN ES CARACTERÍSTICO DEL TUMOR DE CÉLULAS GIGANTES, MUY FRECUENTE EN  MUJRES DE 20‐40 AÑOS] CTO TRAUMATLOGÍA PAG 56  El tumor de células gigantes es un tumor raro, constituye el 5% de los tumores óseos primarios. Ocurre en pacientes entre los 20 y 40  años, siendo raro en los menores de 10 y en mayores de 50 años. Tiene un ligero predominio del sexo femenino y más frecuente en países  orientales que occidentales. Habitualmente se loalizan en huesos largos, con afectación de la epífisis y de localización excéntrica, pueden  propagarse a metáfisis, provocar destrucción cortical y extensión eventual a tejidos blando y al especio articular.    Los tres sitios más habituales de localización en orden de frecuencia son: 1 extremo distal del fémur, 2 extremo proximal de tibia, 3  extremo distal del radio.    Clínicamente, el dolor es el signo más freucente de presentación, tanto si se asocia o no a fractura patológica. Localmente la piel puede  estar hiperémica, eritematosa y si el tumoer crece puede aparecer ciruclación colateral con presencia de masa palpable. La clínica es  infrecuente y si el TCG tiene localización raquídea o sacra presenta signos y síntomas neurológicos.    El  diagnóstico  de  TCG  suele  realizarse  por  laws  manifestaciones  clínicas  y  radiográficas,  teniendo  en  cuenta  lo  spisible  sdiagnóstico  diferenciales  Cmo  pruebas  complementarias,  la  RMN  es  el  método  más  aprovechoso  para  determinar  la  extensión  y  l  estadiaje,  la  gammagrafía es uilizada para detectar TCG multicéntricos que son raros (500cc es alto gasto y nos indica que está obstruído de forma completa.      En incompletas o parciales mecánicas, tx con medio de contraste 200‐250ml, es osmótico, resuelve obstrucción.        HOMBRE DE 38 AÑOS DE EDAD, CON ANTECEDENTE DE APENDICECTOMÍA HACE 6 MESES Y EN ESTA OCASIÓN INGRESA CON DATOS DE  OBSTRUCCIÓN INTESTINAL.  LA CAUSA MÁS PROBABLE POR LA CUAL EL PACIENTE PRESENTA ESTE CUADRO CLÍNICO ES:      A) ADHERENCIAS INTESTINALES.  B) ABSCESO RESIDUAL.  C) CÁNCER DE COLÓN.  D) ADENITIS MESENTÉRICA.    RESPUESTAS A  El  bloqueo  de  la  luz  intestinal  se  produce  en  su  mayoría  por  factores  extrínsecos  (adherencias,  hernia  inguinal,  hernia  de  pared  estrangulada) y en pocas ocasiones por factores intrínsecos.    Las adherencias intraabdominales por intervenciones quirúrgicas previas  representan el 75% de los casos de obstrucción intestinal.         [COLECISTITIS CRÓNICA LITIÁSICA] 

MUJER  DE  46  AÑOS  DE  EDAD,  SORDOMUDA,  CON  SOBREPESO  EVIDENTE.  LLEGA  SOLA  A  URGENCIAS  POR  DOLOR  ABDOMINAL.  SE  DESCONOCEN ANTECEDENTES. DURANTE SU ESTANCIA EN URGENCIAS PRESENTA DOS VÓMITOS DE CONTENIDO GÁSTRICO. SE SOLICITA  UNA RADIOGRAFÍA DE ABDOMEN DE PIE OBTENIENDO LA SIGUIENTE IMAGEN. 

    EL DIAGNÓSTICO MÁS PROBABLE DE ESTA PACIENTE ES:      A) LITIASIS URETERAL.  B) LITIASIS RENAL.  C) COLECISTITIS AGUDA LITIÁSICA.  D) TUMOR RENAL.    RESPUESTA C    La ausencia de sintomatología urinaria y la localización de los litos nos hace descartar litiasis ureteral y renal. Recordar que los litos renales  puden  ser  de  calcio,  ác  urico,  estuvita  y  cisteína.  Los  de  ác  úrico  se  forman  en  orina  ácida  y  son  radiotranspartentes,  el  resto  son  radioopacos.  Pueden  originarse  por  hipocitraturia,  hiperoxaluria,  hipercalciuria  e  hiperuricosuria.  Un  pH  >7.6  nos  hace  pensar  en  bacterias  productoras  de  ureasa.  Si  pH  6‐7  será  necesario  considerar  acidosis  tubular  como  causa  de  nefrocalcinosis.  Si  pH hiperparatiroidismo (medir parathormona)  Los cálculos no radioopacos se pueden observar como un defecto de llenado cuando se usa medio de contraste.      El 70‐80% de los tumores malignos del riñón corresponde a carcinoma de células claras, 50‐60% hace mets a pulmones, 25 a ganglios  hiliaries renales, 12 suprarrenal ipsilateral, 30‐40% con lesiones líticas en huesos. Principal manifestación es hematuria macro o micro  (60%). Dx de imagen de elección TAC abdominal.      [ABSCESO PERIANAL]  MUJER DE 33 AÑOS DE EDAD, DE OCUPACIÓN SECRETARIA, POR LO QUE LA MAYOR PARTE DE SU DÍA LABORAL LA PASA EN POSICIÓN  SENTADA. USTED CONFIRMA EL DIAGNÓSTICO DE ABSCESO PERIANAL AGUDO  EL MECANISMO FISIOPATOLÓGICO QUE MÁS PROBABLEMENTE ANTECEDÍO A LA FORMACIÓN DEL ABSCESO PERIANAL DE LA PACIENTE  FUE:      A) UNA FISURA ANAL.  B) UNA FÍSTULA PERIANAL.  C) INFLAMACIÓN CRIPTOGLANDULAR.  D) COLITIS ISQUÉMICA.      LA COMPLICACIÓN ESPERADA DEL ABSCESO PERIANAL DE LA PACIENTE EN CASO DE NO TENER UNA RESOLUCIÓN PRONTA SERÍA:   

A) FISURA ANAL  B) FÍSTULA PERIANAL  C) SEPSIS ABDOINAL  D) PROLAPSO RECTA.      RESPUESTAS C, B    El Absceso anal    frecuente entre 20‐60 años, en ambos sexos. Causas específicas:    cuerpos extraños, traumatismos, enfermedades  inflamatorias intestinales (Crohn) procesos infecciosos específicos, tumores, secuelas de tratamientos radioterápico y otras enf anales  como la fisura anal. Causas inespecíficas: como la enf criptoglandular.    La teoría criptoglandular es la base fisiopatológica más aceptada  del absceso perianal.    La Fisura anal    es una úlcera longitudina por debajo de la línea dentada. Generalmente posterior, menos habitual anterior y en 3%  pueden  coexistir  las  dos.    Incidencia  igual  en  ambos  sexos,  frec  en  edad  media  de  la  vida.  Generalmente  idiopáticas.  Etiología  más  probable traumatismo agudo del conducto anal durante la defecación.    Si fístulas múltiples pensar en tuberculosis, sífilis, Crohn o sx de  inmunodeficiencia.    Si es crónica se verá como úlcera con bordes indurados, acompañada de pliegue cutáneo indurado en el extremo  distal  (hemorroide  centinela)  y  una  papila  hipertrófica  en  el  borde  proximal  (pólipo  de  Lane).  La  físura  puede  acompañar  al  absceso  perianal agudo e incluso favorecerlo, pero no forma parte de la fisiopatología.      La Fístula perianal es un conducto que comunica una cripta anal o el lumen del recto con la piel. Las fístulas simples son normalmente  interesfinterianas  [B]  o  transesfinterianas  bajas  [C],  generalmente  de  trayecto  único.    La  fístula  compleja  tiene  un  trayecto  que  comprende más del 30‐50% del esfínter externo (transesfintérica alta [C] , supraesfintérica [D] o extraesfintérica[E]), es de localización  anterior en mujeres.    Se presentan en 50%    de los pacientes con absceso anorrectal desarrollan una fístula.    Los antecedentes de  importancia  en  este  caso  son:    patología  intestinal  asociada,  eventos  que  comprometan  el  esfínter,  cirugía  anorrectal,  trauma  obstétrico,  procesos  infecciosos  anorrectales.  Presentan  como  descarga  a  través  del  orificio  externo,  en  ocasiones  dolor.  EF  orificio  externo con descarga y se palpa trayecto con fibrosis. La fístula perianal es más bien una complicación del absceso.      La Colitis isquémica es la forma de presentación más frecuente de isquemia intestinal (70‐75%). Afecta princilamente a adultos mayores,  y en jovenes asocia DM2, LES o crisis de células falciformes. La pancreatitis se ha asociado como factor al ocluir en ocasiones los vaso  mesocólicos.    La  fisiopatlogía  de  la  colitis  isquémica  yace  en  una  insuficiencia  venosa  o  arterial  previa.  La  colitis  isquémica  derecha  aislada de relaciona con    cardiopatía crónica, en especial estenosis aórtica. Se manifiesta como un dolor abdominal agudo, hemorragia  gastro intesitnal y diarrea aguda. No se relaciona con abscesos perianales, patología distinta.        Causas de sepsis abdominal:      Colecistopancreatitis      27.5%    Perforación de colon    17.2    Heridas penetrantes abd.  13.7    Apendicitis aguda    13.7    Politraumtizado    10..3    Abscesos peripancráticos  6.9    Trombosis mesentéricas  3.4    El prolapso rectal se asocia a: demensia senil, parasitosis (esquistosomiasis, tricocefalosis, amebiasis), enf neurológias (espina bífida),  multiparidad, histectomía, estreñimiento de larga evolución, hipermotilidad sigmdea.     

                         [CÁNCER DE PRÓSTATA]  HOMBRE DE 64 AÑOS DE EDAD QUE ACUDE A LA CONSULTA PORQUE DESDE HACE 3 MESES PRESENTA DISMINUCIÓN DEL CALIBRE DEL  CHORRO URINARIO, GOTEO TERMINAL, POLAQUIURIA Y DISURIA. A LA EXPLORACIÓN SE REALIZA TACTO RECTAL ENCONTRANDOSE LA  PRÓSTATA CON UN NÓDULO PÉTREO EN LA PARTE POSTERIOR DE ESTA GLÁNDULA.  PARA CONFIRMAR EL DIAGNÓSTICO SE DEBERÁ INDICAR EL SIGUIENTE ESTUDIO:  A) ULTRASONIDO PROSTÁTICO.  B) FOSFATASA ALCALINA EN SUERO.  C) NIVELES DE ANTÍGENO PROSTÁTICO ESPECÍFICO.  D) BIOPSIAS TRANSRECTALES DE PRÓSTATA.    RESPUESTA D  No se recomienda tamizaje para próstata a población asintomática entre 50‐69 años    de edad empleando la prueba de APE y TR (ya que  no asocia reducción de mortalidad).  La detección basada en APE demostró reducción del 20% pero se relaciona con importante riesgo de sobrediagnóstico.    La mayor parte de cánceres se presentan en la parte posterior de la glándula en forma de nódulos indoloros y pétreos.    La biopsia de próstata guida por USG es la única prueba confirmatoria, idelmente tres de la derecha, tres de la izquieda y si procede  clínicamente 1 de la zona transicional. No se recomienda si prostatitis, a menos que haya recibido un ciclo de antibioticoterapia.    TR VPP 21%. 17% de pacientes con PSA 2.5‐4ng/ml y datos normales en TR tiene cáncer.        [r]  HOMBRE  DE  60  AÑOS  DE  EDAD  QUE  ACUDE  A  CONSULTA  CON  SINTOMATOLOGÍA  URINARIA  IMPORTANTE.  USTED  SOSPECHA  LA  POSIBILIDAD DE CÁNCER DE PROSTATA.  EN ESTE CASO DEBERÁ IDENTIFICAR EL SIGUIENTE FACTOR DE RIESGO:      A) CONSUMO DE DIETA CON ELEVADO CONTENIDO DE GRASAS.  B) CONSUMO OLIGOELEMENTOS.  C) EL CONTACTO PERMANENTE CON SUBSTANCIAS DERIVADAS DE LA ACTIVIDAD INDUSTRIAL.  D) LA RAZA O GRUPO ÉTNICO    RESPUESTA D  Factores ambientales: zona geográfica, dieta alta en grasa, exposición a humo de automóviles, polución del aire, cadmio, fertilizantes, 

sustnacias de la industria de la goma, imprenta y pintura.    Factores genéticos: gen HPC 1.  Factores infecciosos: ITS.  Factores hormonales: andrógenos (en eunucos no aparecen).    El consumo elevado de grasa si es FR pero no el más importante, los oligolementos no tienen nada que ver.    La sustnacias de la industria de pueden generar riesgo pero la respuesta es vaga, no especifica que sea de pintura y la goma.    La raza negra es el mayor factor de riesgo de neoplasia prostática intraepilial multifocal y con gran inestabilidad. Quizás por altos niveles  de testosterona.  En términos generales son los factores de riesgo más imprtantes para Ca prostático: Edad avanzada, antc familiares, raza negra.        HOMBRE DE 35 AÑOS DE EDAD ACUDE A CONSULTA CON MÉDICO GENERAL. EL PACIENTE SE MUESTRA CONSTERNADO DEBIDO A QUE  DOS DE SUS TÍOS LES DETECTARON CÁNCER DE PRÓSTATA A LOS 50 Y EL OTRO A LOS 65 AÑOS, NIEGA SINTOMATOLOGÍA ASOCIADA Y  NO CUENTA CON ANTECEDENTS DE IMPORTANCIA. AL TACTO RETAL SIN PRSENCIA DE NÓDULOS O ALTERACINES. EL APE ES DE 2.3NG/DL.    ¿CUÁL ES LA RECOMENDACIÓN MÁS APROPIADA PARA EL PACIENTE?  A) DETERMINACIONES BIENALES DE APE A PARTIR DE LOS 65‐85 AÑOS  B) DETERMINAICONES ANUALES Y BIENALES DE APE DE LOS 35‐75 AÑOS  C) DETERMINACIONES ANUALES DE APE DE LOS 40‐75 AÑOS  D) DETERMINACIONES ANUALES DE APE DE LOS 50‐75 AÑOS    RESPUESTA C  Ca de próstata tiene fenómeno de anticipación de 7 años. El paciente en cuestión tiene factores de riesgo y por lo tanto el cribado se  inicia con APE a los 40 años, de no tenerlos sería    a los 50.    [info del proedumed]    El antígeno prostático específico es el examen más importante según las guías de la Secretaría de Salud, a pesar de que hay varios países  que lo han retirado de sus guías de práctica clínica, porque aunque detecta    el cánr no modifica la motalidad.  un gram de cáncer produce 3ng/ml de antígeno.    los valores normales van de 0‐4ng/ml. La velocidad de crecimiento anual de más de 1ng/ml por año se considera anormal. El porcentaje  de antígeno libre también se puede detectar, y si este es menor a un 10% en general es debido a áncer y es más de 25% probablemnte  se trate de un adenoma. El estándar de oro para el diag nóstico es la biopsia. Se sugiere realizar determinaciones deanuales de APE, pero  denbido al crecimiento lento del cáncer, se podría realizar también en intervalos de 2 años iniciando desde los 40‐45 años hasta los 75‐ 65 añls si el pacient presentó determinaciones bajas persistentes entre 0.5‐1ng/dl. [reto enarm]      HOMBRE DE 67 AÑOS DE EDAD QUE ACUDE A CONSULTA PORQUE, HACE ALGUNOS MESES, DEBE PUJAR CON ÁS FUERZA PARA PODER  ORINAR Y AUN ASÍ, EL CHORRO URINARIO HA DISMINUIDO DE CALIBRE. AL INTERROGATORIO DIRIGIDO REFIERE QUE SE PARA A ORINAR  MÁS DE DOS VECES POR LA NOCHE. NIEGA DISURIA, HEMTURIA O ALGUNA CONDICIÓN ASOCIADA. TIENE DIAGNÓSTICO DE DM2 HACE  10 AÑOS, EN TRATAMIENTO REGULRA. FC 77, FR 29 T 36 TA 115/67. A LA EXPLORACIÓN FÍSICA SE PALPA PRÓSTATA SIN NÓDULOS O  INDURACIONES PERO AUMENTADA DE TAMAÑO. LOS ESTUDIOS DE LABORATORIO REPORTAN APE 4‐5NG/DL. SE REALZÓ UN USG QUE  DEMOSTRÓ UNA PRÓSTATA DE 45 CC. SIN NÓDULOS NI CALCIFICACIONES.  ¿CUÁL ES EL MANEJO INICIAL DE ESTE PACIENTE?  A) CLONIDINA + FINASTERIDE  B) DANAZOL + DOXAZOSINA  C) PRAZOSINA + DANAZOL  D) DOXAZOSINA + FINASTERIDE    RESPUESTA D  La hiperplasia prostática benigna se reifere a una detección microscópica de la hiperplasia (una proloferación de estroma y epitelio),  crecimiento de próstata detectado por examen rectal digital o por medio de ultrasonido además de presentar síntomas asociados a esta  hiperplasia. Se recuerda que el tamaño de la próstata no siempre correlaciona con los síntomas. La prevalencia aumenta en forma lineal  con la edad, teniendo una prevalencia de 90% en pacientes de 85 años o más. La clásica sintomatología obstructiva es pujo miccional,  dismiinución del calibre de la orina, nicturia, etre otros. Entre los métodos diagnósticos está la USG con medici´n de orina residual. Hay  una relación directa entre lso niveles de APE y el volumen prostático pero no tienen mayor riesgo de desarrollar cáncer de próstata. El  tratamiento farmacológico evita la morbilidad asociada a la cirugía. En pórstata de más de 40cc el tratamiento combiada (alfablqoueador  e inhibidor de la 5 alfa redutasa) es la mejor opciójn. La clonidina es una gonista alfa 2.             [OSTEOMALACIA]  FEMENINA DE 50 AÑOS DE EDAD, DIABÉTICA DESDE HACE 15 AÑOS, CON MAL CONTROL DE SU ENFERMEDAD. ACUDE URGENCIAS POR  PRESENTAR DOLOR INTENSO EN EXTREMIDAD INFERIOR DERECHA ASÍ COMO LIMITACIÓN DE LA MOVILIDAD DE LA MISMA EXTREMIDAD.  GLUCEMIA DE 340 MG/DL. EXAMEN GENERAL DE ORINA CON PROTEINURIA Y ERITROCITOS 1 A 2 POR CAMPO. LA RADIOGRAFÍA DE LA  EXTREMIDAD MUESTRA FISURA EN EL DEDO MEDIO DEL PIE DERECHO Y DESMINERALIZACIÓN EN HUESOS CIRCUNDANTES.      EN ESTA PACIENTE LA DESMINERALIZACIÓN OSEA ES SECUNDARIA A LA DISMINUCIÓN EN: 

    A) VITAMINA A.  B) VITAMINA B12.  C) VITAMINA C.  D) VITAMINA D.    RESPUESTA D    Se trata de paciente con complicaciones microvasculares de DM2, tiene descontrol metabólico y proteinuria. Por otro lado presenta una  fractura (fisura) patológica puesto que no existe antecedente de traumatismo y tiene desmineralización.    La desmineralización se puede explicar por dos mecanismos: a) disminución de matriz protéica (osteoporosis), déficit de sales minerales  (osteomalacia). En ocasines puede haber osteoporomalacia.  La D3 es la forma endógena que se produce en la piel, D2 es la forma exógena. Se metaboliza en el hígado por la 25‐OH conviertiendose  en 25 hidroxivitamina D. Luego pasa al RIÑÓN donde se hidroxila una vez más, conviertiéndose en 1‐25 OH vitamina D que es la forma  de  mayor  actividad  metabólica.  Cuando  existe  daño  renal  se  limita  la  formación  de  este  último  metabolito  altamente  activo  y  en  consecuencia hay desmineralización.        [HERNIA INGUINAL]  MUJER DE 48 AÑOS DE EDAD, CON ANTECEDENTE DE HERNIA INGUINAL BILATERAL. ES VALORADA EN EL SERVICIO DE URGENCIAS POR  PRESENTAR DOLOR INTENSO EN LA REGIÓN INGUINAL DERECHA.  LA POSIBILIDAD DE QUE SE TRATE DE UNA HERNIA ESTRANGULADA SE CONFIRMARA EN CASO DE:    A) QUE HAYAN DATOS DE OCLUSIÓN INTESTINAL.  B) QUE HAYAN DATOS DE PERFORACIÓN INTESTINAL.  C) QUE NO SEA REDUCTIBLE.  D) QUE EXISTA COMPROMISO VASCULAR.    RESPUESTA D  Tanto en hernias incarderadas como estanguladas hay cierto grado de oclusión intestinal.    La perforación intestinal puede ser producto de una isquemia prolongada.    En una hernia estrangulada hay compromiso vascular,    está tensa y muy snesible, la piel que lo cubre tiene color rojizo o azulado. N  existen ruidos intetinales dentro de la hernia. El paciente suele tener leucocitosis con desviación a la izquierda, se encuentra tóxico,  deshidratacio, febril, GASA con acidosis metabólica.    El Tx es Qx de urgencia por riesgo de perforación, el tx de esta última complicación  es la resección del segmento afectado.          [HIPERTENSIÓN INTRACRANEAL]  MUJER OBESA EN LA CUARTA DÉCADA DE LA VIDA CON ANTECEDENTE DE CUADROS REPETITIVOS DE CEFALEA INTENSA. COMO PARTE  DEL PROTOCOLO DE ESTUDIO SE REALIZA EXPLORACIÓN DEL FONDO DE OJO ENCONTRANDO LA SIGUIENTE IMAGEN.    

  EL DIAGNÓSTICO MÁS PROBABLE DE ESTE PACIENTE ES: 

 

    A) DEGENERACIÓN VÍTREA.  B) DRUSEN DEL DISCO ÓPTICO.  C) PAPILEDEMA.  D) UVEITIS.    CORRESPONDE AL MECANISMO FISIOPATOLÓGICO QUE PRODUCE ESTA ENFERMEDAD:    A) INFECCIÓN SIFILÍTICA.  B) ENVEJECIMIENTO.  C) DEPÓSITOS CÁLCICOS EN EL NERVIO ÓPTICO.  D) HIPERTESIÓN ENDOCRANEAL    RESPUESTAS C, D  La degeneración vítrea se presenta a la par del envejecimiento y presenta depósitos lipídicos en el gel vítreo.    El drusen del NO se caracteriza por depósitos refrigentes situados dentro del parénquima de la cabeza del nervio óptico. A la EF partículas  brillantes en la superficie del NO. Clínicamente oscurecimiento de la visión. Puede coexistir papiledema y drusen óptico.  El papiledema se manifiesta como resultado de hipertensión intracraneal, la papila luce con borde deshilachados, exudados algodonosos  peripapilares, hermorragias intra y extrapapilares,    exudados color cereza. Puede asociar cefalea pero no es un requisito. Clíniamente  oscurecimiento transitorio de la visión es un signo clásico,    afección a uno o ambos ojos al mismo tiempo.      La uveítis hace referencia a la inflamación aguda o crónica intraocular. Puede tener causa infecciosa (sífilis‐imagen sal y pimienta +  papiledema) o inmunológica.    Uveítis anterior: células inflamatorias en humor acuoso, células en endotelio corneal (imagen en grasa de carnero), nódulos en iris. Es  causa de ojo rojo doloroso, asocia fotofobia, pérdida visual unilateral.    Uveítis posterior: pérdida visual gradual. Células en vítreo con inflmación en retina y corides.        [DMI]  HOMBRE  DE  22  AÑOS  DE  EDAD,  CON  DIAGNÓSTICO  DE  DIABETES  TIPO  I  DESDE  HACE  TRES  MESES.  A  ÉSTE  PACIENTE,  SE  LE DEBERÁ  RECOMENDAR REALIZAR UNA REVISIÓN OFTALMOLÓGICA EN EL SIGUIENTE LAPSO DE TIEMPO:      A) AL MOMENTO DEL DIAGNÓSTICO.  B) UN AÑO.  C) TRES AÑOS.  D) CINCO AÑOS.    RESPUESTA C  En DM2 la revisión oftalmológica se hace al diagnóstico y cada año, se justifica porque al diagnóstico 39% tiene retinopatía diabética y 4‐ 8% tienen comprometida la visión.    Si tiene DM1 o 2 y desea embarazo, deberá relizarse cada 3 meses de forma rutinaria o si tiene retinopatía diabética leve , cada mes si  retipotía diabética moderada a grave.  El tamizaje en DM1 se realiza      a) A partir de los 12 años de edad    b) A partir del 5to año de diagnóstico y cada año.      c) A partir del 3er año del diagnóstico una vez pasada la pubertad      UVEITIS ANTERIOR  [r]HOMBRE DE 30 AÑOS DE EDAD CON LUMBALGIA INFLAMATORIA. ACUDE A URGENCIAS POR UN CUADRO DE 24 HRS DE EVOLUCIÓN,  CARACTERIZADO PR: DOLOR, FOTOFOBIA, BLEFAROSPAMO, VISIÓN BORROSA EN OJO IZQUIERDO. A LA EF SE OSBSRVAN OJ IZQUIERDO  CON  HIPEREMIA  PERICORNEAL,  PUPILA  IZQUIRDA  MIÓTICA  CON  BRADICORIA,  FENÓMENO  DE  TYNDALL,  PRECIPITADOS  RETRO  CORENALES PQUEÑOS, FONDO DE OJO APARENTEMENTE SIN ALTERACIONES Y AGUDEZA VISUAL DISMINUIDA.  SEGÚN TU DIAGNÓSTICO PRESUNTIVO, ¿ QUÉ ENFERMEDAD REUMATOLÓGIA PADECE CON MAYOR PROBABILIDAD ESTE PACIENTE?  A) ESPONDILITS ANQUILOSANTE  B) SARCOIDSIS  C) BEHCET  D) ARTRITIS IDIOPÁTICA JUVENIL    RESPUESTA A    El paciente tiene una uveitis anterior secundaria a espondilitis anquilosante. El Dx se apoya en: antecedente personal de espndilitis  anquilosante,  ojo  doloroso,  tiempo  de  evolución,  ,  fotofobia,  miosis  y  efecto  de  tyndall.  Las  enfermedades  que  más  frecunetente  se  relacionada con uveítis anterior son: espondiloartritis (y de la más común la espondilitis aquilsante);, artritis idiopática juvenil, Behcet,  Sarcoidosis,  Tuberculosis.  Sin  embargo,  en  este  caso  el  diangóstico  más  probable  es  espondilitis  anquilosante  por  el  antecedente  de  lumbalgia inflamatoria y la suencia de otro dqato sistémico. La uveítis es debida a la inflamación del iris y cuerpo ciliar, lo cual provoca  exudados en cámara anterio,; lo que nos da el fenóemno de Tyndall (células y proteínas fltante en cámara anterior que s eobservan con  la lámpara de hendidura). La inflamación de las estructuas produce un síndrome ciliar, caracterizado por: dolor, fotfobia, blefarospasmo 

e inyección pericorneal. La inflmación del iris produce una miosis con bradicoria (reacción lenta). El manejo es con esteroides, midriáticos  y tratando la causa si que s se cnoce. RECUERDA Repasar diagnótico difernecia de ojo rojo. [reto enarm]      HOMBRE DE 32 AÑOS, SIN ANTECEDENTES PATOLÓGICOS DE INTERÉS, QUE ACUDE A LA CONSULTA POR VISIÓN BORROSA EN SU OJO  DERECHO DE 3 DÍAS DE EVOLUCIÓN. LA AGUDEZA VISUAL ES DE 0., EL EXAMEN DEL POLO ANTERIOR NO MUESTRA ALTERACIONES, Y EN  EL FONDO DE OJO SE OBSERVA CICATRIZ CORIORRETINIANA Y CÉLULAS EN LA CAVIDAD VITREA. EL PACIENTE RELATA DOS EPIDOSIOS  OCULARES  SIMILARES  NO  DIAGNOSTICADOS.  ¿QUIÉ  EXPLORACIONES  COMPLEMENTARIAS  SOLICITARÍA  PARA  ESTABLECER  EL  DIAGNÓSTICO?  A) TEST DE MANTOUX Y TINCIÓN DE ZIEHL‐NIELSEN EN ESPUTO  B) SEROLOGÍA TOXOPLÁSMICA  C) RADIOGRAFÍA DE TÓRAX  D) SEROLOGÍA LUÉTICA    RESPUESTA    UNA MUJER DE 40 AÑOS QUE PADECE ARTRITIS REUMATOIDE, TRATADA CON SALES DE ORO, PRESENTA HIPEREMIA E INYECCIÓN CILIAR  EN OJO DERECHO, CON VISIÓN BORROSA, MIOSIS DERECHA IRREGULAR Y DOLOR OCULAR, CON TENSIÓN OCULAR NORMAL. ¿CUÁL DE  LOS DIAGNÓSTICOS QUE A CONTINUACIÓN SE ENUMERAN ES EL CORRECTO?  A) DESPRENDIMIENTO DE RETINA  B) GLAUCOMA CRÓNICO SIMPLE    C) CONJUNTIVITIS AGUDA  D) UVEÍTIS ANTERIOR    RESPUESTA    A 27            [RETINOPATÍA DIABÉTICA][r]  HOMBRE  DE  63  AÑOS  DE  EDAD  CON  DIAGNÓSTICO  DE  DIABETES  MELLITUS  DESDE  18  AÑOS.  ACUDE  A  CITA  DE  CONTROL  OFTALMOLÓGICA. A LA EXPLORACIÓN DEL FONDO DE OJO SE OBSERVA LA SIGUIENTE IMAGEN. 

      CON BASE EN LA IMAGEN DE FONDO DE OJO MOSTRADA SE PUEDE CONCLUIR QUE EL PACIENTE PRESENTA:      A) RETINOPATÍA NO PROLIFERATIVA.  B) UNA HEMORRAGIA VÍTREA.  C) RETINOPATÍA PROLIFERATIVA.  D) EDEMA DE PAPILA. 

  EL TRATAMIENTO DE ELECCIÓN EN ESTE CASO INCLUYE:      A) CONTROL METABÓLICO + APLICACIÓN DE LÁSER.  B) UNICAMENTE CONTROL METABÓLICO.  C) CONTROL METABÓLICO + DERIVACIÓN DE LÍQUIDO CEFALORRAQUÍDEO.  D) CONTROL METABÓLICO + REPOSO EN SEMIFOWLER    RESPUESTA A, B  En la retinopatía diabética podemos encontrar varias lesiones: microaneurismas, hemorragias, exudados duros, manchas algodonosas,  alteraciones microvasculares, arrosaraiamiento venoso, neovasos, tejido fibroso.    Se clasfica en RDP y RDNP.    La RDNP se caracteriza por aneurismas vasculares retinianos, manchas hemorrágicas, dilatación venosa y exudados algonosos.    RDNP mínima: microanerusismas  RDNP moderada: microaneurismas + exudados duros + engrosamiento de capilares  RDNP avanzada: Más de 20 hermorragias retinianas x cuadrante o Arrosaramiento venoso en 2 o más cuadrantes o AMIR en un cuadrante.        [GLAUCOMA]  MUJER DE 75 AÑOS DE EDAD, QUE ACUDE A LA CONSULTA CON SINTOMATOLOGÍA COMPATIBLE CON GLAUCOMA.  PARA ENTENDER LA FISIOPATOLOGÍA DEL GLAUCOMA, ES IMPORTANTE SABER QUE EL DAÑO GLACOMATOSO SE DA EN LA SIGUIENTE  PORCIÓN DEL NERVIO ÓPTICO:      A) INTRAOCULAR.  B) ORBITARIA.  C) INTRACANALICULAR.  D) INTRACRANEAL.    RESPUESTA A  El glaucoma es una neuropatía óptica crónica y progresiva, hay pérdida de fibras nerviosas de la retina, excavación y palidez progresivas  de la papila y defectos del campo visual. Va asociado a hipertensión intraocular pero NO en todos los casos. De hecho hay glaucoma  normotenso, y hay hipertensión intraocular (>21mmHg) sin glaucoma.    La papila corresponde a la parte intraocular del nervio óptico.  El  glaucoma  de  ángulo  abierto  (GPAA)  es  el  más  freucente  con  un  60%.  El  ángulo  esta  abierto  y  suele  ser  simétrico.    Etiología  desconocida.    Pero el principal defecto fisiopatológico es la resistencia al paso del humor por la malla trabecular.    Factores de reisgo: 1) PIC, 2) antecdenetes familiares (4‐16% de riesgo de padercerla si son de primer grado, 3)>40 años.    Epitelio  de  cuerpo  ciliar===>  humor  acuoso  en  cámara  posterior==>  pupila==>  cámara  anterior  ==>ándulo  iridocorneal==>  trabeculum===> canal de Schlemm==>vaso espiesclerales==> circulación sanguínea general (eliminación del 90% del humor acuoso)    Solo 10% se eliminará vía uveoescleral atravesando directamente el cuerpo ciliar.    En el glaucoma de ángulo cerrado el origen está en un bloqueo pupilar relativo a una mayor aposición del iris y el cristalino que dificutla  el paso del humor acuoso de la cámara posterior a la anterior a través de la pupila (empujando a la periferia el iris).      Independientemente de la causa, habrá un pérdida progresiva de la capa de fibras nerviosas ya sea de forma mecánica directa o por  compromiso vasculaar.    Tratatamiento farmacológico:  1)betabloqueadores: disminuyen la producción del humor acuoso al actuar sobre los receptores beta de los procesos ciliares [timolol,  carteolol, betaxolol (selectivo B1)]  2)Mióticos parasimpaticomiméticos    y    anticolinesterásicos: disminuyen la resistencia de la malla trabecular. [pilocarpina y aceclidina]  3) Simpaticomiméticos A y B adrenérgicos:    [adrenalina] aumentan el flujo de salida del humor acuoso y disminuyendo la producción en  el cuerpo ciliar. [NO en ángulo cerrado]  4) Agonistas alfa 2 adrenérigcos: disminución de la prodccuón de humor acuoso  5)  Prostaglandinas:  [latanaprost  al  0.005%]  aumenta  la  salida  de  humor  acuoso  a  través  de  vía  uveoescleral.  [NO  en  glaucoma  inflamatorio]  6) Inhibidores de anhidrasa carbónica    Tratamiento láser: trabeculoplastia con lárser de argón en GPAA y trabeculectomía, iridotomía periférica con láser argón o lsa Ng‐YAG en  GPAC.  Tratamiento quirúrgico: trabeculectomía simple o combinada cirugía de catarata (facoemulsificación)    ¿CUÁL ES LA CAUSA MÁS FRECUENTE DE UNA EXCAVACIÓN PAPILAR CON RECHAZO NASAL DE LOS VASOS EMERGENTES DE LA MISMA?  A) HIPERTENSIÓN INTRACRANEAL  B) UVEÍTIS POSTERIOR  C) CONJUNTIVITIS CRÓNICA 

D) GLAUCOMA CRÓNICO SIMPLE    RESPUESTA D  No entendí la pregunta :(    UNA  MUJER  DE  64  AÑOS,  HIPERMÉTROPE  Y  CON  CATARATAS  EN  AMBOS  OJOS,  ACUDE  A  CONSULTA  CON  DOLOR  INTENSO  EN  OJO  IZQUIERDO,    DE UNAS HORAS DE EVOLUCIÓN. LA EXPLORACIÓN DE ES OJO PONE DE MANIFIESTO UNA TENSIÓN OCULAR DE 40MMHG,  REACCIÓN HIPERÉMICA CILIO‐CONJUNTIVAL, MIDRIASIS Y EDEMA CORNEAL. ¿CUÁL SERÍA LA ACTITUD INMEDIATA MÁS CORRECTA DE  LAS QUE SE ENUMERAN A CONTINUACIÓN?  A) MANITOL INTRAVENOSO, ASOCIADO A TRATAMIENTO MIÓTICO Y CORTICOIDES SISTÉMICOS  B) PRACTICAR TRABECULECTOMÍA  C) CORTICOIDES TÓPICOS, ASOCIADOS A TRATAMIENTO MIDRIÁTICO  D) OPERAR CON URGENCIA LA CATARATA DE ESE OJO, DESENCADENANTE EL CUADRO.    RESPUESTA A    ACUDE ACONSULTA UN PACIENTE PORQUE DICE QUE EN LOS ÚLTIMOS MESES LE HA CAMBIADO EL COLOR DEL OJO IZQUIERDO. EN LA  ANAMENSIS DEBE INSISTIR EN POSIBLE OCNSUMO DE ALGIUNA DE LAS SIGUIENTES MEDICAMENONES POR VÍA TÓPICA:  A) COLIRIOS PARA GLAUCOMA  B) AINE  C) CORTICOIDES  D)  ACICLOVIR,  PUES  ESTÁ  DEMOSTRADO  QUE  PUEDE  PRODUCIR  HIPERPIGMENTACIÓN  EN  EL  IRIS  CUANDO  SE  MANTIENE  EL  TRATAMIENTO DURANTE UN PERIODO MUY LARGO DE TIEMPO    RESPUESTA A  Hay que recordar que los análogos de prostaglandinas F2 pueden producir como efectos secundarios cambios en la coloración del iris  (hiperpigmentación) e hipermia y aumento del tamaño de las pestañas.    A pesar de ello son los medicamentos mejor tolerados y eficacez  en el manejo de glaucoma de ángulo abierto.    A 93‐YEAR OLD WOMAN, COMES TO THE EMERGENCY ROOM COMPAINING ABOUT ACUTE PAIN AND REDNESS IN HER RIGHT EYE. TEN  YEARS AGO SHE WAS SUBMITTED TO CATARACT SURGERY OF HER LEFT EYE. HOWEVER THEY "DIDN'T OPERATE MY RIGHT EYE, BECAUSE  IT IS A LAZY EYE". VISUAL ACUITY IS COUNTING FINGERS IN HER RIGHT    EYE AND 8/20 IN HER LEFT EYE. IOP IS 48MMHG IN HER RIGHT  EYE  AND  15MMHG  IN  HER  LEFT  EYE.  SLIT  LAMP  EXAMINATION  SHOWS  CORNEAL  EDEMA,  HYPERMATURE  CATARACT  AND  A  DEEP  ANTERIOR  CHAMBER  WITH  INTENSE  FLARE,  AND  GROSS  TYNDALL.  AGGREGATES  OF  WHIT  MATERIAL  ARE  PRESENT  OVER  THE  ENDOTHELIUM AND THE ANTERIOR FACE OF THE LENS. FUNDUS EXAMINATION REVEALS THE PRESENCE OF DRUSEN IN HER LEF EYE, BUT  IT IUS NOT POSSIBLE IT IN HER RIGHT EYE, DUE TO THE DENSITY OF THE CATARACT. THE MOST PROBABLE DIAGNOSIS IS:  A) ACUDE ANGLE‐CLOSURE GLAUCOMA  B) PHACOLYTIC GLAUCOMA  C) PHACOMORPHIC GLAUCOMA  D) CHOROIDAL MELANOMA    THE MOST APROPIATE TREATMENT FOR THIS PATTIEN IS:    A) OCULAR HYPOTENSIVE DRUGS, TOPICAL STEROIDS AND THE CATARACT SURGERY ON HER RIGHT EYE.  B) OCULAR HYPOTENSIVE DRUGS, AND TOPICAL STEROIDS ON HER RIGHT EYE  C) ENUCLEATION OF HER RIGHT EYE  D) EVISCERATION OF HER RIGHT EYE    ANSWERS  B, A    A 45‐YEAR‐OLD HEALTHY MAN HAS STARTED TREATMENT WITH LATANAPROST AND TIMOLOL 0.5% FOR A RECENTLY DIAGNOSED OPEN  ANGLE GLAUCOMA. HE TELLS YOU THAT HE CAN'T KEEP ON WITH HIS FRIENDS WHEN HE IS PLAYING PADDLE. WICH OF THE FOLLOWING  WOULD BE THE WISER ATTITUDE?  A) CHANGE LATANAPROST FOR A DIFFERENT DRUG  B) CHANGE TIMOLOL FOR A DIFFERENT DRUG  C) CHANGE BOTH DRUGS  D) SEND HIM T THE CARDIOLOGIST    ANSWER B  No entendí la pregunta pero es poco factible que latanaprost tenga que suspenderse. Timolol debe usarse con cuidado en cardiopatas y  broncópatas. Probablemente este paciente tenga alguna contraindicación.          [ÚLCERA CORNEAL]  HOMBRE DE 35 AÑOS DE EDAD. ACUDE A SERVICIO DE URGENCIAS POR CUADRO DE 12 HORAS DE EVOLUCIÓN, CARACTERIZADO POR 

DOLOR  OCULAR  DERECHO  INTENSO,  ACOMPAÑADO  DE  FOTOFOBIA,  LAGRIMEO,  PRESENCIA  DE  SECRECIÓN  AMARILLENTA  Y  DISMINUCIÓN IMPORTANTE DE LA AGUDEZA VISUAL ÚNICAMENTE EN ES OJO. AL INTERROGATORIO EL PACIENTE REFIERE QUE HACE 24  HRS TUVO ENTRADA DE ARENILLA EN OJO DERECHO. A LA EXPLORAICÓN FÍSICA: EL OJO DERECHO SE OBSERVA CON HIPEREMIA MIXTA  (PERIFÉRICA Y PERICORNEAL) INTENSA, QUEMOSIS CONJUNTIVAL Y DE LOS PÁRPADOS, CÓRNEA OPACA, BLEFAROESPASMO Y PUPILAS  REACTIVAS. OJO IZQUIERDO SIN ALTERACINES.  ¿CUÁL DE LOS SIGUIENTES DIAGNÓSTICO ES EL MÁS PROBABLE?  A) ÚLCRA CORNEAL  B) UVETITIS AGUDA  C) CONJUNTIVITIS BACTERIANA  D) GLAUCOMA    RESPUESTA A  El  diagnóstico  se  apoya  en  ojo  doloroso  (unilateral),  hiperemia  mixta,  córnea  opaca,  quemosis  y  pupilas  reactivas.  No  se  trata  de  conjuntivitis bacteriana, ya que esta no da dolor ocular. El glaucoma es una causa de ojo rojo doloroso, pero los reflejos pupilares se  encuentran alterados (midriasis fija), y en este paciente no hay alteración pupilar. La uveítis es una causa frecuente de ojo rojo doloroso,  pero hay alteración del reflejo pupilar (miosis) y fenóemnos de Tyndall (humor acuoso turbio por celularidad). La úlcera corneal es una  urgencia oftalmológica, ya que los pacientes corren riesgo de perforación, se debe de inciar antibiótico, quitar el dolor y referir lo antes  posible al oftalmólogo para su manejo.        [INSUFICIENCIA ARTERIAL PERIFÉRICA]  HOMBRE  DE  64  AÑOS  DE  EDAD,  TABAQUISMO  CRÓNICO,  DIABÉTICO  E  HIPERTENSO.  ACUDE  A  CONSULTA  POR  DOLOR  EN  LAS  EXTREMIDADES INFERIORES. A LA EXPLORACIÓN FÍSICA FC 80 FR 19 T 37 TA 130/78, PULSOS PEDIOS, TIBIAL POSTERIOR DISMINUIDOS Y  DOLOR DEL PIE. REALIZA PRUEBA DE ÍNDICE TOBILLO‐BRAZO.    ¿CUÁL ES EL VALOR APROXIMADA QUE PROBABLMENTE TENDRÁ EL PACIENTE EN ESTA PRUEBA?  A) 1.2  B) 1.5  C) 0.9  D) 0.6    RESPUESTA D  El  paciente  tiene  enfermedad  arterial  periférica.  La  prevalencia  de  enfermeda  arterial  periférica  aumenta  en  pacientes  >70  años,  pacientes entre 50‐69 años con historia de tabaquismo o diabetes,    como en    este paciente . Se concen otros factores de riesgo como  historia familiar de ateroesclerosis, sexo masculino, hipertensión, hiperlipidemia, entre otros. Hay una pruba simple que se puede realizar  para orientar el diagnóstico de esta entidad, y se le llama índice tobillo brazo, el cual es la presión sistólica del tobillo dividido entre la  presión sistólica braquial.    El valor normal es >0.9 por lo que en este caso encontraremos valores menores a 0.9. Valores mayores a 1.3  sugieren la presencia de vasos calcificados por lo que se necesitarán estudios adicionales.                   

[             [LUXACIÓN DE CADERA] [r]  FEMENINA DE 64 AÑOS DE EDAD, SUFRE CAÍDA. A SU LLEGADA A URGENCIAS REFIERE DOLOR INTENSO EN CADERA DERECHA. A LA  EXPLORACIÓN SE OBSERVA ACORTAMIENTO DE UNA DE LAS EXTREMIDADES CON SENSACIÓN DE INESTABILIDAD AL MOVER LA  EXTREMIDAD.    SE SOSPECHA UNA LUXACIÓN POSTERIOR DE CADERA. LA URGENCIA EN LA ATENCIÓN DE ESTE PACIENTE ES DEBIDO A QUE ESTE  PROCESO PUEDE OCASIONAR:  A) OSTEOMIELITIS.  B) RIGIDEZ ARTICULAR.  C) FRACTURA SECUNDARIA.  D) OSTEONECROSIS.    LA SIGUIENTE POSICIÓN DE LA EXTREMIDAD LESIONADA DIRIGIRÍA EL DIAGNÓSTICO DE LUXACIÓN POSTERIOR DE CADERA.  A) FLEXIÓN, ROTACIÓN INTERNA Y ABDUCCIÓN.  B) FLEXIÓN, ROTACIÓN, INTERNA Y ADUCCIÓN.  C) FLEXIÓN, ROTACIÓN EXTERNA Y ABDUCCIÓN.  D) FLEXIÓN, ROTACIÓN EXTERNA Y ADUCCIÓN    CORRESPONDE AL TRATAMIENTO DE ELECCIÓN EN ESTE CASO:  A) TRACCIÓN CON REDUCCIÓN QUIRÚRGICA.  B) TRACCIÓN CON SEDACIÓN Y ROTACIÓN EXTERNA.  C) TRACCIÓN E INMOVILIZACIÓN DE LA EXTREMIDAD.  D) TRACCIÓN LONGITUDINAL CON PACIENTE EN SUPINO    RESPUESTAS D, B, D  La luxación posterior de cadera es la más frecuente, corresponde hasta el 95% de las luxaciones de cadera. A la exploración se  encuentra acortamiento de la extremidad, cadera flexionada, rotación interna y aducción.    La luxación anterior de la cadera es la menos frecuente y se traduce en flexión, rotación externa y abducción.    [Posterior AD, anterior  ABD]    En el proceso de luxación de cadera puede haber interrupción vascular asociada y se debe evitar de forma urgente Osteonecrosis. La  osteonecrosis es una complicación tanto de luxación anterior como posterior por la cual ameritan reducción urgente.    Si en la  pregunta hubieran pregunta por complicación específica habría que considerar lesión del nervio ciático puest que es una complicación 

del 20%    de las luxaciones posteriores.    La rigidez articular sí puyede ser una complicación pero crónica y por luxaciones recurrentes, por lo tanto no es urgencia.    La fractura deberá buscarse asociada a la luxación y de forma impsilateral, pero NO es una complicación de la luxación.      No está inciada la rotación de la cadera en el manejo de la luxación y tampoco inmovilización ( a menos que asocia fracturas).  Lo ideal es la reducción con tracción de la extremidad estando el paciente supino: luego de ello deberá flexionarse cadera a 90° en  posición neutral y aplicar fuerza hacia atrás en caso de alteración o subluxación está indicada la reducción quirúrgica urgente.      [FRACTURA DE MUÑECA] [r]    FEMENINA DE 75 AÑOS DE EDAD, ES LLEVADA A URGENCIAS POR SUFRIR CAÍDA. A LA EXPLORACIÓN PRESENTA LA MUÑECA DERECHA  FORMA DE TENEDOR. USTED INFIERE QUE EL MECANISMO DE LA LESION FUE:      A) ANTEBRAZO EN SUPINACIÓN.  B) MUÑECA EN ANTEFLEXIÓN.  C) BRAZO EN ABDUCCIÓN.  D) MANO EN HIPEREXTENSIÓN.    RESPUESTA C  Cuando hablamos de deformidad en dorso de tenedor, se hace referencia a una fractura del extremo distal del radio, o fractura de  Colles. El mecanismo para que se produzca esta lesión es caída sobre la mano con muñeca en hiperextensión o dorsiflexión.  La fractura de Colles se observan tres desviaciones:    Enclavamiento de la epífisis en la metáfisis    Desviación dorsal de la epífisis dando la silueta en "dorso de tenedor"    Desviación radial de la epífisis dando la forma de "desviación en bayoneta"    [PARÁLISIS DE CUERDAS VOCALES] [r]  FEMENINA DE 44 AÑOS, CURSA POSTQUIRÚRGICO SECUNDARIO A TIROIDECTOMÍA TOTAL. PRESENTA SUBITAMENTE ESTRIDOR  INTENSO Y SENSACIÓN DE FALTA DE AIRE. SE SOLICITA LARINGOSCOPÍA QUE MUESTRA PARÁLISIS DE CUERDAS VOCALES.  EN ESTE MOMENTO ESTARÁ INDICADO REALIZAR:      A) INTUBACIÓN OROTRAQUEAL.  B) CORDECTOMÍA.  C) TRAQUEOSTOMÍA.  D) ARITENOIDECTOMÍA.    RESPUESTA C  Hay que tomar en cuenta que la parálisis de cuerdas vocales impide el paso de la cánula de intubación orotraqueal, razón por la cual  habrá que asegurar la vía aérea mediante traqueostomía.    La cordectomía consiste en extirpar parte de las cuerdas voales, requiere especialistas altamente capacitados, poco factible usarla en  urgencias.    La aritenoidectomía también permite liberar la vía aérea, pero requiere especialistas altamente capacitados, poco factible usarl en  urgencias.    El 90% de las parálisis son de causa periférica, 10% central.  Dentro de las periféricas, cirugía tiroidea y cervical (40‐60%), tumoraciones torácicos y cervicales (5‐25%), idiopáticas (5‐20%),  sistémicas y otras causas (5‐15%), traumatismo cervical y anestésico 1‐5%)   

        [QUEMADURAS]  EN EL CENTRO DE SALUD SE DAN PLATICAS ORIENTANDO A LOS USUARIOS SOBRE LA PREVENCION DE LAS QUEMADURAS EN EL  HOGAR.  EN LA PLÁTICA SE COMENTA QUE HAY ALGUNAS QUEMADURAS QUE SON MAS GRAVES DE LO QUE APARENTAN. ESTE TIPO DE  QUEMADURAS SON LAS:      A) ELÉCTRICAS.  B) QUÍMICAS.  C) POR RADIACIÓN.  D) POR LÁSER.    RESPUESTA D    Tipos de traumatismo eléctrico.  Directo: 80% de todos los traumatismos eléctricos.    De bajo voltaje 1000 v produce lesiones graves como quemaduras, afetación multiorgánica, destrucción tisular, etc.            Similar a sx por aplastamiento  Indirecto o arco voltáico: causado por campo magnético al rededor de líneas de alta tensión    (>10 000v)  Flash eléctrico: realmente es una quemadura por llama, es una lesión térmicas.    Rayo: produce parada respiratoria y muerte inmediata. Lesiones cutáneas en arborización típica.    La rabdomiólisis causa liberación de mioglobina e insuficiencia renal. Tx inmediado con manejo de vía aérea, línea IV, monitorea EKG,  sonda vesical. Si orina oscura pensar==> hemocromógenos. De hecho no se debe esperar a la confirmación de mioglobulinuria para  empezar a administrar líquidos, deberá mantenerse una diuresis cuando menos de 100ml/hr en adulto. Si no se aclara el pigmento  habrá que aumentar líquidos, 25gr de manitol, 12.5 a cada litro subsecuente.    [OTITIS EXTERNA]  HOMBRE DE 27 AÑOS DE EDAD QUE ACUDE A CONSULTA POR PRESENTAR OTALGIA DERECHA SEVERA. EL APCIENTE REFIERE QUE EL  DOLOR SE IRRADIA HACIA LA REGIÓN TPM Y TAMBIÉN TIEN PRURIT. NIEGA ANTECEDENTES DE IMPORTNAICA. REFIERE REALZIAR  NATACIÓN 3 VECES POR SEMANA. FC 67, FR 13, T37.5 TA 114/76.. TIENE ERITEMA Y EDEMA DIFUSO EN CONDUTO AUTIVIO DERECHO.    ¿CUÁL ES EL TRATAMIENTO MÁS ADEUCADA PARA ESTE PACIENTE?  A) AMOXICILINA/CLAGVULANATO ORAL + GOTAS ÓTICAS + AC. AC´TICO  B) NEOMICINA ÓTICA + DEXAMETASONA + AC ACÉTICO 

 

C) CIPROFLOACINO ORAL    + AINES  D) GOITAS ÓTICAS CON DEXAMETASONA Y ÁC. AC´TICO    RESPUESTA A  Ciertos factores se asocian con la presencia de otitis externa, que es el diangóstico de este caso: la alteración del ph del conducto  (seuncario a exposición a agua cmo pr ejempo antación), tauma local (cotonetes), limpieza agresiva entre otros. La manifestación  clínica de esta patología es la hiperestesia locorregional a la manipulación de pabellón auricular (o también cnocido com signo del  trago). La exploración física por medio de otoscopia demuesta un entorno inflamatorio (ritema, edema). De acuero a las guinas del  CENETEC, el tratamiento es a base de gotas óticas que combinan esteroide o antibiótico +    ac acético. El uso de ác acético sólo es  menos efectivo que la combinaci´n previamente mencinada. recordar que los agentes etiológicos más frecuentes son: peusomonas  aeruginosa, S. aureus y el tratmiento tópico es de elección.  [OTITIS MEDIA]  MUJER DE 17 AÑOS QUE INICIA SU PADECIMIENTO HACE 7 DÍAS CON LA PRESENCIA DE CEFALEA Y OTALGIA IZQUIERDA. EL DÍA DE AYER  SE AGREGA FIEBRE DE 38.5°C, HIPOACUSIA. A LA EXPLORACIÓN SE ENCUENTRA MEMBRANA TIMPÁNICA IZQUIERDA INTEGRA,  HIPERÉMICA, ABOMBADA E INMÓVIL.  EL DIAGNÓSTICO CLÍNICO MÁS PROBABLE ES:      A) OTITIS MEDIA AGUDA.  B) OTITIS MEDIA CRÓNICA.  C) OTITIS SEROSA.  D) OTITIS NECROTIZANTE.    EL FACTOR DESENCADENANTE MÁS COMÚN EN ESTE TIPO DE PACIENTES ES:  A) NADAR SIN PROTECCIÓN.  B) LA ACUMULACIÓN DE CERUMEN.  C) LAS INFECCIONES DE VÍAS RESPIRATORIAS SUPERIORES.  D) LA PERFORACIÓN DE LA MEMBRANA TIMPÁNICA.      ADEMÁS DE ADMINISTRAR ANALGÉSICOS Y ANTIPIRÉTICOS SE DEBERÁ AGREGAR:  A) TRIMETOPRIMA MÁS SULFAMETOXAZOL.  B) CIPROFLOXACINA.  C) AMOXICILINA.  D) PENICILINA PROCAÍNICA.    RESPUESTA A,C,C  Otitis media aguda: principal agente etiológico 1) S. pneumoniae, 2)H. influenzae no tipable. Es una enfermedad íntimente relacionada  con resfraido común del que es complicación habitual. La infección vírica de vías respiratorias altas provoca una toxicidad sobre el  epitelio respiratorios, causante d discinesia ciliar en la trompa de Eustaquio, con alteración subsecuente del aclaramiento de moco.  La otitis serosa es muy frecuente en la infancia es una condición muy frecuente, se caracteriza por la pesencia de líquido detrás del  tímpano. Generalmente se asocia a resfraido o a otitis aguda previa, aunque no significa que el oido esté infectado. Mejora  espontáneamente en unos meses.  La osteomilitis de la base del cráneo también conocida como otitis externa maligna o necrotizante. Comineza como infección del  conducto auditivo externo, lugo del temporal, meningitis, septicemia,    muerte.    Se observa en inmunocomprometidos (diabéticos de  edad avanzada). Etiología: pseudomonas aeruginosa. Clínica: a parte de los síntomas óticos puede haner hipoacusia, trismus, parálisis  facial o datos de septicemia. (en sus inicios hay síntomas de otitis externa aguda).    Respecto al tatamiento amoxi es de primera elección cefalosporina de 2da generación es segunda eleccion, y cloranfenicol es de  tercera. En otits media S.pneumonae tiene 50% de resistencia. TMP/SMX no ha    mostrado ser mejor que amoxicilina,     

ENFERMEDADES ANORRECTALES  [Resumen la Salle]  DEFINICIÓN    Recto==> hemorroides internas NO duelen==> se pueden tratar endoscópicamente    Línea dentada ==> son válvulas para continencia, tiene columnas de Morgani c glándulas, si se tapan se abscedan ==> enf  criptoglandular  Ano==> hemorroides externas sí duelen    Hemorroides:    Son venas o várices    Cuadrante lateral izquierdo    Cuadrante anterior derecho    Cuadrante posterior derecho    Internas (más frecuentes)  Si ingurgitan da síntomas, prolapso al ano 

Externas (menos frecuentes)      Solo cuando hay trombosis deja colgajo si se resuelve.    Mixta      PATOGENIA  Causas: estreñimiento, hipertensión portal.      DIAGNÓSTICO Y TRATAMIENTO  Hemorroides‐ absceso/fístula ‐ fisura  Hemorragia, dolor, secresión, cambios de hábitos  EF: co  lación permeabilidad, pedir que puje y valorar prolapso de hemorroides o recto, y tacto rectal.    Anoscopia y/o proctosigmoidoscopia, si quiere saber si tiene fístula o saber si enf inflamatoria          Tratamiento  Grado I  Indoloro      Mx  Grado II  Reducción espontánea    Mx  Grado III  Reducción manual    Qx  Grado IV  No reductible    Qx    Mx: dieta con fibra, ejercicio.  Qx: Ligadura c banda elástica, escleroterapia, hemorroidectomía cerrada.      Si están trombosadas ==> trombectomía (anestesia local y drenaje de coágulo.      Dx dif Ca, enf inflamatoria, divertículos, pólipos, fisuras (duelen mucho) ==> sangre fresca    Complicaciones: sepsis perianal (etiología principal Clostrium perfringens), sangrado, dolor, necrosis, .  Si no cambian de hábitos recurren tras la Qx.      Fisuras:    Ocurren en línea media, 90% posteriorres, 10% anteriores, 1% anteroposteriores.    Si fuera de  Síntoma pivote al estar debajo de línea dentada "dolor", no requiere defecr.  El recto se le dilata==> se desgarran más. clínica:; dolor desgarrantes, gotas de sangre rojos urtilante cojn las heces. Paraclínicps: quizas  manomería (esfínter hipertónico) Complicación: sangrado, no malignizan fuera de la línea media si hago biopsia.    Tratamiento: sediluvios (agua tibia), 90% curan, nitroglicerina 2%, isosorbide SL x cada mg /kg /d? de 2/3 diosmina, paciente, se mueve,  calider, lido (8disparos) 1/3 ketanserina (sufrexal) luego de cada sediluvio. Lidocaina, esteroide tópico x 10 días al igual que en  hemorroides.    Toxina botulínica. Si    Tx    Qx si crónicidad o recurrencia.    fisurectomía + esfinterotomía lateral izquierda parcial.      Absceso y fístula:        Perianal      Se ven    Interesfinteriano    Se ven      Isquioanal      Se ven    Supraelevador    Se palpa, es el más riesgoso, si se rompe hace peritonitis, hay que hacer diagnóstico rápido.        Explorar bajo antetesia, tx drenaje.    Clínica: pueden hacer spesis, en DM2 Fournier Paraclínica: USG solo si requiere detectar el trayecto. Si transesfinteriana ‐Setón       

HEMORRAGIA DE TUBO DIGESTIVO ALTO  [Resumen la Salle]  DEFINICIÓN  1) Lipotimia, síncope, diaforesis, melena, hematemesis.    2) Sospecha! ¿Cuál es su estado de TA, FC, ortostatismo, hiptrmia=  Sangrado pero TA, FC, temperatura normales, no ortostatismo ( pérdida  pérdida de 10‐20%  Palidez, hipotermina, hipotensión    >15mmHg, ortostatimo. ==> pérdida 20‐35%VC==> joven, sano, canalizo y vena permeables,  normal en sala de observación.    Si FC >100, TA >15 de pérdida, diaforesis, hipotermina==> 40‐.50% de VC perdido==> cateter venosos ce ntral, terapia intensiva.        Indicación de transfusión:    Hemorragia activa    ==> sangre total    Sangra con tratamiento  ==> sangre total    Caída del 10% del Htc (33%)    ==> concentrado eritrocitario    Dolor torácico    cambios en EKG (isquemia) ==> concentrado eritrocitario  Luego de 10 paquetes de lo que sea debo pasar 1 concentrado plaquetarios (aportan 10,000 plaquetas)    STD con pérdida de 20‐30% de VC 

Joven y adulto joven      Enf ác péptica (úlcera duodenal, enf inflamatria intestinal, pólipos  Aultos